Vous êtes sur la page 1sur 641

GATE – 2016

Question Paper
AglaSem Admission
GATE 2016 General Aptitude - GA Set-2

Q. 1 – Q. 5 carry one mark each.


Q.1 The volume of a sphere of diameter 1 unit is ________ than the volume of a cube of side 1 unit.
(A) least (B) less (C) lesser (D) low

Q.2 The unruly crowd demanded that the accused be _____________ without trial.
(A) hanged (B) hanging (C) hankering (D) hung

Q.3 Choose the statement(s) where the underlined word is used correctly:

(i) A prone is a dried plum.


(ii) He was lying prone on the floor.
(iii) People who eat a lot of fat are prone to heart disease.

(A) (i) and (iii) only (B) (iii) only (C) (i) and (ii) only (D) (ii) and (iii) only

Q.4 Fact: If it rains, then the field is wet.

Read the following statements:


(i) It rains
(ii) The field is not wet
(iii) The field is wet
(iv) It did not rain

Which one of the options given below is NOT logically possible, based on the given fact?
(A) If (iii), then (iv). (B) If (i), then (iii).

(C) If (i), then (ii). (D) If (ii), then (iv).

Q.5 A window is made up of a square portion and an equilateral triangle portion above it. The base of
the triangular portion coincides with the upper side of the square. If the perimeter of the window is
6 m, the area of the window in m2 is ___________.

(A) 1.43 (B) 2.06 (C) 2.68 (D) 2.88

1/3
AglaSem Admission
GATE 2016 General Aptitude - GA Set-2

Q. 6 – Q. 10 carry two marks each.


Q.6 Students taking an exam are divided into two groups, P and Q such that each group has the same
number of students. The performance of each of the students in a test was evaluated out of 200
marks. It was observed that the mean of group P was 105, while that of group Q was 85. The
standard deviation of group P was 25, while that of group Q was 5. Assuming that the marks were
distributed on a normal distribution, which of the following statements will have the highest
probability of being TRUE?
(A) No student in group Q scored less marks than any student in group P.

(B) No student in group P scored less marks than any student in group Q.

(C) Most students of group Q scored marks in a narrower range than students in group P.

(D) The median of the marks of group P is 100.

Q.7 A smart city integrates all modes of transport, uses clean energy and promotes sustainable use of
resources. It also uses technology to ensure safety and security of the city, something which critics
argue, will lead to a surveillance state.

Which of the following can be logically inferred from the above paragraph?

(i) All smart cities encourage the formation of surveillance states.


(ii) Surveillance is an integral part of a smart city.
(iii) Sustainability and surveillance go hand in hand in a smart city.
(iv) There is a perception that smart cities promote surveillance.

(A) (i) and (iv) only (B) (ii) and (iii) only

(C) (iv) only (D) (i) only

Q.8 Find the missing sequence in the letter series.

B, FH, LNP, _ _ _ _.

(A) SUWY (B) TUVW (C) TVXZ (D) TWXZ

Q.9 The binary operation □ is defined as a □ b = ab+(a+b), where a and b are any two real numbers.
The value of the identity element of this operation, defined as the number x such that a □ x = a, for
any a, is .

(A) 0 (B) 1 (C) 2 (D) 10

2/3
AglaSem Admission
GATE 2016 General Aptitude - GA Set-2

| (| |)|
Q.10 Which of the following curves represents the function = ln(| |) for | |<2 ?
Here, represents the abscissa and represents the ordinate.

(A)

(B)

(C)

(D)

END OF THE QUESTION PAPER


3/3
AglaSem Admission
GATE 2016 SET-2 Mechanical Engineering (ME)

Q. 1 – Q. 25 carry one mark each.


Q.1 The condition for which the eigenvalues of the matrix

2 1
𝐴=
1 𝑘
are positive, is
(A) 𝑘 > 1/2 (B) 𝑘 > −2 (C) 𝑘 > 0 (D) 𝑘 < −1/2

Q.2 The values of x for which the function


𝑥 2 − 3𝑥 − 4
𝑓 𝑥 =
𝑥 2 + 3𝑥 − 4
is NOT continuous are
(A) 4 and −1 (B) 4 and 1 (C) −4 and 1 (D) −4 and −1

Q.3 Laplace transform of cos( t ) is

s
(A)
s 2
2


(B)
s 22

s
(C) 2
s 2

(D)
s 2
2

Q.4 A function 𝑓of the complex variable 𝑧 = 𝑥 + 𝑖 𝑦, is given as 𝑓 𝑥, 𝑦 = 𝑢 𝑥, 𝑦 + 𝑖 𝑣 𝑥, 𝑦 , where


𝑢 𝑥, 𝑦 = 2𝑘𝑥𝑦 and 𝑣 𝑥, 𝑦 = 𝑥 2 − 𝑦 2 . The value of k, for which the function is analytic, is _____

Q.5 Numerical integration using trapezoidal rule gives the best result for a single variable function,
which is
(A) linear (B) parabolic (C) logarithmic (D) hyperbolic

Q.6 A point mass having mass M is moving with a velocity V at an angle to the wall as shown in the
figure. The mass undergoes a perfectly elastic collision with the smooth wall and rebounds. The
total change (final minus initial) in the momentum of the mass is

V

(A) −2𝑀𝑉 cos 𝜃 𝑗 (B) 2𝑀𝑉 sin 𝜃 𝑗 (C) 2𝑀𝑉 cos 𝜃 𝑗 (D) −2𝑀𝑉 sin 𝜃 𝑗

ME (Set-2) 1/14
AglaSem Admission
GATE 2016 SET-2 Mechanical Engineering (ME)

Q.7 A shaft with a circular cross-section is subjected to pure twisting moment. The ratio of the
maximum shear stress to the largest principal stress is
(A) 2.0 (B) 1.0 (C) 0.5 (D) 0

Q.8 A thin cylindrical pressure vessel with closed-ends is subjected to internal pressure. The ratio of
circumferential (hoop) stress to the longitudinal stress is
(A) 0.25 (B) 0.50 (C) 1.0 (D) 2.0

Q.9 The forces F1 and F2 in a brake band and the direction of rotation of the drum are as shown in the
figure. The coefficient of friction is 0.25. The angle of wrap is 3/2 radians. It is given that R = 1 m
and F2 = 1 N. The torque (in N-m) exerted on the drum is _________

Q.10 A single degree of freedom mass-spring-viscous damper system with mass m, spring constant k and
viscous damping coefficient q is critically damped. The correct relation among m, k, and q is

2𝑘 𝑘
(A) 𝑞 = 2𝑘𝑚 (B) 𝑞 = 2 𝑘 𝑚 (C) 𝑞 = (D) 𝑞 = 2
𝑚 𝑚

Q.11 A machine element XY, fixed at end X, is subjected to an axial load P, transverse load F, and a
twisting moment T at its free end Y. The most critical point from the strength point of view is

F
T
P
X Y

(A) a point on the circumference at location Y


(B) a point at the center at location Y
(C) a point on the circumference at location X
(D) a point at the center at location X

ME (Set-2) 2/14
AglaSem Admission
GATE 2016 SET-2 Mechanical Engineering (ME)

Q.12 For the brake shown in the figure, which one of the following is TRUE?

(A) Self energizing for clockwise rotation of the drum


(B) Self energizing for anti-clockwise rotation of the drum
(C) Self energizing for rotation in either direction of the drum
(D) Not of the self energizing type

Q.13 The volumetric flow rate (per unit depth) between two streamlines having stream functions 1 and
2 is
(A) 1+2 (B) 12 (C) 1/2 (D) 1−2

Q.14 Assuming constant temperature condition and air to be an ideal gas, the variation in atmospheric
pressure with height calculated from fluid statics is
(A) linear (B) exponential (C) quadratic (D) cubic

Q.15 A hollow cylinder has length L, inner radius r1, outer radius r2, and thermal conductivity k. The
thermal resistance of the cylinder for radial conduction is
ln(r2 / r1 ) ln(r1 / r2 ) 2 kL 2 kL
(A) (B) (C) (D)
2 kL 2 kL ln(r2 / r1 ) ln(r1 / r2 )

Q.16 Consider the radiation heat exchange inside an annulus between two very long concentric cylinders.
The radius of the outer cylinder is 𝑅o and that of the inner cylinder is 𝑅i . The radiation view factor
of the outer cylinder onto itself is
𝑅i 𝑅i 𝑅i 1/3 𝑅
(A) 1 − (B) 1 − (C) 1 − (D) 1 − 𝑅 i
𝑅o 𝑅o 𝑅o o

Q.17 The internal energy of an ideal gas is a function of


(A) temperature and pressure
(B) volume and pressure
(C) entropy and pressure
(D) temperature only

Q.18 The heat removal rate from a refrigerated space and the power input to the compressor are 7.2 kW
and 1.8 kW, respectively. The coefficient of performance (COP) of the refrigerator is ______

ME (Set-2) 3/14
AglaSem Admission
GATE 2016 SET-2 Mechanical Engineering (ME)

Q.19 Consider a simple gas turbine (Brayton) cycle and a gas turbine cycle with perfect regeneration. In
both the cycles, the pressure ratio is 6 and the ratio of the specific heats of the working medium is
1.4. The ratio of minimum to maximum temperatures is 0.3 (with temperatures expressed in K) in
the regenerative cycle. The ratio of the thermal efficiency of the simple cycle to that of the
regenerative cycle is _________

Q.20 In a single-channel queuing model, the customer arrival rate is 12 per hour and the serving rate is
24 per hour. The expected time that a customer is in queue is _______ minutes.

Q.21 In the phase diagram shown in the figure, four samples of the same composition are heated to
temperatures marked by a, b, c and d.

At which temperature will a sample get solutionized the fastest?


(A) a (B) b (C) c (D) d

Q.22 The welding process which uses a blanket of fusible granular flux is
(A) tungsten inert gas welding (B) submerged arc welding
(C) electroslag welding (D) thermit welding

Q.23 The value of true strain produced in compressing a cylinder to half its original length is
(A) 0.69 (B) − 0.69 (C) 0.5 (D) − 0.5

Q.24 The following data is applicable for a turning operation. The length of job is 900 mm, diameter of
job is 200 mm, feed rate is 0.25 mm/rev and optimum cutting speed is 300 m/min. The machining
time (in min) is __________

ME (Set-2) 4/14
AglaSem Admission
GATE 2016 SET-2 Mechanical Engineering (ME)

Q.25 In an ultrasonic machining (USM) process, the material removal rate (MRR) is plotted as a function
of the feed force of the USM tool. With increasing feed force, the MRR exhibits the following
behavior:
(A) increases linearly
(B) decreases linearly
(C) does not change
(D) first increases and then decreases

Q. 26 – Q. 55 carry two marks each.

Q.26 A scalar potential 𝜑 has the following gradient: 𝛻𝜑 = 𝑦𝑧𝑖 + 𝑥𝑧𝑗 + 𝑥𝑦𝑘 . Consider the integral

𝐶
∇𝜑. d𝑟 on the curve 𝑟 = 𝑥𝑖 + 𝑦𝑗 + 𝑧𝑘 .
𝑥=𝑡
The curve C is parameterized as follows: 𝑦 = 𝑡 2 and 1 ≤ 𝑡 ≤ 3.
𝑧 = 3𝑡 2
The value of the integral is ________

Q.27 3 𝑧−5
The value of
Γ
d𝑧 along a closed path Γ is equal to 4 𝜋 𝑖 , where 𝑧 = 𝑥 + 𝑖𝑦 and
𝑧−1 𝑧−2

𝑖 = −1. The correct path Γ is

(A) 𝑦 (B) 𝑦
Γ Γ

𝑥 𝑥
0 1 2 0 1 2

(C) 𝑦 (D) 𝑦

Γ
Γ
𝑥 𝑥
0 1 2 0 1 2

Q.28 The probability that a screw manufactured by a company is defective is 0.1. The company sells
screws in packets containing 5 screws and gives a guarantee of replacement if one or more screws
in the packet are found to be defective. The probability that a packet would have to be replaced
is _________

ME (Set-2) 5/14
AglaSem Admission
GATE 2016 SET-2 Mechanical Engineering (ME)
𝜋
Q.29 The error in numerically computing the integral 0 (sin 𝑥 + cos 𝑥) d𝑥 using the trapezoidal rule
with three intervals of equal length between 0 and 𝜋 is ___________

Q.30 A mass of 2000 kg is currently being lowered at a velocity of 2 m/s from the drum as shown in the
figure. The mass moment of inertia of the drum is 150 kg-m2. On applying the brake, the mass is
brought to rest in a distance of 0.5 m. The energy absorbed by the brake (in kJ) is __________

2m

2 m/s

2000 kg

Q.31 A system of particles in motion has mass center G as shown in the figure. The particle i has mass mi
and its position with respect to a fixed point O is given by the position vector ri. The position of the
particle with respect to G is given by the vector i. The time rate of change of the angular
momentum of the system of particles about G is

(The quantity 𝝆𝑖 indicates second derivative of 𝝆𝑖 with respect to time and likewise for 𝒓𝑖 ).

(A) 𝑖 𝒓𝑖 × 𝑚𝑖 𝝆𝑖 (B) 𝑖 𝝆𝑖 × 𝑚𝑖 𝒓𝑖
(C) 𝑖 𝒓𝑖 × 𝑚𝑖 𝒓𝑖 (D) 𝑖 𝝆𝑖 × 𝑚𝑖 𝝆𝑖

ME (Set-2) 6/14
AglaSem Admission
GATE 2016 SET-2 Mechanical Engineering (ME)

Q.32 A rigid horizontal rod of length 2L is fixed to a circular cylinder of radius R as shown in the figure.
Vertical forces of magnitude P are applied at the two ends as shown in the figure. The shear
modulus for the cylinder is G and the Young’s modulus is E.

P
L

R
L
P
L
A

The vertical deflection at point A is


(A) 𝑃𝐿3 /(𝜋𝑅 4 𝐺) (B) 𝑃𝐿3 /(𝜋𝑅 4 𝐸) (C) 2𝑃𝐿3 /(𝜋𝑅 4 𝐸) (D) 4𝑃𝐿3 /(𝜋𝑅 4 𝐺)

Q.33 A simply supported beam of length 2L is subjected to a moment M at the mid-point x = 0 as shown
in the figure. The deflection in the domain 0 ≤ x ≤ L is given by

−𝑀𝑥
𝑤= 𝐿−𝑥 𝑥+𝑐 ,
12 𝐸𝐼𝐿

where E is the Young’s modulus, I is the area moment of inertia and c is a constant (to be
determined) .
y
M

L L
The slope at the center x = 0 is
(A) 𝑀𝐿/(2𝐸𝐼) (B) 𝑀𝐿/(3𝐸𝐼) (C) 𝑀𝐿/(6𝐸𝐼) (D) 𝑀𝐿/(12𝐸𝐼)

ME (Set-2) 7/14
AglaSem Admission
GATE 2016 SET-2 Mechanical Engineering (ME)

Q.34 In the figure, the load P = 1 N, length L = 1 m, Young’s modulus E = 70 GPa, and the cross-section
of the links is a square with dimension 10 mm × 10 mm. All joints are pin joints.

The stress (in Pa) in the link AB is ___________

(Indicate compressive stress by a negative sign and tensile stress by a positive sign.)

Q.35 A circular metallic rod of length 250 mm is placed between two rigid immovable walls as shown in
the figure. The rod is in perfect contact with the wall on the left side and there is a gap of 0.2 mm
between the rod and the wall on the right side. If the temperature of the rod is increased by 200 o C,
the axial stress developed in the rod is __________ MPa.

Young’s modulus of the material of the rod is 200 GPa and the coefficient of thermal expansion is
10−5 per oC.

ME (Set-2) 8/14
AglaSem Admission
GATE 2016 SET-2 Mechanical Engineering (ME)

Q.36 The rod AB, of length 1 m, shown in the figure is connected to two sliders at each end through pins.
The sliders can slide along QP and QR. If the velocity VA of the slider at A is 2 m/s, the velocity of
the midpoint of the rod at this instant is ___________ m/s.

Q.37 The system shown in the figure consists of block A of mass 5 kg connected to a spring through a
massless rope passing over pulley B of radius r and mass 20 kg. The spring constant k is 1500 N/m.
If there is no slipping of the rope over the pulley, the natural frequency of the system
is_____________ rad/s.

Q.38 In a structural member under fatigue loading, the minimum and maximum stresses developed at the
critical point are 50 MPa and 150 MPa, respectively. The endurance, yield, and the ultimate
strengths of the material are 200 MPa, 300 MPa and 400 MPa, respectively. The factor of safety
using modified Goodman criterion is
3 8 12
(A) (B) (C) (D) 2
2 5 7

ME (Set-2) 9/14
AglaSem Admission
GATE 2016 SET-2 Mechanical Engineering (ME)

Q.39 The large vessel shown in the figure contains oil and water. A body is submerged at the interface of
oil and water such that 45 percent of its volume is in oil while the rest is in water. The density of
the body is _________ kg/m3.

The specific gravity of oil is 0.7 and density of water is 1000 kg/m3.

Acceleration due to gravity g = 10 m/s2.

Q.40 Consider fluid flow between two infinite horizontal plates which are parallel (the gap between them
being 50 mm). The top plate is sliding parallel to the stationary bottom plate at a speed of 3 m/s.
The flow between the plates is solely due to the motion of the top plate. The force per unit area
(magnitude) required to maintain the bottom plate stationary is _________ N/m2.

Viscosity of the fluid µ = 0.44 kg/m-s and density = 888 kg/m3.

ME (Set-2) 10/14
AglaSem Admission
GATE 2016 SET-2 Mechanical Engineering (ME)

Q.41 Consider a frictionless, massless and leak-proof plug blocking a rectangular hole of dimensions
2𝑅 × 𝐿 at the bottom of an open tank as shown in the figure. The head of the plug has the shape of
a semi-cylinder of radius R. The tank is filled with a liquid of density  up to the tip of the plug.
The gravitational acceleration is g. Neglect the effect of the atmospheric pressure.

g
R R R

L
F F
Sectional view A-A
A
The force F required to hold the plug in its position is

𝜋 𝜋
(A) 2𝜌𝑅 2 𝑔𝐿 1 − (B) 2𝜌𝑅 2 𝑔𝐿 1 +
4 4
𝜋
(C) 𝜋𝑅 2 𝜌𝑔𝐿 (D) 2
𝜌𝑅 2 𝑔𝐿

Q.42 Consider a parallel-flow heat exchanger with area 𝐴p and a counter-flow heat exchanger with area
𝐴c . In both the heat exchangers, the hot stream flowing at 1 kg/s cools from 80 C to 50 C. For the
cold stream in both the heat exchangers, the flow rate and the inlet temperature are 2 kg/s and
10 C, respectively. The hot and cold streams in both the heat exchangers are of the same fluid.
Also, both the heat exchangers have the same overall heat transfer coefficient. The ratio 𝐴c /𝐴p
is _________

Q.43 Two cylindrical shafts A and B at the same initial temperature are simultaneously placed in a
furnace. The surfaces of the shafts remain at the furnace gas temperature at all times after they are
introduced into the furnace. The temperature variation in the axial direction of the shafts can be
assumed to be negligible. The data related to shafts A and B is given in the following Table.

Quantity Shaft A Shaft B


Diameter (m) 0.4 0.1
Thermal conductivity (W/m-K) 40 20
Volumetric heat capacity (J/m3-K) 2×106 2×107

The temperature at the centerline of the shaft A reaches 400 C after two hours. The time required
(in hours) for the centerline of the shaft B to attain the temperature of 400 C is _______

Q.44 A piston-cylinder device initially contains 0.4 m3 of air (to be treated as an ideal gas) at 100 kPa
and 80 oC. The air is now isothermally compressed to 0.1 m3. The work done during this process
is ________ kJ.

(Take the sign convention such that work done on the system is negative)

Q.45 A reversible cycle receives 40 kJ of heat from one heat source at a temperature of 127 C and 37 kJ
from another heat source at 97 C. The heat rejected (in kJ) to the heat sink at 47 C is __________

ME (Set-2) 11/14
AglaSem Admission
GATE 2016 SET-2 Mechanical Engineering (ME)

Q.46 A refrigerator uses R-134a as its refrigerant and operates on an ideal vapour-compression
refrigeration cycle between 0.14 MPa and 0.8 MPa. If the mass flow rate of the refrigerant is 0.05
kg/s, the rate of heat rejection to the environment is _________ kW.

Given data:
At P = 0.14 MPa, h  236.04 kJ/kg, s  0.9322 kJ/kg-K
At P = 0.8 MPa, h  272.05 kJ/kg (superheated vapour)
At P = 0.8 MPa, h  93.42 kJ/kg (saturated liquid)

Q.47 The partial pressure of water vapour in a moist air sample of relative humidity 70% is 1.6 kPa, the
total pressure being 101.325 kPa. Moist air may be treated as an ideal gas mixture of water vapour
and dry air. The relation between saturation temperature (𝑇s in K) and saturation pressure (𝑝s in
kPa) for water is given by ln(𝑝s /𝑝𝑜 ) = 14.317 − 5304/𝑇s , where 𝑝𝑜 = 101.325 kPa. The dry
bulb temperature of the moist air sample (in C) is __________

Q.48 In a binary system of A and B, a liquid of 20% A (80% B) is coexisting with a solid of 70% A
(30% B). For an overall composition having 40% A, the fraction of solid is

(A) 0.40 (B) 0.50 (C) 0.60 (D) 0.75

Q.49 Gray cast iron blocks of size 100 mm × 50 mm × 10 mm with a central spherical cavity of diameter
4 mm are sand cast. The shrinkage allowance for the pattern is 3%. The ratio of the volume of the
pattern to volume of the casting is _________

Q.50 The voltage-length characteristic of a direct current arc in an arc welding process is
𝑉 = 100 + 40𝑙 , where 𝑙 is the length of the arc in mm and V is arc voltage in volts. During a
welding operation, the arc length varies between 1 and 2 mm and the welding current is in the range
200-250 A. Assuming a linear power source, the short circuit current is_________ A.

Q.51 For a certain job, the cost of metal cutting is Rs. 18𝐶/𝑉 and the cost of tooling is Rs. 270𝐶/(𝑇𝑉),
where 𝐶 is a constant, 𝑉 is the cutting speed in m/min and 𝑇 is the tool life in minutes. The Taylor's
tool life equation is 𝑉𝑇 0.25 = 150. The cutting speed (in m/min) for the minimum total cost
is _______

ME (Set-2) 12/14
AglaSem Admission
GATE 2016 SET-2 Mechanical Engineering (ME)

Q.52 The surface irregularities of electrodes used in an electrochemical machining (ECM) process are
3 µm and 6 µm as shown in the figure. If the work-piece is of pure iron and 12V DC is applied
between the electrodes, the largest feed rate is ___________mm/min.

Conductivity of the electrolyte 0.02 ohm-1mm-1


Over-potential voltage 1.5 V
Density of iron 7860 kg/m3
Atomic weight of iron 55.85 gm

Assume the iron to be dissolved as Fe+2 and the Faraday constant to be 96500 Coulomb.

Q.53 For the situation shown in the figure below the expression for H in terms of r, R and D is

(A) 𝐻= 𝐷 + 𝑟2 + 𝑅2
(B) 𝐻= 𝑅 + 𝑟 + (𝐷 + 𝑟)
(C) 𝐻= 𝑅 + 𝑟 + 𝐷2 − 𝑅2
(D) 𝐻= 𝑅 + 𝑟 + 2𝐷 𝑅 + 𝑟 − 𝐷 2

Q.54 A food processing company uses 25,000 kg of corn flour every year. The quantity-discount price of
corn flour is provided in the table below:

Quantity (kg) Unit price (Rs/kg)


1-749 70
750-1499 65
1500 and above 60

The order processing charges are Rs. 500/order. The handling plus carry-over charge on an annual
basis is 20% of the purchase price of the corn flour per kg. The optimal order quantity (in kg)
is _____________

ME (Set-2) 13/14
AglaSem Admission
GATE 2016 SET-2 Mechanical Engineering (ME)

Q.55 A project consists of 14 activities, A to N. The duration of these activities (in days) are shown in
brackets on the network diagram. The latest finish time (in days) for node 10 is __________
E(4)
3 10

J(2) M(3)
B(3) 6 8
K(3)
F(2)

A(2) L(2) N(2)


C(4) 9 11 12
1 2 4
G(4) I(5)

D(2) 7 H(4)

END OF THE QUESTION PAPER

ME (Set-2) 14/14
GATE Previous Year
Solved Paper
Mechanical
(Fully Solved)

GATExplore.com

(2013, 2015, 2016, 2017)


Free Download
|ME-GATE-2013 PAPER|

∂ ∂ ∂
+ =
∂ ∂ ∂

! " #$$ %$
$ & " $ ! #
$ & ! " $ ! #
'
( $ ! #

) * % +'',
!
( -$ . )/0 ' 1
2 & ' - %
' -$ . /) ' )
( 32 3' ) ( )3 2 3' ( 32 3' ) ( )3 2 3'

4 ! " ! $ % % 56 %
% ( ! 5 7 8 !. $ $
" # $ , , ! ! 9 $
" % --
-
, ,

( (

-
/

( ( (, − , ) (, (,
(, + , ) , ,

: 5 $ $ ! 56 % %
7 % !# 5 4 $ 5
$ $ 5 $$9 $ $ 5 $$
; 4 ); 40

GATEFORUM- India’s No.1 institute for GATE training


|ME-GATE-2013 PAPER|

(
, ,
= =
, (
,

4=

= × = 4

; < + ! % ! " % # / %
% %< % 5 %< ! # $
" % # = >: $/ % 9 ! 5 7
+ ? $9 $ ! 5 %% 5 %< %
$/

) 4 : ;

,
α = "ω = × = >: × =)
"
α = =4

∴' α= ) +4 =:

> 1 #9 # " ! $ 9 ! % !
! " # % 9 ∆ " ! τ
5#
∆ ∆ ∆ 4∆
τ = τ = τ = τ =
4 4

,
π ∆
τω ( π ) =4 ⋅∆ τω =
4

GATEFORUM- India’s No.1 institute for GATE training


|ME-GATE-2013 PAPER|

0 9 # 5 5 $ 9 " $ # $
5 9 )=@ >=A % " # 7 $ )=@
4 :<( 9 % % $ # $ <! " / <! #
= ==0) == = == B == )

,
(= 5 = =: ( = == C(

( =4 :C(

=; ×= >×4 :
- $ #= === B
( == − 4 :)

B $ # % % 9 ! 9
3 5 # % ! 5
% % " # !" 5# < = < = + 5 3 <= 5 "
% $

>

% 9 $ ! ( / )
' $ % & 7 % %

= 7 ! % 9 $ ! ! $
( % $ (
$

* % +'',
! ## # # # $ %&&
( 1 % D ! % $ 5 %
2 E * %D ! F ! %<
' ! 7 E D ! ) $ 5 %
- , % !D ! 4 F ! %# % $ $
( 432 )3' 3- ( 432 3' )3-
( 32 )3' 43- ( 32 43' 3- )

GATEFORUM- India’s No.1 institute for GATE training


"
|ME-GATE-2013 PAPER|

$ % % $$ ! ;=@ % %
$ !) $ $ 5 & $$
= 0;; === :4 ===

, 1 ;= ! 95 & ?= :> ): (? :4

) $ " %< % := %<


" " ! $ < ! %<
$ $ ! % $ " $ ( % "% $
# 5 9 " ! ! $ $
) 4 : ;

,
λ
λ = := / µ = ;= / D = = = =0) = :$
µ (µ − λ )

4 7 $ $ ! % !9 !" ! ! ! %
$ $ 9 $ !% $ 5 %
& =:

, G α% " $ ! % !" !
! % % !

: 5 == $$ $ = : $$/ "
% 4 $$ < % ;= $ $
$ " $$) /
;= ;> ; ;== ;>: :

, × ×"
π × == × ;=
= (= :) ( 4 ) × = ;>: :
;=

; % 5 % ! : $ % $ $
% 5 $ $ 9 % 0 $ " ! % !
5
= = 4 4=

G G
, = 3 =:=

G = 0G
D % $ % ! ! 5 - =4

G G
=% =% = 4× = 4×: = =$
4

GATEFORUM- India’s No.1 institute for GATE training


'
|ME-GATE-2013 PAPER|

> 1 % $ 9 ! $
' % % % !
5 # 5 5 !# %
5 # 5 5 !# % $
' %

, - % 9 ! 5 # 5 5 !# %

0 7 " $ $ $ $ ( 5 9 5 %< $
5
, 6 , 6
, % 6 7 6

, - % 9" % # 5 %< ?H $ " % # 6

B $ # % % ! (= ≤ ≤ )9
! 5 # % ( == ? ) $
$ =@ ==@ I ! $# !
+'', $
% 5 $ % ==@ %
=@
$ $ $ $ $ 5 ! ==@
$ 5
$ 5 #$$ % 5 $

= %# % :$) ! 5 !
% $ " 5 $ % % : 5
< <F %
0=4 > B:) B0 > =

(
, (G =
(

:
= =: × : = 0=4> 0 B: = 0=4 > <F

! %# % 9% 5 9 56 %
% % $ !
"
=: = = 4=

GATEFORUM- India’s No.1 institute for GATE training


|ME-GATE-2013 PAPER|

(
, σ =

(
σ ! =
4
σ
=
σ !

7 5 " %# 0== $ ! ! $ #
! ! 9 % % $
== 4:= ;== B==

, - % $ # % % 5

) % < $ %% $ ! < (2 ? =$9 2' ? ) =$9


'- ? :$ -( ? >$ " 6 < 5 5
5 %< %< %< $ % $
(2 2' '- -(

, - % ' %< J ' %< $ % $ < $ <


$ 5

4 K5 $ " 5 $ " % 4 5 5 # ( ( K < =)


=:
E & =:
E =:
=

−µ =−µ
, (( < =) = ( < − ( ( L < −= :)
σ σ
= ( ( L > = :) = = : − ( ( = < L < = :) 9
% ! & =:

: +'', % 9 % #
9 % % 9
% 9 % % 9 %

,
% =% −

∴% % $5 % ∴% 9

∴ % % 9 % #

GATEFORUM- India’s No.1 institute for GATE training


(
|ME-GATE-2013 PAPER|

( *+ #

; ! % ! 5 " " !"


# " % # " % 9 1 = + #6 + &< %
% # $ " ! 9 69 < 5 " %

1
4
D - 9 +# +& = $ " %
" % !
π
π π ) 4π
4

, "1 " M ! " ! % $


4 ∨

)
= ) "= ×" $
4 ∨
4
) 4 )
= × × =π =
4 )

> % + == #

% 9 = = =9 = =4 % $ !" 5#

4 4
4
+ + + +

,
E" , N + ==

N + = =
O
1 − = − = +1 ==
( + )1 =4
4
=
+

0 - % % # " % # $ $ 5 9
! # 9 !" 5
- % % # <F/<! G % # $/
7 $% ) := 0=
, $% );= :
$ 5 <! $ : <D
! % ! % ! !# $9 " <D 5#
$ 5 <! $ 9
B= B >=> : >=0 :

GATEFORUM- India’s No.1 institute for GATE training


)
|ME-GATE-2013 PAPER|

,
G G
+ + 2= + +

= () := − );= ) +
( 0= − : ) +:
= >=> : <D

B D % $ ! $ " % #
!9 $ $ =$$ $ " % # $/
%% ! " # B 0 $/ $ ! #9 " % 9
% $ % " # ! % ! % " %
% 9 $ $$ $ = :$ 5
$ #
= : = :

)= 5 $ ;= $$ # $ 5 $ =)= °
% 7 # $ " $ % % $5
)= ° 9 % " % " % % ? = D /$ C $
)
# % # ρ = >0== <! / $ 9 % % " #
D
< = 4= % % %?;== F/<!C $ % %
$C
5 $ =)= ° 4)= °
: B B) B: 44

,

− % - G G
= = = =
− %

4)= − )= −
> == × = = × ;==
=
=)= − )=

= 44

) # % % % ! $ % # !# 4== $
! $ ! = / 7 %
% ± A9 $ $ $ #
<! − $
: := == :

,
, =7 -
4==
7= = :<! − $
= × = =4

GATEFORUM- India’s No.1 institute for GATE training


,
|ME-GATE-2013 PAPER|

) % $ ! ! (9 29 ' - $5 =9 4=9
: =9 % " # E 2 ' $ $
! 5 $ ! 2 % !
' 7 $ ! ' $$9 % % $$ 5
! ( - 2
4= -
(
0=
+ + +
=
;= '

' =$× ' = × : = )=

2= ' = ;=
( ( =
= ( = × ;= = )=
2 2 4=

' ' - )= × =
= = '× = = 4=
- - ' :

( 2 ' -
% % = + + +

= : + )= + : + = = 0=

)) 6 $ ! ! D ! & #
5# % 1 ! 5 % 1 #
$ " % $ " 9 $ ! " % %
"

D D
& D
4

'

D
7= 3 D× = 7α
) !
)!
α=

GATEFORUM- India’s No.1 institute for GATE training


-
|ME-GATE-2013 PAPER|

)!
∴ %% % ? α× =
4
)
∴7 % % D?
4
)D D
∴' % =D− =
4 4

)4 % ! 5 ! % $ $ % !

;
5 G = )===
= ;
I-- G = ==

D G % ! $/$ $ % 5
" ! % ! $/$ %

:= )B 4 4B ) ;= =

,
;
G
=;
= :
G
= :
;
G × ( :) = )===
G = )B 4

): 7 %< ! 9 $ $ ! ( :9 = 5 5 5
% ! ! $ < ! 4: ° % %<
K % $ 5

> :9 : =9 : > :9 : =9 :

); ! $$ ! 5 $ 5
* $& ) + >#

) + ># ≤ =
- 56 % 4 + ;# ≤ 0
9# ≥ =

5 56 % " % % # $

% # $ #$ # $

GATEFORUM- India’s No.1 institute for GATE training


.
|ME-GATE-2013 PAPER|
#

, =9 4

=9 ))
) + ># ≤ =
4 + ;# ≤ 0

9= ) ))9=

+= = =
)> # % : += = = $$ $ % %<
+ =
! )= $% ! % ! ! ! % 9 & E+ ! !
$$ % % $
: =4 : =: : =>4 : =04

+= = =

, : += = =
+ = =)±= ==
+= =
+= =
: =)

E ! ! ?$ $ ? : =:

)0 ! % % $% $ % ! , * $% ! ?:;9
" %#? % === B=A % % %#9 $
$ " 5 " 5 = ; !$/ 7 $ $% ! ? 409
" %#?) $ % 5# , * % % === B=A
% % %#9 % $ $ " !$/ 5
= = ) = )= =:

,
7 = B × 40 × ===
2 = =
1 ) × B;:== × )
2 == )

)B ! ! $ # $ " ! $ <! =< /$


# 56 % $ % $ ! : <
= −4 % $ $$ ! "5
=: = := ==

4= 5 56 % % ! $ =< == < $
# ! 4= *( % $ " 5 ! ;=
*( %% ! - 5 ! % 9 % % $$
5 % #
4== ;== >:= ===

GATEFORUM- India’s No.1 institute for GATE training


|ME-GATE-2013 PAPER|

σ$ σ"
+ =
σ# σ 1-

;= 4=
=== + =
× 4= × ;=
= ===

4 $ # 5 $ ! 56 % " # ! 5
)π $− 9 % $ $

$ ! " % %

&
)π π π


, =
$


=
=

− )π
= % −% = = +
)π )π

- % 5 # 5

' =' = =

4 ! ! # 9 5# $ % 9 "
% $ 4== C )== C 7 $ " %
=0 - &$ % : ;> × = −0 D / $ C 4 9
% ! <D / $ 5
= ;; = >B = BB ) B;

2=
σ ( 4
− 4
)3 ε =ε ==0
+ −
ε ε

2 = = ;; <D /

4) ! ! ! : $9 % )= ° &
$ 9 ! 5 # === <! / $
$ $ $ $ ! <! %
9 < %

GATEFORUM- India’s No.1 institute for GATE training


|ME-GATE-2013 PAPER|

:$

:=== ;;== >:4; B; )

,
5 $$ $ ! P=.? * $ % P+.
* $ +9 × =1 ×#
1 = ! 3 = ! = :×
= % E E $ %

θ = )= °3 )= ° = = :
: #
7! 0$ θ
= % ? Q
:$
5 )
× :) :
7 = = = = 4
:
= 4 ×
= :+ = ;;
(: × ) :
1
% θ= = ;:
:
;;;
θ= # = =) )
# )=
× ;: = × ! × × # = === × B 0 × : × × :×) )
= B4= B
= $! B:04 B; )

44 9 $ " % # ! : 5 9 :== C
:= $/ 9 % " # % % %
% " $ ==: <F/<!C = > 0 <F/<!C9 % " # ! %
!# 7 $ ! 5
)== C9 % " #9 " 5 !# <F/<! $
>= 0> B )

4: 5 5 # < % % $ % %
7 < 9
)
! 5 5 # ! 5 ! % %
4
E" % % #9 %
5 5 # < % %
) : 0
) 4 ; B

GATEFORUM- India’s No.1 institute for GATE training


"
|ME-GATE-2013 PAPER|

? % % #
, =- < % %

, =- ! % %

, = 9 , =
) )

= , × + , ×
, ,
)
= × + × =
) ) 4 4
,
M ! # $9

(, ) × , ×
0
=) =
) B
4

4; −< == < % 9

56 % 5 #% = ?= ?M9
<

=M =M <

−< −<
− +
=M −<
=M −<
− +

−C ==

−< == −C = =
= =9 =C
= <
= +
<
= +
= ==
∴ + ==
=
<
= + =M
" !
M < −M
= 9
− < − <
− <
=M
− <

GATEFORUM- India’s No.1 institute for GATE training


'
|ME-GATE-2013 PAPER|

4> " !

4 ) ) 4 ) 4 4 )
+ − + −
B B B B B B B B

) )

= × − ×
)
)
) )
4
= × × − ×
) B
) 4
= +
B B

/ # #0 ', 1 '-

! " 6 $ ! 5
!! $ % $

( (

D ? == $$9 %< ?: $$9 $5 "


?)9 $ " = = $$9 $ " = $$9
5 σ = == *( 9 5
" σ = == *( 5 5 ! σ % = := *(

40 7 " 5 ! " % ! 9 $ $ $
$ 5 ( <
> := : == := )= ==

= σ% × × × = := × ) × = × : = :<

4B 7 5 ! " ! 9 $ $ $
$ 5 ( <
0) : ;> :=

= σ ×( −) )× = == × ( == − ) × )×: = ;><

GATEFORUM- India’s No.1 institute for GATE training


|ME-GATE-2013 PAPER|

/ # #0 .1

D % % 9 % =4 0 <F / <!C == <!/


$ =° 9 $ := $$ ! )$9
N D
56 %
$
N
:= 7 = :== 9 $ % ?= 9
5 <$ $ " ! °
4 ; >4 =4

N
, = :==
N
== ==
N
=) = :== × ) = >:==
= + >:==
"! = = )>:=

"! × = $% × ∆
)>:= × π × = =: × ) = = = × 4 0 × =) − =
=4 + = = ; °

N
: 7 = :===9 % " % % %
=== D / $ C 9 $ ° %

> >; >B 0

N
, = :=== %
N
× =* (∆
2 = :=== × π × = =: × ) = = = × 4 0 × =) − =
= − = = :; ) = = >; )°
I 5 # % $
2 = −
2 N
= = :=== = −

:=== = === − >; )


= >; ) + : = 0 )

2 % #+ # #0 *+ #

2 % #+ # #0 1 "

7 ! ! 5 == $$ $ = : $$/ "9
% 4 $$ % ! " % # B= $/$ 9 5 "
$ ! % ! % % % % % !
% % $ ! % ! % :==

GATEFORUM- India’s No.1 institute for GATE training


(
|ME-GATE-2013 PAPER|

: ! < ! % ! !
L ) :0 : > ;

, % ! 1 −1
' < ! α==

:) $ % % ! % %
=== :== === :==

, $ % = 1I % α − 1G α

= :== × % = − 1" =
= :==

2 % #+ # #0 '1

7 $ # %#% 9 0 $
% % $ 5 )== C 4== C9 % " #
% $ ! 5 " % % % =0 1
! 9 $ % " % % % %
<F/<!C % % 4 ! % % ! < %
!

:4 5# % $ <D/<! !
B4 > 4) 4 )=4 ) )>0 :

,
= 03 = )==C3 ) = 4==C
γ= 4
=4
γ− )
=( ( ) =( ) 4 = :4) 4)C − O
γ
− :4) 4) − )
η = =0= 4
− − )== 4
γ− =4
γ 4
4
= ;=4 B <3 = =
) ( 0
4 = >> 0;<

) − 4 4 − 4
η = =0=
) − 4 4== − >> 0;

4 = 0B0 B

= ( ( − )= × ( ;=4 B − )== ) = )=4 )

GATEFORUM- India’s No.1 institute for GATE training


)
|ME-GATE-2013 PAPER|

:: $ % %# %#% % ! A
40 )0 ; 44 0 :)

= ( ( ) − 4 )= ×( 4 − 0B0 B ) = := >


$ % %# =
θ := >
:B: > − )=4 ) := > − )=4 )
= = = 4 0A
( ( ) − ) × ( 4== − ;=4 B )

( (.

:; $ %
M " $ % $ RRRRRRRR
% % # # ! #

#$

:> D # 5 9# RRRRRRRRRRR <#


# #
# "

:0 D % ! % $ ! !"
5 S

I! * $ 7 !

% "

:B ! $$ % #7 +'', %
I !
#5 ! %
- ,
#$! $7

;= D 5 $ $ $ $ 449 4 9 4=9 S
:= :=4 :=; :==

GATEFORUM- India’s No.1 institute for GATE training


,
|ME-GATE-2013 PAPER|

$ $ $ $ $ 449 4 9
$ P . $ (

= +( − )
7 % 9 ? 9 ? ?
∴- $= 4+ × = × 4; = :=;

( ( *+ #

; + ! ! 5 ==9 ! $5 5
% $ D 5 5 # % $5
" 5 5# >S
)/B= /B= >0/B= >>/B=

$5 ! $ >? )
∴ ( 5 5 # % ! $5
B= − ) >>
" 5 5# > = =
B= B=

; % " 6 # 5# ;= <$/ 9 $ 5#
5 )= <$/ 5# %#% = <$/ " !
<$/ ! 6 #
); )= 4 0

% % " 5 P .

9 " ! ?
$ <
=
= = = = 4 <$ /
:
+ +
+ 4 + 4 = = 4=
; )= =

;) 1 $

+ + + +
+ + ) )+ 4 0= + 0

> 0 B =

GATEFORUM- India’s No.1 institute for GATE training


-
|ME-GATE-2013 PAPER|

% 5

( ) − ( ) + ( )) − ( ) +−−−−−
( 0 ) −( 0= )
+ + ) 0= + 0

=
( − )( + )+−−−−−−+( 0 − 0= )( 0 + 0= )
( + ) 0= + 0

;4 % % % % ' )9 == 7 5 !
# % 5# /: % ! < ! % 5#
/ 4 % 9 % % '
;9:== :9 0= 9=== =9 =

PD. 5 5 ! 9 P .5 < !
9 % % D×
7 % ! ? =A
∴ ' " ! ? D
==
% 5 $ ? A
4
)
∴ ' " $ = − =
4 4
)
∴ ' " % = × D = :D
4
= : × ) == = : 0=

;: " 9 ' 5 % % $$
% F 5 % $$ ! % 9 % $ % $ !
# " $ ! 5
$ $ < ! $ - % $ 5# $ %
% 9 % 5 " ! $ 5 !
" M $ #9 %% " ! "
- % % % ! % % $$ %
9 % 5 %< ! $ # 5 9 #
D % ! 5 5# 5 " $ S
1 %%
I # 5 %#
5 ! "
" #6 !" !

GATEFORUM- India’s No.1 institute for GATE training


.
ME-GATE-2015 PAPER-01| www.gateforum.com

General Aptitude
Q. No. 1 – 5 Carry One Mark Each

1. What is the adverb for the given word below?


Misogynous
(A) Misogynousness (B) Misogynity
(C) Misogynously (D) Misogynous

Answer: (C)

2. Ram and Ramesh appeared in an interview for two vacancies in the same department. The
probability of Ram‟s selection is 1/6 and that of Ramesh is 1/8. What is the probability that
only one of them will be selected?
47 1 13 35
A  B C  D
48 4 48 48

Answer: (B)
Exp: P  Ram   1 ; p  Ramesh   1
6 8
7 1 5
p  only at   p  Ram   p  not ramesh   p  Ramesh   p  n 0  R am   1   
6 8 8 6
12 1
 
40 4

3. Choose the appropriate word/phrase, out of the four options given below, to complete the
following sentence:
Dhoni, as well as the other team members of Indian team, _____ present on the occasion.
(A) were (B) was (C) has (D) have
Answer: (A)

4. An electric bus has onboard instruments that report the total electricity consumed since the
start of the trip as well as the total distance covered. During a single day of operation, the bus
travels on stretches M, N, O and P, in that order. The cumulative distances travelled and the
corresponding electricity consumption are shown in the table below.

Stretch Comulative Electricity


distance(km) used (kWh)
M 20 12
N 45 25
O 75 45
P 100 57

The stretch where the electricity consumption per km is minimum is


(A) M (B) N (C) O (D) P
Answer: (B)

 India’s No.1 institute for GATE Training  1 Lakh+ Students trained till date  65+ Centers
across India
1
ME-GATE-2015 PAPER-01| www.gateforum.com

Exp: For M  12 20  6

N  25  555
45
O  45 6
75
P  57  57
200

5. Choose the word most similar in meaning to the given word:


Awkward
(A) Inept (B) Graceful (C) Suitable (D) Dreadful

Answer: (D)

6. In the following sentence certain parts are underlined and marked P, Q and R. One of the
parts may contain certain error or may not be acceptable in standard written communication.
Select the part containing an error. Choose D as your Answer: if there is no error.
The student corrected all the errors that the instructor marked on the answer book
P Q R
(A) P (B) Q (C) R (D) No Error

Answer: (B)
Exp: The is not required in „Q‟

7. Given below are two statements followed by two conclusions. Assuming these statements to
be true, decide which one logically follows.
Statement:
I. All film stars are playback singers.
II. All film directors are film stars.
Conclusions:
I. All film directors are playback singers.
II. Some film stars are film directors.
(A) Only conclusion I follows
(B) Only conclusion II follows
(C) Neither conclusion I nor II follows
(D) Both conclusions I and II follow
Answer: (D)

8. Lamenting the gradual sidelining of the arts in school curricula, a group of prominent artists wrote to
the Chief Minister last year, asking him to allocate more funds to support arts education in
schools. However, no such increase has been announced in this year‟s Budget. The artists
expressed their deep anguish at their request not being approved, but many of them remain
optimistic about funding in the future.

 India’s No.1 institute for GATE Training  1 Lakh+ Students trained till date  65+ Centers
across India
2
ME-GATE-2015 PAPER-01| www.gateforum.com

Which of the statement(s) below is/are logically valid and can be inferred from the above
statements?
i. The artists expected funding for the arts to increase this year.
ii. The Chief Minister was receptive to the idea of increasing funding for the arts.
iii. The Chief Minister is a prominent artist.
iv. Schools are giving less importance to arts education nowadays.
(A) iii and iv (B) i and iv
(C) i, ii and iv (D) i and iii
Answer: (C)

If a  b  c  1, then ab + bc + ac lies in the interval


2 2 2
9.
 2  1   1
A 1, 3   B  2 ,1  C  1, 2   D 2, 4
     

Answer: (B)

10. A tiger is 50 leaps of its own behind a deer. The tiger takes 5 leaps per minute to the deer‟s 4.
If the tiger and the deer cover 8 metre and 5 metre per leap respectively, what distance in
meters will the tiger have a run before it catches the deer?

Answer: 800
Exp: Tiger  1 leap  8 meter
Speed  5leap hr  40m min
Deer  1 leap  5meter
speed  4hr  20m min
Let at time „t‟ the tiger catches the deer.
 Distance travelled by deer + initial distance between them
50  8  400m = distance covered by tiger.
 40  t  400  20t
400
t  20 min
200
 total dis tance  400  20  t  800 ms

 India’s No.1 institute for GATE Training  1 Lakh+ Students trained till date  65+ Centers
across India
3
ME-GATE-2015 PAPER-01| www.gateforum.com

Mechanical engineering

4 7 8
1. If any two columns of a determinant P  3 1 5 are interchanged, which one of the following
9 6 2
statements regarding the value of the determinant is CORRECT?
(A) Absolute value remains unchanged but sign will change
(B) Both absolute value and sign will change
(C) Absolute value will change but sign will not change
(D) Both absolute value and sign will remain unchanged
Answer: (A)
Q
2. A wheel of radius r rolls without slipping on a horizontal surface
shown below. If the velocity of point P is 10 m/s in the
horizontal direction, the magnitude of velocity of point Q (in P
m/s) is ______.
r
Answer: 20
A
Exp: t
At p 'A ', the velocity v
 10  10  20 m s
v

v0

3. Which one of the following types of stress-strain relationship best describes the behavior of
brittle materials, such as ceramics and thermosetting plastics, (σ = stress and ε = strain)?

A  B
 

 

C  D
 



Answer: (D)

 India’s No.1 institute for GATE Training  1 Lakh+ Students trained till date  65+ Centers
across India
4
ME-GATE-2015 PAPER-01| www.gateforum.com

4. The function of interpolator in a CNC machine controller is to


(A) control spindle speed
(B) coordinate feed rates of axes
(C) control tool rapid approach speed
(D) perform Miscellaneous (M) functions (tool change, coolant control etc.)
Answer: (B)

5. Holes of diameter 25.00.040


0.020 mm are assembled interchangeably with the pins of diameter

25.00.005
0.008 mm. The minimum clearance in the assembly will be

(A) 0.048 mm (B) 0.015 mm


(C) 0.005 mm (D) 0.008 mm

Answer: (B)
Exp: Minimum clearance
 minimu m hole  max ium shaft
 25  .020  25  .005
 0.015 mm

1 3 9
6. Simpson‟s rule is used to integrate the function f  x   x 2  between x = 0 and x = 1
3 5 5
using the least number of equal sub-intervals. The value of the integral is ___________
Answer: 0.0208
1
Exp: x 0 1
2
3 9 9 39 12
y  f  x   x2 
5 5 5 20 5

1
1    9 12
2    39  
 y dx        4   
0
2  5 5   20  
 0.0208

7. Consider fully developed flow in a circular pipe with negligible entrance length effects.
Assuming the mass flow rate, density and friction factor to be constant, if the length of the
pipe is doubled and the diameter is halved, the head loss due to friction will increase by a
factor of
(A) 4 (B) 16 (C) 32 (D) 64
Answer: (D)
2
Q
2 fl  
  
fLv A
Exp: head loss 
2gd 2gd

 India’s No.1 institute for GATE Training  1 Lakh+ Students trained till date  65+ Centers
across India
5
ME-GATE-2015 PAPER-01| www.gateforum.com

L
 head loss 
d5
L
h1*
d5
.L L
h *2    64
d 2
5
d5

h1 1 h
  2  64
h2 64 h1

8. For an ideal gas with constant values of specific heats, for calculation of the specific enthalpy,
(A) it is sufficient to know only the temperature
(B) both temperature and pressure are required to be known
(C) both temperature and volume are required to be known
(D) both temperature and mass are required to be known
Answer: (A)

9. A Carnot engine (CE-1) works between two temperature reservoirs A and B, where TA = 900
K and TB = 500 K. A second Carnot engine (CE-2) works between temperature reservoirs B
and C, where TC = 300 K. In each cycle of CE-1 and CE-2, all the heat rejected by CE-1 to
reservoir B is used by CE-2. For one cycle of operation, if the net Q absorbed by CE-1 from
reservoir A is 150 MJ, the net heat rejected to reservoir C by CE-2 (in MJ) is _________.
Answer: 50
1  T1 1  500 900
Exp: 1    4.44
T1 900 Q1
Q 2  1  1   Q1  53.33 MJ HE W
Q2
1  T3 300
2  1  0.4 500
T2 500
Q2
Q3  1  2   Q 2  50 MJ
HE W
Q3
300

kg
10. A stream of moist air (mass flow rate = 10.1 kg/s) with humidity ratio of 0.01
kg dry air
mixes with a second stream of superheated water vapour flowing at 0.1 kg/s. Assuming
proper and uniform mixing with no condensation, the humidity ratio of the final stream
 kg 
 in  is ___________.
 kg dry air 
Answer: 0.0197
m11  m 2 2
Exp: new 
m1  m 2
0.1 10.1  .1 1
  .0197 kg kg dry air
10.1  .1

 India’s No.1 institute for GATE Training  1 Lakh+ Students trained till date  65+ Centers
across India
6
ME-GATE-2015 PAPER-01| www.gateforum.com

11. Consider a steel (Young‟s modulus E = 200 GPa) column hinged on both sides. Its height is
1.0m and cross-section is 10 mm × 20 mm. The lowest Euler critical buckling load (in N) is
______.
Answer: 3289.86
2 EI 2  200  109  .02  .013
Exp: Euler 's critical load  
l2 12
 3289.8681 N

12. Air enters a diesel engine with a density of 1.0 kg/m3. The compression ratio is 21. At steady
state, the air intake is 30 × 10–3 kg/s and the net work output is 15 kW. The mean effective
pressure (kPa) is _______.
Answer: 420
work output
Exp: mep 
swept volume
12

 1 
30  103 1  
 21 
 420 kPa

13. Match the following products with preferred manufacturing processes:

Product Process
P Rails (1)Blow
molding
Q Engine crankshaft (2) Extrusion
R Aluminium channels (3) Forging
S PET water bottles (4) Rolling

 A  P  4, Q  3, R  1, S  2  B P  4, Q  3, R  2, S  1
 C P  2, Q  4, R  3, S  1  D P  3, Q  4, R  2, S  1
Answer: (B)

th
1
14. Under certain cutting conditions, doubling the cutting speed reduces the tool life to   of
 16 
the original. Taylor‟s tool life index(n) for this tool-workpiece combination will be _______
Answer: 0.25
Exp: VT n  C

 16
2
T1
V1T1n  2V1 

on solving we get
n  0.25

 India’s No.1 institute for GATE Training  1 Lakh+ Students trained till date  65+ Centers
across India
7
ME-GATE-2015 PAPER-01| www.gateforum.com

15. Consider a slider crank mechanism with nonzero masses and inertia. A constant torque  is applied on
the crank as shown in the figure. Which of the following plots best resembles variation of crank angle,
 versus time


m

A  B
Time
 

Time

C  D
 

Time Time

Answer: (B)

1  cos  x 2 
16. The value of lim x 0 is
2x 4
1 1
A 0  B C  D undefined
2 4
Answer: (A)

Exp: lim
1  cos x 2 0
x 0 4
2x 0
Using L Hospital Rule

lim
 sin x  2x  0
2

x 0 8x 3 0

lim
 cos x  2x2x  sin x  2
2 2

x 0 24x 2

 India’s No.1 institute for GATE Training  1 Lakh+ Students trained till date  65+ Centers
across India
8
ME-GATE-2015 PAPER-01| www.gateforum.com

lim
 cos x  4x
2 2
 2sin x 2
x 0 24x 2

 lim
  sin x  4x
2 2
 
 cos x 2  8x   2 cos x 2 2x
x 0 48x

 lim
  cos x  2x  4x     sin x  8x     sin x   2x 8x   12cos x    sin x   2x  4x
2 2 2 2 2 2

x 0 48
0
 0
48

17. Two identical trusses support a load of 100 N as shown in the figure. The length of each truss
is 1.0 m, cross-sectional area is 200 mm2; Young‟s modulus E = 200 GPa. The force in the
truss AB (in N) is ______

A C

B
30O 30O

100 N
Answer: 100
Exp:  2f sin30  100 2Fsin 30

 f  100 N F
F

100

IV

18. Among the four normal distributions


with probability density functions as
shown below, which one has the lowest
variance? III

A I  B II
 C III  D  IV II
I
Answer: (D)
Exp: We have Probability distribution
2 1 0 1 2
function of Normal Distribution

 India’s No.1 institute for GATE Training  1 Lakh+ Students trained till date  65+ Centers
across India
9
ME-GATE-2015 PAPER-01| www.gateforum.com


 x  
1
f x  e 2 2 ________(1)
 
Variance = σ2 is lowest
 σ also lowest
 If σ decreases  f(x) increases (∵ from (1))
 Curve will have highest peak

19. Given two complex numbers Z1  5  5 3 i and z 2    2


3
 2i, the argument of
z1
z2
in

degrees is
(A) 0 (B) 30 (C) 60 (D) 90

Answer: (A)
Exp:  
z1  5  5 3 i

5 3
arg z1  tan 1 
5
  tan
1
 3   60
 
2
z2   2i
3
 
 2 
arg z 2  tan 1 
 2
  tan 1

 3   60
 
 3
z 
arg  1   arg  z1   arg  z 2 
 z2 
 60  60  0

20. The Blasius equation related to boundary layer theory is a


(A) third-order linear partial differential equation
(B) third-order nonlinear partial differential equation
(C) second-order nonlinear ordinary differential equation
(D) third-order nonlinear ordinary differential equation

Answer: (D)

21. A swimmer can swim 10 km in 2 hours when swimming along the flow of a river. While
swimming against the flow, she takes 5 hours for the same distance. Her speed in still water
(in km/h) is ________.
Answer: 35
Exp: Let Swimmer = x
River = y

 India’s No.1 institute for GATE Training  1 Lakh+ Students trained till date  65+ Centers
across India
10
ME-GATE-2015 PAPER-01| www.gateforum.com

10
2
xy
10
5
xy
On solving we get x  35 km h

22. For flow of viscous fluid over a flat plate, if the fluid temperature is the same as the plate
temperature, the thermal boundary layer is
(A) thinner than the velocity boundary layer
(B) thicker than the velocity boundary layer
(C) of the same thickness as the velocity boundary layer
(D) not formed at all
Answer: (D)

23. Which one of the following is the most conservative fatigue failure criterion?
(A) Soderberg (B) Modified Goodman
(C) ASME Elliptic (D) Gerber
Answer: (A)

24. In a linear arc welding process, the heat input per unit length is inversely proportional to
(A) welding current (B) welding voltage
(C) welding speed (D) duty cycle of the power source
Answer: (D)

25. Consider a stepped shaft subjected to a twisting moment applied at B as shown in the figure.
Assume shear modulus, G = 77 GPa. The angle of twist at C (in degrees) is _____

10Nm All dimensions


in mm

 20 10
B C
A 500 500

Answer: 0.236
Exp: Angle of twist at (C) = Angle of twist at (B)
TL

GJ
10  0.5  32
  0.236050
77  109    .024

 India’s No.1 institute for GATE Training  1 Lakh+ Students trained till date  65+ Centers
across India
11
ME-GATE-2015 PAPER-01| www.gateforum.com

26. A 10 mm diameter electrical conductor is covered by an insulation of 2 mm thickness. The


conductivity of the insulation is 0.08 W/m-K and the convection coefficient at the insulation
surface is 10 W/m2-K. Addition of further insulation of the same material will
(A) increase heat loss continuously
(B) decrease heat loss continuously
(C) increase heat loss to a maximum and then decrease heat loss
(D) decrease heat loss to a minimum and then increase heat loss

Answer: (C)
Exp: rc = 8mm
∴ the heat lost increases to maximum and then decreases.

27. A machine element is subjected to the following bi-axial state of stress; σx = 80 MPa; σy = 20
MPa; xy  40MPa. If the shear strength of the material is 100 MPa, the factor of safety as
per Tresca‟s maximum shear stress theory is
(A) 1.0 (B) 2.0 (C) 2.5 (D) 3.3
Answer: (B)

2
80  20  80  20 
Exp: 1      40
2

2  2 

 50  502
 100
2  0
1   2
  50
7
100
FOS  2
50

28. The probability of obtaining at least two “SIX” in throwing a fair dice 4 time is
 A  425 432  B  19 144  C 13 144  D 125 432
Answer: (B)
1
Exp: n = 4; p
6
1 5
 q 1 
6 6
p  x  2  1  p  x  2
 1   p  x  0   p  x  1
 0
1 5
4
 1   5   19
1 3
 1   4C0      4C1      
 6 6  6   6   144

 India’s No.1 institute for GATE Training  1 Lakh+ Students trained till date  65+ Centers
across India
12
ME-GATE-2015 PAPER-01| www.gateforum.com

29. A horizontal plate has been joined to a vertical post using four rivets arranged as shown in
figure. The magnitude of the load on the worst loaded rivet (in N) is ________

40mm
500mm

40mm

400 N

Answer: 1839.83
400
Exp: Shear load on all rivets   100N
4
Secondary shear load, due to bending moment
Pe r1

r  r  r32  r42
1
2
1
2

400  .5  .02 2
  1767.766953 N
.02 2 
2
4

P = 1839.837 N.

30. Temperature of nitrogen in a vessel of volume 2 m3 is 288 K. A U-tube manometer connected


to the vessel shows a reading of 70 cm of mercury (level higher in the end open to
atmosphere). The universal gas constant is 8314 J/kmol-K, atmospheric pressure is 1.01325
bar, acceleration due to gravity is 9.81 m/s2 and density of mercury is 13600 kg/m3. The mass
of nitrogen (in kg) in the vessel is _______.
Answer: 4.55
Exp: p  gh  0.7  1360  9.81  9339.12pa
Actual pressure  atmospheric pressure  110664.12 pa
pv  mrT  m  pv RT
110664.12  2
  4.5539kg
288  8314

2
V
31. The solidification time of a casting is proportional to   , where V is the volume of the
A
casting and A is the total casting surface area losing heat. Two cubes of same material and
size are cast using sand casting process. The top face of one of the cubes is completely
insulated. The ratio of the solidification time for the cube with top face insulated to that of the
other cube is
25 36 6
A  B  C 1  D
36 25 5
Answer: (B)

 India’s No.1 institute for GATE Training  1 Lakh+ Students trained till date  65+ Centers
across India
13
ME-GATE-2015 PAPER-01| www.gateforum.com

 A
2
Exp: t min  k V
2
 93 
t1  k  
 502 
 93 
t2  k  
 5 
t1 36

t 2 25

32. Match the following pairs:

Equation Physical Interpretation



(P) V0 (I) Incompressible continuity equation

(Q) .V0 (II) Steady flow

DV (III) Irrotational flow
(R) 0
Dt

V (IV) Zero acceleration of fluid particle
(S) 0
t

 A  P  IV,Q  I, R  II,S  III  B P  IV,Q  III, R  I,S  II


 C P  III,Q  I, R  IV,S  II  D P  III,Q  I, R  II,S  IV
Answer: (C)

33. Steam enters a well insulated turbine and expands isentropically throughout. At an
intermediate pressure, 20 percent of the mass is extracted for process heating and the
remaining steam expands isentropically to 9 kPa.
Inlet to turbine: P = 14 MPa, T = 560°C, h = 3486 kJ/kg, s = 6.6 kJ/(kg.K)
Intermediate stage: h = 27776 kJ/kg
Exit of turbine: P = 9 kPa, hf = 174 kJ/kg, hg = 2574 kJ/kg,
sf = 0.6 kJ/(kg.K), sg = 8.1 kJ/(kg.K)
If the flow rate of steam entering the turbine is 100 kg/s, then the work output (in MW) is
_______.

Answer: 125.56
1
Exp: h1 = 3486 kJ/kg
5 m
h2 = 2776 kJ/kg
20% 2
s1 = s3 =6.6
6.6  .6  x  0.1  .6  4 1 m 3
x  0.8

 India’s No.1 institute for GATE Training  1 Lakh+ Students trained till date  65+ Centers
across India
14
ME-GATE-2015 PAPER-01| www.gateforum.com

h  174  .8  2574  174 


 2094 kJ kg
w   3486  2776   .8  2776  2094 
 1255.6 kJ kg
 125.56 MW

34. Water (ρ = 1000 kg/m3) flows through a venturimeter with inlet diameter 80 mm and throat
diameter 40 mm. The inlet and throat gauge pressures are measured to be 400 kPa and 130
kPa respectively. Assuming the venturimeter to be horizontal and neglecting friction, the inlet
velocity (in m/s) is ________.
Answer: 6
p1 v12 p 2 v 22  p1  400000 
Exp:     
g 2g g 2g  p 2  130000 
v1  802  v 2  402
v 2  4v1
Substituting v 2 and solving for v1 we get
v1  6 m s

35. For a canteen, the actual demand for disposable cups was 500 units in January and 600 units
in February. The forecast for the month of January was 400 units. The forecast for the month
of March considering smoothing coefficient as 0.75 is _______.
Answer: 560.75
Exp: Forecast for Feb  400    500  400   400  .25  100  475
Forecast for march  475    600  475  560.75

36. Consider a spatial curve in three-dimensional space given in parametric form by


2 
x  t   cos t, y  t   sin t,z  t   t, 0  t 
 2
The length of the curve is ______
Answer: 1.86
Exp: The length of the curve
2 2 2 2
 dx   dy   dz 
 
0
        dt
 dt   dt   dt 
2 2
2
   sin t    cos t 
2 2
    dt
0 
2 2
4 4
 
0
sin 2 t  cos 2 t 
2
dt  
0
1
2
dt

4 2 4 
 1 . t 0  1   1.8622
 2
2 2

 India’s No.1 institute for GATE Training  1 Lakh+ Students trained till date  65+ Centers
across India
15
ME-GATE-2015 PAPER-01| www.gateforum.com

37. Considering massless rigid rod and small oscillations, the natural frequency (in rad/s) of
vibration of the system shown in the figure is

k  400 N m
m  1kg

r 2r

400 400 400 400


A  B  C  D
1 2 3 4
Answer: (D)
Exp: Form  dx = 2rθ
d2 x
2
 2r. 
dt
Tak ing moments
2r .m.2 r.  400   r   r  0
 4m.   400    0
400

4

38. A triangular facet in a CAD model has vertices: P1(0, 0, 0); P2(1, 1, 0) and P3(1, 1, 1). The area of the
facet is
(A) 0.500 (B) 0.707
(C) 1.414 (D) 1.732

Answer: (B)
y
1
Exp: Area   b  h
2
1
 a 2 a
2
a 1 x
1
  0.7071
2
z

 India’s No.1 institute for GATE Training  1 Lakh+ Students trained till date  65+ Centers
across India
16
ME-GATE-2015 PAPER-01| www.gateforum.com

39. In a slab rolling operation, the maximum thickness reduction (Δhmax) is given by Δhmax = μ2R,
where R is the radius of the roll and μ is the coefficient of friction between the roll and the
sheet. If μ = 0.1, the maximum angle subtended by the deformation zone at the centre of the
roll (bite angle in degree) is ______.
Answer: 5.71
h
Exp:   tan 1  tan 1 
R
 5.7106

d2 y  3
40. Fine the solution of  y which passes through the origin and the point  ln 2,  ,
dx 2  4

 B y   e x  e x 
1 1
 A  y  e x  e x
2 2
 C  y   e  e x 
1 x 1
 D  y  ex  e x
2 2

Answer: (C)
d2 y
Exp:
dx 2

 y  D2  1 y  0 
D2-1 = 0
 D = ±1
y  c1ex  c2e x
 3
Passes through (0,0) and 142, 
 4
(0,0)
 0 = C1 + C2 __________(1)
 3
 142, 
 4
3 C
 C1e142  C 2 e 142  C1 2  2
4 2
1 3
 2C1  C 2  ______  2 
2 4
solving 1 and  2 
1
 C1 
2
1
C2  
2
1 1
y  e x  e  x
2 2
1 x

 e  e x
2

 India’s No.1 institute for GATE Training  1 Lakh+ Students trained till date  65+ Centers
across India
17
ME-GATE-2015 PAPER-01| www.gateforum.com

41. For the truss shown in figure, the magnitude of the force in member PR and the support reaction at R
are respectively 100kN
(A) 122.47 kN and 50 kN 60O P

(B) 70.71 kN and 100 kN

(C) 70.71 kN and 50 kN Q 45O R

4m
(D) 81.65 kN and 100 kN

Answer: (C)
Exp: M Q 0 100sin 60
100cos60 FPR
100  cos 60  4  R a  4
 R a  50kN
FPR  cos 45  100cos 60  FPR  70.71 kN
Q
36.6 50

42. A ball of mass 0.1 kg, initially at rest, is dropped from height of 1 m. Ball hits the ground and
bounces off the ground. Upon impact with the ground, the velocity reduces by 20%. The
height (in m) to which the ball will rise is ______
Answer: 0.64
Exp: v  2gh  2  9.01  1  4.4294 m s
v '  0  v  3.5435m s
v2
h  0.64m
2g

43. A DC welding power source has a linear voltage-current (V-I) characteristic with open circuit
voltage of 80 V and a short circuit current of 300 A. For maximum arc power, the current (in
Amperes) should be set as _____.
Answer: 150
80
Exp: v  80  I
300
80 2
p  v 7  80I  I
300
Differentiating and equating to '0'
I  150A

 India’s No.1 institute for GATE Training  1 Lakh+ Students trained till date  65+ Centers
across India
18
ME-GATE-2015 PAPER-01| www.gateforum.com

44. A well insulated rigid container of volume 1 m3 contains 1.0 kg of an ideal gas [Cp = 1000
J/(kg.K) and Cv = 800 J/(kg.K)] at a pressure of 105 Pa. A stirrer is rotated at constant rpm in
the container for 1000 rotations and the applied torque is 100 N-m. The final temperature of
the gas (in K) is _________.
Answer: 628.31

Exp: Work  T.


 100  100  2 Cp T
T  628.3105K

45. A pinion with radius r1, and inertia I1 is driving a gear with radius r2 and inertia I2. Torque 1
is applied on pinion. The following are free body diagrams of pinion and gear showing
important forces (F1 and F2) of interaction. Which of the following relations hold true?

2
1
1 F2
1 , 2  Angular
r1 Displacements
F1 r2
Inertia  l1
Inertia  l2
r1 
 A  F1  F2 ; 1  I11 :F2  I2 1
r22

  r1   
2
r
 B F1  F2 ; 1  I1  I2    1 : F2  I2 12 1
  r2   r2

1 
 C F1  F2 ; 1  I1
1 :F2  I2 2
r2

  r1   
2
1
 D  F1  F2 ; 1  I1  I2    1;F2  I2 2
  r2   r2

Answer: (D)

Exp: F1 = F2 [Vertical equilibrium]

For Gear 1 For Gear 2

 
 1 – F1r1 = I-1 1 f2r2 = I2  2

 India’s No.1 institute for GATE Training  1 Lakh+ Students trained till date  65+ Centers
across India
19
ME-GATE-2015 PAPER-01| www.gateforum.com


 r  I 2
1  I 2 2 1  I1 1 .....using F2  2 and F1 
r2 r2
  r1
1  I1 1  I  2
r2
 
 
 2 r
  I1  I 2  1  1
 r 
 1 2 
   
 r1   
2
1 r
1   I1  I 2    1 .....    2 
 r1 
  r2  
 2 

I 2
and clearly F2  2
r2

46. A cantilever beam with flexural rigidity of 200 Nm2 is loaded as shown in the figure. The
deflection (in mm) at the tip of the beam is _____.

500 N

50 mm
100mm

Answer: 0.26
Exp: Deflection = Deflection at load + Slope × Distance
Wl3 Wl2
   .05
3EI 2EI
W  500
l  .05
EI  200
  0.2604 mm

47. In the assembly shown below, the part dimensions are:

L1  22.0 0.01mm,
L2  L3  10.00.005 mm.

Assuming the normal distribution of part dimensions, the dimension L4 in mm for assembly
condition would be:

 India’s No.1 institute for GATE Training  1 Lakh+ Students trained till date  65+ Centers
across India
20
ME-GATE-2015 PAPER-01| www.gateforum.com

L4
L2 L3
L1

 A  2.00.008  B 2.00.012  C 2.00.016  D 2.00.020


Answer: (D)

Exp: Since all dimensions have bilateral tolerances

L 4  L1  L 2  L3
 22  10  10
L 4  2 mm
Tolerance  0.01  0.005  0.005
 0.02

L 4  2.00.02 mm

Tolerance is calculated assuming L4 to be sink and tolerance of sink will be cumulative sum
of all tolerances

48. A mobile phone has a small motor with an eccentric mass used for vibrator mode. The
location of the eccentric mass on motor with respect to center of gravity (CG) of the mobile
and the rest of the dimensions of the mobile phone are shown. The mobile is kept on a flat
horizontal surface.
CG Motor
10cm Eccentric
P Q Mass

6cm 3cm

Given in addition that the eccentric mass = 2 grams, eccentricty = 2.19 mm, mass of the
mobile = 90 grams, g = 9.81 m/s2. Uniform speed of the motor in RPM for which the mobile
will get just lifted off the ground at the end Q is approximately
(A) 3000 (B) 3500 (C) 4000 (D) 4500
Answer: (B)

 India’s No.1 institute for GATE Training  1 Lakh+ Students trained till date  65+ Centers
across India
21
ME-GATE-2015 PAPER-01| www.gateforum.com

Exp: When lifted from ground at Q


Reaction = 0
∴ taking moments about „p‟ and equating to 0
.09×.06 = mr  2×.09
9.01×.09×.06 = .002×2.19×10-3×  2×.09
  = 366.50 rad/s
= 3500 rpm

49. A precision instrument package (m = 1 kg) needs to be mounted on a surface vibrating at 60


Hz. It is desired that only 5% of the base surface vibration amplitude be transmitted to the
instrument. Assuming that the isolation is designed with its natural frequency significantly
lesser than 60 Hz, so that the effect of damping may be ignored. The stiffness (in N/m) of the
required mounting pad is ________.
Answer: 6767.6
Exp:   2N  2   60  376.99 rad second
2 2
1    
.05   20  1        21
 n   n 
2
 
1  
 n 
2  21n2
n  82.266 rad s
k
n   82.266
m
l  6767.6005 N m
50. Following data refers to the activities of a project, where, node 1 refers to the start and node 5
refers to the end of the project

Activity Duration (days)


1-2 2
2-3 1
4-3 3
1-4 3
2-5 3
3-5 2
4-5 4

The critical path (CP) in the network is


(A) 1-2-3-5 (B) 1-4-3-5 (C) 1-2-3-4-5 (D) 1-4-5

Answer: (B)

 India’s No.1 institute for GATE Training  1 Lakh+ Students trained till date  65+ Centers
across India
22
ME-GATE-2015 PAPER-01| www.gateforum.com

Exp:
2 5
0 2 2 5
2 3 6 8
1 3
0 3 3 6

4 3 7

Critical path-1-4-3-5
Time taken = 8 days

51. For flow through a pipe of radius R, the velocity and temperature distribution are as follows:
  r 3 
u  r, x   C1 , and T  r, x   C2 1     , where C1 and C2 are constants
 R 
 
2
 u  r, x  T  r, x  rdr,
R
The bulk mean temperature is given by Tm 
umR 2 0

with Um being the mean velocity of flow. The value of Tm is

0.5C2 0.6C2
A  B 0.5C2  C 0.6C2  D
Um Um

Answer: (C)
2
R
  r 3 
Exp: Tm 
umR 2 0 
C1C2 1     r dr
 R 
 
2C1C2  r4 
R
  r 

umR 2 0  R3 
 dr

2C C  3  0.6C1C2
 1 22  R 2  
u m R  10  um

Since u(r,x) is constant, um = C1

∴ tm = 0.6C2

52. Consider an ant crawling along the curve (x – 2)2 + y2 = 4, where x and y are in meters. The
ant starts at the point (4, 0) and moves counter-clockwise with a speed of 1.57 meters per
second. The time taken by the ant to reach the point (2, 2) is (in seconds) _______.

 India’s No.1 institute for GATE Training  1 Lakh+ Students trained till date  65+ Centers
across India
23
ME-GATE-2015 PAPER-01| www.gateforum.com

Answer: 2
Exp:
1
 circumference
4
1
   4
4
time 

 2sec
 0,0   2,0   4,0 
1.5 

53. Air (ρ = 1.2 kg/m3 and kinematic viscosity, v = 2 × 10–5 m2/s) with a velocity of 2 m/s flows
over the top surface of a flat plate of length 2.5 m. If the average value of friction coefficient
1.328
is Cf  , the total drag force (in N) per unit width of the plate is ________.
Re x
Answer: 0.0159
1.320
Exp: Cf 
Re x
vd vd.
R ex  
 
v  2m / s
l  2.5m
  2  105 m 2 / s
1
F  Cf A 2
2
A  2.5  1
On substituting we get
F = 0.0159N

54. The velocity field of an incompressible flow is given by


V = (a1x + a2y + a3z)i + (b1x + b2y + b3z)j + (c1x + c2y + c3z)k, where a1= 2 and c3 = – 4. The
value of b2 is ________.
Answer: 2
u v w
Exp:   0
x y z
a1  b 2  c3  0
2  4  b2  0
b2  2

 India’s No.1 institute for GATE Training  1 Lakh+ Students trained till date  65+ Centers
across India
24
ME-GATE-2015 PAPER-01| www.gateforum.com

55. An orthogonal turning operation is carried out under the following conditions: rake angle =
5°, spindle rotational speed = 400 rpm; axial feed = 0.4 m/min and radial depth of cut = 5
mm. The chip thickness tc, is found to be 3 mm. The shear angle (in degrees) in this turning
process is ________.

Answer: 32.239
3
Exp: Chip thickness ratio(r) =  0.6
5

To find shear angle   


r cos  0.6cos5
tan     0.6306
1  r sin  1  0.6sin 5
  tan 1  0.6306   32.24o

 India’s No.1 institute for GATE Training  1 Lakh+ Students trained till date  65+ Centers
across India
25
ME-GATE-2015 PAPER-02| www.gateforum.com

General Aptitude

Q. No. 1 – 5 Carry One Mark Each

1. Find the missing sequence in the letter series below:


A, CD, GHI,?, UVWXY
(A) LMN (B) MNO (C) MNOP (D) NOPQ
Answer: (C)
Exp:
A, C D, G H I, M N O P, UVWXY

B E, F JKL QRST
1 2 3 4

2. Choose the correct verb to fill in the blank below:


Let us ______________.
(A) Introvert (B) alternate (C) atheist (D) altruist
Answer: (B)

3. Choose the most appropriate word from the options given below to complete the following
sentence?
If the athlete had wanted to come first in the race, he ___________several hours every day.
(A) Should practice (B) Should have practiced
(C) Practised (D) Should be practicing
Answer: (B)
Exp: For condition regarding something which already happened, should have practiced is the
correct choice.

4. Choose the most suitable one word substitute for the following expression
Connotation of a road or way
(A) Pertinacious (B) Viaticum
(C) Clandestine (D) Ravenous
Answer: (A)
Exp: No word is relevant. Least irrelevant word is pertinacious.

 India’s No.1 institute for GATE Training  1 Lakh+ Students trained till date  65+ Centers cross
India
1
ME-GATE-2015 PAPER-02| www.gateforum.com

5. If x>y>I, which of the following must be true?


 i  ln x  ln y  ii  ex  ey  iii  yx  x y  iv  cos x  cos y
 A   i  and  ii   B  i  and  iii 
 C  iii  and iv   D ii  and iv 
Answer: (A)

Exp: For whole numbers, greater the value greater will be its log.
Same logic for power of e.

Q. No. 6 – 10 Carry Two Marks Each

6. From a circular sheet of paper of radius 30cm, a sector of 10% area is removed. If the
remaining part is used to make a conical surface, then the ratio of the radius and height of the
cone is________.
Answer: 13.08
Exp: 90% of area of sheet = Cross sectional area of cone
 0.9    30  30    r1  30
 27 cm  r1

 height of the cone  302  27 2


 13.08 cm

7. In the following question, the first and the last sentence of the passage are in order and
numbered 1 and 6. The rest of the passage is split into 4 parts and numbered as 2,3,4, and 5.
These 4 parts are not arranged in proper order. Read the sentences and arrange them in a
logical sequence to make a passage and choose the correct sequence from the given options.
1. One Diwali, the family rises early in the morning.
2. The whole family, including the young and the old enjoy doing this,
3. Children let off fireworks later in the night with their friends.
4. At sunset, the lamps are lit and the family performs various rituals
5. Father, mother, and children visit relatives and exchange gifts and sweets.
6. Houses look so pretty with lighted lamps all around.
(A) 2, 5, 3, 4 (B) 5, 2, 4, 3 (C) 3, 5, 4,2 (D) 4, 5, 2, 3
Answer: (B)

 India’s No.1 institute for GATE Training  1 Lakh+ Students trained till date  65+ Centers cross
India
2
ME-GATE-2015 PAPER-02| www.gateforum.com

8. Ms X will be in Bagdogra from 01/05/2014 to 20/05/2014 and from 22/05/2014 to


31/05/2014. On the morning of 21/05/204, she will reach Kochi via Mumbai
Which one of the statements below is logically valid and can be inferred from the above
sentences?
(A) Ms. X will be in Kochi for one day, only in May
(B) Ms. X will be in Kochi for only one day in May
(C) Ms. X will be only in Kochi for one day in May
(D) Only Ms. X will be in Kochi for one day in May.
Answer: (A)
Exp: Second sentence says that Ms. X reaches Kochi on 21/05/2014. Also she has to be in
Bagdogora on 22/05/2014.
 She stays in Kochi for only one day in may.

9. log tan1O  log tan 2O  ......  log tan89O is .........


A 1  B 1 2  C 0  D 1
Answer: (C)
Exp: log tan1o  log tan 89o  log  tan1o  tan 89o 
 log  tan1o  cot1o 
 log1
0
Using the same logic total sum is ‘0’.

10. Ram and Shyam shared a secret and promised to each other that it would remain between
them. Ram expressed himself in one of the following ways as given in the choices below.
Identify the correct way as per standard English.
(A) It would remain between you and me.
(B) It would remain between I and you
(C) It would remain between you and I
(D) It would remain with me.
Answer: (A)

 India’s No.1 institute for GATE Training  1 Lakh+ Students trained till date  65+ Centers cross
India
3
ME-GATE-2015 PAPER-02| www.gateforum.com

Mechanical Engineering
Q. No. 1 – 25 Carry One Mark Each

1. The uniaxial yield stress of a material is 300 MPa. According to von Mises criterion, the shear
yield stress (in MPa) of the material is ______
Answer: 173.28
Exp. If there is uniaxial loading yield stress is  y
As per Von Mises failure theory
y 2  12  12  22
Under pure shear stress loading
1 = - 2 = τ
then
 y 2  2  2  2
 32
y y
 hence   0.577 y  173.28
3 3
where  isshear yield stress

2. The primary mechanism of material removal in electrochemical machining (ECM) is


(A) Chemical corrosion
(B) etching
(C) ionic dissolution
(D) spark erosion
Answer: (B)

3. Curl of vector V(x,y,z) = 2x 2i  3z2 j  y3 k at x  y  z  1 is


 A  3i  B 3i  C 3i  4j  D 3i  6k
Answer: (A)

i j k
  
Exp: Curl of V  x, y, z  
x y z
2x 2 3z 2 y3
 i 3y 2  6z   j 0  0  k 0  0


 3y 2  6z i  x  y  z 1

 3i

 India’s No.1 institute for GATE Training  1 Lakh+ Students trained till date  65+ Centers cross
India
4
ME-GATE-2015 PAPER-02| www.gateforum.com

4. A small ball of mass 1kg moving with a velocity of 12m/s undergoes a direct central impact
with a stationary ball of mass 2 kg. The impact is perfectly elastic. The speed (in m/s) of 2 kg
mass ball after the impact will be ________
Answer: 8
Exp: For elastic collision
m1u1+m2u2 = m1v1+m2v2 (1) _________ moment conservation
m1 = 1 kg u1 = 12 m/s
m2 = 2 kg u2 = 0 m/s
1 1 1 1
m1u12  m2 u 22  m1v12  m2 v 22  2  ____ energyconservation
2 2 2 2
From (1) equation
12 = v1+2v2 ________(3)
From (2) equation
1 1 1 1
 1  144   2  0   1  v12   2  v 22
2 2 2 2
 144  v1  2v 2 ____  4 
2 2

From (3) and (4)


144  144  4v 22  48v 2  2v 22
 6v 22  48v 2  0
6v 2  v 2  8   0
 v 2  8m s

5. A rod is subjected to a unit-axial load within linear elastic limit. When the change in the stress
is 200 MPa, the change in the strain is 0.001. If the Poisson’s ratio of the rod is 0.3, the
modulus of rigidity (in GPa) is _____________
Answer: 77
Exp: Modulus of rigidity (G)
E
G
2 1   
It has given change in stress = 200 MPa
Change in strain = 0.001
Here
200 = E×0.003
200 MPa
E  200  103 MPa
0.002
 200GPa
200 200 100 1000
G     77 GPa
2 1  0.3 2  1.3 1.3 13

 India’s No.1 institute for GATE Training  1 Lakh+ Students trained till date  65+ Centers cross
India
5
ME-GATE-2015 PAPER-02| www.gateforum.com

6. Within a boundary layer for a steady incompressible flow, the Bernoulli equation
(A) holds because the flow is steady
(B) holds because the flow is incompressible
(C) holds because the flow is transitional
(D) does not hold because the flow is frictional
Answer: (D)
Exp: Bernoulli equation does not hold because it is for non viscous flow

7. The atomic packing factor for a material with body centered cubic structure is _______
Answer: 0.64

8. If a foam insulation is added to a 4cm outer diameter pipe as shown in the figure, the critical
radius of insulation (in cm) is _____________

 4cm
Foam h 0  2W m2 .K
kfoam  0.1W m.K hi  15W m2  K

Pipe
k pipe  15W m.K

Answer: 5
k 0.1
Exp: Critical radius rc    .05m
h0 2
 5cm

9. During the development of a product an entirely new process plan is made based on design
logic, examination of geometry and tolerance information. This type of process planning is
known as
(A) Retrieval (B) Generative
(C) Variant (D) Group technology based
Answer: (D)

 India’s No.1 institute for GATE Training  1 Lakh+ Students trained till date  65+ Centers cross
India
6
ME-GATE-2015 PAPER-02| www.gateforum.com

10. Annual demand of a product is 50000 units and the ordering cost is Rs. 7000 per order
considering the basic economic order quantity model, the economic order quantity is 10000
units. When the annual inventory cost is minimized, the annual inventory holding cost (in Rs.)
is _______
Answer: 35000
Exp: At optimum total inventory cost (TIC), annual inventory hold is cost is equal to annual
inventory ordering cost
= Number of orders × ordering cost per order
50000
  7000
10000
 5  7000
 35000

11. Sales data of a product is given in the following table:

Month January February March April May


Number of unit 10 11 16 19 25
sold

Regarding forecast for the month of June, which one of the following statements is TRUE?
(A) Moving average will forecast a higher value compared to regression
(B) Higher the value of order N, the greater will be the forecast value by moving average.
(C) Exponential smoothing will forecast a higher value compared to regression.
(D) Regression will forecast a higher value compared to moving average
Answer: (B)
Exp: In case of increasing trend, regression will give increasing value than moving average.

 a 
12. The Vander Waals equation of state is  p  2   v  b   RT, where p is pressure, v is
 v 
specific volume, T is temperature and R is characteristic gas constant. The SI unit of a is
 A J kg.K  B m kg  C m5 kg  s2  D Pa kg
Answer: (C)
a
Exp: p both term should gave same unit since they are getting added
2
2
N  kg 
 2  a 3 
m m 
m6 m m5 kg m5
 a  unit   .kg.  
kg 2 s 2 m 2 kg 2s 2 kg s 2

 India’s No.1 institute for GATE Training  1 Lakh+ Students trained till date  65+ Centers cross
India
7
ME-GATE-2015 PAPER-02| www.gateforum.com

13. Which of the following statements regarding a Rankine cycle with reheating are TRUE?
(i) increase in average temperature of heat addition
(ii) reduction in thermal efficiency
(iii) drier steam at the turbine exit
(A) only (i) and (ii) are correct (B) only (ii) and (iii) are correct
(C) only (i) and (iii) are correct (D) (i), (ii) and (iii) are correct
Answer: (C)
Exp: With reheat average temperature of heat addition increases. Hence, efficiency of cycle also
increases. Further, the quality of steam is higher at turbine exit.

14. In a spring-mass system, the mass is m and the spring constant is k. The critical damping
coefficient of the system is 0.1kg/s. In another spring-mass system, the mass is 2m and the
spring constant is 8K. The critical damping coefficient (in kg/s) of this system is __________
Answer: 1.6
2S
Exp: CC   2 Sm
n
S2 m 2
 CC 2   CC 1
S1m1
 0.4 8  2
 CC 2  1.6

15. The COP of a cannot heat pump operating between 6OC and 37OC is ___________
Answer: (10)
T1 310
Exp:  COP c.p.    10
T1  T2 31

16. The number of degrees of freedom of the planetary gear train shown in the figure is

20teeth gear

arm

50 teeth
gear

(A) 0 (B) 1 (C) 2 (D) 3

Answer: (C)
Exp: A planetary gear train has 2 DOF and hence requires too input to get desired output.

 India’s No.1 institute for GATE Training  1 Lakh+ Students trained till date  65+ Centers cross
India
8
ME-GATE-2015 PAPER-02| www.gateforum.com

17. A rope-brake dynamometer attached to the crank shaft of an I.C. engine measures a brake
power of 10kW when the speed of rotation of the shaft is 400 rad/s. The shaft torque (in N-m)
sensed by the dynamometer is _______
Answer: 25
Exp: P = T
10000
T  25 N  m
400

18. At least one eigen value of a singular matrix is


(A) Positive (B) Zero (C) Negative (D) Imaginary
Answer: (B)

19. If the fluid velocity for a potential flow is given by V  x, y   u  x, y  i  v  x, y  j with usual
notations, then the slope of the potential line at (x,y) is
v u v2 u
A  B  C  D
u v u2 v
Answer: (D)
Exp: It has given
V(x,y) = u(x,y)i + v(x,y)J
u component = u (x,y)
v component = v(x,y)
but we know that

u
x

   u  x, y  ___ 1
x
and

  v  x, y  ____  2 
y
from 1 and  2 
y  y  u  x, y  u
 .  
x x   v  x, y  v

20. Which one of the following statements is TRUE?


(A) The ‘GO’ gage controls the upper limit of a hole
(B) The ‘NO’ gage controls the lower limit of a shaft
(C) The ‘GO’ gage controls the lower limit of a hole
(D) The ‘NO GO’ gage controls the lower limit of a hole
Answer: (A)
Exp: Go gauge represents lower limit in hole

 India’s No.1 institute for GATE Training  1 Lakh+ Students trained till date  65+ Centers cross
India
9
ME-GATE-2015 PAPER-02| www.gateforum.com

21. There vendors were asked to supply a very high precision component. The respective
probabilities of their meeting the strict design specifications are 0.8, 0.7 and 0.5. Each vendor
supplies one component. The probability that out of total three components supplied by the
vendors, at least one will meet the design specification is _________
Answer: 0.97
Exp: Probability (at least one will meet specification) = 1-probability (none will meet specification)
= 1-(1-0.8)×(1-0.7)×(1-0.5)
= 1-0.2×0.3×0.5
= 1-0.03
= 0.97

22. The Laplace transform of ei5t where i  1 , is


s  5i s  5i s  5i s  5i
A  B  C  D
s 2  25 s 2  25 s 2  25 s 2  25
Answer: (A)
Exp:  
L ei5t  L  cos5t  isin 5t 
s  5i
 L  cos5t   iL  sin 5t  
s 2  25

23. A gas is stored in a cylindrical tank of inner radius 7 m and wall thickness 50 mm. The gage
pressure of the gas is 2MPa. The maximum shear stress (in MPa) in the wall is
(A) 35 (B) 70 (C) 140 (D) 280
Answer: (B)
pd 2  14
Exp: c    280 MPa
2t 2  0.05
pd 2  14
l    140 MPa
4t 4  0.05
   2  c  l
Maximumshear stress max  l   70 MPa
2 2
Note: This stress τmax is the maximum in plane shear stress.
c
Absolute τmax will be  140 MPa
2

 India’s No.1 institute for GATE Training  1 Lakh+ Students trained till date  65+ Centers cross
India
10
ME-GATE-2015 PAPER-02| www.gateforum.com

24. In the laminar flow of air  Pr  0.7  over a heated plate if  and T denote, respectively, the
hydrodynamic and thermal boundary layer thicknesses, then
 A   T  B   T  C   T  D   0 but T  0
Answer: (C)
Exp: When Pr < 1 δt > δ
Pr > 1 δt < δ
Pr = 1 δt = δ

25. At x = 0, the function f(x) = x has


(A) A minimum (B) A maximum
(C) A point of inflexion (D) neither a maximum nor minimum
Answer: (A)
Exp: For negative values of x, f(x) will be positive
For positive values of x, f(x) will be positive
∴ minimum value of f(x) will occur at x = 0

26. The total emissive power of a surface is 500 W/m2 at a temperature T1 and 1200 W/m2 at a
temperature T2. Where the temperatures are in Kelvin. Assuming the emissivity of the surface
T
to be constant, the ratio of the temperatures 1 is
T2
(A) 0.308 (B) 0.416 (C) 0.803 (D) 0.874
Answer: (C)
Exp: If temperature is T1 then
Emissive power  T14

500  T14 ___ 1


or
500  k T14 ___ 1
similarly
1200  kT24 ___  2 
from 1  2 
4
 T1  500
  
 T2  1200
14
T1  500 
   0.803
T2  1200 

 India’s No.1 institute for GATE Training  1 Lakh+ Students trained till date  65+ Centers cross
India
11
ME-GATE-2015 PAPER-02| www.gateforum.com

27. A hallow shaft of 1m length is designed to transmit a power of 30 kW at 700 rpm. The
maximum permissible angle of twist in the shaft is 1O. The inner diameter of the shaft is 0.7
times the outer diameter. The modulus of rigidity is 80 GPa. The outside diameter (in mm) of
the shaft is _______
Answer: 44.52
Exp: P = T
2  700
30  1000  T 
60
T  409.256 N  m
T G

IP l
 l  1m 
409.256 80  109   
 
     radians 
32
 4

1  0.7 d 0
4 1 180
 180 

Solving, we get
d0 = 44.5213 mm

28. In a Rankine cycle, the enthalpies at turbine entry and outlet are 3159kJ/kg. and 2187 kJ/kg,
respectively. If the specific pump work is 2kJ/kg the specific steam consumption (in kg/kW-
h) of the cycle based on net output is __________
Answer: 3.71
3600
Exp: Specific steam consumption 
WT  WP

WT = h 2  h1 = 3159-2187 kJ/kg
WT = 972 kJ/kg
WP = 2 kJ/kg
3600
Thus specific steam consumption  kg kW  h
972  2
 3.71kg kW.h

 India’s No.1 institute for GATE Training  1 Lakh+ Students trained till date  65+ Centers cross
India
12
ME-GATE-2015 PAPER-02| www.gateforum.com

29. A single point cutting tool with 0O rake angle is used in an orthogonal machining process. At
a cutting speed of 180 m/min, the thrust fore is 490N. If the coefficient of friction between the
tool and the chip is 0.7, then the power consumption (in kW) for the machining operation is
__________
Answer: 2.1
F F sin   FT cos 
Exp:   C
N FC cos   FT sin 
Given :   0
FT
 
FC

490
0.7 
FC
FC  700 N
180 1
Power consumption, P  Fc  Vc  700    kW 
60 1000
P  2.1kW

30. The chance of a student passing an exam is 20%. The chance of a student passing the exam
and getting above 90% marks in it is 5% Given that a student passes the examination, the
probability that the student gets above 90% marks is
1 1 2 5
(A) (B) (C) (D)
18 4 9 18
Answer: (B)
Exp: Let Astudent passes the exam
Bstudent gets above 90% marks
Given P(A) = 20%; P(A∩B) = 5%
P  A  B 5% 1
required probabilityis P  B A    
P  A 20% 4

31. A manufacturer has the following data regarding a product:


Fixed cost per month = Rs. 50000
Variable cost per unit = Rs.200
Selling price per unit = Rs.300
Production capacity = 1500 units per month
If the production is carried out at 80% of the rated capacity, that the monthly profit (in Rs.) is
________
Answer: 70000
Exp: Profit per month = 0.8 ×1500 × (300-200) – 50000
= 120000-50000
= 70000

 India’s No.1 institute for GATE Training  1 Lakh+ Students trained till date  65+ Centers cross
India
13
ME-GATE-2015 PAPER-02| www.gateforum.com

32. The head loss for a laminar incompressible flow through a horizontal circular pipe is h1. Pipe
length and fluid remaining the same, if the average flow velocity doubles and the pipe
diameter reduces to half its previous value, the head loss is h2. The ratio h2/h1 is
(A) 1 (B) 4 (C) 8 (D) 16

Answer: (C)
u avg
Exp: head loss h 
D2
2
h D  u
 2   1  avg,2
h1  D 2  u avg,1
 22  2
h2
8
h1

33. A cube and a sphere made of cat iron (each of volume 1000 cm3) were cast under identical
conditions. The time taken for solidifying the cube was 4s. The solidification time (in s) for
the sphere is _________

Answer: 6.15
2
v
Exp: solidification Time  k  
A
2
 a3  a
2
for cube, t  4  k.  2   k.  
 6a  6
a2
k.  4
36
ka 2  36  4 _____ 1
vol.of cube  vol.of sphere
4
 r 3  a 3
3
13
 3 
r   a
 4 
2
4 3
 r   r 2
solidification timefor sphere, t '  k.  3 2    
 4r   3 
 
2
  3 1 3 
 k    .a / 3 
 
  4  
23
 3 
 k.a 2    9
 4 
 6.1573sec

 India’s No.1 institute for GATE Training  1 Lakh+ Students trained till date  65+ Centers cross
India
14
ME-GATE-2015 PAPER-02| www.gateforum.com

34. One kg of air (R = 287 J/kg.K) undergoes an irreversible process between equilibrium state 1
 
20O C, 0.9m3 and equilibrium state 2 (20 OC, 0.6m3). The change in entropy S2 – S1 (in
J/kg.K) is _________
Answer: -116.36
V2
Exp: s 2  s1  mR ln
V1
0.6
 287 ln  116.368J / kgK
0.9

35. In a plane stress condition, the components of stress at point are x  20 MPa, y  80MPa
and xy  40MPa. The maximum shear stress ( in MPa) at the point is
(A) 20 (B) 25 (C) 50 (D) 100
Answer: (C)
2
  2  x   y 
Exp: max  1     xy
2

2  2 
2
 80  20 
    40
2

 2 
 50 MPa

36. Work is done on an adiabatic system due to which its velocity changes from 10 m/s to 20 m/s,
elevation increases by 20 m and temperature increases by 1 K. The mass of the system is 10
kg. CV  100J  kg.K  and gravitational acceleration is 10 m/s2. If there is no change in any
other component of the energy of the system, the magnitude of total work done (in kJ) on the
system is________
Answer: 4.5
Exp: Using SFEE
 v 2 v  2 
m  1  2    z1  z 2  g   h1  h 2    W
 2 2  

102 202 
10    (20) 10  100  1   W
 2 2 
W  4.5kJ
 Work done on the system is 4.5 kJ.

 India’s No.1 institute for GATE Training  1 Lakh+ Students trained till date  65+ Centers cross
India
15
ME-GATE-2015 PAPER-02| www.gateforum.com

37. A hollow shaft do  2di where do and di are the outer and inner diameters respectively)
needs to transmit 20kW power at 3000 RPM. If the maximum permissible shear stress is 30
MPa, dO is
 A  11.29mm  B 22.58mm  C 33.87 mm  D 45.16mm
Answer: (B)
Exp: P  T
2 3000
20 103  T 
60
 T  63.662 N  m
T 
 ,
Ip r
63.662 30 106
 (r0  d),
 4 d
(15d1 ) 1
32
 d1  11.295 mm
 d0  2d1  22.59 mm

38. A cantilever beam OP is connected to another beam PQ with a pin joint as shown in the
figure. A load of 10kN is applied at the mid-point of PQ. The magnitude of bending moment
(in kN-m) at fixed end O is \
10kN
O P Q

2m 1m

(A) 2.5 (B) 5 (C) 10 (D) 25


Answer: (B)
Exp: 10kN
P
Q
C
2m
1m

P-being internal hinge


Mp = 0 _______(1)
Condition ‘PQ’, reaction at Q = 10 kN
Now from (1)
Mp = 0 M0-RQ×1+Rc×0.5 = 0  M0 = 5kN

 India’s No.1 institute for GATE Training  1 Lakh+ Students trained till date  65+ Centers cross
India
16
ME-GATE-2015 PAPER-02| www.gateforum.com

39. The flow stress (in MPa) of a material is given by


  5000.1
Where  is true strain. The Young’s modulus of elasticity of the material is 200 GPa. A block
of thickness 100 mm made of this material is compressed to 95 mm thickness and then the
load is removed. The final dimension of the block (in mm) is _________
Answer: 95.18
100
Exp: Truestrain  ln  0.5129
95
  500   0.5129   371.5147523
.1

Upto elastic limits using hooks law


l
E
l
371.5147523  106  100
 200  109 
l
 l  0.18575mm  considering this for elastic recovery
∴ This will be added to 95 mm
 Final dimension  95.18575 mm

40. The initial velocity of an object is 40m/s. The acceleration a of the object is given by the
following expression: a= 0.1V
Where V is the instantaneous velocity of the object. The velocity of the object after 3 seconds
will be _______
Answer: 29.632
Exp: a=-0.1V
dv
 0.1V
dt
lnv  0.1t  ln k
V  ke 0.1t
at t  0 ; V  40
 k  40
V  40e0.1t
at t  3 sec onds
V  40 e0.13  29.6327 m / s

 India’s No.1 institute for GATE Training  1 Lakh+ Students trained till date  65+ Centers cross
India
17
ME-GATE-2015 PAPER-02| www.gateforum.com

41. A balanced counter flow heat exchanger has a surface area of 20m2 and overall heat transfer
coefficient of 20 W m2  K Air  CP  1000J kg  K  entering at 0.4 kg/s and 280 K is to be

preheated by the air leaving the system at 0.4 kg/s and 300 K. The outlet temperature (in K)
of the preheated air is
(A) 290 (B) 300 (C) 320 (D) 350
Answer: (A)
Exp: Counter flow heat exchanged

Surface Area A  20m mass flowrate  0.4kg s


2

20W TemperatureTci  280K


u 2
m K Tco  ?
J
Cp of air  1000
kgK

Thi
Tco Tho
Tci


Since m is same for both flow = 0.4 kg/s
Assume Cp is same = 1000 J/kg.K
Hence
T1  Ti  T 0  T2  Th0  Tci

T1  300  Tco  Tho  280

Tm  T1  T2



uATm  m C p  Tco  Tci 

20  20   300  Tco   0.4  1000  Tco  280 

2Tco  300  280

580
Tco   290K
2

 India’s No.1 institute for GATE Training  1 Lakh+ Students trained till date  65+ Centers cross
India
18
ME-GATE-2015 PAPER-02| www.gateforum.com

42. The values of function f (x) at 5 discrete point are given below:

x 0 0.1 0.2 0.3 0.4


f(x) 0 10 40 90 160
0.4

Using Trapezoidal role with step size of 0.1, the value of  f  x  dx


0
is _________

Answer: 22

x 0 0.1 0.2 0.3 0.4


Exp: y  f (x) 0 10 40 90 160
y0 y1 y2 y3 y4
h
(y0  y4 )  2(y1  y 2  y3 ) 
0.4 0.4
 0
f (x)dx   ydx 
0 2
0.1
 (0  160)  2(10  40  90)  22
2

43. In a two-stage wire drawing operation, the fractional reduction (ratio of change in cross-
sectional area to initial cross-sectional area) in the first stage is 0.4. The fractional reduction
in the second stage is 0.3. The overall fractional reduction is
(A) 0.24 (B) 0.58 (C) 0.60 (D) 1.00
Answer: (A)
Exp: Since only option (A) is less then 0.3. (A) is the correct answer, as overall reduction will be
less then the reduction in the first stage.

44. A single-degree. Freedom spring-mass system is subjected to a sinusoidal force of 10 N


amplitude and frequency  along the axis of the spring. The stiffness of the spring is 150
N/m, damping factor is 0.2 and the undamped natural frequency is 10. At steady state, the
amplitude of vibration (in m) is approximately
 A  0.05  B 0.07  C 0.70  D  0.90
Answer: (B)

f0 / s
Exp: Amplitude of vibration A=
2 2
    2 
1     
 n   n 
10 / 150
  0.06605  0.07
2
 1 2
  1
2

1       2  0.2  
  10    10 

 India’s No.1 institute for GATE Training  1 Lakh+ Students trained till date  65+ Centers cross
India
19
ME-GATE-2015 PAPER-02| www.gateforum.com

45. For a fully developed laminar flow of water (dynamic viscosity 0.001 Pa-s) through a pipe of
radius 5cm. the axial pressure gradient is – 10Pa/m. The magnitude of axial velocity (in m/s)
at a radial location of 0.2 cm is ________
Answer: 6.24
1 P 2 2
Exp: G
4 x
R  r 
1
  (10)  (0.05)2  (0.002)2 
4  0.001
u  6.24m / s

46. In a certain slider-crank mechanism, lengths of crank and connecting rod are equal. If the
crank rotates with a uniform angular seed of 14 rad/s and the crank length is 300 mm, the
maximum acceleration of the slider (in m/s2) is ___________
Answer: 117.6
Exp: a max  2r2 (when   0) i.e at Inner dead centre

 2  0.3 142
a max  117.6 m / s2

1
   9xi  3yj.ndS over the sphere giveb by x  y2  z 2  9 is ____
2
47. The surface integral
s

Answer: 216
Exp: By Gauss divergence theorem,

 S
F.n ds   divFdV
V

Here F  9x i  3y
div F  9  3  6

 
1

 9x i  3yj .nds   6 dV  6V
1
 V
1

s

1 4 3
 6  r 
 3 
 8(3)3
 216.

 India’s No.1 institute for GATE Training  1 Lakh+ Students trained till date  65+ Centers cross
India
20
ME-GATE-2015 PAPER-02| www.gateforum.com

48. A cantilever beam with square cross-section of 6mm side is subjected to a load of 2kN normal
to the top surface as shown in the figure. The young’s modulus of elasticity of the material of
the beam is 210 GPa. The magnitude of slope. (in radian) at Q (20 mm from the fixed end) is
________

20mm 2kN

O
Q P
100mm
Answer: 0.1587
Px
Exp: Slope. Q  (2L  x)
2EI
where x is measured from the fixed end
2000(0.02)
Q  (2  0.1  0.02)
(0.006)4
2  210 10 
9

12
=-0.1587
The magnitude of slope is 0.1587 radian

49. A cylindrical uranium fuel rod of radius 5 mm in a nuclear reactor is generating heat at the
rate of 4 107 W m3 . The rod is cooled by a liquid (convective heat transfer coefficient 1000
W m2 .K ) at 25O C. At steady state, the surface temperature (in K) of the rod is
(A) 308 (B) 398 (C) 418 (D) 448
Answer: (B)
q g
Exp: The surface temp. (Tw )  Ta  R
2h
4 107
 298   5 103
2 1000
Tw  398 K

50. For the same values of peak pressure, peak temperature and heat rejection, the correct order of
efficiencies for Otto, Dual and Diesel cycles is
 A  otto  Dual  Diesel  B Diesel  Dual  otto
 C Dual  Diesel  otto  D Diesel  Otto  Dual
Answer: (B)
Exp: For same values of peak pressure and temperature. Diesel cycle is most efficient and
Otto cycle is least. Efficiency of dual cycle lies in between.
diesel > dual > otto

 India’s No.1 institute for GATE Training  1 Lakh+ Students trained till date  65+ Centers cross
India
21
ME-GATE-2015 PAPER-02| www.gateforum.com

51. During a TIG welding process, the are current and are voltage were 50 A and 60 V,
respectively, when in the welding speed was 150 mm/mi. In another process, the TIG welding
is carried out at a welding speed of 120 mm/min at the same arc voltage and heat input to the
material so that weld quality remains the same. The welding current (in A) for this process is
(A) 40.00 (B) 44.72 (C) 55.90 (D) 62.25
Answer: (A)
Exp: Total heat input = VIt
time t is inversely proportional to weld speed (S)
V2 I 2 V1I1
 
S2 S1
120
I 2   50
150
I 2  40 A

52. Consider the following differential equation:


dy
 5y; initial condition : y  2 at t  0.
dt
The value of y at t = 3 is
 A  5e10  B 2e10  C 2e15  D  15e2
Answer: (C)
dy
Exp:  5y
dt
dy
  5dt  variablesseparableform 
y

Integrating,
ln y  5t  c _____ 1

when y  2at t  0  initialconditional, 1 gives 


c  ln 2
y
ln y  5t  ln 2  ln    5t  y  2e 5t
2
at t  3, y  2e 15

 India’s No.1 institute for GATE Training  1 Lakh+ Students trained till date  65+ Centers cross
India
22
ME-GATE-2015 PAPER-02| www.gateforum.com

53. For the truss shown in the figure, the magnitude of the force (in kN) in the member SR is

30kN
W V S R

1m

Q
P U T
1m 1m 1m

(A) 10 (B) 14.14 (C) 20 (D) 28.28

Answer: (C)

30kN
Exp:
W V S R

1m

P X
U T
Rx
Rp 2m 1m
3m
Rp + Rx = 30000
Mp = 0
S R
Rx×3 = 2×3000
 Rx = 20kN
20
and Rp = 10 kN x
T
for balance at ‘x’  FRx = 20kN 20

at ‘RT’ FRT cos45 = 20


20
  FRT  ____ (1)
cos 45
Also FSR = FRTcos45 ____ (2)
from (1) and (2)
FSR = 20 kN

 India’s No.1 institute for GATE Training  1 Lakh+ Students trained till date  65+ Centers cross
India
23
ME-GATE-2015 PAPER-02| www.gateforum.com

54. A project consists of 7 activities. The network along with the time durations (in days) for
various activities is shown in the figure.

12 11 10
1 3 5 6

14
12
9

2 7 4

The minimum time (in days) for completion of the project is _____
Answer: 40
Exp: Time taken for 3 paths are as follows
Path 1 = 12+11+10
Path 2 = 14+12+10
Path 3 = 14+7+9+10
∴ Path3 is longest i.e. path 3 is critical path
∴ Project duration = 40 days

55. A resistance-capacitance relaxation circuit is used in an electrical discharge machining


process. The discharge voltage is 100 V. At a spark cycle time of s, the average power input
required is 1 kW. The capacitance  in F  in the circuit is
(A) 2.5 (B) 5.0 (C) 7.5 (D) 10.0
Answer: (A)
Exp: Voltage = 100V
Power = 1kW
V2 100  100
Power  R  10
R 1000
 25
  RC  c    2.5 F
R 10

 India’s No.1 institute for GATE Training  1 Lakh+ Students trained till date  65+ Centers cross
India
24
ME-GATE-2015 PAPER-03| www.gateforum.com

General aptitude

Q. No. 1 – 5 Carry One Mark Each

1. Five teams have to compete in a league, with every team playing every other team exactly
once, before going to the next round. How many matches will have to be held complete
the league round of matches?
(A) 20 (B) 10 (C) 8 (D) 5
Answer: (B)
Exp: For a match to be played, we need 2 teams
No of matches = no. of ways of selections 2 teams out of 5
 5C 2  10

2. Tanya is older than Enc.


Cliff is older than Tanya.
Eric is older than Cliff.
If the first two statements are true, then the third statement is
(A) True (B) False (C) Uncertain (D) Data insufficient
Answer: (B)

3. Choose the appropriate word/phase, out of the four options given below, to complete the
following sentence:
Apparent lifelessness ___________ dormant life.
(A) harbours (b) lead to (c) supports (d) affects
Answer: (A)
Exp: Apparent: looks like
dormant: hidden
Harbour: give shelter
Effect (verb): results in

4. Choose the statement where underlined word is used correctly.


(A) When the teacher eludes to different authors, he is being elusive
(B) When the thief keeps eluding the police, he is being elusive
(C) Matters that are difficult to understand, identify or remember are allusive
(D) Mirages can be allusive, but a better way to express them is illusory
Answer: (B)
Exp: Elusive: Difficult to answer.

© All rights reserved by GATE Forum Educational Services Pvt. Ltd. No part of this booklet may be reproduced or utilized
in any form without the written permission. Discuss this questions paper at www.gatementor.com.
1
ME-GATE-2015 PAPER-03| www.gateforum.com

5. Fill in the blank with the correct idiom/phrase.


That boy from the town was a __________ in the sleepy village.
(A) Dog out of herd (B) Sheep from the heap
(C) Fish out of water (D) Bird from the flock
Answer: (C)
Exp: From the statement, it appears that boy found it tough to adapt to a very different
situation.

Q. No. 6 – 10 Carry Two Marks Each

6. Right triangle PQR is to be constructed in the xy – plane so that the right angle is at P and
line PR is parallel to the-axis. The x and y coordinates of P, Q, and R are to be integers
that satisfy the inequalities: –4 ≤ x ≤ 5 and 6 ≤ y ≤ 16. How many different triangles
could be constructed with these properties?
(A) 110 (B) 1,100 (C) 9,900 (D) 10,000
Answer: (B)

7. Select the appropriate option in place of underlined part of the sentence.


Increased productivity necessary reflects greater efforts made by the employees.
(A) Increase in productivity necessary
(B) Increase productivity is necessary
(C) Increase in productivity necessarily
(D) No improvement required
Answer: (C)

8. Given below are two statements followed by two conclusions. Assuming these statements
to be true, decide which one logically follows:
Statements:
I. No manager is a leader. II. All leaders are executive.
Conclusions:
I. No manager is a executive. II. All executive is a manager.
(A) Only conclusion I follows. (B) Only conclusion II follows.
(C) Neither conclusion I nor II follows. (D) Both conclusion I and II follow.
Answer: (D)
Exp: S  1: S 2:

M L L E

Therefore concluding diagram can be


E E E

L E or L M or L M

© All rights reserved by GATE Forum Educational Services Pvt. Ltd. No part of this booklet may be reproduced or utilized
in any form without the written permission. Discuss this questions paper at www.gatementor.com.
2
ME-GATE-2015 PAPER-03| www.gateforum.com

9. A coin is tossed thrice. Let X be the event that head occurs in each of the first two tosses.
Let Y be the event that a tail occurs on the third toss. Let Z be the event that two tails
occurs in three tosses. Based on the above information, which one of the following
statements is TRUE?
(A) X and Y are not independent
(B) Y and Z are dependent
(C) Y and Z are independent
(D) X and Z independent
Answer: (D)
Exp: x = {HHT, HHH}
y depends on x
z = {TTH, TTT}
∴ D is the correct choice.

10. In the given figure angle Q is a right angle, PS:QS = 3:1, RT:QT = 5:2 and PU:UR = 1:1.
If area of triangle QTS is 20 cm2, then the area of triangle PQR in cm2 is ______.
R

U T

P Q
S
Answer: 280
Exp: Let area of triangle PQR be ‘A’
SQ 1 1
 
PQ 1  3 4
QT 2 2
 
QR 2  5 7
1
Area of  le QTS   SQ  QT
2
1 1  2 
   PQ    QR 
2 4  7 
1 2 1 
     PQ  QR 
4 7 2 
1
  Area of  le PQR
14
1
given 20cm 2  A
14
A  14  20  280cm 2

© All rights reserved by GATE Forum Educational Services Pvt. Ltd. No part of this booklet may be reproduced or utilized
in any form without the written permission. Discuss this questions paper at www.gatementor.com.
3
ME-GATE-2015 PAPER-03| www.gateforum.com

Mechanical Engineering
Q. No. 1 – 25 Carry One Mark Each

1. Three parallel pipes connected at the two ends have flow-rates Q1, Q2 and Q3 respectively,
and the corresponding frictional head losses are hL1, hL2 and hL3 respectively. The correct
expressions for total flow rate (Q) and frictional head loss across the two ends (hL) are
(A) Q = Q1 + Q2 + Q3; hL = hL1 + hL2 + hL3
(B) Q = Q1 + Q2 + Q3; hL = hL1 = hL2 = hL3
(C) Q = Q1 = Q2 = Q3; hL = hL1 + hL2 + hL3
(D) Q = Q1 = Q2 = Q3; hL = hL1 = hL2 = hL3
Answer: (B)
Exp: Total flow rate Q  Q1  Q2  Q3
head loss h  h L1  h L2  h L3

4 2
2. The lowest eigen value of the 2 × 2 matrix   is ________
1 3
Answer: 2
4 2
Exp: Let A   
1 3 
Characteristic equation of A is A  I  0
4 2
 0
1 3
  2  7  10  0    2,5

3. Which two of the following joining processes are autogenous?


i. Diffusion welding
ii. Electroslag welding
iii. Tungsten inert gas welding
iv. Friction welding
(A) i and iv (B) ii and iii (C) ii and iv (D) i and iii
Answer: (A)
Exp: Diffusion welding and friction welding are autogenous welding process as they do not
require any filler material.

4. The strain hardening exponent n of stainless steel SS 304 with distinct yield and UTS
values undergoing plastic deformation is
(A) n < 0 (B) n =0 (C) 0 < n < 1 (D) n = 1
Answer: (C)
Exp: n lies between 0 and 1. 0 means material is a perfectly plastic solid, while 1 represents a
100% elastic solid.

© All rights reserved by GATE Forum Educational Services Pvt. Ltd. No part of this booklet may be reproduced or utilized
in any form without the written permission. Discuss this questions paper at www.gatementor.com.
4
ME-GATE-2015 PAPER-03| www.gateforum.com

5. In the figure, link 2 rotates with constant angular velocity ω 2. A slider link 3 moves
outwards with a constant relative velocity VQ/P, where Q is a point on slider 3 and P is a
point on link 2. The magnitude and direction of Coriolis component of acceleration is
given by
Qon 3 VQP

P on 2
3

2 2
O

(A) 2ω2 VQ/P; direction of VQ/P rotated by 90° in the direction ω2


(B) ω2 VQ/P; direction of VQ/P rotated by 90° in the direction ω2
(C) 2ω2 VQ/P; direction of VQ/P rotated by 90° opposite to the direction of ω2
(D) ω2 VQ/P; direction of VQ/P rotated by 90° opposite to the direction ω2
Answer: (A)
Exp: direction is obtained by rotating velocity vector through 90o in the direction of rotation of
the link.

6. Couette flow is characterized by


(A) steady, incompressible, laminar flow through a straight circular pipe
(B) fully developed turbulent flow through a straight circular pipe
(C) steady, incompressible, laminar flow between two fixed parallel plates
(D) steady, incompressible, laminar flow between one fixed plate and the other moving
with a constant velocity
Answer: (D)
Exp: Coutte flow is steady incompressible, laminar flow between one fixed plate and other
moving with constant velocity.
V

7. If P(X) = 1 4,P  Y   1 3, and P  X  Y  1 12, the value of P  Y X  is


1 4 1 29
A  B C  D
4 25 3 50
Answer: (C)
1
P X  Y 1
Exp: PY X   12 
PX 1 3
4

© All rights reserved by GATE Forum Educational Services Pvt. Ltd. No part of this booklet may be reproduced or utilized
in any form without the written permission. Discuss this questions paper at www.gatementor.com.
5
ME-GATE-2015 PAPER-03| www.gateforum.com

8. In a machining operation, if the generatrix and directix both are straight lines, the surface
obtained
(A) cylindrical (B) helical
(C) plane (D) surface of revolution
Answer: (C)
Exp: The surface obtained is plane.

9. A rigid container of volume 0.5 m3 contains 1.0 kg of water at 120°C (vf = 0.00106
m3/kg, vg = 0.8908 m3/kg). The state of water is
(A) Compressed liquid
(B) Saturated liquid
(C) A mixture of saturated liquid and saturated vapor
(D) Superheated vapor
Answer: (C)
0.5 3
Exp: V m / kg  0.5m3 / kg
1
Since Vf  V  Vg the state of water is mixture of saturated water and saturated vapour.

10. In full mould (cavity-less) casting process, the pattern is made of


(A) expanded polystyrene (B) wax
(C) epoxy (D) plaster of Paris
Answer: (A)
Exp: The pattern is made of expanded polystrene

11. A gear train is made up of five spur gears as shown in the figure. Gear 2 is driver and gear
6 is driven member. N2, N3, N4, N5 and N6 represent number of teeth on gears 2, 3, 5 and 6
respectively. The gear(s) which act(s) as idler(s) is/are

N5
N3
N6
N2 2

4 5 6
Driver 3
Driven
N4
(A) only 3 (B) only 4
(C) only 5 (D) Both 3 and 5
Answer: (C)
2 2 3 5 N3 N5 N6 NN
Exp:    3 6
6 3 5 6 N2 N4 N5 N2 N4
 only Gear(5) is Idle

© All rights reserved by GATE Forum Educational Services Pvt. Ltd. No part of this booklet may be reproduced or utilized
in any form without the written permission. Discuss this questions paper at www.gatementor.com.
6
ME-GATE-2015 PAPER-03| www.gateforum.com

12. Let  be an arbitrary smooth real valued scalar function and V be an arbitrary smooth
vector valued function in a three-dimensional space. Which one of the following is an
identity?
 A   
Cur l V   Div V   B Div V  0

 C DivCurlV  0  D  
Div V  DivV
Answer: (C)

13. Which of the following statements are TRUE for damped vibrations?
P. For a system having critical damping, the value of damping ratio is unity and system
does not undergo a vibratory motion.
Q. Logarithmic decrement method is used to determine the amount do damping in a
physical system.
R. In case of damping due to dry friction between moving surfaces resisting force of
constant magnitude acts opposite to the relative motion.
S. For the case of viscous damping, drag force is directly proportional to the square of
relative velocity.
(A) P and Q only (B) P and S only
(C) P, Q and R only (D) Q and S only
Answer: (C)

  sin x 
14. The value of lim x  0   is ____
 2sin x  x cos x 
Answer: -0.333
  sin x  0 
Exp: lim    form 
x 0  2sin x  x cos x  0 
  cos x 
 lim  
x 0  2cos x  cos x  x sin x 
 L  Hospital Rule 
1

3

15. The ratio of momentum diffusivity (v) to thermal diffusivity (α), is called
(A) Prandtl number (B) Nusselt number
(C) Biot number (D) Lewis number
Answer: (A)
Cp VCp V V
Exp: Pr    
k k k 
cp

© All rights reserved by GATE Forum Educational Services Pvt. Ltd. No part of this booklet may be reproduced or utilized
in any form without the written permission. Discuss this questions paper at www.gatementor.com.
7
ME-GATE-2015 PAPER-03| www.gateforum.com

16. For the given fluctuating fatigue load, the values of stress amplitude and stress ratio are
respectively
MPa
250
200
150
100
50
0 Time

(A) 100 MPa and 5 (B) 250 MPa and 5


(C) 100 MPa and 0.20 (D) 250 MPa and 0.20
Answer: (C)
max  min 250  50
Exp: stress amplitude    100 Mpa
2 2
min 50
Stress ratio    0.2
max 250

2
17. Using a unit step size, the value of integral 1
x ln x dx by trapezoidal rule is ______

Answer: 0.69
x 1 2
Exp:
y  1hx 0 21h2
By Trapezoidal Rule,
2
1
 x1nx dx  2 0  21n2  1n2  0.69
1

18. A weight of 500 N is supported by two metallic ropes as shown in the figure. The values
of tensions T1 and T2 are respectively

30O
T1
90O T2

120O

500 N

(A) 433 N and 250 N (B) 250 N and 433 N


(C) 353.5 N and 250 N (D) 250 N and 353.5 N
Answer: (A)
Exp: Using sine rule

© All rights reserved by GATE Forum Educational Services Pvt. Ltd. No part of this booklet may be reproduced or utilized
in any form without the written permission. Discuss this questions paper at www.gatementor.com.
8
ME-GATE-2015 PAPER-03| www.gateforum.com

T1 T2 500 T1
o
 o
 T2
sin120 sin150 sin 90o 90o
T1  500  sin120o ; T2  500sin150o
T1  433 N T2  250 N 150o 120o

500

19. In the notation (a/b/c) : (d/e/f) for summarizing the characteristics of queueing situation,
the letters ‘b’ and ‘d’ stand respectively for
(A) service time distribution and queue discipline
(B) number of servers and size of calling source
(C) number of servers and queue discipline
(D) service time distribution and maximum number allowed in system
Answer: (A)
Exp: b: Service time distribution (usually represented by ‘m’)
D: Queuing discipline (usually represented by ‘GD’)

20. The thermodynamic cycle shown in figure (T/s diagram) indicates


2
(A) Reversed Cannot cycle T
(B) Reversed Brayton cycle PC

(C) Vapor compression cycle 3 PC 1


(D) Vapor absorption cycle 4
s
Answer: (B)
Exp: T-S diagram represent a reversed brayton cycle used in air conditioning of aero planes
where air is used as a refrigerant.

21. A drill is positioned at point P and its has to proceed to point Q. The coordinates of point
Q in the incremental system of defining position of a point in CNC part program will be
Y

P
12
5

3 4

(A) (3, 12) (B) (5, 7) (C) (7, 12) (D) (4, 7)
Answer: (D)
Exp: In incremental system. Co-ordinates of point Q are (4,7).

© All rights reserved by GATE Forum Educational Services Pvt. Ltd. No part of this booklet may be reproduced or utilized
in any form without the written permission. Discuss this questions paper at www.gatementor.com.
9
ME-GATE-2015 PAPER-03| www.gateforum.com

22. A cylindrical tank with closed ends is filled with compressed air at a pressure of 500 kPa.
The inner radius of the tank is 2m, and it has wall thickness of 10 mm. The magnitude of
maximum in-plane shear stress (in MPa) is ____ .
Answer: 25
pd
Exp: Maximum in-plane shear stress max 
8t
500  4
 MPa  25MPa
8  10

23. An air-standard Diesel cycle consists of the following processes:


1-2: Air is compressed isentropically.
2-3: Heat is added at cosntant pressure.
3-4: Air expands isentropically to the original volume.
4-1: Heat is rejected at constant volume.
If  and T denotes the specific heat ratio and temperature, respectively the efficiency of
the cycle is
T4  T1 T4  T1
(A) 1  (B) 1
T3  T2   T3  T2 

  T4  T1  T4  T1
(C) 1  (D) 1
T3  T2    1 T3  T2 
Answer: (B)

2
Exp: heat applied, Qs  cp  T3  T2  p
3

heat rejected, Qr  cr  T4  T1  4
Qr 1 (T4  T1 )
 1 1
Qs  (T3  T2 ) 1

v

24. Saturated vapor is condensed to saturated liquid in condenser. The heat capacity ratio is
c
Cr  min . The effectiveness () of the condenser is
cmax
1  exp   NTU(1  Cr ) 1  exp   NTU(1  Cr ) 
(A) (B)
1  Cr 1  Cr exp   NTU(1  Cr ) 
NTU
(C) (D) 1  exp(NTU)
1  NTU
Answer: (D)
Exp: E of condenser is given by 1-exp(-NTU)
Cmin
because Cr   0. (as Cmax  )
Cmax

© All rights reserved by GATE Forum Educational Services Pvt. Ltd. No part of this booklet may be reproduced or utilized
in any form without the written permission. Discuss this questions paper at www.gatementor.com.
10
ME-GATE-2015 PAPER-03| www.gateforum.com

25. For the same material and the mass, which of the following configurations of flywheel
will have maximum mass moment of inertia about the axis of rotation OO’ passing
through the center of gravity

O  O
A  B O'

C  D O
O'

Answer: (B)
Exp: Rim med wheel has maximum mass located away from the axis of rotation Thus will have
maximum moment of inertia.

Q. No. 26 – 55 Carry Two Marks Each

26. For ball bearings, the fatigue life L measured in number of revolutions and the radial load
F are related by FL1/3 = K, where K is a constant. It withstands a radial load of 2 kN for a
life of 540 million revolutions. The load (in kN) for a life of one million revolutions is
_______
Answer: 16.286
1
Exp: FL3  k
1 1
F1L31  F2 L32
1 1
2  540  F2 (1)
3 3

 F2  16.286 kN

© All rights reserved by GATE Forum Educational Services Pvt. Ltd. No part of this booklet may be reproduced or utilized
in any form without the written permission. Discuss this questions paper at www.gatementor.com.
11
ME-GATE-2015 PAPER-03| www.gateforum.com

27. The torque (in N-m) exerted on the crank shaft of a two stroke engine can be described as
T = 10000 + 1000 sin 1θ – 1200 cos 2θ, where θ is the crank angle as measured from
inner dead center position. Assuming the resisting torque to be constant, the power (in
kW) developed by the engine at 100 rpm is __________.
Answer: 16.67
2

Exp: Tmean 
0
Td

10000
Nm
2 2
10000 100  2 1
P  Tmean      16.67 kW
2 60 1000

28. The value of moment o inertia of the section shown in the figure about the ais-XX is

60
30

All dimensions 45
are in mm 120
15
X X
15

45

30

 A 8.5050 106 mm4  B 6.88.50 105 mm4

 C 7.7625 106 mm4  D 8.5725 106 mm4


Answer: (B)
1 1 
Exp: Moment of Inertia. I xx  (120)3  60  2   (30)4  30  30  30
12  12 
 6.885 106 mm4

29. The value of

 3x  8y  dx   4y  6xy  dy ,


2
C (where C is boundary of th region bounded by x = 0,
y = 0 and x + y = 1 is) is _____________

Answer: 3.66
Exp: x  0 to x  1  y
&
y  0 to y  1

© All rights reserved by GATE Forum Educational Services Pvt. Ltd. No part of this booklet may be reproduced or utilized
in any form without the written permission. Discuss this questions paper at www.gatementor.com.
12
ME-GATE-2015 PAPER-03| www.gateforum.com

 3x  8y  dx   4y  6xy dy
2

By Green’s theorem, 
C
m N
 N m 
   x  y dxdy
1 1 y
  6y   16y   dxdy
    0,1
y 0 x 0
x  y 1
1 1 y 
 
  22ydx  dy
 x 0
y 0  
1  0,0  1,0 

1 y
 22 yx 0 dy
y 0
1
 y 2 y3 
1
 22 y 1  y   0  dy  22   

y 0  2 3 
0

 1 1  11
 22      3.66
3 3 3

 W 
30. A brick wall  k  0.9  of thickness 0.18 m separates the warm air in a room from the
 m.k 
cold ambient air. On a particular winter day, the outside air temperature is – 5°C and the
room needs to be maintained at 27°C. The heat transfer coefficient associated with
W
outside air is 20 2 . Neglecting the convective resistance of the air inside the room,
m K'
 w 
the heat loss, in  2  is
m 
(A) 88 (B) 110 (C) 128 (D) 160
Answer: (C)
Exp:

5o C 27o C
h  20w / m 2 K

1 1
Total thermal resistance 
R th 1  l
h k
1 1
  4 W / m2 K
R th 1 0.18

20 0.9
T
Q   27  (5)  4  128 W / m 2
R th

© All rights reserved by GATE Forum Educational Services Pvt. Ltd. No part of this booklet may be reproduced or utilized
in any form without the written permission. Discuss this questions paper at www.gatementor.com.
13
ME-GATE-2015 PAPER-03| www.gateforum.com

31. A bullet spins as the shot is fired from a gun. For this purpose, two helical slots as shown
in the figure are cut in the barrel. Projections A and B on the bullet engage in each of the
slots

Gun Barrel A
Bullet

0.5m B

Helical slots are such that one turn of helix is completed over a distance of 0.5 m. If
velocity of bullet when it exits the barrel is 20 m/s, its spinning speed in rad/s is _______.

Answer: 251.3
0.5
Exp: Time taken force revolution   0.025 sec.
20
2
The spinning speed is rad / sec
0.02s
 251.3 rad / sec

32. Which of the following statements are TRUE, when the cavitation parameter σ = 0?
i. the local pressure is reduced to vapor pressure
ii. cavitation starts
iii. boiling of liquid starts
iv cavitation stops
(A) i, ii and iv (B) only ii and iii (C) only i and iii (D) i, ii and iii

Answer: (D)
Exp: σ = 0 implies (i), (ii) and (iii)

33. In a CNC milling operation, the tool has to machine the circular arc from point (20, 20) to
(10, 10) at sequence number 5 of the CNC part program. If the center of the arc is at (20,
10) and the machine has incremental mode of defining position coordinates, the correct
tool path command is (A) N 05 G 90 G01 X–10 Y–10 R10
(B) N 05 G91 G03 X–10 Y–10 R10
(C) N 05 G90 G03 X20 Y20 R10
(D) N 05 G91 G02 X20 Y20 R10

Answer: (B)
Exp: for incremental coordinates (G91) and coordinates of final point are (10, 10). The tool
moves CCW (counter clockwise), So G03.

© All rights reserved by GATE Forum Educational Services Pvt. Ltd. No part of this booklet may be reproduced or utilized
in any form without the written permission. Discuss this questions paper at www.gatementor.com.
14
ME-GATE-2015 PAPER-03| www.gateforum.com

34. Ratio of solidification time of a cylindrical casting (height =radius) to the cubic
casting of side two times the height of cylindrical casting is _____.
Answer: 0.25
2
V
Exp: ts  k  
A
for cylindrical cavity (1)
2
  2 
 d h  2
d
  d  h 
t1  k  4 
 k
 dh  2 d 2  6
 4 
for cubic casting (2)
2
 a3  a
2
d
2
t 2  k  2   k    k    a  2d 
 6a  6 3
t1 32
   0.25
t 2 62

35. The number of degrees of freedom of the linage shown in the figure is

(A) -3 (B) -0 (C) 1 (D) 2

Answer: (C)
Exp: Number of links, N = 6
Total number as binary joints, j = 7
F = 3 (N-1) – 2j
= 15-14 = 1.

36. Figure shows a single degree of freedom system. The system consists of a massless rigid
bar OP hinged at O and a mass m at end P. The natural frequency of vibration of the
system i
1 k 1 k k
A fn   B fn  O
P
2 4m 2 2m Q
m
1 k 1 2k a a
 C fn   D fn 
2 m 2 m
Answer: (A)
Exp: force in the spring F= 2mg [from equilibrium]

© All rights reserved by GATE Forum Educational Services Pvt. Ltd. No part of this booklet may be reproduced or utilized
in any form without the written permission. Discuss this questions paper at www.gatementor.com.
15
ME-GATE-2015 PAPER-03| www.gateforum.com

Deflection as mass at P,
a a
x  P
x1   2a  2x O
a
X
2mg 4mg
 2  X1
k k
g g k
wn    ;
s x1 4m
1 1 k
fn  n 
2 2 4m

37. For the linear programming problem:

Maximize Z = 3X1 + 2X2

Subject to

– 2X1 + 3X2 ≤ 9

X1 – 5X2 ≥ – 20

X1, X2 ≥ 0

The above problem has


(A) unbounded solution
(B) infeasible solution
(C) alternative optimum solution
(D) degenerate solution
Answer: (A)
Exp: Plotting the graph for the given constraints as shown in figure.
From figure we can see that LPP has unbounded solution.

x2

(0,4)

(20,0) x1 
(45,0)

© All rights reserved by GATE Forum Educational Services Pvt. Ltd. No part of this booklet may be reproduced or utilized
in any form without the written permission. Discuss this questions paper at www.gatementor.com.
16
ME-GATE-2015 PAPER-03| www.gateforum.com

38. Air in a room is at 35° and 60% relative humidity (RH). The pressure in the room is 0.1
MPa. The saturation pressure of water at 35°C is 5.63 kPa. The humidity ratio of the air
(in gram/kg of dry air) is _____.
Answer: 21.74
P P
Exp:   w  0.6  w
Ps 5.63
 Pw  3.378kPa
Pw
humidity Ratio, w  0.622
Pa  Pw
3.378
 0.622 
100  3.378
 0.021745kg kg of dry air
or 21.745 g/kg of dry air

39. A solid sphere 1 of radius ‘r’ is placed inside a hollow, closed hemispherical surface 2 of
radius ‘4r’. The shape factor F2-1 is

2r 2
1

8r

1 1
A  B  C 2  D  12
12 2
Answer: (A)
Exp: f11  f12  1
f12  1
f 21 A 2  f12 A1
f12 A1 1  4r 2 1
f 21   
1
A2 4  4r     4r  12
2 2

40. Newton-Raphson method is used to find the roots of the equation, x3 + 2x2 + 3x – 1 = 0. If
the initial guess is x0 = 1, then the value of x after 2nd iteration is ___________.
Answer: 0.30
Exp: By Newton-Raphson Method,
f  x0 
1st iteration, x1  x 0 
f  x0 
f 1 5 1
1 1 
f  1 10 2

© All rights reserved by GATE Forum Educational Services Pvt. Ltd. No part of this booklet may be reproduced or utilized
in any form without the written permission. Discuss this questions paper at www.gatementor.com.
17
ME-GATE-2015 PAPER-03| www.gateforum.com

Where f  x   x 3  2x 2  3x  1  f 1  5
f   x   3x 2  4x  3  f  1  10

f  x1 
2nd iteration, x 2  x1 
f   x1 
f  0.5 
 0.5   0.3043
f   0.5 

41. The annual requirement of rivets at a ship manufacturing company is 2000 kg. The rivets
are supplied in units of 1 kg costing Rs. 25 each. If the costs Rs. 100 to place an order and
the annual cost of carrying one unit is 9% of its purchase cost, the cycle length of the
order (in days) will be_____
Answer: 76.94
2DC0
Exp: EOQ 
Ch

2  2000  100
  421.637
0.09  25
365
length of cycle   76.948 days
No. of orders
2000
No. of orders 
EOQ


 f t e
 st
42. Laplace transform of the function f(t) is given by F(s) = L{f(t)} = dt.
0

Laplace transform of the function shown below is given by

f t

1 t

1  e2s 1  es 2  2es


A  B  C
s 2s s
1  2es
 D
s
Answer: (C)
Exp: f  t   2; 0  t 1
0; otherwise
1
est 2  2es
1
 L f  t   2e dt  2

 st

0
s 0
s

© All rights reserved by GATE Forum Educational Services Pvt. Ltd. No part of this booklet may be reproduced or utilized
in any form without the written permission. Discuss this questions paper at www.gatementor.com.
18
ME-GATE-2015 PAPER-03| www.gateforum.com

43. Orthogonal turning of a mild steel tube with a tool of rake angle 10° carried out at a feed
of 0.14 mm/rev. If the thickness of the chip produced is 0.28 mm, the values of shear
angle and shear strain will be respectively
(A) 28°20′ and 2.19 (B) 22°20′ and 3.53
(C) 24°30′ and 3.53 (D) 37°20′ and 5.19
Answer: (A)
0.14
Exp: r  0.5
0.28
r cos 
tan  
1  r sin 
  28.3345o
or 28o 20
Shear strain, Y  cot   tan(  )  2.1859  2.19

44. Steam enters a turbine at 30 bar, 300°C (u = 2750 kJ/kg, h = 2993 kJ/kg) and exits the
turbine as saturated liquid at 15 kPa (u = 225 kJ/kg, h = 226 kJ/kg). Heat loss to the
surrounding is 50 kJ/kg of steam flowing through the turbine. Neglecting changes in
kinetic energy and potential energy, the work output of the turbine (in kJ/kg of steam) is
_______.
Answer: 2717
Exp: Work output  (2993  226.50)kJ / kg =2717 kJ/kg

 4  3i i 
45. For a given matrix  , where is i  1, the inverse of matrix P is
 i 4  3i 

1  4  3i i  1  i 4  3i 
A   B
24  i 4  3i  
25  4  3i i 

1  4  3i i  1  4  3i i 
 C  D 

24  i 4  3i  25  i 4  3i 
Answer: (C)
Exp: |P| = (4+3i)(4-3i)-(i)(-i) = 16+9-1 = 24
 4  3i i 
adjP  
 i 4  3i 
1  4  3i i 
P 1  
24  i 4  3i 

© All rights reserved by GATE Forum Educational Services Pvt. Ltd. No part of this booklet may be reproduced or utilized
in any form without the written permission. Discuss this questions paper at www.gatementor.com.
19
ME-GATE-2015 PAPER-03| www.gateforum.com

46. For the overhanging beam shown in figure, the magnitude of maximum bending moment
(in kN-m) is _____.
20kN
10kN m
B
A C
4m 2m

Answer: 40 kN-m
Exp: BMD:

5
M
B
(KN  m) A C
X

40
R A  10kN
R B  50kN
Maximum bending momentum occurs at reaction B an has a magnitude of 40 kN-m.

47. Figure shows a wheel rotating about O2. Two points A and B located along the radius of
wheel have speeds of 80 m/s and 140 m/s respectively. The distance between the points A
and B is 300 mm. The diameter of the wheel (in mm) is ________

Answer: 1400
Exp: VA  80 m / s, VB  140 m / s 2

rB  rA  300 … (i)
A B
 rA  80 O2
 rB  140
rB
  1.75 ...(2)
rA
Solving (1) & (2), rB=700 mm.
 diameter of wheel is 1400 mm.

© All rights reserved by GATE Forum Educational Services Pvt. Ltd. No part of this booklet may be reproduced or utilized
in any form without the written permission. Discuss this questions paper at www.gatementor.com.
20
ME-GATE-2015 PAPER-03| www.gateforum.com

48. The dimensions of a cylindrical side riser (height = diameter) for a 25 cm × 15 cm × 5 cm


steel casting are to be determined. For the tabulated shape factor values given below,
diameter of the riser (in cm) is ______.
Shape Factor 2 4 6 8 10 12
Riser Volume / Casting Volume 1.0 0.70 0.55 0.50 0.40 0.35

Answer: 13.36
l  w 25  15
Exp: Shapefactor    8 then from the table
h 5
Vr
  0.5
Vc
 2
d h  0.5  25  15  5
4
or d 3  2387.324cm3
d  13.365cm

49. A Prandtl tube (Pitot-static tube with C = 1) is used to measure the velocity of water. The
differential manometer reading is 10 mm of liquid column with a relative density of 10.
Assuming g = 9.8 m/s2, the velocity of water (in m/s) is _______.
Answer: 1.32
Exp: Velocity as water  Cv 2gh
Cv = 1 (Given)
 sg 
h  x   1  0.0110  1  0.09 m
 s0 
velocityof flow  2  9.8  0.09  1.328m s

50. In a rolling operation using rolls of diameter 500 mm if a 25 mm thick plate cannot be
reduced to less than 20 mm in one pass, the coefficient of friction between the roll and the
plate is __________
Answer: 0.1414

 h max
Exp: 
R
 h max  25  20  5mm
R  250 mm
5
   0.1414
250

© All rights reserved by GATE Forum Educational Services Pvt. Ltd. No part of this booklet may be reproduced or utilized
in any form without the written permission. Discuss this questions paper at www.gatementor.com.
21
ME-GATE-2015 PAPER-03| www.gateforum.com

51. Refrigerant vapor enters into the compressor of a standard vapor compression cycle at –
10°C (h = 402 kJ/kg) and leaves the compression at 50°C(h = 432 kJ/kg). It leaves the
condenser at 30°C (h = 237 kJ/kg). The COP of the cycle is __________.
Answer: 5.5
Exp: work done=432-402=30 kJ/kg
Refrigerating effect =402-237=165 kJ/kg.
165
COP   5.5
30

52. A cantilever bracket is bolted to a column using three M12 × 1.75 bolts, P, Q and R. The
value of maximum shear stress developed in the bolt P (in MPa) is _______.

9kN

40 P
30
Q
30 10mm thick
R
40

250 All dimension


Answer: 341 are in mm
Exp: P=9kN, e=250 mm
Primary shear force
p q
Pp'  Pq'  Pr'    3kN
3 3 P
Secondary shear force:
Pp''
By symmetry C.G lies at the centre of bolt Q.
 rp  30 mm Pp'
rr  30 mm q
rq  0 C.G
Pe 9000  250
C  2 2  1250 Pq'
r  rq  rr 30  0  302
2
q
2 2

Pp''  Pr''  C R
Pp''  Crp  37.5 kN Pr''

Pr'  Crr  37.5 kN


Pr'
Pq''  0

© All rights reserved by GATE Forum Educational Services Pvt. Ltd. No part of this booklet may be reproduced or utilized
in any form without the written permission. Discuss this questions paper at www.gatementor.com.
22
ME-GATE-2015 PAPER-03| www.gateforum.com

Resultant shear force


Due to symmetry stress in P Q R will be equal in magnitude

Pp  Pr  (3) 2  (37.5) 2  38.5648 kN


P 38.5648
   340.987 MPa 341MPa
A  (0.012) 2
4

53. A mixture of ideal gases has the following composition by mass:

N2 O2 CO2
60% 30% 10%

If the Universal gas constant is 8314 J/mol-K, the characteristic gas constant of the mixture
(inJ/kg.K) is _________.
Answer: 274.99
Universal gas constant
Exp: gas constant of mixture, R m 
Average molar mass

100
Average molar mass   30.233 kg / kmol
60 30 10
 
28 32 44
8314
Rm   274.996J / kg  K
30.233

54. A shaft of length 90 mm has a tapered portion of length 55 mm. The diameter of the taper is
80 mm at one end and 65 mm at the other. If the taper is made by tailstock set over method,
the taper angle and the set over respectively are
(A) 15°32′ and 12.16 mm (B) 15°32′ and 15.66 mm
(C) 11°22′ and 10.26 mm (D) 10°32′ and 14.46 mm
Answer: (A)
80  65
Exp: Rate of taper, T   0.27
55
T  L 0.27  90
Set over=   12.15
2 2
Taper angle  Tan 1 (0.27) =15.10

© All rights reserved by GATE Forum Educational Services Pvt. Ltd. No part of this booklet may be reproduced or utilized
in any form without the written permission. Discuss this questions paper at www.gatementor.com.
23
ME-GATE-2015 PAPER-03| www.gateforum.com

55. One side of a wall is maintained at 400 K and the other at 300 K. The rate of heat transfer
through the wall is 1000 W and the surrounding temperature is 25°C. Assuming no
generation of heat within the wall, the irreversibility (in W) due to heat transfer through the
wall is ________.
Answer: 0.833
Exp: Q = 1000W
ds wall  
 Stransfer  Sgen, wall
dt

Q 
0
T
 Sgen, wall

1000 1000 
   Sgen, wall
400 300

Sgen, wall  0.833W / K

© All rights reserved by GATE Forum Educational Services Pvt. Ltd. No part of this booklet may be reproduced or utilized
in any form without the written permission. Discuss this questions paper at www.gatementor.com.
24
|ME| GATE-2016-PAPER-01 www.gateforum.com

General Aptitude
Q. No. 1 – 5 Carry One Mark Each

1. Which of the following is CORRECT with respect to grammar and usage?


Mount Everest is .
(A) the highest peak in the world
(B) highest peak in the world
(C) one of highest peak in the world
(D) one of the highest peak in the world
Key: (A)

2. The policeman asked the victim of a theft, “What did you ?”


(A) loose (B) lose (C) loss (D) louse
Key: (B)

3. Despite the new medicine‟s____________in treating diabetes, it is not __________ widely.


(A) effectiveness --- prescribed (B) availability --- used
(C) prescription -- available (D) acceptance --- proscribed
Key: (A)

4. In a huge pile of apples and oranges, both ripe and unripe mixed together, 15% are unripe fruits. Of the
unripe fruits, 45% are apples. Of the ripe ones, 66% are oranges. If the pile contains a total of 5692000
fruits, how many of them are apples?
(A) 2029198 (B) 2467482 (C) 2789080 (D) 3577422
Key: (A)
Exp: 5692000  Total fruits 

15% unripe 85% ripe

853800 4838200

45% 55% 34% 66%

apples oranges apples oranges

384210 469590 1644988 3193212


Total number of apples = 384210 + 1644988 = 2029198

 ICP–Intensive Classroom Program  eGATE-Live Internet Based Classes DLP  TarGATE-All India Test Series
Leaders in GATE Preparations  65+ Centers across India
© All rights reserved by Gateforum Educational Services Pvt. Ltd. No part of this booklet may be reproduced or utilized in any form without the written permission.

1
11/11
|ME| GATE-2016-PAPER-01 www.gateforum.com

5. Michael lives 10 km away from where I live. Ahmed lives 5 km away and Susan lives 7 km away from
where I live. Arun is farther away than Ahmed but closer than Susan from where I live. From the
information provided here, what is one possible distance (in km) at which I live from Arun‟s place?
(A) 3.00 (B) 4.99 (C) 6.02 (D) 7.01
Key: (C)

Q. No. 6 – 10 Carry Two Marks Each

6. A person moving through a tuberculosis prone zone has a 50% probability of becoming infected.
However, only 30% of infected people develop the disease. What percentage of people moving
through a tuberculosis prone zone remains infected but does not show symptoms of disease?
(A) 15 (B) 33 (C) 35 (D) 37
Key: (C)
Exp:

0.3 develop disease

0.5 Infected
0.7
not develop
0.5 not Infected
P(a person infected but does not show symptoms)  0.50  0.70  0.35
The percentage is 35%

7. In a world filled with uncertainty, he was glad to have many good friends. He had always assisted them
in times of need and was confident that they would reciprocate. However, the events of the last week
proved him wrong.
Which of the following inference(s) is/are logically valid and can be inferred from the above passage?
(i) His friends were always asking him to help them.
(ii) He felt that when in need of help, his friends would let him down.
(iii) He was sure that his friends would help him when in need.
(iv) His friends did not help him last week.
(A) (i) and (ii) (B) (iii) and (iv) (C) (iii) only (D) (iv) only
Key: (B)

8. Leela is older than her cousin Pavithra. Pavithra‟s brother Shiva is older than Leela. When Pavithra and
Shiva are visiting Leela, all three like to play chess. Pavithra wins more often than Leela does.
Which one of the following statements must be TRUE based on the above?
(A) When Shiva plays chess with Leela and Pavithra, he often loses.
(B) Leela is the oldest of the three.
(C) Shiva is a better chess player than Pavithra.
(D) Pavithra is the youngest of the three.
Key: (D)
 ICP–Intensive Classroom Program  eGATE-Live Internet Based Classes DLP  TarGATE-All India Test Series
Leaders in GATE Preparations  65+ Centers across India
© All rights reserved by Gateforum Educational Services Pvt. Ltd. No part of this booklet may be reproduced or utilized in any form without the written permission.

2
22/12
|ME| GATE-2016-PAPER-01 www.gateforum.com

1 1 1
9. If q  a  and r  b  and s  C  , the value of abc is_____ .
r s q
(A) ( r q s ) −1 (B) 0 (C) 1 (D) r+q+s
Key: (C)
1 1 1
Exp: q a  , r b  , sc 
r s q
q a  r, r b  s, s c  q
r  q a   sc   sac
a

s  r b   s ac   s abc  abc  1
b

10. P, Q, R and S are working on a project. Q can finish the task in 25 days, working alone for 12 hours
a day. R can finish the task in 50 days, working alone for 12 hours per day. Q worked 12 hours a day
but took sick leave in the beginning for two days. R worked 18 hours a day on all days. What is the ratio
of work done by Q and R after 7 days from the start of the project?
(A) 10:11 (B) 11:10 (C) 20:21 (D) 21:20
Key: (C)
1
Exp: Q's one hour work 
25  12
1
R 's one hour work 
50  12
Since Q has taken 2 days sick leave, he has worked only 5 days on the end of seventh day.
1
Work completed by Q on 7th day= (5  12)
25  12
1
Work completed by R on 7th day= (7  18)
50  12
5  12 7  18 20
Ratio of their work    20 : 21
25  12 50  12 21
Mechanical Engineering

Q. No. 1 – 25 Carry One Mark Each

1. The solution to the system of equations


2 5
   x  2 
      is
   y  30
 4 3
(A) 6, 2 (B) −6, 2 (C) −6, −2 (D) 6, −2

Key: (D)
 2 5  6   2 
Exp: By verification method;      
 4 3  2 30

 ICP–Intensive Classroom Program  eGATE-Live Internet Based Classes DLP  TarGATE-All India Test Series
Leaders in GATE Preparations  65+ Centers across India
© All rights reserved by Gateforum Educational Services Pvt. Ltd. No part of this booklet may be reproduced or utilized in any form without the written permission.

3
33/13
|ME| GATE-2016-PAPER-01 www.gateforum.com

2. If f(t) is a function defined for all t ≥ 0, its Laplace transform F(s) is defined as

(A)  est f (t)dt  est f (t)dt

(B)
0 0

 
(C) 0
eist f (t)dt (D)  0
eist f (t)dt
Key: (B)
Exp: Definition of Laplace transform of f  t  v t  0.

3. f(z)=u(x,y)+iv(x,y) is an analytic function of complex variable z=x+iy where i  1. If


u(x,y)=2xy, then v(x,y) may be expressed as
(A) –x2 + y 2 + constant (B) x 2 − y 2 + constant
(C) x 2 + y 2 + constant (D) − (x 2 + y 2 ) + constant
Key: (A)
Exp: Given u  x, y   2xy
By total derivative definition; we have
v dv
dv  dx  dy
x y
u u
 dx  dy  by C  R equations … (1)
y x
u u
 u  x, y   2xy   2y;  2x
x y
From (1) dv  2xdx  2ydy … (2)

 x 2   y2 
 V  2    2    constant  equation 2 is exact D.E 
 2   2
 V   x 2  y 2  constant
4. Consider a Poisson distribution for the tossing of a biased coin. The mean for this distribution is µ. The
standard deviation for this distribution is given by
(A)  (B)  2 (C)  (D) 1/ 
Key: (A)
Exp: Given mean of a poisson distribution for the tossing of a biased coin is .
We know that Mean  Variance  

 Standard deviation  variance   .


5. Solve the equation x = 10 cos (x) using the Newton-Raphson method. The initial guess is

x   / 4 . The value of the predicted root after the first iteration, up to second decimal, is
Key: 1.56

By Newton-Raphson method; the iterative formula for finding approximate root at  n  1 iteration is
th
Exp:

f  xn 
x n 1  x n  ; where x = 0, 1, 2 …….
f  xn 
 ICP–Intensive Classroom Program  eGATE-Live Internet Based Classes DLP  TarGATE-All India Test Series
Leaders in GATE Preparations  65+ Centers across India
© All rights reserved by Gateforum Educational Services Pvt. Ltd. No part of this booklet may be reproduced or utilized in any form without the written permission.

4
44/14
|ME| GATE-2016-PAPER-01 www.gateforum.com

Putting n = 0; then

f  x0 
x1  x 0  … (1)
f  x0 

    10
Let f  x   x  10cos x  f  x 0   f     .
4 4 2

 f   x   1  10sin x

 10
From (1);  f   x 0   f     1 
4 2

  10 

 4 2   1.56
 x1    
4  1  10 
 2 

6. A rigid ball of weight 100 N is suspended with the help of a string. The ball is pulled by a
horizontal force F such that the string makes an angle of 30o with the vertical. The magnitude of force
F (in N) is .

Key: 57.735
Exp: Applying Lami‟s theorem
T 30
T 100 F 120
 
sin90 sin 90  30 sin 180  30 
   F
150
90
F  57.735 N

100
7. A point mass M is released from rest and slides down a spherical bowl (of radius R) from a height H as
shown in the figure below. The surface of the bowl is smooth (no friction). The velocity of the mass at
the bottom of the bowl is

(A) gH (B) 2gR (C) 2gH (D) 0

 ICP–Intensive Classroom Program  eGATE-Live Internet Based Classes DLP  TarGATE-All India Test Series
Leaders in GATE Preparations  65+ Centers across India
© All rights reserved by Gateforum Educational Services Pvt. Ltd. No part of this booklet may be reproduced or utilized in any form without the written permission.

5
55/15
|ME| GATE-2016-PAPER-01 www.gateforum.com

Key: (C)
Exp: Since there is no friction. Therefore there will be no loss of energy of system. Hence energy
remains conserved.
P.E1+K.E1 = P.E2+K.E2
1 1
mgH  m  0   mg  0  mVb2
2

2 2
Vb  2gH

8. The cross sections of two hollow bars made of the same material are concentric circles as shown in the
figure. It is given that r3 > r1 and r4 > r2 , and that the areas of the cross-sections are the same. J1 and J2
are the torsional rigidities of the bars on the left and right, respectively. The ratio J2/J1 is

r3
r1

r2 r4

(A) > 1 (B) < 0.5 (C) =1 (D) between 0.5 and 1
Key: (A)
Exp:
r3
r1

r2 r4

Smaller Ring 1 Bigger Ring  2 

Given A1 = A2
  r22  r12     r42  r32 

r22  r12  r42  r32 ________(1)


We know , Torsional Rigidity = J= Shear Modulus× Polar moment of Inertia
 4 4
J2 2
G
r4  r3 J2 
r42  r32 
r32  r42 r32  r42      
    2 2  2 2  2 2
J1 
G  r24  r14 J1

r2  r1 r1  r2
 r1  r2      
2
But, r4  r2 and r3  r1
J2
 1
J1

 ICP–Intensive Classroom Program  eGATE-Live Internet Based Classes DLP  TarGATE-All India Test Series
Leaders in GATE Preparations  65+ Centers across India
© All rights reserved by Gateforum Educational Services Pvt. Ltd. No part of this booklet may be reproduced or utilized in any form without the written permission.

6
66/16
|ME| GATE-2016-PAPER-01 www.gateforum.com

9. A cantilever beam having square cross-section of side a is subjected to an end load. If a is increased by
19%, the tip deflection decreases approximately by
(A) 19% (B) 29% (C) 41% (D) 50%
Key: (D)
p3 1 1
Exp:      4
3EI I a
1 a 24
 where, a 2  1.19a,
2 a14
1 1
 1.19   2 
4
  0.51
2 1.19 4
So, deflection decrease by 50%

10. A car is moving on a curved horizontal road of radius 100 m with a speed of 20 m/s. The rotating masses
of the engine have an angular speed of 100 rad/s in clockwise direction when viewed from the front of
the car. The combined moment of inertia of the rotating masses is 10 kg-m2. The magnitude of the
gyroscopic moment (in N-m) is .
Key: 200
Exp: Given: Spin velocity ( s )= 100 rad/sec
Moment of Inertia (MOI) = 10 kg-m2
V  20 m sec

Precision Angular Velocity  p  


linear speed
R R  100 m

 p
20
  0.2 rad sec
100
Gyroscopic moment = MOI s × p
 10  100  0.2
 200 Nm
11. A single degree of freedom spring mass system with viscous damping has a spring constant of 10
kN/m. The system is excited by a sinusoidal force of amplitude 100 N. If the damping factor (ratio) is
0.25, the amplitude of steady state oscillation at resonance is mm.
Key: 20
Exp: Given: Spring constant (k) = 10 kN/m = 10,000 N/m
Magnitude of force (F0) = 100N
Damping factor    0.25
Forcing frequency (ω) = Natural frequency (ωn)
F 100
Static deflection of spring  0  4  102 m
k 10
 10 mm

 ICP–Intensive Classroom Program  eGATE-Live Internet Based Classes DLP  TarGATE-All India Test Series
Leaders in GATE Preparations  65+ Centers across India
© All rights reserved by Gateforum Educational Services Pvt. Ltd. No part of this booklet may be reproduced or utilized in any form without the written permission.

7
77/17
|ME| GATE-2016-PAPER-01 www.gateforum.com

Staticdeflection
Dynamic deflection 
   2   
2

1       2 
  n    n 
10

1  1  2 2
  2  0.25  1
2

10
  20 mm
2  0.25
12. The spring constant of a helical compression spring DOES NOT depend on
(A) coil diameter
(B) material strength
(C) number of active turns
(D) wire diameter
Key: (B)
Exp: For Helical compression spring.
64 WR 3n
Deflection,  
Gd4
W W Gd 4
Stiffness, or spring constant   
 64 WR 3 n 64R 3 n
Gd 4
From the above formula we can say that spring constant depends on coil diameter (D), wire diameter (d),
No. of active turns (n) and modulus of rigidity (G) and is independent of material strength.

13. The instantaneous stream-wise velocity of a turbulent flow is given as follows:


u(x, y, z, t) = u (x, y, z)  u  ( x, y, z, t)
The time-average of the fluctuating velocity u  ( x, y, z, t) is
(A) u  / 2 (B) u / 2 (C) zero (D) u / 2
Key: (C) u'
Exp:
u

time  t 
Given u  u  u ' _____(1)
T
1
T 0
where u  u dt T = period after which pattern will repeat

 ICP–Intensive Classroom Program  eGATE-Live Internet Based Classes DLP  TarGATE-All India Test Series
Leaders in GATE Preparations  65+ Centers across India
© All rights reserved by Gateforum Educational Services Pvt. Ltd. No part of this booklet may be reproduced or utilized in any form without the written permission.

8
88/18
|ME| GATE-2016-PAPER-01 www.gateforum.com

Rewriting equation (1)


u '  u u
taking average of fluctuating component
1T 
 
T T T T
1 1 1 1
T 0 T 0 T 0 T 0
u'  u 'dt  u  u dt  u dt  u dt  u  u   dt 
T 0 
 uu
u'  0

14. For a floating body, buoyant force acts at the


(A) centroid of the floating body
(B) center of gravity of the body
(C) centroid of the fluid vertically below the body
(D) centroid of the displaced fluid
Key: (D)
15. A plastic sleeve of outer radius r0 = 1 mm covers a wire (radius r = 0.5 mm) carrying electric
current. Thermal conductivity of the plastic is 0.15 W/m-K. The heat transfer coefficient on the outer
surface of the sleeve exposed to air is 25 W/m2-K. Due to the addition of the plastic cover, the heat
transfer from the wire to the ambient will
(A) increase
(B) remain the same
(C) decrease
(D) be zero
Key: (A)
Exp: Given
r0  1mm
r  0.5mm
K  0.15W/mK
h  25W/m2 K
0.15
Critical radius  k/h   6mm
25
So radius of wire is less than critical radius, addition of plastic sleeve will increase the heat transfer.

16. Which of the following statements are TRUE with respect to heat and work?
(i) They are boundary phenomena
(ii) They are exact differentials
(iii) They are path functions
(A) both (i) and (ii) (B) both (i) and (iii) (C) both (ii) and (iii) (D) only (iii)
Key: (B)

 ICP–Intensive Classroom Program  eGATE-Live Internet Based Classes DLP  TarGATE-All India Test Series
Leaders in GATE Preparations  65+ Centers across India
© All rights reserved by Gateforum Educational Services Pvt. Ltd. No part of this booklet may be reproduced or utilized in any form without the written permission.

9
99/19
|ME| GATE-2016-PAPER-01 www.gateforum.com

17. Propane (C3H8) is burned in an oxygen atmosphere with 10% deficit oxygen with respect to the
stoichiometric requirement. Assuming no hydrocarbons in the products, the volume percentage of CO
in the products is
Key: 14.286
Exp: C3H8  5(0.9)O2  5(3.76)0.9N2  aCO  bCO2  4H2O  16.92N2
Carbon balance : 3  a  b
Qxygen balance : 9  a  2b  4
6  b  4
a  1, b  2
1
100  14.286%
7
18. Consider two hydraulic turbines having identical specific speed and effective head at the inlet. If the
speed ratio (N1/N2) of the two turbines is 2, then the respective power ratio (P1/P2) is
____.
Key: 0.25
Exp: Given: Specific speed of Turbine “1” (NS1) = Specific Speed of Turbine “2” (NS2)
Effective Head at inlet of Turbine “1” (H1) = Effective head at inlet of turbine “2” (H2)
and N1/N2 = 2
N P
Specific Speed of Turbine  NS  
H5 4
NS1 = NS2
N1 P1 N 2 P2

H15 4 H52 4
2
P1  N 2   1  1
2

       0.25
P2  N1   2  4
19. The INCORRECT statement about regeneration in vapor power cycle is that
(A) it increases the irreversibility by adding the liquid with higher energy content to the steam
generator
(B) heat is exchanged between the expanding fluid in the turbine and the compressed fluid before heat
addition
(C) the principle is similar to the principle of Stirling gas cycle
(D) it is practically implemented by providing feed water heaters
Key: (A)
20. The “Jominy test” is used to find
(A) Young‟s modulus (B) hardenability
(C) yield strength (D) thermal conductivity
Key: (B)

 ICP–Intensive Classroom Program  eGATE-Live Internet Based Classes DLP  TarGATE-All India Test Series
Leaders in GATE Preparations  65+ Centers across India
© All rights reserved by Gateforum Educational Services Pvt. Ltd. No part of this booklet may be reproduced or utilized in any form without the written permission.

10
1010/
|ME| GATE-2016-PAPER-01 www.gateforum.com

21. Under optimal conditions of the process the temperatures experienced by a copper work piece in fusion
welding, brazing and soldering are such that
(A) Twelding > Tsoldering> Tbrazing (B) Tsoldering > Twelding > Tbrazing
(C) Tbrazing >Twelding > Tsoldering (D) Twelding > Tbrazing > Tsoldering
Key: (D)

22. The part of a gating system which regulates the rate of pouring of molten metal is
(A) pouring basin (B) runner (C) choke (D) ingate
Key: (C)

23. The non-traditional machining process that essentially requires vacuum is


(A) electron beam machining (B) electro chemical machining
(C) electro chemical discharge machining (D) electro discharge machining
Key: (A)
Exp: Electron beam machining requires vacuum, to avoid deflection of electrons

24. In an orthogonal cutting process the tool used has rake angle of zero degree. The measured cutting force
and thrust force are 500 N and 250 N, respectively. The coefficient of friction between the tool and
the chip is
Key: 0.5
Ft
Exp: Tan      
FC
250
Tan    0  
500
Tan  1  0.5
2

25. Match the following:


P. Feeler gauge I. Radius of an object
Q. Fillet gauge II. Diameter within limits by comparison
R. Snap gauge III. Clearance or gap between components
S. Cylindrical plug gauge IV. Inside diameter of straight hole

(A) P–III, Q–I, R–II, S–IV (B) P–III, Q–II, R–I, S–IV
(C) P–IV, Q–II, R–I, S–III (D) P–IV, Q–I, R–II, S–III
Key: (A)
Exp: Feeler gauge: Clearance or gap between components
Fillet gauge: Radius of an object
Snap gauge: Diameter within limits by comparison
Cylindrical plug gauge: Inside diameter of straight hole.
 ICP–Intensive Classroom Program  eGATE-Live Internet Based Classes DLP  TarGATE-All India Test Series
Leaders in GATE Preparations  65+ Centers across India
© All rights reserved by Gateforum Educational Services Pvt. Ltd. No part of this booklet may be reproduced or utilized in any form without the written permission.

11
1111/
|ME| GATE-2016-PAPER-01 www.gateforum.com

Q. No. 26 – 55 carry Two Marks Each

26. Consider the function f (x)  2x 3  3x 2 i n the domain [−1, 2]. The global minimum of f(x) is
_______
Key: -5
Exp: Given that, f  x   2x3  3x 2
 f   x   0  6x 2  6x  0
 x 2  x  0  x  x  1  0
 x  0; x  1
are Stationary points.
 f   x   12x  6
f   0  6  0
 f(x) has maximum at x = 0.
f  1  12 1  6  6  0
 f(x) has minimum at x = 1.
 f 1  2  3  1  local minimum value

But f  1  2  3  5
 Global minimum of f(x) = -5

27. If y=f(x)satisfies the boundary value problem y  9y  0, y(0)  0, y( / 2)  2, , then y( / 4) is
_________
Key: -1
Exp: Given D.E is y  9y  0

  D2  9 y  0

The A.E is D2  9  0
 D2  9  D  3i
 y  c1 cos3x  c2 sin 3x … (1)
Given
y   / 2  2
y 0  0 and
i.e; x  0, y  0 i.e; x   / 2; y  2
From(1); 0  C1 From(1); 2  0  C2 (1)
 C2   2
 From (1); y   2 sin3x

   3   1 
 y     2 sin     2    1
4  4  2

 y    1
4
 ICP–Intensive Classroom Program  eGATE-Live Internet Based Classes DLP  TarGATE-All India Test Series
Leaders in GATE Preparations  65+ Centers across India
© All rights reserved by Gateforum Educational Services Pvt. Ltd. No part of this booklet may be reproduced or utilized in any form without the written permission.

12
1212/
|ME| GATE-2016-PAPER-01 www.gateforum.com

28. The value of the integral


 sin x
  x 2  2x  2
dx

evaluated using contour integration and the residue theorem is


(A) –  sin 1 / e (B)  cos 1 / e (C) sin(1)/e (D) cos(1)/e
Key: (A)
Exp: We know that sin x is the imaginary part of eix
eiz
∴ We consider the function f  z  
z  2z  2
2

Now, the poles of f(z) are given by z2  2z  2  0


2  4  4  2  2  4
z 
2 2
2  i2
  1 i
2
But Z  1  i is the only pole (simple) lie in the upper half of the Z-plane.
eiz
∴ Resf  z   Lt z   1  i  .

z 1i z 1i   z   1 i   z   1  i 
eiz ei 1i  ei 1 ei
 Lt   
z1i z  1  i  1  i  1  i 2i 2ie

eiz  ei  ei


Thus e z2  2z  2 dz  2 i  
 2ie  e
Equating imaginary parts on both sides we get
sin z    sin 1   sin 1
z
e
2
 2z  2
dz 
e

e

29. Gauss-Seidel method is used to solve the following equations (as per the given order):
x1  2x 2  3x 3  5
2x1  3x 2  x 3  1
3x1  2x 2  x 3  3
Assuming initial guess as x1  x 2  x 3  0, the value of x3 after the first iteration is ____
Key: -6
Exp: x11  0  0  5 ...1

2x11  3x 21  0  1 ...(2)


3x11  2x 21  x 31  3 ...  3

∴ From equation (1) x1   5


1

 ICP–Intensive Classroom Program  eGATE-Live Internet Based Classes DLP  TarGATE-All India Test Series
Leaders in GATE Preparations  65+ Centers across India
© All rights reserved by Gateforum Educational Services Pvt. Ltd. No part of this booklet may be reproduced or utilized in any form without the written permission.

13
1313/
|ME| GATE-2016-PAPER-01 www.gateforum.com

From equation (2), 2x11  3x 21  1


 3x 21 1  2x11
 1  2  5
 3x 21  9
9
 x 21   3  x 21  3
3
From equation (3), x 31  3  3x11  2x 21
 3  3  5   2  3
 3  15  6  6
 x 31  6
∴ After the first iteration, the value of x3 is -6.
30. A block of mass m rests on an inclined plane and is attached by a string to the wall as shown in the
figure. The coefficient of static friction between the plane and the block is 0.25. The string can withstand
a maximum force of 20 N. The maximum value of the mass (m) for which the string will not break and
the block will be in static equilibrium is kg.
Take cos   0.8and sin   0.6.

Acceleration due to gravity g = 10 m/s2

Key: 5 R
Exp: F  R   mg cos 
T
 0.25  m  10  0.8 m F
 2m
mgsin 
For equilibrium, 

T  F  mgsin 
 20  2m  m  10  0.6  m  5kg
mg mgsin 

 ICP–Intensive Classroom Program  eGATE-Live Internet Based Classes DLP  TarGATE-All India Test Series
Leaders in GATE Preparations  65+ Centers across India
© All rights reserved by Gateforum Educational Services Pvt. Ltd. No part of this booklet may be reproduced or utilized in any form without the written permission.

14
1414/
|ME| GATE-2016-PAPER-01 www.gateforum.com

31. A two-member truss PQR is supporting a load W. The axial forces in members PQ and QR are
respectively

(A) 2W tensile and 3W compressive

(B) 3W tensile and 2W compressive

(C) 3W compressive and 2W tensile


(D) 2Wcompressive and 3W tensile
Key: (B)
Exp: F.B.D of point Q
Fx  0
FQR sin 60
 FPQ  FQR sin 60  0 ...... 1 Q
FPQ
Fy  0 30
 FQR cos60  W  0 ....  2  60
FQR cos 60
W
 FQR   FQR  2W  compressive  FQR
cos60
From equation (1) W
FPQ  2Wsin 60  0

3
 FPQ  2W   3W  Tensile 
2

32. A horizontal bar with a constant cross-section is subjected to loading as shown in the figure. The
Young‟s moduli for the sections AB and BC are 3E and E, respectively.

For the deflection at C to be zero, the ratio P/F is


Key: 4 A B B C
Exp:

FP 3E FP F E F
F.B.D

 
Since, net deflection at C is zero
 ICP–Intensive Classroom Program  eGATE-Live Internet Based Classes DLP  TarGATE-All India Test Series
Leaders in GATE Preparations  65+ Centers across India
© All rights reserved by Gateforum Educational Services Pvt. Ltd. No part of this booklet may be reproduced or utilized in any form without the written permission.

15
1515/
|ME| GATE-2016-PAPER-01 www.gateforum.com

 AB  BC  0


 F  P    F
0
A.3E AE
FP
 F  0
3
 4F  P  0
P
 4
F
33. The figure shows cross-section of a beam subjected to bending. The area moment of inertia

(in mm4) of this cross-section about its base is _________.

Key: 1873 - 1879


34. A simply-supported beam of length 3L is subjected to the loading shown in the figure.

It is given that P = 1 N, L = 1 m and Young‟s modulus E = 200 GPa. The cross-section is a square with
dimension 10 mm × 10 mm. The bending stress (in Pa) at the point A located at the top surface of the
beam at a distance of 1.5L from the left end is
(Indicate compressive stress by a negative sign and tensile stress by a positive sign.)
Key: 0 P P
Exp: A L L L B
A
RA RB

Taking moment about B


MB  0  R A  3L  P  2L  PL  0  R A   P 3

 ICP–Intensive Classroom Program  eGATE-Live Internet Based Classes DLP  TarGATE-All India Test Series
Leaders in GATE Preparations  65+ Centers across India
© All rights reserved by Gateforum Educational Services Pvt. Ltd. No part of this booklet may be reproduced or utilized in any form without the written permission.

16
1616/
|ME| GATE-2016-PAPER-01 www.gateforum.com

Fy  0  R B  R A  0  R B  P 3

Taking moment about A


M A  0

R A  1.5L  0.5PL  M A  Assuming MA anticlockwise 


P
   1.5L  0.5PL  M A
3
 MA  0

M b
we know,   b  Bendingstress   0 since, M A  0
I y
35. A slider crank mechanism with crank radius 200 mm and connecting rod length 800 mm is shown. The
crank is rotating at 600 rpm in the counterclockwise direction. In the configuration shown, the crank
makes an angle of 90o with the sliding direction of the slider, and a force of 5 kN is acting on the slider.
Neglecting the inertia forces, the turning moment on the crank (in kN-m) is _____

Key: 1
Exp: T cos   5
Moment about crank shaft  M   T  r sin  90   
  T cos   r T
 5  0.2 T

 1kN m
5
Alternate method:
H
  
M  5  0.2 sin  90   sin 180 
 2 
 1kN m
36. In the gear train shown, gear 3 is carried on arm 5. Gear 3 meshes with gear 2 and gear 4. The number of
teeth on gear 2, 3, and 4 are 60, 20, and 100, respectively. If gear 2 is fixed and gear 4 rotates with an
angular velocity of 100 rpm in the counterclockwise direction, the angular speed of arm 5 (in rpm) is

(A) 166.7 counterclockwise (B) 166.7 clockwise


(C) 62.5 counterclockwise (D) 62.5 clockwise
Key: (C)

 ICP–Intensive Classroom Program  eGATE-Live Internet Based Classes DLP  TarGATE-All India Test Series
Leaders in GATE Preparations  65+ Centers across India
© All rights reserved by Gateforum Educational Services Pvt. Ltd. No part of this booklet may be reproduced or utilized in any form without the written permission.

17
1717/
|ME| GATE-2016-PAPER-01 www.gateforum.com

Exp:
Arm Gears
2 3 4
All gear locked to
60 60
Arm & 2 is given x 0 x x x
20 100
Rotations in C.C.W
Arm also given y yx 60 60
y y x y x
Rotations in C.C.W 20 100
Given: x  y 0 ...(1)
3
y  x 100 ...(2)
5
3 5  100
y  y  100  y   62.5 counter clockwise
5 8
37. A solid disc with radius a is connected to a spring at a point d above the center of the disc. The other
end of the spring is fixed to the vertical wall. The disc is free to roll without slipping on the ground. The
mass of the disc is M and the spring constant is K. The polar moment of inertia for the disc about its
centre is J  Ma 2 / 2.

The natural frequency of this system in rad/s is given by

2K(a  d)2 2K 2K(a  d)2 K(a  d)2


(A) (B) (C) (D)
3Ma 2 3M Ma 2 Ma 2
Key: (A)
Exp: Apply D'Alembert Principle:
 k  a  d    a  d    I cm  Ma 2  
 0 cm

k a  d
2

 0
 Ma 2 
  Ma 2  x
 2 
2k  a  d 
2

n 
3Ma 2 k a  d 
cm

 ICP–Intensive Classroom Program  eGATE-Live Internet Based Classes DLP  TarGATE-All India Test Series
Leaders in GATE Preparations  65+ Centers across India
© All rights reserved by Gateforum Educational Services Pvt. Ltd. No part of this booklet may be reproduced or utilized in any form without the written permission.

18
1818/
|ME| GATE-2016-PAPER-01 www.gateforum.com

38. The principal stresses at a point inside a solid object are 1  100 MPa, 2  100 MPa and 3  0
MPa. The yield strength of the material is 200 MPa. The factor of safety calculated using Tresca
(maximum shear stress) theory is nT and the factor of safety calculated using von Mises (maximum
distortional energy) theory is nV. Which one of the following relations is TRUE?

(A) n T   3 / 2 n v

(B) n T   3n v

(C) n T  n v

(D) n v   3n T

Key: (C)

  1  3   2  3   3  1  
Exp: max  max  ,  ,     50 Mpa
 2   2   2  

max 
S yt / 2
 50 
 200 / 2    2
T T
T

  1  2 2   2  3 2   3  1 2   Syt 2
   
 2   v 

 12  22  1 2   Syt / v 


2

1  2  100 & Syt  200


2
 200 
100      v  2
2

 v 
 T  v

39. An inverted U-tube manometer is used to measure the pressure difference between two pipes A and B, as
shown in the figure. Pipe A is carrying oil (specific gravity = 0.8) and pipe B is carrying water. The
densities of air and water are 1.16 kg/m3 and 1000 kg/m3, respectively. The pressure difference between
pipes A and B is kPa.

Acceleration due to gravity g = 10 m/s2.

 ICP–Intensive Classroom Program  eGATE-Live Internet Based Classes DLP  TarGATE-All India Test Series
Leaders in GATE Preparations  65+ Centers across India
© All rights reserved by Gateforum Educational Services Pvt. Ltd. No part of this booklet may be reproduced or utilized in any form without the written permission.

19
1919/
|ME| GATE-2016-PAPER-01 www.gateforum.com

Key: -2.199

Exp: PA  oil gh1  air gh 2  PB water g  h1  h 2  h 3 


PA  PB  0.8 103  10  0.2 1.16  10  0.08 103 10  0.2  0.08  0.1
 1600  0.928  3800
  2199.072 Pa
PA  PB   2.199 kPa

40. Oil (kinematic viscosity, Voil  1.0 × 10−5 m2/s) flows through a pipe of 0.5 m diameter with a
velocity of 10 m/s. Water (kinematic viscosity, vw= 0.89 × 10−6 m2/s) is flowing through a model pipe
of diameter 20 mm. For satisfying the dynamic similarity, the velocity of water (in m/s) is _____ .

Key: 22.25
Prototype
Exp:
oil  1.0 105 m2 /s
V0 =10m/s oil doil  0.5m

model
 w  0.89 106 m2 /s
Vw = ? water
d w  0.02m

There is no free surface inside pipe flow, therefore only Reynold numbers are equal in both the cases.

 R e oil   R e water  
VD   VD 
  
  mod el    prototype
Vw  0.02 10  0.5
  Vw  22.25m/sec
0.89 106 1.0 105

41. A steady laminar boundary layer is formed over a flat plate as shown in the figure. The free stream
velocity of the fluid is Uo. The velocity profile at the inlet a-b is uniform, while that at a
  y   y 2 
downstream location c-d is given by u  U0  2       .
       

 bd , leaving through the horizontal section b-d to that entering


The ratio of the mass flow rate, m
through the vertical section a-b is .
Key: 0.33
 ICP–Intensive Classroom Program  eGATE-Live Internet Based Classes DLP  TarGATE-All India Test Series
Leaders in GATE Preparations  65+ Centers across India
© All rights reserved by Gateforum Educational Services Pvt. Ltd. No part of this booklet may be reproduced or utilized in any form without the written permission.

20
2020/
|ME| GATE-2016-PAPER-01 www.gateforum.com

Exp:
This is steady state process, so time derivative will be zero. Choose b-a-c-d-b as Control Volume (CV)
   
  V.n  dA  0

cs 

where C.S  stands for control surface &



n is unit vector perpendicular to Area of flow which is always taken outside of side of cross-
sectional Area.
Let fluid is incompressible, ρ= constant

               
a  d b
  V.n  dA    V.n  dA    V.n  dA    V.n  dA  0
b  a  c  d 

     y 2  y 2 
  
  U0 b.dy    0.n  dA   U 0 2       bdy  m
 bd  0
0 a  c        

  
 U 0 b  0  V b     m bd  0
 3
2 
 U 0 b  bV0   m bd  0
3
 1
m bd   U0 b  
3

m bd
 0.33
 U 0 b
42. A steel ball of 10 mm diameter at 1000 K is required to be cooled to 350 K by immersing it in a water
environment at 300 K. The convective heat transfer coefficient is 1000 W/m2-K. Thermal
conductivity of steel is 40 W/m-K. The time constant for the cooling process τ is 16 s. The time
required (in s) to reach the final temperature is _____
Key: 42.22
Exp: Given
d = 10 mm = 0.01 m
W W
t i  1000K;t  350K;t   300K; k  40 ;h  1000 2 ; th  16s
mK mK
t  t 
 e th
ti  t
 t  t   
ln    th
 ti  t 
 350  300  
ln    16
 1000  300 
  42.22s

 ICP–Intensive Classroom Program  eGATE-Live Internet Based Classes DLP  TarGATE-All India Test Series
Leaders in GATE Preparations  65+ Centers across India
© All rights reserved by Gateforum Educational Services Pvt. Ltd. No part of this booklet may be reproduced or utilized in any form without the written permission.

21
2121/
|ME| GATE-2016-PAPER-01 www.gateforum.com

43. An infinitely long furnace of 0.5 m × 0.4 m cross-section is shown in the figure below. Consider all
surfaces of the furnace to be black. The top and bottom walls are maintained at temperature T1 = T3
= 927oC while the side walls are at temperature T2 = T4 = 527 oC. The view factor, F1-2 is 0.26. The
net radiation heat loss or gain on side 1 is W/m.

Stefan-Boltzmann constant = 5.67 × 10−8 W/m2-K4

Key: 24530.688
44. A fluid (Prandtl number, Pr = 1) at 500 K flows over a flat plate of 1.5 m length, maintained at
300 K. The velocity of the fluid is 10 m/s. Assuming kinematic viscosity, ν = 30 × 10−6 m2/s, the
thermal boundary layer thickness (in mm) at 0.5 m from the leading edge is .
Key: 6.124
Exp: Pr = 1
ux 10  0.5
Re x  
 30  106
 166666.67

 1.67  105
Hydrodynamic boundary layer thickness
5x 5  0.5
h x  
Re x 1.67  105

 6.124  103 m
If Pr = 1
h x  Tx  6.124  103 m

 6.124 mm
 Thermal boundary layer thickness = 6.124 mm.
45. For water at 25oC, dps / dTs  0.189kPa / K (ps is the saturation pressure in kPa and Ts is the saturation
temperature in K) and the specific volume of dry saturated vapour is 43.38 m3/kg. Assume that the
specific volume of liquid is negligible in comparison with that of vapour. Using the Clausius- Clapeyron
equation, an estimate of the enthalpy of evaporation of water at 25oC (in kJ/kg) is ______.
Key: 2443.24

 ICP–Intensive Classroom Program  eGATE-Live Internet Based Classes DLP  TarGATE-All India Test Series
Leaders in GATE Preparations  65+ Centers across India
© All rights reserved by Gateforum Educational Services Pvt. Ltd. No part of this booklet may be reproduced or utilized in any form without the written permission.

22
2222/
|ME| GATE-2016-PAPER-01 www.gateforum.com

dPs h fg h fg
Exp:   0.189 
dTs Ts  g   f   25  273 43.38  0 
 h fg  2443.248kJ kg

46. An ideal gas undergoes a reversible process in which the pressure varies linearly with volume. The
conditions at the start (subscript 1) and at the end (subscript 2) of the process with usual notation are:
p1  100kPa, V1  0.2m3 and p 2  200 kPa, V2  0.1m3 and the gas constant, R = 0.275 kJ/kg-K. The
magnitude of the work required for the process (in kJ) is .
Key: 15
Exp: Pressure varies linearly with volume.
P = a + bv
P1  a  bv1

 100  a  b  0.2      (1)

P2  a  bv2

 200  a  b  0.1      (2)

Solving (1) & (2)

100
100  0.1b  b   1000
0.1
b  1000

Substituting in any of the equations to get „a‟.


 100  a   1000  0.2   a  300
2 2
W   pdv    a  bv  dv
1 1
2
 v 2  v12 
   300  1000v  dv  300  v2  v1   1000  2 
1  2 
  0.12   0.2 2 
 300  0.1  0.2   1000    30   15  15kJ
 2 
 
 Magnitude of work required is 15 kJ.
47. In a steam power plant operating on an ideal Rankine cycle, superheated steam enters the turbine at 3
MPa and 350oC. The condenser pressure is 75 kPa. The thermal efficiency of the cycle is
________ percent.
Given data:
For saturated liquid, at P = 75 kPa, h f  384.39kJ / kg, vf  0.001037m3 / kg, sf  1.213kJ / kg  K
At 75 kPa, hfg = 2278.6 kJ/kg, sfg = 6.2434 kJ/kg-K

At P = 3 MPa and T = 350oC (superheated steam), H  3115.3kJ / kg, s  6.7428kJ / kg  K


 ICP–Intensive Classroom Program  eGATE-Live Internet Based Classes DLP  TarGATE-All India Test Series
Leaders in GATE Preparations  65+ Centers across India
© All rights reserved by Gateforum Educational Services Pvt. Ltd. No part of this booklet may be reproduced or utilized in any form without the written permission.

23
2323/
|ME| GATE-2016-PAPER-01 www.gateforum.com

Key: 25.99%
Exp: 1

3 2

Given:- P1  P4  3MPa, T1  350o C  350  273  623K


h1  3115.3kJ / kg, S1  6.7428kJ / kgK
P2  P3  75kPa.
h f2  h f3  384.39kJ / kg, h fg2  2278.6kJ / kg
sf2  sf3  1.213kJ / kgK, sfg2  6.2434kJ / kgK
vf3  0.001037m3 / kg
S1  S2
 S1  Sf2  x 2Sfg2
6.7428  1.213  x 2  6.2434
x 2  0.886
h 2  h f2  x 2 h fg2  384.39  0.886  2278.6
 2403.2296kJ / kg
 2403.23
Turbine work, WT  h1  h 2  3115.3  2403.23  712.07kJ / kg
Pump work, Wp  vf3  P4  P3   0.001037(3000  75)
 3.03k J / kg
h 4  h 3  Wp  h f3  Wp  384.39  3.03
 387.42kJ / kg
Heat supplied to boiler
 h1  h 4  3115.3  387.42
 2727.88kJ / kg
Net work done  WT  Wp  712.07  3.03  709.04kJ / kg
Net work done 709.04
Thermal efficiency      100  25.99%
Heat supplied 2727.88

 ICP–Intensive Classroom Program  eGATE-Live Internet Based Classes DLP  TarGATE-All India Test Series
Leaders in GATE Preparations  65+ Centers across India
© All rights reserved by Gateforum Educational Services Pvt. Ltd. No part of this booklet may be reproduced or utilized in any form without the written permission.

24
2424/
|ME| GATE-2016-PAPER-01 www.gateforum.com

48. A hypothetical engineering stress-strain curve shown in the figure has three straight lines PQ, QR, RS
with coordinates P(0,0), Q(0.2,100), R(0.6,140) and S(0.8,130). 'Q' is the yield point, 'R' is the UTS
point and 'S' the fracture point.

The toughness of the material (in MJ/m3) is ________ .


Key: 0.85
Exp:

R  0.6,140 
160
140 4
3
2
S  0.8,130 
120
100
Q  0.2,100 
Stress in

80 1
MPa

60
40
20
0
P  0,0 0.2 0.4 0.6 0.8 1

Strainin %

Toughness of material
Total area – [Area of 1 + Area of 2 + Area of 3 + Area of 4]

 140 

0.8   1
    100 
100   2  0.2
100
 40 
0.2
100
1

  40 
2
0.4
100
1
  10 
2
0.2 
100   
 1.12   0.1  0.08  0.08  0.01  0.85MJ m 3

 ICP–Intensive Classroom Program  eGATE-Live Internet Based Classes DLP  TarGATE-All India Test Series
Leaders in GATE Preparations  65+ Centers across India
© All rights reserved by Gateforum Educational Services Pvt. Ltd. No part of this booklet may be reproduced or utilized in any form without the written permission.

25
2525/
|ME| GATE-2016-PAPER-01 www.gateforum.com

49. Heat is removed from a molten metal of mass 2 kg at a constant rate of 10 kW till it is completely
solidified. The cooling curve is shown in the figure.

Assuming uniform temperature throughout the volume of the metal during solidification, the latent heat
of fusion of the metal (in kJ/kg) is .
Key: (50)
Exp: Given 20sec
m  2 kg; Q  10 kcal 10s
873k
Since heat is removed at constant rate 873k
10kw is removed per second.
So, latest heat = mL  10 kW  10sec  10 sec is time requiredfor phasechange
2 L  100 kJ
L  50kJ / kg

The tool life equation for HSS tool is VT f d  Constant. The tool life (T) of 30 min is obtained
0.14 0.7 0.4
50.
using the following cutting conditions:
V = 45 m/min, f = 0.35 mm, d = 2.0 mm
If speed (V), feed (f) and depth of cut (d) are increased individually by 25%, the tool life (in min) is
(A) 0.15 (B) 1.06 (C) 22.50 (D) 30.0
Key: (B)
Exp: VT 0.14 f 0.7 d 0.4  constant
V1  45m/min; f1  0.35mm; d1  2mm
T1  30min; V2  1.25V1;
f 2  1.25f1 ; d 1 1.25d1 ;
T2  ?

 45  T10.14 f10.7 d10.4  1.25  45  T20.14 1.250.7 f10.7  1.250.4 d10.4


 T10.14 1.25 1.250.7 1.250.4  T20.14
 30 
0.14

 T2   1.055 1.06
1.252.1 / 0.14

 ICP–Intensive Classroom Program  eGATE-Live Internet Based Classes DLP  TarGATE-All India Test Series
Leaders in GATE Preparations  65+ Centers across India
© All rights reserved by Gateforum Educational Services Pvt. Ltd. No part of this booklet may be reproduced or utilized in any form without the written permission.

26
2626/
|ME| GATE-2016-PAPER-01 www.gateforum.com

51. A cylindrical job with diameter of 200 mm and height of 100 mm is to be cast using modulus
method of riser design. Assume that the bottom surface of cylindrical riser does not contribute as cooling
surface. If the diameter of the riser is equal to its height, then the height of the riser (in mm) is
(A) 150 (B) 200 (C) 100 (D) 125
Key: (A)
Exp: dc  200 mm dr  h r C  Casting
h c  100 mm hr  ? R  Rises

M r  1.2 MC

V V
   1.2  
 S r  S C
 2  2
d r hr dc h c
4  1.2 4
 
d r h r  d r2 d c h c  d c2 x 2
4 4
dr 2 h r 1.2d c2 h c
 
4d r h r  d r2 4d c h c  2d c2
1.2   200   100
2
h 3r
   d v  h r 
4h 2r  h 2r 4  200  100  2  2002
hr 200  10
  1.2 
5 4  100  2  200
200  100
 h r  1.2  5   125  1.2  150
4  200
52. A 300 mm thick slab is being cold rolled using roll of 600 mm diameter. If the coefficient of
friction is 0.08, the maximum possible reduction (in mm) is .
Key: 1.92
Exp:  h max   2 R

  0.08  300
2

 1.92 mm

53. The figure below represents a triangle PQR with initial coordinates of the vertices as P(1,3),
Q(4,5) and R(5,3.5). The triangle is rotated in the X-Y plane about the vertex P by angle θ in clockwise
direction. If sin θ = 0.6 and cos θ = 0.8, the new coordinates of the vertex Q are

(A) (4.6, 2.8) (B) (3.2, 4.6) (C) (7.9, 5.5) (D) (5.5, 7.9)
 ICP–Intensive Classroom Program  eGATE-Live Internet Based Classes DLP  TarGATE-All India Test Series
Leaders in GATE Preparations  65+ Centers across India
© All rights reserved by Gateforum Educational Services Pvt. Ltd. No part of this booklet may be reproduced or utilized in any form without the written permission.

27
2727/
|ME| GATE-2016-PAPER-01 www.gateforum.com

Key: (A)
Exp: y

Q  4,5

P 1,3

x
0

Rotate PQ by  in clockwise direction


y Q  xo , y0 

Q'  x, y 

P 1,3, 

x
0
Rotation of point Q about point P in anticlockwise is given as

x n  x p   x o  x p  cos    yo  yp  sin  ...(1)

yn  yp   x o  x p  sin    yo  yp  cos  ...(2)

For clockwise rotation  will be   . So the desired equation will be

x n  x p   x o  x p  cos    yo  yp  sin  ...(3)

1   4  1  0.8   5  3  0.6
 1 2.4 1.2  4.6

yn  yp   yo  yp  sin    yo  yp  cos  ...(4)

 3   4  1  0.6   5  3  0.8
 3  1.8  1.6

yn  2.8

54. The annual demand for an item is 10,000 units. The unit cost is Rs. 100 and inventory carrying charges
are 14.4% of the unit cost per annum. The cost of one procurement is Rs. 2000. The time between two
consecutive orders to meet the above demand is ______ month(s).
Key: 2

 ICP–Intensive Classroom Program  eGATE-Live Internet Based Classes DLP  TarGATE-All India Test Series
Leaders in GATE Preparations  65+ Centers across India
© All rights reserved by Gateforum Educational Services Pvt. Ltd. No part of this booklet may be reproduced or utilized in any form without the written permission.

28
2828/
|ME| GATE-2016-PAPER-01 www.gateforum.com

Exp: D= 10,000, Cu = Rs 100, Ch= 0.144×Cu, Co=Rs 2,000.


2DC0 2  10,000  2000 Q*
Q*    1666.67 units
Ch 0.144  100

Q * 1666.67
We know T    0.1667 years = 2 months
D 10,000
T

55. Maximize Z=15X1 + 20X2


subject to
12X1+ 4X2 ≥ 36
12X1 − 6X2 ≤ 24
X1, X2 ≥ 0
The above linear programming problem has
(A) infeasible solution (B) unbounded solution
(C) alternative optimum solutions (D) degenerate solution
Key: (B)
Exp: Max Z  15x1  20x 2
Subject to
12x1  4x 2  36
12x1  6x 2  24
x1 , x 2  0
Since, there is no limitation of boundary for the feasible region therefore, the LPP has unbounded
solution.

 ICP–Intensive Classroom Program  eGATE-Live Internet Based Classes DLP  TarGATE-All India Test Series
Leaders in GATE Preparations  65+ Centers across India
© All rights reserved by Gateforum Educational Services Pvt. Ltd. No part of this booklet may be reproduced or utilized in any form without the written permission.

29
2929/
|ME| GATE-2016-PAPER-02 www.gateforum.com

General Aptitude
Q. No. 1 – 5 Carry One Mark Each

1. The volume of a sphere of diameter 1 unit is than the volume of a cube of side 1 unit.
(A) least (B) less (C) lesser (D) low
Key: (B)

2. The unruly crowd demanded that the accused be without trial.


(A) hanged (B) hanging (C) hankering (D) hung
Key: (A)

3. Choose the statement(s) where the underlined word is used correctly:


(i) A prone is a dried plum.
(ii) He was lying prone on the floor.
(iii) People who eat a lot of fat are prone to heart disease.
(A) (i) and (iii) only (B) (iii) only (C) (i) and (ii) only (D) (ii) and (iii) only
Key: (D)

4. Fact: If it rains, then the field is wet.


Read the following statements:
(i) It rains
(ii) The field is not wet
(iii) The field is wet
(iv) It did not rain
Which one of the options given below is NOT logically possible, based on the given fact?
(A) If (iii), then (iv). (B) If (i), then (iii).
(C) If (i), then (ii). (D) If (ii), then (iv).
Key: (C)

5. A window is made up of a square portion and an equilateral triangle portion above it. The base of the
triangular portion coincides with the upper side of the square. If the perimeter of the window is 6 m, the
area of the window in m2 is .
(A) 1.43 (B) 2.06 (C) 2.68 (D) 2.88

Key: (B) x
x
Exp:
3x+2x=6
5x=6
x=6/5 x
x
6 6 36
Area of square   
5 5 25
3 2 3 6 6 3 36
Area of triangle       .
4 4 5 5 4 25 x
 3  36
1     2.06
 4  25

 ICP–Intensive Classroom Program  eGATE-Live Internet Based Classes DLP  TarGATE-All India Test Series
Leaders in GATE Preparations  65+ Centers across India
© All rights reserved by Gateforum Educational Services Pvt. Ltd. No part of this booklet may be reproduced or utilized in any form without the written permission.

1
11/11
|ME| GATE-2016-PAPER-02 www.gateforum.com

Q. No. 6 – 10 Carry Two Marks Each

6. Students taking an exam are divided into two groups, P and Q such that each group has the same number
of students. The performance of each of the students in a test was evaluated out of 200 marks. It was
observed that the mean of group P was 105, while that of group Q was 85. The standard deviation of
group P was 25, while that of group Q was 5. Assuming that the marks were distributed on a normal
distribution, which of the following statements will have the highest probability of being TRUE?
(A) No student in group Q scored less marks than any student in group P.
(B) No student in group P scored less marks than any student in group Q.
(C) Most students of group Q scored marks in a narrower range than students in group P.
(D) The median of the marks of group P is 100.
Key: (C)
68-95-97 rule
P Q

80 125 150 75 80 90 95
      2   2       2
105 85
 
95% of students in P scores between 65 to 150
95% of students in Q score between 75 to 95.
D is not correct
median = mean for normal distribution.
C is correct answer.
7. A smart city integrates all modes of transport, uses clean energy and promotes sustainable use of
resources. It also uses technology to ensure safety and security of the city, something which critics argue,
will lead to a surveillance state.
Which of the following can be logically inferred from the above paragraph?
(i) All smart cities encourage the formation of surveillance states.
(ii) Surveillance is an integral part of a smart city.
(iii) Sustainability and surveillance go hand in hand in a smart city.
(iv) There is a perception that smart cities promote surveillance.
(A) (i) and (iv) only (B) (ii) and (iii) only
(C) (iv) only (D) (i) only
Key: (C)

 ICP–Intensive Classroom Program  eGATE-Live Internet Based Classes DLP  TarGATE-All India Test Series
Leaders in GATE Preparations  65+ Centers across India
© All rights reserved by Gateforum Educational Services Pvt. Ltd. No part of this booklet may be reproduced or utilized in any form without the written permission.

2
22/12
|ME| GATE-2016-PAPER-02 www.gateforum.com

8. Find the missing sequence in the letter series.


B, FH, LNP, _ _ _ _.
(A) SUWY (B) TUVW (C) TVXZ (D)TWXZ
Key: (C)

9. The binary operation  is defined as a  + b = ab+(a+b), where a and b are any two real numbers.
The value of the identity element of this operation, defined as the number x such that a  x = a, for any a,
is .
(A) 0 (B) 1 (C) 2 (D) 10
Key: (A)
Exp: ax  a  ax   a  x   a
 x 1  a   0  x  0 is the identity element

Which of the following curves represents the y  ln  e  for x  2 ?


 sin  x  

10. 
 
Here, x represents the abscissa and y represents the ordinate.

(A)

(B)

 ICP–Intensive Classroom Program  eGATE-Live Internet Based Classes DLP  TarGATE-All India Test Series
Leaders in GATE Preparations  65+ Centers across India
© All rights reserved by Gateforum Educational Services Pvt. Ltd. No part of this booklet may be reproduced or utilized in any form without the written permission.

3
33/13
|ME| GATE-2016-PAPER-02 www.gateforum.com

(C)

(D)

Key: (C)

Mechanical Engineering

Q. No. 1 – 25 Carry One Mark Each

1. The condition for which the eigen values of the matrix


2 1 
A  are positive, is
1 k 
(A) k > 1/2 (B) k > −2 (C) k> 0 (D) k < −1/2
Key: (A)
Exp: By the properties of eigen values & eigen vectors, if all the principal minors of „A‟ are +Ve then all the
eigen values of „A‟ are also +Ve.
1
 A 22  0 for k 
2
1
So k 
2

 ICP–Intensive Classroom Program  eGATE-Live Internet Based Classes DLP  TarGATE-All India Test Series
Leaders in GATE Preparations  65+ Centers across India
© All rights reserved by Gateforum Educational Services Pvt. Ltd. No part of this booklet may be reproduced or utilized in any form without the written permission.

4
44/14
|ME| GATE-2016-PAPER-02 www.gateforum.com

2. The values of x for which the function


x 2  3x  4
f (x)  is NOT continuous are
x 2  3x  4
(A) 4 and −1 (B) 4 and 1 (C) −4 and 1 (D) −4 and −1
Key: (C)
x 2  3x  4
Exp: The function f (x)  2 is not continuous at x  4 &1; since f(x) does not exists at x=-4 &1.
x  3x  4

3. Laplace transform of cos(ωt) is


s  s 
(A) (B) 
(C) (D)
s  2
2
s  2 s  2
2
s  2
2

Key: (A)
Exp: By the L.T of standard functions

4. A function f of the complex variable z  x  iy, is given as f (x, y)  u(x, y)  iv(x, y), where
u(x, y)  2kxy and v(x, y)  x 2  y 2 . The value of k, for which the function is analytic, is _____ .
Key: -1
Exp: From C-R equation; we have
u v u v
 & 
x y y dx
 u(x, y)  2kxy v(x, y)  x 2  y 2
u u v v
  2ky  2kx   2x;  2y
x y x y
u v
 
x y
 2ky  2y
 k  1

5. Numerical integration using trapezoidal rule gives the best result for a single variable function, which is
(A) linear (B) parabolic (C) logarithmic (D) hyperbolic
Key: (A)
6. A point mass having mass M is moving with a velocity V at an angle θ to the wall as shown in the figure.
The mass undergoes a perfectly elastic collision with the smooth wall and rebounds. The total change
(final minus initial) in the momentum of the mass is

(A) 2MV cos ˆj (B) 2MVsin ˆj (C) 2MVcos ˆj (D) 2MVsin ˆj
Key: (D)
 ICP–Intensive Classroom Program  eGATE-Live Internet Based Classes DLP  TarGATE-All India Test Series
Leaders in GATE Preparations  65+ Centers across India
© All rights reserved by Gateforum Educational Services Pvt. Ltd. No part of this booklet may be reproduced or utilized in any form without the written permission.

5
55/15
|ME| GATE-2016-PAPER-02 www.gateforum.com

Exp: Let w is the velocity after collision

V sin j
w cos  i

V

 w

V cos  i  w sin  j
Beforecollision After collision

velocity of separation
e  1  e  1, for perfectly elastic collision
velocity of approach
 w sin ˆj  Vsin ˆj ______ (1)
Change in momentum = final momentum – initial momentum


P   w sin j  V sin j M  
 2MV sin  j  w sin j  V sin j
7. A shaft with a circular cross-section is subjected to pure twisting moment. The ratio of the
maximum shear stress to the largest principal stress is
(A) 2.0 (B) 1.0 (C) 0.5 (D) 0
Key: (B)
Exp: Mohr 's circle
xy xy

xy xy
xy
xy
max  xy
1  xy where, 1 is largest principal stress
max
 1
1

8. A thin cylindrical pressure vessel with closed-ends is subjected to internal pressure. The ratio of
circumferential (hoop) stress to the longitudinal stress is
(A) 0.25 (B) 0.50 (C) 1.0 (D) 2.0
Key: (D)

 ICP–Intensive Classroom Program  eGATE-Live Internet Based Classes DLP  TarGATE-All India Test Series
Leaders in GATE Preparations  65+ Centers across India
© All rights reserved by Gateforum Educational Services Pvt. Ltd. No part of this booklet may be reproduced or utilized in any form without the written permission.

6
66/16
|ME| GATE-2016-PAPER-02 www.gateforum.com

pd
Exp: Circumferential stress  c  
2t
pd
Longitudinal stress    
4t
where, p is internal pressure
d is internal diameter
t is thickness
pd

 c  2t  2
 pd
4t

9. The forces F1 and F2 in a brake band and the direction of rotation of the drum are as shown in the figure.
The coefficient of friction is 0.25. The angle of wrap is 3π/2 radians. It is given that R = 1 m and
F2 = 1 N. The torque (in N-m) exerted on the drum is _____ .

Key: 2.248
F2
Exp:  exp   
F1
Torque   F2  F1  R

 1  exp   
 F2 R 

  3  
 1  1  1  exp  0.25     2.248 Nm
  2 
10. A single degree of freedom mass-spring-viscous damper system with mass m, spring constant k and
viscous damping coefficient q is critically damped. The correct relation among m, k, and q is

2k k
(A) q  2km (B) q  2 km (C) q  (D) q 
m m
Key: (B)
Exp: We know that
q
n
2m
k
q  2m   2 km
m

 ICP–Intensive Classroom Program  eGATE-Live Internet Based Classes DLP  TarGATE-All India Test Series
Leaders in GATE Preparations  65+ Centers across India
© All rights reserved by Gateforum Educational Services Pvt. Ltd. No part of this booklet may be reproduced or utilized in any form without the written permission.

7
77/17
|ME| GATE-2016-PAPER-02 www.gateforum.com

11. A machine element XY, fixed at end X, is subjected to an axial load P, transverse load F, and a twisting
moment T at its free end Y. The most critical point from the strength point of view is

(A) a point on the circumference at location Y


(B) a point at the center at location Y
(C) a point on the circumference at location X
(D) a point at the center at location X
Key: (C)
Exp:
F.B.D
F
F T
T T
P P
P

Y MF M  F 
X

At location Y
At circumference
 Direct stress due to direct load P is 0  p A where A is cross section area.
 Due to shear load F is 1  F A
 d
 Due to torsion T is 2  16T d3 r  
 2
 Due to B.M F. dx is b  0
At centre of location Y
 0  P A

 1  F A

T 
 2  0 r  0  in relation 
J r
 b  0
At location X
At circumference
 0  P A

 1  F A

 ICP–Intensive Classroom Program  eGATE-Live Internet Based Classes DLP  TarGATE-All India Test Series
Leaders in GATE Preparations  65+ Centers across India
© All rights reserved by Gateforum Educational Services Pvt. Ltd. No part of this booklet may be reproduced or utilized in any form without the written permission.

8
88/18
|ME| GATE-2016-PAPER-02 www.gateforum.com

 d
 2  16T / d3  r  
 2
My 32M
 b   ( y  d / 2 and M  F  )
I d3
At center
 o  P / A
 1  F / A
 2  0  r  0 
 b  0  y  0 
 The most critical point is at circumference of location X.

12. For the brake shown in the figure, which one of the following is TRUE?

(A) Self energizing for clockwise rotation of the drum


(B) Self energizing for anti-clockwise rotation of the drum
(C) Self energizing for rotation in either direction of the drum
(D) Not of the self energizing type F
Key: (A) 
b
Exp: FBD of Lever
taking moment about hinge for clockwise rotation of wheel
f  N
Nb  F  Nc  0 c

N
F
 F N
 b  c  N

 b  c  
If b c  self energizing
So for clockwise rotation of the drum, the brake is self energizing.

13. The volumetric flow rate (per unit depth) between two streamlines having stream functions 1 and  2 is
(A) 1  2 (B) 1 2 (C) 1 /  2 (D) 1  2
Key: (D)
Exp: Volume flow rate per unit depth between two streamlines is given by 1   2
14. Assuming constant temperature condition and air to be an ideal gas, the variation in atmospheric pressure
with height calculated from fluid statics is
(A) linear (B) exponential (C) quadratic (D) cubic
Key: (B)
 ICP–Intensive Classroom Program  eGATE-Live Internet Based Classes DLP  TarGATE-All India Test Series
Leaders in GATE Preparations  65+ Centers across India
© All rights reserved by Gateforum Educational Services Pvt. Ltd. No part of this booklet may be reproduced or utilized in any form without the written permission.

9
99/19
|ME| GATE-2016-PAPER-02 www.gateforum.com

15. A hollow cylinder has length L, inner radius r1, outer radius r2, and thermal conductivity k. The thermal
resistance of the cylinder for radial conduction is

ln  r2 / r1  ln  r1 / r2  2kL 2kL
(A) (B) (C) (D)
2kL 2kL ln  r2 / r1  ln  r1 / r2 
Key: (A)
Exp: For a hollow cylinder
2kL  T1  T2  k
Q
r 
ln  2  r1 r2
 r1 
T1  T2 T1  T2
 
 r2  R th
ln  
 r1 
2kL

16. Consider the radiation heat exchange inside an annulus between two very long concentric cylinders. The
radius of the outer cylinder is R0 and that of the inner cylinder is Ri. The radiation view factor of the
outer cylinder onto itself is
1/3
Ri R R  Ri
(A) 1  (B) 1 i (C) 1   i  (D) 1 
R0 R0  R0  R0

Key: (D) 2
1
Exp: F11  0
Ri
F12  1
A1 2R i L R i R0
A 2 F21  A1F12  F2 1   
A 2 2R 0 L R 0

Ri
 F2 2  1  F21  1 
R0

17. The internal energy of an ideal gas is a function of


(A) temperature and pressure
(B) volume and pressure
(C) entropy and pressure
(D) temperature only
Key: (D)
Exp: According to Joule‟s law,
Internal energy of an ideal gas is a function of temperature only.

 ICP–Intensive Classroom Program  eGATE-Live Internet Based Classes DLP  TarGATE-All India Test Series
Leaders in GATE Preparations  65+ Centers across India
© All rights reserved by Gateforum Educational Services Pvt. Ltd. No part of this booklet may be reproduced or utilized in any form without the written permission.

10
1010/
|ME| GATE-2016-PAPER-02 www.gateforum.com

18. The heat removal rate from a refrigerated space and the power input to the compressor are 7.2 kW and
1.8 kW, respectively. The coefficient of performance (COP) of the refrigerator is .
Key: 4
Q2 7.2
Exp: C.O.PR   4
W 1.8

Source

Q1

W  1.8kW
R
Q2  7.2kW

Refrigerated
Sink
space

19. Consider a simple gas turbine (Brayton) cycle and a gas turbine cycle with perfect regeneration. In both
the cycles, the pressure ratio is 6 and the ratio of the specific heats of the working medium is 1.4. The
ratio of minimum to maximum temperatures is 0.3 (with temperatures expressed in K) in the regenerative
cycle. The ratio of the thermal efficiency of the simple cycle to that of the regenerative cycle is ____ .
Key: 0.8021
Exp: Brayton cycle:-
3
rp  6
  1.4 T 2
1
brayton  1  1 4

(rp )
1 1
1 1.4 1
(6) 1.4
 0.4006 S
Gas Turbine cycle with perfect regeneration:-
rp  6
  1.4
  1  Cp  T5  T2   Cp  T3  T2  4
3
 T5  T3 2
T
T1
 0.3 5
T4
6
Heat supplied  Cp  T4  T3  1

 ICP–Intensive Classroom Program  eGATE-Live Internet Based Classes DLP  TarGATE-All India Test Series
Leaders in GATE Preparations  65+ Centers across India
© All rights reserved by Gateforum Educational Services Pvt. Ltd. No part of this booklet may be reproduced or utilized in any form without the written permission.

11
1111/
|ME| GATE-2016-PAPER-02 www.gateforum.com

Work done  WT  Wc
 Cp  T4  T5   Cp  T2  T1 
 Cp  T4  T3   Cp  T2  T1 

Work done Cp  T4  T3   Cp  T2  T1 
regenerative  
Heat supplied Cp  T4  T3 

T   T2 
T1  2  1  1
 T2  T1   1
T   1  T1  T1
1   1 
T
 4  T3   T3  T4  1  T5 
T4 1    T 
 T4   4 

 
 1 
T1  p 
 r  1  1 1.4 1
  1   rp   1  0.3(6) 1.4  0.4994
T1
1  
T4  1  1  T4
 1

  rp   
 
brayton 0.4006
  0.8021
regenerative 0.4994

20. In a single-channel queuing model, the customer arrival rate is 12 per hour and the serving rate is 24 per
hour. The expected time that a customer is in queue is minutes.
Key: 2.5
 12
Exp:   12 hr ,   24 hr      0.5
 24
Let, expected time that a customer spend in queue is w q

  0.5  0.5
LqL   1 
wq   s   0.5  60
   12
0.5
  60  2.5mins
12

21. In the phase diagram shown in the figure, four samples of the same composition are heated to
temperatures marked by a, b, c and d.

At which temperature will a sample get solutionized the fastest?


(A) a (B) b (C) c (D) d
Key: (C)
 ICP–Intensive Classroom Program  eGATE-Live Internet Based Classes DLP  TarGATE-All India Test Series
Leaders in GATE Preparations  65+ Centers across India
© All rights reserved by Gateforum Educational Services Pvt. Ltd. No part of this booklet may be reproduced or utilized in any form without the written permission.

12
1212/
|ME| GATE-2016-PAPER-02 www.gateforum.com

22. The welding process which uses a blanket of fusible granular flux is
(A) tungsten inert gas welding (B) submerged arc welding
(C) electroslag welding (D) thermit welding
Key: (B)
Exp: Submerged arc welding uses a blanket of fusible granular flux

23. The value of true strain produced in compressing a cylinder to half its original length is
(A) 0.69 (B) − 0.69 (C) 0.5 (D) − 0.5
Key: (B)
Exp: Final length = L/2
Initial length = L
Lf L 1
T  ln  ln  ln  0.69
L0 2L 2

24. The following data is applicable for a turning operation. The length of job is 900 mm, diameter of job is
200 mm, feed rate is 0.25 mm/rev and optimum cutting speed is 300 m/min. The machining time (in min)
is ___.
Key: 7.539
Exp: L  900mm
d  200mm
f  0.25mm / rev
v  300m / min
t ?
v  DN m/min
1000
300 m/min =  0.2  N
300
N  478 RPM
 0.2
L 900
t   7.539min
fN 0.25  478

25. In an ultrasonic machining (USM) process, the material removal rate (MRR) is plotted as a function of
the feed force of the USM tool. With increasing feed force, the MRR exhibits the following behavior:
(A) increases linearly
(B) decreases linearly
(C) does not change
(D) first increases and then decreases
Key: (D)

 ICP–Intensive Classroom Program  eGATE-Live Internet Based Classes DLP  TarGATE-All India Test Series
Leaders in GATE Preparations  65+ Centers across India
© All rights reserved by Gateforum Educational Services Pvt. Ltd. No part of this booklet may be reproduced or utilized in any form without the written permission.

13
1313/
|ME| GATE-2016-PAPER-02 www.gateforum.com

Exp: In USM,

MRR Increase
Decrease due to Crushing
of abrasives

Feed force of tool

Q. No. 26 – 55 carry Two Marks Each

26. A scalar potential  has the following gradient.   yziˆ  xzjˆ  xyk.
ˆ Consider the integral

 
 .dr on the curve r  xiˆ  yjˆ  zk.
ˆ
c

x  t

The curve C is parameterized as follows:  y  t 2 and 1  t  3.
z  3t 2

The value of the integral is______ .

Key: 726

 .dr    yziˆ  xzjˆ  xykˆ .dxiˆ  dyjˆ  dzkˆ 



Exp:
c c

  yzdx  xzdy  xydz … (1)


c

 x  t; y  t 2 ; z  3t 2

 dx  dt  dy  2tdt  dz  6tdt


From (1);  .dr   t 2 (3t 2 )dt  t  3t 2  2tdt  t  t 2  6tdt
c c

3
  3t   6t 4  6t 4 dt
t 1

3
3  t5 
  15t dt  15    3 35  1
4
t 1
 5 1

 726

 ICP–Intensive Classroom Program  eGATE-Live Internet Based Classes DLP  TarGATE-All India Test Series
Leaders in GATE Preparations  65+ Centers across India
© All rights reserved by Gateforum Educational Services Pvt. Ltd. No part of this booklet may be reproduced or utilized in any form without the written permission.

14
1414/
|ME| GATE-2016-PAPER-02 www.gateforum.com

3z  5
27. The value of  (z 1)(z 2) dz
r
along a closed path  is equal to (4i), where z=x+iy and i  1. The

correct path  is
(A) (B)

(C) (D)

Key: (B)
Exp: Since if z=1 lies inside the closed path  and z=2 lies outside of the closed path  then by cauchy‟s
formula.
3z  5
3z  5
M (z 1)(z 2) dz  n zz  12 dz
 3z  5 
 2i  
 z  2  at z 1
 2 
 2i    4i
 1 

28. The probability that a screw manufactured by a company is defective is 0.1. The company sells screws in
packets containing 5 screws and gives a guarantee of replacement if one or more screws in the packet are
found to be defective. The probability that a packet would have to be replaced is .
Key: 0.40951
Exp: Probability that a packet would have to be replaced i.e., P  X  1  ? [ Let „x‟ denote the number of
defective screws]
 P  X  1  1  P  X  1
 1  P  X  0
 1  5C0 (0.1)0 (0.9)5
 1   0.9   0.40951
5

Since by the Binomial distribution when P=probability of defective screw.



29. The error in numerically computing the integral   sin x  cos x  dx using the trapezoidal rule with three
0
intervals of equal length between 0 and  is_______ .
Key: 0.178
 ICP–Intensive Classroom Program  eGATE-Live Internet Based Classes DLP  TarGATE-All India Test Series
Leaders in GATE Preparations  65+ Centers across India
© All rights reserved by Gateforum Educational Services Pvt. Ltd. No part of this booklet may be reproduced or utilized in any form without the written permission.

15
1515/
|ME| GATE-2016-PAPER-02 www.gateforum.com

ba 0 
Exp: h   ; f (x)  sin x  cos x
n 3 3
 2
x 0 
3 3
y  f (x) 1 1.37 0.37 1
By trapezoidal rule; we have the approximate value of the integral is
 /3
0 (sin x  cos x)dx  2 1  (1)  2(1.37  0.37)
 1.822
Exact value of the integral is

(sinx  cosx)dx    cos x  sin x 0  1  (1)  2

0

Error  Exact value  Approximate value


 2  1.822  0.178

30. A mass of 2000 kg is currently being lowered at a velocity of 2 m/s from the drum as shown in the figure.
The mass moment of inertia of the drum is 150 kg-m2. On applying the brake, the mass is brought to rest
in a distance of 0.5 m. The energy absorbed by the brake (in kJ) is

Key: 14.11
Exp:
V  R
V 2.0
i  i   2rad/sec
R 1

J  i2  f2 
1
Loss in kinetic energy of Drum =
2
1
 KE drum  150   2   300 Joule
2

2
1
Loss in kinetic energy of block =  m vi2  vf2
2
 
1
  2000   2   02 
2

2  
KE block  4000 Joule
Loss of Potential energy of block = mgh
 2000 9.81  0.5
PE block  9810 Joule

 ICP–Intensive Classroom Program  eGATE-Live Internet Based Classes DLP  TarGATE-All India Test Series
Leaders in GATE Preparations  65+ Centers across India
© All rights reserved by Gateforum Educational Services Pvt. Ltd. No part of this booklet may be reproduced or utilized in any form without the written permission.

16
1616/
|ME| GATE-2016-PAPER-02 www.gateforum.com

Total energy loss  KEdrum  KE block  PE block


 300  4000  9810 14110 joule
 14.11 kJ

31. A system of particles in motion has mass center G as shown in the figure. The particle i has mass mi and
its position with respect to a fixed point O is given by the position vector ri. The position of the particle
with respect to G is given by the vector  i . The time rate of change of the angular momentum of the
system of particles about G is
(The quantity i indicates second derivative of  i with respect to time and likewise for ri ).

(A) i ri  mii (B) i i  miri (C) i ri  miri (D) i i  mii


Key: (B)
Exp: By definition of Torque
 
Torque  rPOR  Fext
where

rPOR = position vector with respect to point of rotation (POR) of the particle on which force is acting.

Fext  External force acting on ith particle

 miri
 
i  i  mi ri
For complete Rigid body
 
 i   i  mi ri
i

32. A rigid horizontal rod of length 2L is fixed to a circular cylinder of radius R as shown in the figure.
Vertical forces of magnitude P are applied at the two ends as shown in the figure. The shear modulus for
the cylinder is G and the Young‟s modulus is E.

The vertical deflection at point A is


(A) PL3 / R 4 G   (B) PL3 / R 4 E  (C) 2PL3 / R 4 E   (D) 4PL3 / R 4G  
Key: (D)

 ICP–Intensive Classroom Program  eGATE-Live Internet Based Classes DLP  TarGATE-All India Test Series
Leaders in GATE Preparations  65+ Centers across India
© All rights reserved by Gateforum Educational Services Pvt. Ltd. No part of this booklet may be reproduced or utilized in any form without the written permission.

17
1717/
|ME| GATE-2016-PAPER-02 www.gateforum.com

Exp: F.B.D
T

T  2PL

T
Because of torsion angle of twist    will be there.
T 2  32 P2 4 P2
Where    
G.J G  d 4 R 4G
Due to angle of twist, A will reach at A ' and B will reach at B'
let A 'A'' be the vertical displacement
A' B
 
x

O
A'' 
A
B'
In  A 'O A''
A 'A '' x
sin    where, x is vertical deflection
A 'O 
x
When  is very small, sin      x  

4P 3
x
R 4G
33. A simply supported beam of length 2L is subjected to a moment M at the mid-point x = 0 as shown in the
figure. The deflection in the domain 0 ≤ x ≤ L is given by
Mx
W  L  x  (x  c),
12EIL
where E is the Young‟s modulus, I is the area moment of inertia and c is a constant (to be determined) .

The slope at the center x = 0 is


(A) ML / (2EI) (B) ML / (3EI) (C) ML / (6EI) (D) ML / (12EI)
Key: (C)

 ICP–Intensive Classroom Program  eGATE-Live Internet Based Classes DLP  TarGATE-All India Test Series
Leaders in GATE Preparations  65+ Centers across India
© All rights reserved by Gateforum Educational Services Pvt. Ltd. No part of this booklet may be reproduced or utilized in any form without the written permission.

18
1818/
|ME| GATE-2016-PAPER-02 www.gateforum.com

Exp: As we know that



M2dx
U
0 2EI

U
and slope at mid-span where moment is applied will be (according to Costigliano‟s theorem).
M
Mx
M 0  x  
2

Mx
M M    x  2 
2 x

 
   Mx 2 2
 Mx 
2
 1
 U     dx     M  dx  
 0  2    2   2EI x
M M M

 x  2  dx  2
M2  x 2
 2 2 2
U    2 dx    x
2EI  0 4  4 2 

M2   3   3  M 2 
     
2EI  4 2  3  3  12EI

 U  M
 slopeat  x  0     
 M  x 0 6EI
34. In the figure, the load P = 1 N, length L = 1 m, Young‟s modulus E = 70 GPa, and the cross-section of the
links is a square with dimension 10 mm × 10 mm. All joints are pin joints.

The stress (in Pa) in the link AB is ________ .


(Indicate compressive stress by a negative sign and tensile stress by a positive sign.)
Key: 0

 ICP–Intensive Classroom Program  eGATE-Live Internet Based Classes DLP  TarGATE-All India Test Series
Leaders in GATE Preparations  65+ Centers across India
© All rights reserved by Gateforum Educational Services Pvt. Ltd. No part of this booklet may be reproduced or utilized in any form without the written permission.

19
1919/
|ME| GATE-2016-PAPER-02 www.gateforum.com

Exp: F.B.D of point B


FBC

45
45 B

P
FAB

Applying Lame‟s theorem


FAB P F
  BC
sin180 sin135 sin 45
 FAB  0
Stress in AB  0

35. A circular metallic rod of length 250 mm is placed between two rigid immovable walls as shown in the
figure. The rod is in perfect contact with the wall on the left side and there is a gap of 0.2 mm between
the rod and the wall on the right side. If the temperature of the rod is increased by 200 o C, the axial stress
developed in the rod is MPa.
Young‟s modulus of the material of the rod is 200 GPa and the coefficient of thermal expansion is 10−5
per oC.

Key: 240
Exp: t  200C
  105 C
E  2 105 MPa
  0.2 mm
We know that, Axial stress gets induced in the rod when some gap „  ‟ is provided is
 t      105  200  250  0.2 
  E     2 10
5

    250 
 240 MPa

 ICP–Intensive Classroom Program  eGATE-Live Internet Based Classes DLP  TarGATE-All India Test Series
Leaders in GATE Preparations  65+ Centers across India
© All rights reserved by Gateforum Educational Services Pvt. Ltd. No part of this booklet may be reproduced or utilized in any form without the written permission.

20
2020/
|ME| GATE-2016-PAPER-02 www.gateforum.com

36. The rod AB, of length 1 m, shown in the figure is connected to two sliders at each end through pins. The
sliders can slide along QP and QR. If the velocity VA of the slider at A is 2 m/s, the velocity of the
midpoint of the rod at this instant is m/s.

Key: 1
VA =2m/sec 2cos60  1
Exp: Given AB  2 60 2sin60  3
A
Since Rod AB is rigid, so A
Axial velocity of A & B should be same C
C 2cos60  1
VA cos60  VB cos60 60 60 B
Q B
VA  VB  2m/sec VB

C is mid point of AB 2sin60  3

A 3
A

C
 C  Vc  1

B
1 B
 3

Velocity corresponding to Velocity corresponding to


pure translation part Rotational part

Alternate Method:
 2   2  x 2
2 2
y a
cos120  x
30o
2 2 2 c
2
2
1  x  120o
 1    30o
2 2 2 b o
2
3
x  2 22 3
2
x
y  3,so oc will be perpendicular toab.
2
v
sin  30   c  vc 1
2

 ICP–Intensive Classroom Program  eGATE-Live Internet Based Classes DLP  TarGATE-All India Test Series
Leaders in GATE Preparations  65+ Centers across India
© All rights reserved by Gateforum Educational Services Pvt. Ltd. No part of this booklet may be reproduced or utilized in any form without the written permission.

21
2121/
|ME| GATE-2016-PAPER-02 www.gateforum.com

37. The system shown in the figure consists of block A of mass 5 kg connected to a spring through a massless
rope passing over pulley B of radius r and mass 20 kg. The spring constant k is 1500 N/m. If there is no
slipping of the rope over the pulley, the natural frequency of the system is rad/s.

Key: 10
Exp: Displace the block “A” & Release

 k r r   
1 2
Mr  mr 2  
 0
2 
 
  k 
   0
 1 M m 
2 
 
 k  1500
n     10 rad/sec
 1Mm  10  5
2 

Alternate method:
1 1 1
Energy of system remain conserved, E  J2  mv 2  mg y  k  y    …(1)
2

2 2 2

Rotational Translational Gravitational P.E stored


K.E K.E K.E in spring
where,
  Static elongation of spring at equilibrium which is calculated as follows:

mg
mg  k   
k
Differentiating Eqn.(1)w.r.t time, which will be zero because E  constant
dE
0
dt
d dv
J  mv  mgv  k  y    v  0 ...(2)
dt dt
Since there is no slipping between rope & pulley
v  r

 ICP–Intensive Classroom Program  eGATE-Live Internet Based Classes DLP  TarGATE-All India Test Series
Leaders in GATE Preparations  65+ Centers across India
© All rights reserved by Gateforum Educational Services Pvt. Ltd. No part of this booklet may be reproduced or utilized in any form without the written permission.

22
2222/
|ME| GATE-2016-PAPER-02 www.gateforum.com

1 2  v  1 dv  dv
Mr     mv  mgv  k  y    v  0
2  r  r dt  dt
M d y
2

  m  2  ky  0
 2  dt
k 1500
n   10rad/sec
M  10  5
  m
 2 

38. In a structural member under fatigue loading, the minimum and maximum stresses developed at the
critical point are 50 MPa and 150 MPa, respectively. The endurance, yield, and the ultimate strengths of
the material are 200 MPa, 300 MPa and 400 MPa, respectively. The factor of safety using modified
Goodman criterion is

3 8 12
(A) (B) (C) (D) 2
2 5 7
Key: (D)
Exp: Given

max 150MPa, min  50MPa, m  100MPa

a  50MPa,Se  200MPa,Sut  300MPa

Sut  400MPa

Equation of line 1

a  m
 1 ...(1)
Se Sut

Equation of line 2

a 1
 tan   ...(2)
m 2

Solving above two equations to get co-ordinates of point P (Sm, Sa)

Sa S
 m 1
200 400

2 Sa  Sm  400 ...(3)

Sm  2Sa ...(4)

from (3) & (4)

Sa  100MPa
Sa 100
f.o.s   2
a 50

 ICP–Intensive Classroom Program  eGATE-Live Internet Based Classes DLP  TarGATE-All India Test Series
Leaders in GATE Preparations  65+ Centers across India
© All rights reserved by Gateforum Educational Services Pvt. Ltd. No part of this booklet may be reproduced or utilized in any form without the written permission.

23
2323/
|ME| GATE-2016-PAPER-02 www.gateforum.com

39. The large vessel shown in the figure contains oil and water. A body is submerged at the interface of oil
and water such that 45 percent of its volume is in oil while the rest is in water. The density of the body is
______kg/m3.
The specific gravity of oil is 0.7 and density of water is 1000 kg/m3.
Acceleration due to gravity g = 10 m/s2.

Key: 865
Exp: Given water  1000 kg m3
oil  700 kg m3
body  ?
Voil  0.45Vbody
Vwater  0.55Vbody
oil Voil g  water Vwater g  body Vbody g
700  0.45Vbody  1000  0.55Vbody  body Vbody
body  865kg m3

40. Consider fluid flow between two infinite horizontal plates which are parallel (the gap between them being
50 mm). The top plate is sliding parallel to the stationary bottom plate at a speed of 3 m/s. The flow
between the plates is solely due to the motion of the top plate. The force per unit area (magnitude)
required to maintain the bottom plate stationary is N/m2.
Viscosity of the fluid µ = 0.44 kg/m-s and density ρ = 888 kg/m3.
Key: 26.4
Exp: Vtop  3m / s
50 mm
  0.44 kg m  s
  888kg m3

du Vtop  Vbottom 3
   60
dy 0.05 0.05
du
J wall    0.44  60 kg m.s 2
dy
 26.4 N m2
 ICP–Intensive Classroom Program  eGATE-Live Internet Based Classes DLP  TarGATE-All India Test Series
Leaders in GATE Preparations  65+ Centers across India
© All rights reserved by Gateforum Educational Services Pvt. Ltd. No part of this booklet may be reproduced or utilized in any form without the written permission.

24
2424/
|ME| GATE-2016-PAPER-02 www.gateforum.com

41. Consider a frictionless, massless and leak-proof plug blocking a rectangular hole of dimensions 2R  L
the bottom of an open tank as shown in the figure. The head of the plug has the shape of a semi-cylinder
of radius R. The tank is filled with a liquid of density ρ up to the tip of the plug. The gravitational
acceleration is g. Neglect the effect of the atmospheric pressure.

The force F required to hold the plug in its position is


     2
(A) 2R 2 gL 1   (B) 2R 2 gL 1   (C) R 2gL (D) R gL
 4  4 2
Key: (A)
Exp:

2R
Downward force due to water = weight of water above curved surface
  R 2 L  
 2  g  R 2 L  
  4 
 
 2gR 2 L 1   N
 4
Weight of plug is neglected.

42. Consider a parallel-flow heat exchanger with area Ap and a counter-flow heat exchanger with area Ac.
In both the heat exchangers, the hot stream flowing at 1 kg/s cools from 80 ℃ to 50 ℃. For the cold
stream in both the heat exchangers, the flow rate and the inlet temperature are 2 kg/s and 10 ℃,
respectively. The hot and cold streams in both the heat exchangers are of the same fluid. Also, both the
heat exchangers have the same overall heat transfer coefficient. The ratio Ac / Ap is ________ .
Key: 0.928
Exp: mh  1kg / s
mc  2kg / s
CPh  Cpc (as both are same fluids)

 
mk Cpk t k1  t k 2  mc Cpc t c2  t c1  
 
1(80  50)  2 t c2  10  t c2  25 C o

 ICP–Intensive Classroom Program  eGATE-Live Internet Based Classes DLP  TarGATE-All India Test Series
Leaders in GATE Preparations  65+ Centers across India
© All rights reserved by Gateforum Educational Services Pvt. Ltd. No part of this booklet may be reproduced or utilized in any form without the written permission.

25
2525/
|ME| GATE-2016-PAPER-02 www.gateforum.com

Parallel flow
T
T Counter flow

t h1  80o C t h1  80o C
1  t k1  t c2
t h2  50o C t c2  25o C
t h 2  50o C
1  t k1  t c1 2  t k2  t c2 2  t k2  t c1
t c2  25o C t c2  10o C

t c1  10o C

L
L

1  70o C 1  80  25  55o C
2  25o C 2  50  10  40o C
  1  2
mp  1 2 mc 
   
ln  1  ln  1 
 2   2 
70  25 55  40
 
 70   55 
ln   ln  
 25   40 
 43.705  47.1
Q  U.Ap mp Q  U.A c .mc
 A p mp  A c .mc
A c mp 43.705
   0.928
A p mc 47.1

43. Two cylindrical shafts A and B at the same initial temperature are simultaneously placed in a furnace.
The surfaces of the shafts remain at the furnace gas temperature at all times after they are introduced into
the furnace. The temperature variation in the axial direction of the shafts can be assumed to be negligible.
The data related to shafts A and B is given in the following Table.

Quantity Shaft A Shaft B


Diameter (m) 0.4 0.1
Thermal conductivity (W/m-K) 40 20
Volumetric heat capacity (J/m3-K) 2×106 2×107

The temperature at the centerline of the shaft A reaches 400℃ after two hours. The time required
(in hours) for the centerline of the shaft B to attain the temperature of 400℃ is _____ .
Key: 2.4 – 2.6

 ICP–Intensive Classroom Program  eGATE-Live Internet Based Classes DLP  TarGATE-All India Test Series
Leaders in GATE Preparations  65+ Centers across India
© All rights reserved by Gateforum Educational Services Pvt. Ltd. No part of this booklet may be reproduced or utilized in any form without the written permission.

26
2626/
|ME| GATE-2016-PAPER-02 www.gateforum.com

44. A piston-cylinder device initially contains 0.4 m3 of air (to be treated as an ideal gas) at 100 kPa and
80oC. The air is now isothermally compressed to 0.1 m3. The work done during this process is
kJ.
(Take the sign convention such that work done on the system is negative)
Key: -55.45
Exp: V1  0.4m3
P1  100kPa
T1  80o C  353K
V2  0.1 m3
Ideal gas & process is isothermal.
V 
 W  P1V1 ln  2 
 V1 
 0.1 
 100  0.4ln  
 0.4 
 55.45kJ

45. A reversible cycle receives 40 kJ of heat from one heat source at a temperature of 127 ℃ and 37 kJ from
another heat source at 97 ℃. The heat rejected (in kJ) to the heat sink at 47 ℃ is .
Key: 64
Exp: Reversible cycle.
T1  127  273 T2  97  273
 400K  370K

Q1  40kJ Q2  37kJ

Q3

T3  47  273
 320K
Q
 0
T
Q1 Q2 Q3 40 37 Q3
  0   0
T1 T2 T3 400 370 320
Q3 1 320
  Q3   64kJ
320 5 5

46. A refrigerator uses R-134a as its refrigerant and operates on an ideal vapour-compression
refrigeration cycle between 0.14 MPa and 0.8 MPa. If the mass flow rate of the refrigerant is 0.05 kg/s,
the rate of heat rejection to the environment is kW.
Given data:
At P = 0.14 MPa, h = 236.04 kJ/kg, s=0.9322 kJ/kg-K
At P = 0.8 MPa, h = 272.05 kJ/kg (superheated vapour)
At P = 0.8MPa, h = 93.42 kJ/kg (saturated liquid)
Key: 8.9315
 ICP–Intensive Classroom Program  eGATE-Live Internet Based Classes DLP  TarGATE-All India Test Series
Leaders in GATE Preparations  65+ Centers across India
© All rights reserved by Gateforum Educational Services Pvt. Ltd. No part of this booklet may be reproduced or utilized in any form without the written permission.

27
2727/
|ME| GATE-2016-PAPER-02 www.gateforum.com

Exp: Given
h1  236.04kJ / kg P 0.8MPa 3
s1  0.9322 kJ / kg 2
h 2  272.05kJ / kg
h 3  93.42kJ / kg
0.14MPa 4 1
m  0.05kg / s

  h 2  h3 
 Heat Rejection to environment  m
 0.05  272.05  93.42  h

 8.9315kW

47. The partial pressure of water vapour in a moist air sample of relative humidity 70% is 1.6 kPa, the total
pressure being 101.325 kPa. Moist air may be treated as an ideal gas mixture of water vapour and dry air.
The relation between saturation temperature (Ts in K) and saturation pressure (ps in kPa) for water is
given by ln  ps / p0  14.317  5304 / Ts , where p0  101.325 kPa. The dry bulb temperature of the moist
air sample (in ℃) is
Key: 19.89
Exp: PV  1.6 kPa
T
  70% Ps  2.2857 kPa
Po  101.325 kPa PV  1.6 kPa
P  5304
ln  s  14.317   Ps  ? TSat
d.b.t  TSat  ?
 po  Ts  2
Pv

Psat
1 d.p.t
1.6 1.6
0.7   Ps   2.2857 kPa
Ps 0.7 S

 2.2857  5304
ln   14.317 
 101.325  Tsat
5304
Tsat   292.898 K
 2.2857 
14.317  n  
 101.325 
d.b.t  t sat  19.89o C

48. In a binary system of A and B, a liquid of 20% A (80% B) is coexisting with a solid of 70% A (30% B).
For an overall composition having 40% A, the fraction of solid is
(A) 0.40 (B) 0.50 (C) 0.60 (D) 0.75
Key: (A)

 ICP–Intensive Classroom Program  eGATE-Live Internet Based Classes DLP  TarGATE-All India Test Series
Leaders in GATE Preparations  65+ Centers across India
© All rights reserved by Gateforum Educational Services Pvt. Ltd. No part of this booklet may be reproduced or utilized in any form without the written permission.

28
2828/
|ME| GATE-2016-PAPER-02 www.gateforum.com

 0.2L  0.7S  0.4 20% A 70% A


 L  S 
80% B 30% B
0.2L  0.7S  0.4L  0.4S
Liquid (L) Solid (S)
0.2L  0.3S ...(1)
S
Fraction of solid 
S L  0.2L  0.7S % A


S  0.8L  0.3S % B
0.3
S S Mixture of liquid&solid
0.2
0.2
  0.40
0.5

49. Gray cast iron blocks of size 100 mm × 50 mm × 10 mm with a central spherical cavity of diameter 4 mm
are sand cast. The shrinkage allowance for the pattern is 3%. The ratio of the volume of the pattern to
volume of the casting is ______ .
Key: 1.08 – 1.10

50. The voltage-length characteristic of a direct current arc in an arc welding process is
V  100 + 40  , where l is the length of the arc in mm and V is arc voltage in volts. During a welding
operation, the arc length varies between 1 and 2 mm and the welding current is in the range 200-250 A.
Assuming a linear power source, the short circuit current is A.
Key: 424.6
Exp: V  100  40 1  1mm &  2  2mm
V1  140V I1  250A
V2  180V I2  200A
V I
 1
0CV SCC
140 250
 1 …(a)
OCV SCC
180 200
 1 …(b)
OCV SCC
On solving equation a and b, we have
S.C.C=424.6A

51. For a certain job, the cost of metal cutting is Rs. 18C/V and the cost of tooling is Rs. 270 C/(TV), where
C is a constant, V is the cutting speed in m/min and T is the tool life in minutes. The Taylor's tool life
equation is VT0.25  150. The cutting speed (in m/min) for the minimum total cost is .
Key: 57.9
Exp: Cost of metal cutting = Rs 18 C/V
Cost of Tooling = Rs 270 C/TV
 ICP–Intensive Classroom Program  eGATE-Live Internet Based Classes DLP  TarGATE-All India Test Series
Leaders in GATE Preparations  65+ Centers across India
© All rights reserved by Gateforum Educational Services Pvt. Ltd. No part of this booklet may be reproduced or utilized in any form without the written permission.

29
2929/
|ME| GATE-2016-PAPER-02 www.gateforum.com

C= Constant ,V = Cutting Speed , T = tool life

C = 150, VT0.25 = 150  T  150 V 


1 0.25

T  150 V 
4

C C
Total cost  18  270
V TV
18C 270CV 4
 
V 1504 V
18C 270C V3
 
V 1504
On, differentiating total cost
18C 270 C3V 2
 
V2 1504
18 1504
V4 
3  270
 V  57.914 m min

52. The surface irregularities of electrodes used in an electrochemical machining (ECM) process are 3 µm
and 6 µm as shown in the figure. If the work-piece is of pure iron and 12V DC is applied between the
electrodes, the largest feed rate is mm/min.
Conductivity of the electrolyte 0.02 ohm-1mm-1
Over-potential voltage 1.5 V
Density of iron 7860 kg/m3
Atomic weight of iron 55.85 gm
+2
Assume the iron to be dissolved as Fe and the Faraday constant to be 96500 Coulomb.

Key: 51.51
Exp: Given V = 12V V  1.5V

 ICP–Intensive Classroom Program  eGATE-Live Internet Based Classes DLP  TarGATE-All India Test Series
Leaders in GATE Preparations  65+ Centers across India
© All rights reserved by Gateforum Educational Services Pvt. Ltd. No part of this booklet may be reproduced or utilized in any form without the written permission.

30
3030/
|ME| GATE-2016-PAPER-02 www.gateforum.com

1
k  0.02 ohm 1 mm 1  0.02  0.2ohm 1 cm 1
ohm101 cm
  7860 kg m3
103
 7860  gm cm3
106
  55.85gm
 7.860 gm cm3
AI 55.85I
MRR : Q :   3.68 105 cm3 sec
ZF 7.86  2  96600
Inter electrode gap given  9 m  9 104 cm
k  V  V 
current density J 
y
0.2 12  1.5
  2333.33
9 104
I  J  S.A
I  2333.33  S.A  S.A  I 2333.33
Electrode feed rate = MRR surface area cm sec
3.68 105 I  2333.33
 cm sec
I
 0.086 10  60 mm min
 51.51mm min
53. For the situation shown in the figure below the expression for H in terms of r, R and D is

(A) H  D  r 2  R 2 (B) H  (R  r)  (D  r)

(C) H  (R  r)  D 2  R 2 (D) H  (R  r)  2D(R  r)  D2


Key: (D)

 ICP–Intensive Classroom Program  eGATE-Live Internet Based Classes DLP  TarGATE-All India Test Series
Leaders in GATE Preparations  65+ Centers across India
© All rights reserved by Gateforum Educational Services Pvt. Ltd. No part of this booklet may be reproduced or utilized in any form without the written permission.

31
3131/
|ME| GATE-2016-PAPER-02 www.gateforum.com

Exp:

a c
H

a  R  r, b  D   R  r  , C  a 2  b 2

 R  r   D   R  r 
2 2
C 

H  R  r  C  R  r  R  r   D2   R  r   2D  R  r 
2 2

H   R  r   2D  R  r   D2

54. A food processing company uses 25,000 kg of corn flour every year. The quantity-discount price of corn
flour is provided in the table below:

Quantity (kg) Unit price (Rs/kg)


1-749 70
750-1499 65
1500 and above 60

The order processing charges are Rs. 500/order. The handling plus carry-over charge on an annual basis
is 20% of the purchase price of the corn flour per kg. The optimal order quantity (in kg) is .
Key: 1500
Exp: D = 25000 kg, C0 = Rs 500/order, Ch = 20% of Cu

Qty (kg) Cu (Rs/kg) Ch (Rs/Kg/year)


1  Q1  750 70 0.2  70  14
750  Q2  1500 65 0.2  65  13
Q3  1500 60 0.2  60  12

This problem belongs to inventory model with two price break.


2DCo
Q 
Ch
 first checking for least unit price

2  25000  500
Q*3   1443.37
12
Now, 1443.37 < 1500 therefore, the company will not get the item at Rs 60/kg
Now, checking for second minimum unit price
 ICP–Intensive Classroom Program  eGATE-Live Internet Based Classes DLP  TarGATE-All India Test Series
Leaders in GATE Preparations  65+ Centers across India
© All rights reserved by Gateforum Educational Services Pvt. Ltd. No part of this booklet may be reproduced or utilized in any form without the written permission.

32
3232/
|ME| GATE-2016-PAPER-02 www.gateforum.com

2  25000  500
Q*2   1386.75
13
Since, 1386.75 lies between 750 and 1500
Therefore, we need to find

 
Total cost Q2  25000  65 
25000
1386.75
 500 
1386.75
2
13

 Rs1643027.755

 D Q 
T.C  D  Cu  Q  C0  2  Ch 
 
25000 1500
Totalcost 1500   25000  60   500  12  Rs 1517333.33
1500 2
Since, T.C (1500) < T.C Q*2 0  
Therefore, optimal order quantity is 1500.

55. A project consists of 14 activities, A to N. The duration of these activities (in days) are shown in brackets
on the network diagram. The latest finish time (in days) for node 10 is

Key: 14
Exp:
E5
E9
L  10
L  14
3 4
10
3 E  10
8 L  12
2 3
E2 6
L2 2 E 8
3
2 L  10 2 2
4 4 9 12
1 2 11
E0 E6 4 5 E  15 E  17 E  19
L0 L6 L  15 L  17 L  19
7
2 E  10 4
L  10

5
E4
L  11
The latest finish time for node 10 is 14 days.
 ICP–Intensive Classroom Program  eGATE-Live Internet Based Classes DLP  TarGATE-All India Test Series
Leaders in GATE Preparations  65+ Centers across India
© All rights reserved by Gateforum Educational Services Pvt. Ltd. No part of this booklet may be reproduced or utilized in any form without the written permission.

33
3333/
|ME| GATE-2016-PAPER-03 www.gateforum.com

General Aptitude
Q. No. 1 – 5 Carry One Mark Each

1. Based on the given statements, select the appropriate option with respect to grammar and
usage. Statements
(i) The height of Mr. X is 6 feet.
(ii) The height of Mr. Y is 5 feet.
(A) Mr. X is longer than Mr. Y.
(B) Mr. X is more elongated than Mr. Y.
(C) Mr. X is taller than Mr. Y.
(D) Mr. X is lengthier than Mr. Y.
Key: (C)

2. The students the teacher on teachers’ day for twenty years of dedicated teaching.
(A) facilitated (B) felicitated (C) fantasized (D) facillitated
Key: (B)

3. After India’s cricket world cup victory in 1985, Shrotria who was playing both tennis and
cricket till then, decided to concentrate only on cricket. And the rest is history.
What does the underlined phrase mean in this context?
(A) history will rest in peace (B) rest is recorded in history books
(C) rest is well known (D) rest is archaic
Key: (C)

Given  9 inches    0.25yards 


1/2 1/2
4. , which one of the following statements is TRUE?
(A) 3 inches = 0.5 yards (B) 9 inches = 1.5 yards
(C) 9 inches = 0.25 yards (D) 81 inches = 0.0625 yards
Key: (C)

5. S, M, E and F are working in shifts in a team to finish a project. M works with twice the
efficiency of others but for half as many days as E worked. S and M have 6 hour shifts in a
day, whereas E and F have 12 hours shifts. What is the ratio of contribution of M to
contribution of E in the project?
(A) 1:1 (B) 1:2 (C) 1:4 (D) 2:1
Key: (B)

 ICP–Intensive Classroom Program  eGATE-Live Internet Based Classes DLP  TarGATE-All India Test Series
Leaders in GATE Preparation  65+ Centers across India
© All rights reserved by Gateforum Educational Services Pvt. Ltd. No part of this booklet may be reproduced or utilized in any form without the written permission.

1
|ME| GATE-2016-PAPER-03 www.gateforum.com

Q. No. 6 – 10 Carry Two Marks Each

6. The Venn diagram shows the preference of the student population for leisure activities.

29

From the data given, the number of students who like to read books or play sports is .
(A) 44 (B) 51 (C) 79 (D) 108
Key: (D)
Exp: From Venn diagram
n(A)  no of persons reading books  13  44  12  7  76
n(B)  no of persons playing  15  44  7  17  83
n(A  B)  51
n(A  B)  n(A)  n(B)  n(A  B)  76  83  51  108

7. Social science disciplines were in existence in an amorphous form until the colonial period
when they were institutionalized. In varying degrees, they were intended to further the colonial
interest. In the time of globalization and the economic rise of postcolonial countries like India,
conventional ways of knowledge production have become obsolete.
Which of the following can be logically inferred from the above statements?
(i) Social science disciplines have become obsolete.
(ii) Social science disciplines had a pre-colonial origin.
(iii) Social science disciplines always promote colonialism.
(iv) Social science must maintain disciplinary boundaries.
(A) (ii) only (B) (i) and (iii) only
(C) (ii) and (iv) only (D) (iii) and (iv) only
Key: (A)

8. Two and a quarter hours back, when seen in a mirror, the reflection of a wall clock without
number markings seemed to show 1:30. What is the actual current time shown by the clock?
(A) 8:15 (B) 11:15 (C) 12:15 (D) 12:45
Key: (D)
Exp: If reflection is seen as Actual will be

1: 30
10 : 30
Thus present time will be 10:30  2:15 12: 45

 ICP–Intensive Classroom Program  eGATE-Live Internet Based Classes DLP  TarGATE-All India Test Series
Leaders in GATE Preparation  65+ Centers across India
© All rights reserved by Gateforum Educational Services Pvt. Ltd. No part of this booklet may be reproduced or utilized in any form without the written permission.

2
|ME| GATE-2016-PAPER-03 www.gateforum.com

9. M and N start from the same location. M travels 10 km East and then 10 km North-East. N
travels 5 km South and then 4 km South-East. What is the shortest distance (in km) between
M and N at the end of their travel?
(A) 18.60 (B) 22.50 (C) 20.61 (D) 25.00
Key: (C)

10. A wire of length 340 mm is to be cut into two parts. One of the parts is to be made into a
square and the other into a rectangle where sides are in the ratio of 1:2. What is the length of
the side of the square (in mm) such that the combined area of the square and the rectangle is a
MINIMUM?
(A) 30 (B) 40 (C) 120 (D) 180

Key: (B)
Exp: x  y  340
 x 2x 
Perimeter of rectangle  2     2x
3 3 
Perimeter of square  340  2x x 2 x/ 3

340  2x
Length of square 
4 x x/ 3
2
 340  2x  2 2
Totalarea     x  f (x)
 4  9
Square Rectangle
4 2x  340
f '(x)  x  0
9 4
4 1
 x   340  2x   x  90
9 4
340  2x
Length of square   40mm
4
Mechanical Engineering
Q. No. 1 – 25 Carry One Mark Each

1 A real square matrix A is called skew-symmetric if


(A) AT = A
(B) AT = A-1
(C) AT = A
(D) AT = A+A-1
Key: (C)

log e (1  4 x)
2 lt is equal to
x 0 e3x  1
1 4
(A) 0 (B) (C) (D) 1
12 3

Key: (C)

 ICP–Intensive Classroom Program  eGATE-Live Internet Based Classes DLP  TarGATE-All India Test Series
Leaders in GATE Preparation  65+ Centers across India
© All rights reserved by Gateforum Educational Services Pvt. Ltd. No part of this booklet may be reproduced or utilized in any form without the written permission.

3
|ME| GATE-2016-PAPER-03 www.gateforum.com

log e (1  4x) 0


Exp: lim  
x 0 e3x  1 0
1
.4
lim 1 3x4x 
4 4

x 0 e .3 (1  4.0)e .3 3
0

3. Solutions of Laplace’s equation having continuous second-order partial derivatives are called
(A) biharmonic functions
(B) harmonic functions
(C) conjugate harmonic functions
(D) error functions
Key: (B)

4. The area (in percentage) under standard normal distribution curve of random variable Z within
limits from −3 to +3 is _____
Key: 99.74
Exp:
3  99.74%

0
1  68.4% 
2  95.45%

5. The root of the function f(x) = x3+x1 obtained after first iteration on application of Newton-
Raphson scheme using an initial guess of x0=1 is
(A) 0.682 (B) 0.686 (C) 0.750 (D) 1.000
Key: (C)
f  xn 
Exp: We have x n 1  x n 
f  xn 
f  x0 
For n=0, x1  x 0 
f  x0 
f (x)  x 3  x  1  f (x)  3x 2  1
given x 0  1
f  x 0   f (1)  1, f   x 0   f (1)  4
1 3
 x1  1    0.75
4 4

 ICP–Intensive Classroom Program  eGATE-Live Internet Based Classes DLP  TarGATE-All India Test Series
Leaders in GATE Preparation  65+ Centers across India
© All rights reserved by Gateforum Educational Services Pvt. Ltd. No part of this booklet may be reproduced or utilized in any form without the written permission.

4
|ME| GATE-2016-PAPER-03 www.gateforum.com

6. A force F is acting on a bent bar which is clamped at one end as shown in the figure.

The CORRECT free body diagram is

(A) (B)

(C) (D)

Key: (A)

7. The cross-sections of two solid bars made of the same material are shown in the figure. The
square cross-section has flexural (bending) rigidity I1, while the circular cross-section has
flexural rigidity I2. Both sections have the same cross-sectional area. The ratio I1/I2 is
(A) 1/ 
(B) 2 / 
(C)  / 3
(D)  / 6
Key: (C)
Exp: Flexural rigidity = EI
Both have same cross-section area

 a 2  d2
4
Where a is side of square and d is diameter of circle.
a4 2 d 4
E1 
 4 I 2
12  12  16  
 a4  d  1 ( E1  E2 because of same material)
16 I2 d 4  4 3
E2  d
64 64

 ICP–Intensive Classroom Program  eGATE-Live Internet Based Classes DLP  TarGATE-All India Test Series
Leaders in GATE Preparation  65+ Centers across India
© All rights reserved by Gateforum Educational Services Pvt. Ltd. No part of this booklet may be reproduced or utilized in any form without the written permission.

5
|ME| GATE-2016-PAPER-03 www.gateforum.com

8. The state of stress at a point on an element is shown in figure (a). The same state of stress is
shown in another coordinate system in figure (b).
The components (xx,yy,xy) are given by


(A) p / 2,  p / 2, 0 
(B) (0, 0, p)

(C)  p, p, p / 2 
(D)  0,0, p / 2 
Key: (B) a  b
Exp: We know,
 x   y   x   y 
     cos 2  xy sin 2
 2   2 
Where,  is the location of any oblique plane which making an angle  in CCW direction.
 When   45o ,   xx ,  x  p,  y  p

pp pp
  xx      cos90  0
o
 2   2 
When   45o ,  yy

pp pp
      cos90  0
o
 2   2 
When   45o ,   xy

 x   y 
We know     sin 2  xy cos 2
 2 
pp
   xy    sin 90  p xx , yy , xy is 0, 0,p
 2 

9. A rigid link PQ is undergoing plane motion as shown in the figure (VP and VQ are non-zero). VQP
is the relative velocity of point Q with respect to point P.

Which one of the following is TRUE?


(A) VQP has components along and perpendicular to PQ
(B) VQP has only one component directed from P to Q
(C) VQP has only one component directed from Q to P
(D) VQP has only one component perpendicular to PQ
Key: (D)
 ICP–Intensive Classroom Program  eGATE-Live Internet Based Classes DLP  TarGATE-All India Test Series
Leaders in GATE Preparation  65+ Centers across India
© All rights reserved by Gateforum Educational Services Pvt. Ltd. No part of this booklet may be reproduced or utilized in any form without the written permission.

6
|ME| GATE-2016-PAPER-03 www.gateforum.com

Exp: Let VP & VQ make an angle  and  with axis of link PQ respectively.

VQ cos Q

 VQ sin 
VQ sin 
VQ VQ cos
VP sin  VP sin 

VP
 P

VP cos VP cos

Since link PQ is rigid, so the distance between P & Q will never change. Hence relative
velocity between P & Q along axial direction should be zero.
Vp cos   VQ sin 
VQ sin 
Relative velocity between P & Q
 VP sin   PQ r to PQ
VP sin 
 VQ sin  PQ r to PQ
  VP sin   VQ sin   PQ r to PQ

10. The number of degrees of freedom in a planar mechanism having n links and j simple hinge
joints is
(A) 3(n  3)  2j (B) 3(n  1)  2j (C) 3n  2j (D) 2j  3n  4
Key: (B)
Exp: DOF F  3 n  1  2j
Where, n  total number of links
j = Effective number of binary points

11. The static deflection of a spring under gravity, when a mass of 1 kg is suspended from it, is 1
mm. Assume the acceleration due to gravity g =10 m/s2. The natural frequency of this spring-
mass system (in rad/s) is ______
Key: (100)
Exp: st  103 m
g 10
n    100 rad/sec
st 103

12. Which of the bearings given below SHOULD NOT be subjected to a thrust load?
(A) Deep groove ball bearing
(B) Angular contact ball bearing
(C) Cylindrical (straight) roller bearing
(D) Single row tapered roller bearing
Key: (C)

 ICP–Intensive Classroom Program  eGATE-Live Internet Based Classes DLP  TarGATE-All India Test Series
Leaders in GATE Preparation  65+ Centers across India
© All rights reserved by Gateforum Educational Services Pvt. Ltd. No part of this booklet may be reproduced or utilized in any form without the written permission.

7
|ME| GATE-2016-PAPER-03 www.gateforum.com

13. A channel of width 450 mm branches into two sub-channels having width 300 mm and 200
mm as shown in figure. If the volumetric flow rate (taking unit depth) of an incompressible
flow through the main channel is 0.9 m3/s and the velocity in the sub-channel of width 200
mm is 3 m/s, the velocity in the sub-channel of width 300 mm is m/s.
Assume both inlet and outlet to be at the same elevation.

Key: 1
Exp: Apply Mass conservation and taking incompressibility
We have
300mm V2  ?
A1V1  A 2 V2  A3 V3 Given A1V1  0.9m3 /s
450mm 2
0.9  0.3V2  0.2  3
0.9  0.6  0.3V2 1
200mm
 V2  1m / s 3
0.9m /s 3

V3  3m/s

14. For a certain two-dimensional incompressible flow, velocity field is given by 2xyiˆ  y2ˆj . The
streamlines for this flow are given by the family of curves
(A) x 2 y 2  constant (B) xy 2  constant
(C) 2xy  y 2 = constant (D) xy  constant
Key: (B)
Exp: v  2xyiˆ  y 2 ˆj
 
u v
y x
2xy dy  d
On integrating
 = xy 2  f  x 

  y2  f '  x      y2 
x
 y2  f '  x   y2
f '  x   0  f  x   constant
  =xy 2  constant

 ICP–Intensive Classroom Program  eGATE-Live Internet Based Classes DLP  TarGATE-All India Test Series
Leaders in GATE Preparation  65+ Centers across India
© All rights reserved by Gateforum Educational Services Pvt. Ltd. No part of this booklet may be reproduced or utilized in any form without the written permission.

8
|ME| GATE-2016-PAPER-03 www.gateforum.com

15. Steady one-dimensional heat conduction takes place across the faces 1 and 3 of a composite
slab consisting of slabs A and B in perfect contact as shown in the figure, where kA , kB denote
the respective thermal conductivities. Using the data as given in the figure, the interface
temperature T2 (in °C) is .

Key: 67.5
T1  T2 T2  T3
Exp: 
LA LB
kA  A kB  A
130  T2 T2  30

0.1 0.3
20 100
390  3T2  5T2  150
8T2  540
T2  67.5o C
 Interface temperature  67.5o C

16. Grashof number signifies the ratio of


(A) inertia force to viscous force
(B) buoyancy force to viscous force
(C) buoyancy force to inertia force
(D) inertia force to surface tension force
Key: (B)
Inertia force  Buoyant force
Exp: Grashof number (Gr) 
(Viscous force) 2

17. The INCORRECT statement about the characteristics of critical point of a pure substance is
that
(A) there is no constant temperature vaporization process
(B) it has point of inflection with zero slope
(C) the ice directly converts from solid phase to vapor phase
(D) saturated liquid and saturated vapor states are identical
Key: (C)
Exp: The process of conversion from solid phase to vapour phase is called sublimation and this
does not happen at critical point. All the other statements are true at Critical point.

 ICP–Intensive Classroom Program  eGATE-Live Internet Based Classes DLP  TarGATE-All India Test Series
Leaders in GATE Preparation  65+ Centers across India
© All rights reserved by Gateforum Educational Services Pvt. Ltd. No part of this booklet may be reproduced or utilized in any form without the written permission.

9
|ME| GATE-2016-PAPER-03 www.gateforum.com

18. For a heat exchanger, ∆Tmax is the maximum temperature difference and ∆Tmin is the minimum
temperature difference between the two fluids. LMTD is the log mean temperature difference.
Cmin and Cmax are the minimum and the maximum heat capacity rates. The maximum possible
heat transfer (Qmax) between the two fluids is
(A) Cmin LMTD (B) Cmin ∆Tmax (C) Cmax ∆Tmax (D) Cmax ∆Tmin
Key: (B)
Exp: In a heat exchanger, maximum possible heat transfer will be.
Qmax  Cmin Tmax

19. The blade and fluid velocities for an axial turbine are as shown in the figure.

The magnitude of absolute velocity at entry is 300 m/s at an angle of 65 to the axial direction,
while the magnitude of the absolute velocity at exit is 150 m/s. The exit velocity vector has a
component in the downward direction. Given that the axial (horizontal) velocity is the same at
entry and exit, the specific work (in kJ/kg) is _____
Key: 52.80
Exp: Given:   90  65  25o.
Let ‘1’ & ‘2’ denotes inlet & outlet of vane.
Vane velocity (u) = u1  u 2  150m / s.
Inlet:
Velocity of whirling Vw1  V1 cos  
 300cos25  271.89m / s

 
Velocity of flow Vf1  V1 sin 
 300sin 25  126.7854m / s
 
Absolute velocity of Inlet = Vw1  Vf1
It is given that horizontal velocity is same at entry & exit.
So,
Vf1  Vf 2
126.7859  V2 cos 

126.7854 
  cos 1    32.3o
 150 
Vw2  V2 sin(32.3)  80.16m / s

 ICP–Intensive Classroom Program  eGATE-Live Internet Based Classes DLP  TarGATE-All India Test Series
Leaders in GATE Preparation  65+ Centers across India
© All rights reserved by Gateforum Educational Services Pvt. Ltd. No part of this booklet may be reproduced or utilized in any form without the written permission.

10
|ME| GATE-2016-PAPER-03 www.gateforum.com
u  150m / s
Vf2


V2 Vw 2
Vr2
Vf1

u2

Vr1
Vw1 V1

u1

65

Absolute velocity at outlet


  
V2  Vf2  Vw 2
Specific work done on the fluid per unit weight (wsp )
 
 wsp   u. Vw2  Vw1
 
Let ĵ unit vector positive toward upward.
()Ve

Vw1  271.89m / sec ()Ve

u  150m / sec

Vw2  80.16

()Ve


w  (150 ˆj). (271.89 ˆj)  (80.16 ˆj)  52.8075kJ / kg 
20. Engineering strain of a mild steel sample is recorded as 0.100%. The true strain is
(A) 0.010% (B) 0.055% (C) 0.099% (D) 0.101%
Key: (C)
0.1
Exp: 
100
We know,
T  ln 1    , where T is True strain and  is Engineering strain
 0.1 
 T  ln 1    0.0009995 T %  0.0009995 100  0.099%
 100 

 ICP–Intensive Classroom Program  eGATE-Live Internet Based Classes DLP  TarGATE-All India Test Series
Leaders in GATE Preparation  65+ Centers across India
© All rights reserved by Gateforum Educational Services Pvt. Ltd. No part of this booklet may be reproduced or utilized in any form without the written permission.

11
|ME| GATE-2016-PAPER-03 www.gateforum.com

21. Equal amounts of a liquid metal at the same temperature are poured into three moulds made of
steel, copper and aluminum. The shape of the cavity is a cylinder with 15 mm diameter. The
size of the moulds are such that the outside temperature of the moulds do not increase
appreciably beyond the atmospheric temperature during solidification. The sequence of
solidification in the mould from the fastest to slowest is
(Thermal conductivities of steel, copper and aluminum are 60.5, 401 and 237 W/m-K,
respectively. Specific heats of steel, copper and aluminum are 434, 385 and 903 J/kg-K,
respectively.
Densities of steel, copper and aluminum are 7854, 8933 and 2700 kg/m3, respectively.)
(A) Copper - Steel - Aluminum
(B) Aluminum - Steel – Copper
(C) Copper - Aluminum - Steel
(D) Steel - Copper - Aluminum
Key: (C)
Exp: K steel  60.5; steel  7854; Csteel  434 J/kg K
K copper  401; copper  8933; Ccopper  385 J/kg K
K A1  237; A1  2700; CAl  903 J/kg K

Heat capacity c


 c steel  3408.636 kJ/m3K
 c copper  3439.205 kJ/m3K
 c A1  2438.100 kJ/m3K
 k  60.5
  steel     103  0.0177 103 m 2 /s

 steel
c 3408
 k  401
  copper     103  0.1166 103 m 2 /s

 copper
c 3439
 k  23.7
  A1     103  0.1119 103 m 2 /s

 A1
c 2438
So, cooling rates would be copper, Aluminium, steel

22. In a wire-cut EDM process the necessary conditions that have to be met for making a
successful cut are that
(A) wire and sample are electrically non-conducting
(B) wire and sample are electrically conducting
(C) wire is electrically conducting and sample is electrically non-conducting
(D) sample is electrically conducting and wire is electrically non-conducting
Key: (B)
Exp: In this process, a thin metallic wire is fed on to the conducting workpiece, which is submerged
in a tank of dielectric fluid such as de-ionized water. Wire is fed in the programmed path &
material is cut from the workpiece accordingly. Material removal takes place by a series of
discrete discharges between the wire electrode & workpiece in the presence of a dielectric
fluid. The di-electric fluid gets ionized in between the tool electrode gap thereby creating a

 ICP–Intensive Classroom Program  eGATE-Live Internet Based Classes DLP  TarGATE-All India Test Series
Leaders in GATE Preparation  65+ Centers across India
© All rights reserved by Gateforum Educational Services Pvt. Ltd. No part of this booklet may be reproduced or utilized in any form without the written permission.

12
|ME| GATE-2016-PAPER-03 www.gateforum.com

path for each discharge. The area wherein discharge takes place gets heated to very high
temperature such that the surface get melted & removed. The cut particles (debris) get flushed
away by continuous flowing dielectric fluid.
Generally, wire-cut EDM is used for cutting Aluminium, brass, etc. & wire material used for
quicker cutting action is zinc coated brass wires.

23. Internal gears are manufactured by


(A) hobbing (B) shaping with pinion cutter
(C) shaping with rack cutter (D) milling
Key: (B)

24. Match the following part programming codes with their respective functions

Part Programming Codes Functions

P. G01 I. Spindle stop

Q. G03 II. Spindle rotation, clockwise

R. M03 III. Circular interpolation,


anticlockwise
S. M05 IV. Linear interpolation

(A) P – II, Q – I, R – IV, S – III


(B) P – IV, Q – II, R – III, S – I
(C) P – IV, Q – III, R – II, S – I
(D) P – III, Q – IV, R – II, S – I

Key: (C)

25. In PERT chart, the activity time distribution is


(A) Normal (B) Binomial (C) Poisson (D) Beta
Key: (D)

Q. No. 26 – 55 carry Two Marks Each


2 1 0
26.
 
The number of linearly independent eigenvectors of matrix A   0 2 0  is ____ .
0 0 3
Key: 2
Exp: Here   2, 2,3

For   2, No. of L.I eigen vectors

 3  rank of  A  2I   3  2  1
For   3, No. of L.I eigen vectors =1
∴ Total L.I eigen vectors = 2

 ICP–Intensive Classroom Program  eGATE-Live Internet Based Classes DLP  TarGATE-All India Test Series
Leaders in GATE Preparation  65+ Centers across India
© All rights reserved by Gateforum Educational Services Pvt. Ltd. No part of this booklet may be reproduced or utilized in any form without the written permission.

13
|ME| GATE-2016-PAPER-03 www.gateforum.com

4
27. The value of the line integral  F.r ds, where C is a circle of radius

units is .
C

Here, F(x, y)  yiˆ  2xjˆ and r is the UNIT tangent vector on the curve C at an arc length s
from a reference point on the curve. î and ĵ are the basis vectors in the x-y Cartesian
reference. In evaluating the line integral, the curve has to be traversed in the counter-clockwise
direction.

Key: 16
Exp: By Green’s theorem,

 F.r ' ds   ydx  2xdy    2  1 dxdy


c c
2
 4 
   16
 

28. lim x  x 2  x  1  x is
(A) 0 (B) ∞ (C) 1/2 (D) −∞
Key: (C)

Exp: lim
x 
 x2  x 1  x   x2  x 1  x
x2  x 1  x
x2  x 1  x2
 lim
x2  x 1  x
x 

 1
x 1  
 x 1 0 1
 lim  
x  1 1 1 0  0  1 2
x 1  2 1
x x

29. Three cards were drawn from a pack of 52 cards. The probability that they are a king, a queen,
and a jack is
16 64 3 8
(A) (B) (C) (D)
5525 2197 13 16575

Key: (A)
4
C1  4C1  4C1 64 16
Exp: Required probability  52
 
C3 22100 5525

30. An inextensible massless string goes over a frictionless pulley. Two


weights of 100 N and 200 N are attached to the two ends of the
string. The weights are released from rest, and start moving due to
gravity. The tension in the string (in N) is .
Key: 133.33
200N

100N

 ICP–Intensive Classroom Program  eGATE-Live Internet Based Classes DLP  TarGATE-All India Test Series
Leaders in GATE Preparation  65+ Centers across India
© All rights reserved by Gateforum Educational Services Pvt. Ltd. No part of this booklet may be reproduced or utilized in any form without the written permission.

14
|ME| GATE-2016-PAPER-03 www.gateforum.com

Exp: From F.B.D


200
200  T  a …(1)
g

100
T  100  a …(2)
g
Adding equations (1) and (2) T
T
300 a
100  a  a  g/3 200
g
From equation (1) a 100
200 2 200
200  T    F.B.D F.B.D
g 3 3
T T
200
T  200   133.33N
3 a a

200 100

31. A circular disc of radius 100 mm and mass 1 kg, initially at rest at position A, rolls without
slipping down a curved path as shown in figure. The speed v of the disc when it reaches
position B is m/s.

Acceleration due to gravity g = 10 m/s2.


Key: 20
Exp: According to energy conservation principle
Total K.E + P.E = constant
 K.E A  P.E A  K.E B  P.E B
1 1
 0  mg.30  m  v 2  I2  0
2 2
1 1 1
 30 10  1 v 2    1  v 2
2 2 2
 mr 2
v
I  and   
 2 r
3 2
 30 10   v  v  20m/s
4

 ICP–Intensive Classroom Program  eGATE-Live Internet Based Classes DLP  TarGATE-All India Test Series
Leaders in GATE Preparation  65+ Centers across India
© All rights reserved by Gateforum Educational Services Pvt. Ltd. No part of this booklet may be reproduced or utilized in any form without the written permission.

15
|ME| GATE-2016-PAPER-03 www.gateforum.com

32. A rigid rod (AB) of length L  2 m is undergoing translational as well as rotational motion
in the x-y plane (see the figure). The point A has the velocity V1  ˆi  2ˆjm / s. The end B is
constrained to move only along the x direction.
The magnitude of the velocity V2 (in m/s) at the end B is
Key: 3
Exp: v1  i  2j V 5 1
 v x  1 and v y  2 2
V2 cos 45
v1  1  2  5
2 2 =α+45°
45 V2
1
Let ϕ is angle between v1 and x-axis.
 tan   2   63.43 V1
V2 sin 45
   45  63.43    18.43
V1 sin  V1 cos
Since, the rod is rigid 
 v1 cos   v 2 cos 45 45

 5 cos18.43  v 2 cos 45  v 2  3m/s

33. A square plate of dimension L × L is subjected to a uniform pressure load p = 250 MPa on
its edges as shown in the figure. Assume plane stress conditions. The Young’s modulus E =
200 GPa.

The deformed shape is a square of dimension L  2. If L  2 m and   0.001 m, the


Poisson’s ratio of the plate material is ______
Key: 0.2
Exp: According to Hooke’s Law
 y
x  x 
E E
Where,  x   y   p and  x 
2

2  p p 
    
 E E 
21 p 250
  1     1       0.2
2000 E 2  105

 ICP–Intensive Classroom Program  eGATE-Live Internet Based Classes DLP  TarGATE-All India Test Series
Leaders in GATE Preparation  65+ Centers across India
© All rights reserved by Gateforum Educational Services Pvt. Ltd. No part of this booklet may be reproduced or utilized in any form without the written permission.

16
|ME| GATE-2016-PAPER-03 www.gateforum.com

34. Two circular shafts made of same material, one solid (S) and one hollow (H), have the same
length and polar moment of inertia. Both are subjected to same torque. Here, S is the twist
and S is the maximum shear stress in the solid shaft, whereas H is the twist and H is the
maximum shear stress in the hollow shaft. Which one of the following is TRUE?
(A) s  H and s  H (B) s  H and s  H
(C) s  H and s  H (D) s  H and s  H
Key: (D)
Exp: According to pure torsion equation
T  G
 
J R 
Let ds and d1 are diameter of solid shaft and outer diameter of hollow shaft
Ts s

Js R s  
 s  h
Th h Rs Rh

Jh R h
  d
 s  h  s   h . s
d s d1 d1
Since, J s  J h
 d1 must be greater than ds
ds
 must be less than 1  s  h and
d1
Ts G s s

T G Js s
   s  h
J  T G 
h
 h h
Jh h

35. A beam of length L is carrying a uniformly distributed load w per unit length. The flexural
rigidity of the beam is EI. The reaction at the simple support at the right end is _____ .

wL 3wL wL wL
(A) (B) (C) (D)
2 8 4 8
Key: (B)

Exp: 

A B 

RB


2
A B 
1
RB
 ICP–Intensive Classroom Program  eGATE-Live Internet Based Classes DLP  TarGATE-All India Test Series
Leaders in GATE Preparation  65+ Centers across India
© All rights reserved by Gateforum Educational Services Pvt. Ltd. No part of this booklet may be reproduced or utilized in any form without the written permission.

17
|ME| GATE-2016-PAPER-03 www.gateforum.com

4 R 3
1  ; 2  B
8EI 3EI
Since, net vertical deflection at B is zero
 1  2
 4 R B3 3
   RB 
8EI 3EI 8

36. Two masses m are attached to opposite sides of a rigid rotating shaft in the vertical plane.
Another pair of equal masses m1 is attached to the opposite sides of the shaft in the vertical
plane as shown in figure. Consider m = 1 kg, e = 50 mm, e1 = 20 mm, b = 0.3 m, a = 2 m and
a1 = 2.5 m. For the system to be dynamically balanced, m1 should be kg.

Key: 2
Exp: Couple due to m = couple due to m1
mea  m1e1a1
 50   2 
m1  1        2kg
 20   2.5 

37. A single degree of freedom spring-mass system is subjected to a harmonic force of constant
3k
amplitude. For an excitation frequency of , the ratio of the amplitude of steady state
m
response to the static deflection of the spring is .

Key: 0.5
1 1 1
Exp: M.F      0.5
  1 3 2
2
1 
n

 ICP–Intensive Classroom Program  eGATE-Live Internet Based Classes DLP  TarGATE-All India Test Series
Leaders in GATE Preparation  65+ Centers across India
© All rights reserved by Gateforum Educational Services Pvt. Ltd. No part of this booklet may be reproduced or utilized in any form without the written permission.

18
|ME| GATE-2016-PAPER-03 www.gateforum.com

38. A bolted joint has four bolts arranged as shown in figure. The cross sectional area of each bolt
is 25 mm2. A torque T = 200 N-m is acting on the joint. Neglecting friction due to clamping
force, maximum shear stress in a bolt is MPa.

Key: 40
Exp: Let the resisting force in each bolt = F Newton
3
Net resisting torque (TR) = 4F  50 10 N-m
Applied torque (T) = Resisting Torque
T  4F  50103
200
F 1000  1000N
4  50
Let shear stress developed in each bolt =  MPa
It is given that resisting area (AR) = 25mm2
F    AR
1000
  40MPa
25

39. Consider a fully developed steady laminar flow of an incompressible fluid with viscosity µ
through a circular pipe of radius R. Given that the velocity at a radial location of R/2 from the
centerline of the pipe is U1, the shear stress at the wall is kµU1/R, where K is
Key: 2.667
Exp: Given, ,R u at R/2  U1 ,
y wall  ku1 /R
K=?
Velocity profile in horizontal pipe flow is
 r2 
u  u max 1  2 
 R 
 R2 
u1  u max 1  2 
 4R 
3 4
u1  u max  u max  u1
4 3

 u
y wall   
 r r  R
2r 2 2 u max 2 4
   max 2
  u max    u1
R R R R 3
8 u1 8
  k   2.667
3 R 3
 ICP–Intensive Classroom Program  eGATE-Live Internet Based Classes DLP  TarGATE-All India Test Series
Leaders in GATE Preparation  65+ Centers across India
© All rights reserved by Gateforum Educational Services Pvt. Ltd. No part of this booklet may be reproduced or utilized in any form without the written permission.

19
|ME| GATE-2016-PAPER-03 www.gateforum.com

40. The water jet exiting from a stationary tank


through a circular opening of diameter 300 mm
impinges on a rigid wall as shown in the figure.
Neglect all minor losses and assume the water
level in the tank to remain constant. The
net horizontal force experienced by the
wall is kN.
Density of water is 1000 kg/m3.
Acceleration due to gravity g = 10 m/s2

Key: 8.76
Exp: Force exerted by a jet of water striking fixed wall is av 2 N where a is area of jet.


 103   0.32  v 2  v  2gh  2 10  6.2
4


103   0.32  2  10  62  8.76kN
4

 x ˆ y ˆ
41. For a two-dimensional flow, the velocity field is u  i 2 j, where ˆi and ˆj are the
x y
2 2
x  y2
basis vectors in the x-y Cartesian coordinate system. Identify the CORRECT statements from
below.
(1) The flow is incompressible.
(2) The flow is unsteady.
y
(3) y-component of acceleration, a y 
 
2
x  y2
2

  x  y
(4) x-component of acceleration, a x 
x  y2 
2 2

(A) (2) and (3) (B) (1) and (3) (C) (1) and (2) (D) (3) and (4)

Key: (B)
x y
Exp: u ; v
x 2  y2 x 2  y2
 Given flow is independent of time of flow is steady
 Density ‘ρ’ is not present in velocity components, so flow is incompressible
Acceleration along x direction
u u
ax  u v
x y
x
 u
x  y2
2

u  x  y   x2x
2 2
y2  x 2 u 2xy
    
x x  y   x  y  y  x  y2 
2 2 2 2 2 2 2 2

 ICP–Intensive Classroom Program  eGATE-Live Internet Based Classes DLP  TarGATE-All India Test Series
Leaders in GATE Preparation  65+ Centers across India
© All rights reserved by Gateforum Educational Services Pvt. Ltd. No part of this booklet may be reproduced or utilized in any form without the written permission.

20
|ME| GATE-2016-PAPER-03 www.gateforum.com

u
 u  2
x  y  x   xy  x
2 2 2 3
…(a)
x x  y2 x  y  x  y 
2 2 2 2 2 3

u y 2xy 2xy 2
 v  2  …(b)
y  x  y2   x 2  y 2 2  x 2  y 2 3

xy2  x3   2xy2   x  x 2  y2  x
ax  a  b   
x 
2 3
x 
2 3
x  y2 
2
2
y 2
y 2

So, only statement 1 & 3 are correct.

42. Two large parallel plates having a gap of 10 mm in between them are maintained at
temperatures.
T1 = 1000 K and T2 = 400 K. Given emissivity values, 1  0.5, 2  0.25 and Stefan-
Boltzmann constant  = 5.67 × 10−8 W/m2-K4, the heat transfer between the plates (in
kW/m2) is
Key: 11.049
  T14  T24 
Exp: Q1 2  T1  1000K
1 1 T2  400K
 1
1 2
5.67  108 10004  4004 

1

1
1 1  0.5 2  0.25
0.5 0.25
 11049.696 W 2  11.049 kW 2
m m

43. A cylindrical steel rod, 0.01 m in diameter and 0.2 m in length is first heated to 750 0 C and
then immersed in a water bath at 1000C. The heat transfer coefficient is 250 W/m2-K. The
density, specific heat and thermal conductivity of steel are =7801 kg/m3, c = 473
J/kg-K, and k = 43 W/m-K, respectively. The time required for the rod to reach 3000C is
____ seconds.
Key: 43.49
Exp: d = 0.01 m
L = 0.2m, R=7801 kg/m3.
t i  750o C, C  473J / kgK
t   100o C, k  43 W / mK
 2
d
V 4 d
h  250 W / m K, Lc   2

A d 4
 t  t   hA h
ln     
 t i  t   Vc cLc
 t  t   h  4
ln   
 t i  t   c  d
 300  100  250  4
ln       43.49s
 750  100  7801 473  0.01

 ICP–Intensive Classroom Program  eGATE-Live Internet Based Classes DLP  TarGATE-All India Test Series
Leaders in GATE Preparation  65+ Centers across India
© All rights reserved by Gateforum Educational Services Pvt. Ltd. No part of this booklet may be reproduced or utilized in any form without the written permission.

21
|ME| GATE-2016-PAPER-03 www.gateforum.com

44. Steam at an initial enthalpy of 100 kJ/kg and inlet velocity of 100 m/s, enters an
insulated horizontal nozzle. It leaves the nozzle at 200 m/s. The exit enthalpy (in kJ/kg)
is .
Key: 85 1 2
Exp: h1  100kJ / kg
C1  100m / s
C1 C2
C2  200m / s
h1 h2
2 2
C C
h1   h2 
1 2
2000 2000
1002 2002
h 2  100    85kJ / kg
2000 2000

45. In a mixture of dry air and water vapor at a total pressure of 750 mm of Hg, the partial
pressure of water vapor is 20 mm of Hg. The humidity ratio of the air in grams of water vapor
per kg of dry air (gw/kgda) is ____ .
Key: 17
Exp: Pt  750 mm of Hg
Pv  20 mm of Hg
Humidity ratio (or) specific Humidity
PV
w  0.622
Pt  PV
 20 
 0.622   
 750  20 
kg g
 0.017 w.v  17 w.v
kg d.a kg d.a

46. In a 3-stage air compressor, the inlet pressure is p1 , discharge pressure is p4 and the
intermediate pressures are p2 and p3  p2  p3 . The total pressure ratio of the compressor is 10
and the pressure ratios of the stages are equal. If p1  100 kPa, the value of the pressure p3 (in
kPa) is .
Key: 464.16
Exp: 3 stage compressor, P1= 100 kPa
Pressure ratios of all stages are equal
P2 P3 P4
 
P1 P2 P3
P4
Overall pressure ratio   10
P1
P2 P3 P4 3
 rp     10  2.154
P1 P2 P3
P3  (rp )2  P1
 P3  2.514  2.514 100  464.16kPa

 ICP–Intensive Classroom Program  eGATE-Live Internet Based Classes DLP  TarGATE-All India Test Series
Leaders in GATE Preparation  65+ Centers across India
© All rights reserved by Gateforum Educational Services Pvt. Ltd. No part of this booklet may be reproduced or utilized in any form without the written permission.

22
|ME| GATE-2016-PAPER-03 www.gateforum.com

47. In the vapour compression cycle shown in the figure, the


evaporating and condensing temperatures are 260 K and 310 K,
respectively. The compressor takes in liquid-vapour mixture (state
1) and isentropically compresses it to a dry saturated vapour
condition (state 2). The specific heat of the liquid refrigerant is
4.8kJ/kg-K and may be treated as constant. The enthalpy of
evaporation for the refrigerant at 310 K is 1054 kJ/kg.
The difference between the enthalpies at state points 1 and 0 (in kJ/kg) is
Key: 1103.51
Exp: dQ  CpdT
3
Cp dT h fg T
s 2  s0   
0 T 310
3 2
 310  1054
s1  s0  4.8ln   ( s1  s 2 )
 260  310 0
4 1
h1  h 0  310  1054
 4.8ln   s
260  260  310
 h1  h 0  1103.51kJ kg

48. Spot welding of two steel sheets each 2 mm thick is carried out successfully by passing 4 kA
of current for 0.2 seconds through the electrodes. The resulting weld nugget formed between
the sheets is 5 mm in diameter. Assuming cylindrical shape for the nugget, the thickness of the
nugget is mm.

Latent heat of fusion for steel 1400 kJ/kg


Effective resistance of the weld joint 200 
Density of steel 8000 kg/m3
Key: 2.91
Exp: t s  2mm
I  4kA; t  0.2
d  5mm; t n  ?
L.H 1400
R  200
  8000 kg m3
Energy supplied = I2 Rt …(a)

 
2
 4  103  200  106  0.2  640J

Energy required for melting =  V   L.H  … (b)


 1400  103  8000   52  106  t n  219911.4858t n
4
Equating (a) & (b)
640 = 219911.4858t n
t n  2.91mm
 ICP–Intensive Classroom Program  eGATE-Live Internet Based Classes DLP  TarGATE-All India Test Series
Leaders in GATE Preparation  65+ Centers across India
© All rights reserved by Gateforum Educational Services Pvt. Ltd. No part of this booklet may be reproduced or utilized in any form without the written permission.

23
|ME| GATE-2016-PAPER-03 www.gateforum.com

49. For an orthogonal cutting operation, tool material is HSS, rake angle is 22°, chip thickness is
0.8 mm, speed is 48 m/min and feed is 0.4 mm/rev. The shear plane angle (in degrees) is
(A) 19.24 (B) 29.70 (C) 56.00 (D) 68.75
Key: (B)
Exp: Given o  22
Thickness of chip = 0.8mm  t 2 
Feed = 0.4 mm/rev
Speed = 48m/min
In orthogonal cutting = feed = thickness of uncut chip.  t1 
t 0.8
K 2 2
t1 0.4
cos o cos 22
tan     0.57
K  sin o 2  sin 22
  tan 1  0.57   29.7

50. In a sheet metal of 2 mm thickness a hole of 10 mm diameter needs to be punched. The yield
strength in tension of the sheet material is 100 MPa and its ultimate shear strength is 80 MPa.
The force required to punch the hole (in kN) is
Key: 5.0265
Exp: Given
t  2mm
d  10mm
s  80MPa

Syt  100MPa
F  dt  s   10  2  80  5026.5N  5.0265kN

51. In a single point turning operation with cemented carbide tool and steel work piece, it is found
that the Taylor’s exponent is 0.25. If the cutting speed is reduced by 50% then the tool life
changes by ____ times
Key: 16
Exp: n = 0.25
V
V2  1
2
V1T1n  V2T2n
V
V1T1n  1 T2n
2
0.25
 T2  2
 T
 1

T2  T1  24
T2  16  T1
Tool life changes by 16 times

 ICP–Intensive Classroom Program  eGATE-Live Internet Based Classes DLP  TarGATE-All India Test Series
Leaders in GATE Preparation  65+ Centers across India
© All rights reserved by Gateforum Educational Services Pvt. Ltd. No part of this booklet may be reproduced or utilized in any form without the written permission.

24
|ME| GATE-2016-PAPER-03 www.gateforum.com

52. Two optically flat plates of glass are kept at a small angle  as shown in the figure.
Monochromatic light is incident vertically.

If the wavelength of light used to get a fringe spacing of 1 mm is 450 nm, the wavelength of
light (in nm) to get a fringe spacing of 1.5 mm is _______.
Key: 675
Exp: Fringe space by (d1) = 1mm
h
1  450nm t

d2  1.5mm, 2  ?
x
1 
tan    2 L
2d1 2d 2
 1.5 
 2  450     675nm
 1 

53. A point P (1, 3, −5) is translated by 2iˆ  3jˆ  4kˆ and then rotated counter clockwise by 90
about the z-axis. The new position of the point is
(A) (−6, 3, −9) (B) (−6, −3, −9) (C) (6, 3, −9) (D) (6, 3, 9)
Key: (A)
Exp: We can write vector 2iˆ  3jˆ  4kˆ y

as coordinate form as  2, 3,  4  .

OP  ˆi  3jˆ  5kˆ P 1,3, 5 P'  3,6, 9

OP  3iˆ  6ˆj  9kˆ

Now rotate OP' vector about z axis through 90°
in counter clockwise direction.
Since, it is rotated about z-axis, therefore z-
x
coordinate remains same. 0
x n  x o cos   yo sin 
yn  x o sin   yo cos 
z
Where  x o , yo  are co-ordinates corresponding
to old values
 xn , yn  are co-ordinates corresponding to new values

After rotating vector OP through an angle θ in counter clockwise
x n  3 cos90  6sin 90
x n  6
y n  3sin 90  6cos90
yn  3
Hence the new coordinate is  6, 3,  9

 ICP–Intensive Classroom Program  eGATE-Live Internet Based Classes DLP  TarGATE-All India Test Series
Leaders in GATE Preparation  65+ Centers across India
© All rights reserved by Gateforum Educational Services Pvt. Ltd. No part of this booklet may be reproduced or utilized in any form without the written permission.

25
|ME| GATE-2016-PAPER-03 www.gateforum.com

54. The demand for a two-wheeler was 900 units and 1030 units in April 2015 and May
2015, respectively. The forecast for the month of April 2015 was 850 units. Considering a
smoothing constant of 0.6, the forecast for the month of June 2015 is
(A) 850 units (B) 927 units
(C) 965 units (D) 970 units
Key: (D)
Exp:
Month Demand Forecast    D t 1  1    Ft 1 
April 900 850
May 1030  0.6 900  0.4 850  880
June  0.6 1030  0.4 880  970
 FJune  970 units

55. A firm uses a turning center, a milling center and a grinding machine to produce two parts.
The table below provides the machining time required for each part and the maximum
machining time available on each machine. The profit per unit on parts I and II are Rs. 40 and
Rs. 100, respectively. The maximum profit per week of the firm is Rs. .

Machining time required for


the machine part (minutes) Maximum machining time
Type of machine
available per week (minutes)
I II
Turning Center 12 6 6000
Milling Center 4 10 4000
Grinding Machine 2 3 1800

x2
Key: 40,000
Exp: Linear program formulation is
max z  40x1  100x 2  0, 1000
Constraints are
12x1  6x 2  6000  0, 600
4x1  10x 2  4000
2x1  3x 2  1800  0, 400
x1 , x 2  0
375, 250
At  0,400  , z  40,000
At  375, 250  ,z  40,000 x1
500,0 900,0 1000,0
At  500,0  ,z  20,000
∴ Maximum profit per week is Rs. 40,000.

 ICP–Intensive Classroom Program  eGATE-Live Internet Based Classes DLP  TarGATE-All India Test Series
Leaders in GATE Preparation  65+ Centers across India
© All rights reserved by Gateforum Educational Services Pvt. Ltd. No part of this booklet may be reproduced or utilized in any form without the written permission.

26
|ME| GATE-2017-PAPER-II www.gateforum.com

Mechanical Engineering
Q. No. 1 to 25 Carry One Mark Each

1. A mass m of a perfect gas at pressure p1 and volume V1 undergoes an isothermal process. The
final pressure is p2 and volume is V2. The work done on the system is considered positive. If R
is the gas constant and T is the temperature, then the work done in the process is
V2 p1 V2 p2
(A) p1V1 ln (B) p1V1 ln (C) RT ln (D) mRT ln
V1 p2 V1 p1
Key: (B)
Exp: Isothermal work done, W   pdV
For isothermal, pV= C
p1V1  pV  C  constant 
V2
dV
so, W  p1V1 
V1
V
V  p 
W  p1V1 n  2   p1V1 n  1  ( p1V1  p 2 V2 )
 V1   p2 

2. Which one of the following statements is TRUE for the ultrasonic machining (USM) process?
(A) In USM, the tool vibrates at subsonic frequency.
(B) USM does not employ magnetostrictive transducer.
(C) USM is an excellent process for machining ductile materials.
(D) USM often uses a slurry comprising abrasive-particles and water.
Key: (D)

3. The standard deviation of linear dimensions P and Q are 3 m and 4 m, respectively. When
assembled, the standard deviation (in m ) of the resulting linear dimension (P+Q) is ________
Key: 5 to 5
Exp: Given that
Standard deviate of P is 3 m  Variance of P is 9 m
Standard deviation of Q is 4 m  Variance of Q is 16 m
Variance of P + Q = Var (P+Q) = Variance P + Variance Q
= 9 + 16= 25
 Standard deviation of P + Q = + Variance = + 25 = 5

4. The emissive power of a blackbody is P. If its absolute temperature is doubled, the emissive
power becomes.
(A) 2P (B) 4P (C) 8P (D) 16P
Key: (D)
Exp: Emissive power of black body  E b   T 4

Given  E b 1  P  T 4

 ICP–Intensive Classroom Program  eGATE-Live Internet Based Classes DLP  TarGATE-All India Test Series
Leaders in GATE Preparations  65+ Centers across India
© All rights reserved by Gateforum Educational Services Pvt. Ltd. No part of this booklet may be reproduced or utilized in any form without the written permission.

1
|ME| GATE-2017-PAPER-II www.gateforum.com

now T  2T
 Eb 2    2T   16T 4   E b 2  16P
4

5. The state of stress at a point is x   y  z  xz  zx   yz  zy  0 and  xy   yx  50MPa .


The maximum normal stress (in MPa) at that point is _________
Key: 49.9 to 50.1
Exp: It is a pure torsion case in 2D 

xy   yx  50MPa
 0,  xy

 for pure Torsion, 


xy   yx  1
0,   xy
1
So maximum Normal stress 1  50MPa

6. The determinant of a 2×2 matrix is 50. If one eigenvalue of the matrix is 10, the other
eigenvalue is ___________
Key: 5 to 5
Exp: Given that det of 2×2 Matrix is 50 and are Eigen Value is 10.
 Other Eigen value is 5  det  product of eigenvalues 

7. Which one of the following statement is TRUE?


(A) Both Pelton and Francis turbines are impulse turbines.
(B) Francis turbine is a reaction turbine but Kaplan turbine is an impulse turbine.
(C) Francis turbine is an axial – flow reaction turbine.
(D) Kaplan turbine is an axial – flow reaction turbine.
Key: (D)

8. Two coins are tossed simultaneously. The probability (upto two decimal points accuracy) of
getting at least one head is ____________
Key: 0.75 to 0.75
Exp: Total No of outcomes when two coins are tossed is 4 and sample space
S  HH, HT,TH,TT
Favorable out comes for existence of at least one head are HH, HT, TH.
3
Required probability =  0.75
4

9. A cantilever beam of length L and flexural modulus EI is subjected to a point load P at the free
end. The elastic strain energy stored in the beam due to bending (neglecting transverse shear)
P 2 L3 P 2 L3 PL3 PL3
(A) (B) (C) (D)
6EI 3EI 3EI 6EI
Key: (A)

 ICP–Intensive Classroom Program  eGATE-Live Internet Based Classes DLP  TarGATE-All India Test Series
Leaders in GATE Preparations  65+ Centers across India
© All rights reserved by Gateforum Educational Services Pvt. Ltd. No part of this booklet may be reproduced or utilized in any form without the written permission.

2
|ME| GATE-2017-PAPER-II www.gateforum.com

Exp: M x   P.x
x P
L 2 2 2
M dx P x dx
U   x

0
2EI 2EI L
x x
P 2 L3
U
6EI

10. It is desired to make a product having T-shaped cross-section from a rectangular aluminium
block. Which one of the following processes is expected to provide the highest strength of the
product?
(A) Welding (B) Casting (C) Metal Forming (D) Machining
Key: (C)

11. The heat loss from a fin is 6W. The effectiveness and efficiency of the fin are 3 and 0.75,
respectively. The heat loss (in W) from the fin, keeping the entire fin surface at base
temperature, is __________.
Key: 7.9 to 8.1
Exp: Given QLoss  6W
  3,   0.75
Q Qloss
  act 
Q max Heat loss keep entire fin as base temperature
6
Q max   8W
0.75

12. For a single server with Poisson arrival and exponential service time, the arrival rate is 12 per
hour. Which one of the following service rates will provide a steady state finite queue length?
(A) 6 per hour (B) 10 per hour (C) 12 per hour (D) 24 per hour
Key: (D)
Exp:   12 / hour
For steady state finite queue length

So,   24 / hour

13. For the stability of a floating body the


(A) centre of buoyancy must coincide with the centre of gravity
(B) centre of buoyancy must be above the centre of gravity
(C) centre of gravity must be above the centre of buoyancy
(D) metacentre must be above the centre of gravity
Key: (D)
Exp: Stability of floating body is measure with the help of Meta center.
Floating body to be stable, Meta center must be above C.G
I
 MG   BG ,

For stability MG > 0

 ICP–Intensive Classroom Program  eGATE-Live Internet Based Classes DLP  TarGATE-All India Test Series
Leaders in GATE Preparations  65+ Centers across India
© All rights reserved by Gateforum Educational Services Pvt. Ltd. No part of this booklet may be reproduced or utilized in any form without the written permission.

3
|ME| GATE-2017-PAPER-II www.gateforum.com

14. The divergence of the vector yi  xj __________


Key: 0 to 0

Exp: Let F   yi  xj
  
divergence of F    y    x   0
x y

15. For a loaded cantilever beam of uniform cross-section, the bending moment (in N.mm) along
the length is M(x) = 5x2+10x, where x is the distance (in mm) measured from the free end of
the beam. The magnitude of shear force (in N) in the cross-section at x =10 mm is ________.
Key: 110 to 110
d
Exp:  S.F  (B.M)
dx
S.F   5x 2  10x 
d
dx
S.f  10x  10   S.F  x 10mm  10  10   10  110N

16. A sample of 15 data is a follows: 17, 18, 17, 17, 13, 18, 5, 5, 6, 7, 8, 9, 20, 17, 3. The mode of
the data is
(A) 4 (B) 13 (C) 17 (D) 20
Key: (C)
Exp: We know that mode is the value of the data which occurred most of
17 is mode

17. If a mass of moist air contained in a closed metallic vessel is heated, then its
(A) relative humidity decreases (B) relative humidity increases
(C) specific humidity increases (D) specific humidity decreases
Key: (A)
Exp: Given that mass of moist air contained in a closed metallic vessel is heated, means its specific
humidity   is constant.
So, from the psychometric chart,

Relative humidity 1
2
    100%

1 2

DBT
We can say that at constant specific humidity, as temperature increases relative humidity
decreases i.e., 2  1
So, Answer is (A)

 ICP–Intensive Classroom Program  eGATE-Live Internet Based Classes DLP  TarGATE-All India Test Series
Leaders in GATE Preparations  65+ Centers across India
© All rights reserved by Gateforum Educational Services Pvt. Ltd. No part of this booklet may be reproduced or utilized in any form without the written permission.

4
|ME| GATE-2017-PAPER-II www.gateforum.com

18. In a slider-crank mechanism, the lengths of the crank and the connecting rod are 100mm and
160mm, respectively. The crank is rotating with an angular velocity of 10 radian/s counter-
clockwise. The magnitude of linear velocity (in m/s) of the piston at the instant corresponding
to the configuration shown in the figure is _____________

Key: 0.99 to 1.01


 sin 2 
Exp: V  r sin  
 n 
if   90
 sin180o 
V  r  sin 90    0.1  1 sin 90   1m sec
 2n 

19. A machine component made of a ductile material is subjected to a variable loading with
min  50 MPa and max  50 MPa. If the corrected endurance limit and the yield strength for
the material are 'e  100 MPa and  y  300MPa, the factor of safety is __________
Key: 1.99 to 2.01
Exp:  max  50MPa
 min   50MPa
 y  300MPa
e  100MPa
max  min 50  50
max   0
2 2
  min 50   50 
 v  max   50
2 2
max  v 1
 
 yt e F.S
0 50 1
 
300 100 F.S
F.S  2

20. The crystal structure of aluminium is


(A) body-centred cubic (B) face-centred cubic
(C) close-packed hexagonal (D) body-centred tetragonal
Key: (B)

21. A steel bar is held by two fixed supports as shown in the figure and is subjected to an increases
of temperature T  100C. and 200GPa, respectively, the magnitude of thermal stress (in
MPa) induced in the bar is __________.

 ICP–Intensive Classroom Program  eGATE-Live Internet Based Classes DLP  TarGATE-All India Test Series
Leaders in GATE Preparations  65+ Centers across India
© All rights reserved by Gateforum Educational Services Pvt. Ltd. No part of this booklet may be reproduced or utilized in any form without the written permission.

5
|ME| GATE-2017-PAPER-II www.gateforum.com

Key: 218 to 222


Exp: Thermal Stress,
Th  ET  200  103  11 106  100  220MPa

22. The Laplace transform of tet is


s 1 1 s
(A) (B) (C) (D)
 s  1  s  1  s  1 s 1
2 2 2

Key: (B)
 Leat f  t   F  s  a  
Lte  
1
Exp: t
;  
 s  1  where F  s   Lf  t  
2

23. Consider a laminar flow at zero incidence over a flat plate. The shear stress at the wall is
denoted by  w . The axial positions x1 and x2 on the plate are measured from the leading edge
in the direction of flow. If x2 > x1, then
(A) w x1
 w x2
 0 (B) w x1
 w x2
 0 (C) w x1
 w x2
(D) w x1
 w x2

Key: (C)
Exp:  w 1   w 2
 u   u   u 
     w   
 y  
1  y
y 0  y 0 
2  y y 0 

24. A mass m is attached to two identical springs having spring constant k as shown in the figure.
The natural frequency  of this single degree of freedom system is
2k k
(A) (B)
m m

k 4k
(C) (D)
2m m

Key: (A)
Exp: Equivalent stiffness keq = k + k = 2k
k eq 2k
Natural frequency,  n    n 
m m

25. Given the atomic weight of Fe is 56 and that of C is 12, the weight percentage of carbon in
cementite (Fe3C) is _________.
Key: 6.3 to 7.0
12
Exp: Percentage of carbon by weight in cementite =  100  6.67%
56  3  12

 ICP–Intensive Classroom Program  eGATE-Live Internet Based Classes DLP  TarGATE-All India Test Series
Leaders in GATE Preparations  65+ Centers across India
© All rights reserved by Gateforum Educational Services Pvt. Ltd. No part of this booklet may be reproduced or utilized in any form without the written permission.

6
|ME| GATE-2017-PAPER-II www.gateforum.com

Q. No. 26 to 55 Carry Two Marks Each


26. In an orthogonal machining with a tool of 9° orthogonal rake angle, the uncut chip thickness is
0.2mm. The chip thickness fluctuates between 0.25 mm and 0.4 mm. The ratio of the
maximum shear angle to the minimum shear angle during machining is ___________
Key: 1.45 to 1.53
Exp:   9
t1  0.2mm
t c  0.25mm to 0.4mm
r cos  t
 tan   , where r  1
1  r sin  tc
if t c  0.25mm, r  0.8 &   42.08o
if t c  0.4mm, r  0.5 &   28.18o
max 42.08
  1.493
min 28.18

0.005
27. A cylindrical pin of 250.020
0.010 mm diameter is electroplated. Plating thickness is 2.0 mm.
Neglecting the gauge tolerance, the diameter (in mm, up to 3 decimal points accuracy) of the
GO ring gauge to inspect the plated pin is _________.
Key: 29.030 to 29.030
Exp: Diameter of GO end of ring gauge = 25.02 + 2(2.005) = 29.030mm

28. A helical compression spring made of wire of circular cross-section is subjected to a


compressive load. The maximum shear stress induced in the cross-section of the wire is 24
MPa. For the same compressive load, if both the wire diameter and the mean coil diameter are
doubled, the maximum shear stress (in MPa) induced in the cross-section of the wire is _____.
Key: 6 to 6
8P.D
Exp: max 
d 3
Given, 1  24MPa
P1  P2 , d 2  2d1 , D 2  2D1
1 D1 d 32
 2 
24 1 3
so,  . 3
2 D 2 d1 2 2
24
2  MPa
4
2  6MPa

29. In a counter-flow heat exchanger, water is heated at the rate of 1.5kg/s from 40°C to 80°C by
an oil entering at 120°C and leaving at 60°C. The specific heats of water and oil are
4.2kJ/kg.K and 2kJ/kg.K respectively. The overall heat transfer coefficient is 400 W/m2.K.
The required heat transfer surface area (in m2) is
(A) 0.104 (B) 0.022 (C) 10.4 (D) 21.84
Key: (D)
Exp: Given counter flow Heat Exchanger

 ICP–Intensive Classroom Program  eGATE-Live Internet Based Classes DLP  TarGATE-All India Test Series
Leaders in GATE Preparations  65+ Centers across India
© All rights reserved by Gateforum Educational Services Pvt. Ltd. No part of this booklet may be reproduced or utilized in any form without the written permission.

7
|ME| GATE-2017-PAPER-II www.gateforum.com


Water – m c  1.5kg / s Oil - Th i  120C
Tci  40C Th o  60C
Tco  80C C po  2kJ / kg.K
C pc  4.2KJ / kg.K
Overall Heat Transfer Coefficient = 400 W/m2K
Area (A) =?
Q  UA  LMTD  Thi  120C
 
i  40 
Tco  80C

 

mc Cpc Tco  Tci  UA  LMTD  
 Th o  60C 

 

mc Cpc Tco  Tci   o  20
A Tci  40C 
   o 
U i 
 n(i / o ) 
1.5  4.2   80  40   103
  21.83m 2  A  21.83m 2
 
40  20
400  
 n 40
 
20  

30. The rod PQ of length L = 2 m, and uniformly distributed mass of M = 10 kg, is released
from rest at the position shown in the figure. The ends slide along the frictionless faces OP and
OQ. Assume acceleration due to gravity, g = 10 m/s2. The mass moment of inertia of the rod
about its centre of mass and an axis perpendicular to the plane of the figure is (ML 2/12). At
this instant, the magnitude of angular acceleration (in radian/s2) of the rod is ____________

Key: 7.25 to 7.75


Exp: M  10kg, g 10m sec2
0.5
ML2
Ic  I
12 1

 2
2
2 2
M  1  2M M 2M
II   M   12  2  3 1m
12  2
2M
T  I  Mg  0.5   Mg
3
2
 10  0.5    1m
3
15
   7.5rad sec 2
2
 ICP–Intensive Classroom Program  eGATE-Live Internet Based Classes DLP  TarGATE-All India Test Series
Leaders in GATE Preparations  65+ Centers across India
© All rights reserved by Gateforum Educational Services Pvt. Ltd. No part of this booklet may be reproduced or utilized in any form without the written permission.

8
|ME| GATE-2017-PAPER-II www.gateforum.com

31. A steel plate, connected to a fixed channel using three identical bolts A, B and C, carries a
load of 6kN as shown in the figure. Considering the effect of direct load and moment, the
magnitude of resultant shear force (in kN) on bolt C is.

(A) 13 (B) 15 (C) 17 (D) 30


Key; (C)
6
Exp: Pr imary shear   2kN
3
Secondaryshear 1500 kN  mm
 Pe  1500
M 2 2   0.3
 r1  r2   50  50 
2 2

R1A  R1C  C  r1  0.3  50  15kN


Resultant shear at 'C'  2  15 17 kN.
Resultant shear  15  2  17 kN.

32. The volume and temperature of air (assumed to be an ideal gas) in a closed vessel is 2.87 m3
and 300K, respectively. The gauge pressure indicated by a manometer fitted to the wall of the
vessel is 0.5bar. If the gas constant of air is R = 287 J/kg. K and the atmospheric pressure is 1
bar, the mass of air (in kg) in the vessel is
(A) 1.67 (B) 3.33 (C) 5.00 (D) 6.66
Key: (C)
Exp: V  2.87m3 ; T  300K
Pgauge  0.5bar
R  287J / kg.K
Patm  1 bar
Pabs  Pg  Patm  1.5bar
PV  mRT
PV 1.5  105  2.87
So, m    5kg
RT 287  300

33. For the laminar flow of water over a sphere, the drag coefficient CF is defined as
CF  F /  U2 D2  , where F is the drag force,  is the fluid density, U is the fluid velocity and
D is the diameter of the sphere. The density of water is 1000 kg/m3. When the diameter of the
sphere is 100mm and the fluid velocity is 2m/s, the drag coefficient is 0.5. If water now flows
over another sphere of diameter 200mm under dynamically similar conditions, the drag force
(in N) on this sphere is _____________

 ICP–Intensive Classroom Program  eGATE-Live Internet Based Classes DLP  TarGATE-All India Test Series
Leaders in GATE Preparations  65+ Centers across India
© All rights reserved by Gateforum Educational Services Pvt. Ltd. No part of this booklet may be reproduced or utilized in any form without the written permission.

9
|ME| GATE-2017-PAPER-II www.gateforum.com

Key: 19.9 to 20.1


Exp: Given that the condition is dynamic similarity, and in the given condition, Inertia and viscous
force plays major role, hence Reynold‟s number should be same for both model and prototype.
 Re 1   Re 2
In the first case: U1  2m sec, D1  100 mm,   1000 kg m3
In the second case: U2  2m sec, D2  200mm,   1000 kg m3
 UD   UD 
   
  1    2
Since same water is flowing over both sphere
1   2 , 1   2
U1D1  U 2 D 2
  2 100    V2  200 
U 2  1m sec
So, Drage force in second case will be
F2  CFU22 D22   0.51000 (1)2 0.2   20N
2

34. A rod of length 20mm is stretched to make a rod of length 40 mm. Subsequently, it is
compressed to make a rod of final length 10mm. Consider the longitudinal tensile strain as
positive and compressive strain as negative. The total true longitudinal strain in the rod is
(A) –0.5 (B) –0.69 (C) –0.75 (D) –1.0
Key: (B)
Exp:  i  20mm  f  10mm
 truestrain,
   10 
True  n  f   n    0.69
 i   20 

35. A gear train shown in the figure consists of gears P, Q, R and S. Gear Q and gear R are
mounted on the same shaft. All the gears are mounted on parallel shafts and the number of
teeth of P, Q, R and S are 24, 45, 30 and 80, respectively. Gear P is rotating at 400 rpm. The
speed (in rpm) of the gear S is _________.

Key: 120 to 120


Exp: TP  24, TQ  45, TR  30, TS  80
N P  400rpm
NP .TP  NQ TQ

 ICP–Intensive Classroom Program  eGATE-Live Internet Based Classes DLP  TarGATE-All India Test Series
Leaders in GATE Preparations  65+ Centers across India
© All rights reserved by Gateforum Educational Services Pvt. Ltd. No part of this booklet may be reproduced or utilized in any form without the written permission.

10
|ME| GATE-2017-PAPER-II www.gateforum.com

400  24
NQ   213.33rpm
45
N Q TQ  NSTS
213.33  45
NS   120rpm
80

36. In the Rankine cycle for a steam power plant the turbine entry and exit enthalpies are 2803 kJ/kg
and 1800 kJ/kg, respectively. The enthalpies of water at pump entry and exit are 121 kJ/kg and 124
kJ/kg, respectively. The specific steam consumption (in kg/k W.h) of the cycle is ______
Key: 3.5 to 3.7
Exp: Given that, T
h1  2803 kJ / kg, h 2  1800kJ / kg 1
h 3  121kJ / kg, h 4  124kJ / kg
4
WT  h1  h 2 3
2
 2803  1800  1003kJ / kg
Wp  h 4  h 3
S
 124  121  3kJ / kg
Wnet  WT  WP  1000kJ / kg
3600
Specific Steam consumption   3.6kg / kWh
Wnet

37. A calorically perfect gas (specific heat at constant pressure 1000 J/kg.K) enters and leaves a gas
turbine with the same velocity. The temperatures of the gas at turbine entry and exit are 1100 K
and 400 K. respectively. The power produced is 4.6 MW and heat escapes at the rate of 300 kJ/s
through the turbine casing. The mass flow rate of the gas (in kg/s) through the turbine is.
(A) 6.14 (B) 7.00 (C) 7.50 (D) 8.00
Key: (B)
Exp: Given that,
Cp  1000J / kgK
T1  1100K, P  4.6MW P
T2  400K, Q L  300kJ / s
 
E in  E out
V12 V2
h1   gz1  h 2  2  gz 2  Q L  P
2 2
1 2
 V1  V2 , Z1  Z2
So, h1  h 2  Q L  P QL

m Cp  T1  T2   Q L  P

m
 300 10    4.6 10 
3 6

1000  1100  400 



m  7kg / sec

 ICP–Intensive Classroom Program  eGATE-Live Internet Based Classes DLP  TarGATE-All India Test Series
Leaders in GATE Preparations  65+ Centers across India
© All rights reserved by Gateforum Educational Services Pvt. Ltd. No part of this booklet may be reproduced or utilized in any form without the written permission.

11
|ME| GATE-2017-PAPER-II www.gateforum.com

38. Three masses are connected to a rotating shaft supported on bearings A and B as shown in the
figure. The system is in a space where the gravitational effect is absent. Neglect the mass of
shaft and rods connecting the masses. For m1 = 10kg, m2 = 5kg and m3 = 2.5 kg and for a shaft
angular speed of 1000 radian/s, the magnitude of the bearing reaction (in N) at location B is
_________.

Key: 0 to 0
Exp: m1  10kg, r1  0.1m
m 2  5kg , r2  0.2m
m3  2.5kg , r3  0.4m

F x  m1r1 cos 1  m 2 r2 cos 2  m3 r3 cos 3


 10  0.1 cos 0o   5  0.2  cos120o  2.5  0.4  cos 240o
 1  0.5  0.5  0
F y m1r1 sin 1  m 2 r2 sin 2  m3 r3 sin 3  0   5  0.2  sin120o   2.5  0.4sin 240o   0
R A  R B  0kN

39. A strip of 120 mm width and 8mm thickness is rolled between two 300 mm-diameter rolls to
get a strip of 120 mm width and 7.2 mm thickness. The speed of the strip at the exit is 30
m/min. There is no front or back tension. Assuming uniform roll pressure of 200 MPa in the
roll bite and 100% mechanical efficiency, the minimum total power (in kW) required to drive
the two rolls is _________.
Key: 9.4 to 9.8
Exp: Width = 120mm
Initial thickness to = 8mm
Diameter of Roller = 300mm
Radius of Roller = 150mm
Final thickness = 7.2mm
h  t i  t f  8  7.2  0.8 mm
Power require to drive one roller
P  T.  F  L p    .A.L p . Here, A  LP  b

P  L2p .b  here, Lp  Rh

V
P    Rh  .b.  
R
30
 200  106  0.8  103  0.12   4.8 kW
60
So, power require to drive 2 roller = 2P  2  4.8 kW  9.6 kW

 ICP–Intensive Classroom Program  eGATE-Live Internet Based Classes DLP  TarGATE-All India Test Series
Leaders in GATE Preparations  65+ Centers across India
© All rights reserved by Gateforum Educational Services Pvt. Ltd. No part of this booklet may be reproduced or utilized in any form without the written permission.

12
|ME| GATE-2017-PAPER-II www.gateforum.com

40. A product made in two factories p and Q, is transported to two destinations, R and S. The per
unit costs of transportation (in Rupees) from factories to destinations are as per the following
matrix:
Destination
R S
Factory
P 10 7
Q 3 4

Factory P produces 7 units and factory Q produces 9 units of the product. Each destination
requires 8 units. If the north-west corner method provides the total transportation cost as X (in
Rupees) and the optimized (the minimum) total transportation cost Y (in Rupees), then (X-Y),
in Rupees, is
(A) 0 (B) 15 (C) 35 (D) 105
Key: Answer is not matched with IIT Key
Exp: R S
10 7
P 7
3 4
Q 9
8 8
By North West corner Rule
R S
10
P 7 7
Q 3 1 4 8

Total cost, x = 10×7+7×1+4×8 = Rs.105


By VAM
V1  6 V2  7
u1  0 10

7
7 7 1
u 2  3 3
8 4
1 9 1
8 8
7  3
Total minimum Cost, y = 3×8+4×1+7×7=Rs.77
So, x–y = Rs.28

41. One kg of an ideal gas (gas constant R = 287 J/kg.K) undergoes an irreversible process from
state-1 (1 bar, 300 K) to state -2 (2 bar, 300 K). The change in specific entropy (s2 – s1) of the
gas (in J/kg. K) in the process is ___________
Key: –201 to –197
Exp: Given m  1kg, R  287J / kg.K
P1  1bar, P2  2bar
T1  300K, T2  300K
Same Temperature

S2  S1  Rn  P P   287n  21   198.93J / kgK


2
1

 ICP–Intensive Classroom Program  eGATE-Live Internet Based Classes DLP  TarGATE-All India Test Series
Leaders in GATE Preparations  65+ Centers across India
© All rights reserved by Gateforum Educational Services Pvt. Ltd. No part of this booklet may be reproduced or utilized in any form without the written permission.

13
|ME| GATE-2017-PAPER-II www.gateforum.com

42. A 60 mm-diameter water jet strikes a plate containing a hole of 40mm diameter as shown in
the figure. Part of the jet passes through the hole horizontally, and the remaining is deflected
vertically. The density of water is 1000 kg/m3. If velocities are as indicated in the figure, the
magnitude of horizontal force (in N) required to hold the plate is _________

Key: 627 to 629


Exp: Force in X–direction = Rate of change of momentum
  Pi  Pf  x  direction

  

  m1 V1  m 2 V2  D1  6cm
  x direction D2  4cm
 V  A1V1  A 2 V2  V1  V2 
 V 2  A1  A 2 
 20m / s  V2
 1000  20  20   0.06    0.04  
2 2

4   20m / s  V1
 628.32N

43. The arrangement shown in the figure measures the


velocity V of a gas of density 1 kg/m3 flowing
through a pipe. The acceleration due to gravity is
9.81 m/s2. If the manometric fluid is water (density
1000 kg/m3) and the velocity V is 20 m/s, the
differential head h (in mm) between the two arms of
the manometer is ____________
Key: 19 to 21
Exp: Given
g  1kg / m3 , g  9.81m / s 2
m  1000kg / m3 , V  20m / s

 
V  2gH Where H  h  m  1
  
V 2
 
So,  h  m  1
2g   
  20 2 
   h 1000  1
 2  9.81 
h  0.0204m  2.04cm  h  20.4mm

 ICP–Intensive Classroom Program  eGATE-Live Internet Based Classes DLP  TarGATE-All India Test Series
Leaders in GATE Preparations  65+ Centers across India
© All rights reserved by Gateforum Educational Services Pvt. Ltd. No part of this booklet may be reproduced or utilized in any form without the written permission.

14
|ME| GATE-2017-PAPER-II www.gateforum.com

44. A metal ball of diameter 60mm is initially at 220 °C. The ball is suddenly cooled by an air jet
of 20°C. The heat transfer coefficient is 200 W/m2.K and 9000kg/m3, respectively. The ball
temperature (in °C) after 90 seconds will be approximately.
(A) 141 (B) 163 (C) 189 (D) 210
Key: (A)
Exp: D = 60mm = 0.06m
Ti  220C, h  200W / m 2 K,   9000kg / m3
T  20C, Cp  400J / kgK, K  400W / mK
t  90sec T?
 h.A 
 200903 
Ti  T  .t
 .VC  220  20  
 e p    e 0.039000400 
Tf  T Tf  20
Tf  141.3C

45. A single – plate clutch has a friction disc with inner and outer radii of 20mm and 40 mm,
respectively. The friction lining in the disc is made in such a way that the coefficient of
friction  varies radially as   0.01r, where r is in mm. The clutch needs to transmit a friction
torque of 18.85kN.mm. As per uniform pressure theory, the pressure (in MPa) on the disc is
_________
Key: 0.49 to 0.51
Exp: ri  20mm, ro  40mm,   0.01r dr
T  18.85kN  mm
p?
P   p2rdr r
T   p 2r 2 dr   p  0.01r  2r 2 dr   0.0628r 3 .dr  ri
40 r0
40
 r4 
T  0.0628p  r dr  0.0628p  
3

20  4  20
 404 204 
18.85  103  0.0628p   
 4 4 
p  0.5MPa

46. The surface integral  F.ndS


S
over the surface S of the sphere x 2  y 2  z 2  9, where

F=(x+y) i+ (x+z) j+(y+z) k and n is the unit outward surface normal, yields ______.
Key: 225 to 227

Exp: F   x  yi   x  z J   y  z k
   
divF   x  y    x  z    y  z   1  0  1  2
x y z
By divergence theorem,
 ^ 
S
F.n dS   divFdV
V
where V is volume of given surface of sphere x 2  y2  z2  9

4  27 
  2dV  2V  2   72  226.1947
V
3

 ICP–Intensive Classroom Program  eGATE-Live Internet Based Classes DLP  TarGATE-All India Test Series
Leaders in GATE Preparations  65+ Centers across India
© All rights reserved by Gateforum Educational Services Pvt. Ltd. No part of this booklet may be reproduced or utilized in any form without the written permission.

15
|ME| GATE-2017-PAPER-II www.gateforum.com

47. Block 2 slides outward on link 1 at a uniform velocity of 6 m/s as shown in the figure. Link 1
is rotating at a constant angular velocity of 20 radian/s counterclockwise. The magnitude of
the total acceleration (in m/s2) of point P of the block with respect to fixed point O is
________

Key: 243 to 244


Exp: Acceleration of the block,
v  6m / sec,   20rad / sec

  r    2v
2 2
a  a r 2  a cr 2  2

r  OP,
a cr  2v  2  6  20  240m / s 2
a r  2 r   20   0.1  40
2

a r  2402  402  243.31m / sec 2

48. During the turning of a 20mm-diameter steel bar at a spindle speed of 400 rpm, a tool life of
20 minute is obtained. When the same bar is turned at 200 rpm, the tool life becomes 60
minute. Assume that Taylor‟s tool life equation is valid. When the bar is turned at 300 rpm, the
tool life (in minute) is approximately.
(A) 25 (B) 32 (C) 40 (D) 50
Key: (B)
Exp: Taylors Tool life equation,
VT n  C
 V1T1n  V2 T2 n
n
 T2  V1
  
 T1  V2
n
 60  400
  
 20  200
n  0.63
at N 3  300rpm, T3  ?
n
 T3  V1
  
 T1  V3
0.63
 T3  400
 20   300
 
T3  31.57 min utes
T3  32 min utes
 ICP–Intensive Classroom Program  eGATE-Live Internet Based Classes DLP  TarGATE-All India Test Series
Leaders in GATE Preparations  65+ Centers across India
© All rights reserved by Gateforum Educational Services Pvt. Ltd. No part of this booklet may be reproduced or utilized in any form without the written permission.

16
|ME| GATE-2017-PAPER-II www.gateforum.com

50 70 
49. Consider the matrix A =   whose eigenvectors corresponding to eigenvalues 1 and  2
70 80 
 70    80
are x1    and x 2   2 T
 . respectively. The value of x1 x 2 is_________

 1  50   70 
Key: 0 to 0
50 70 
Exp: A 
 70 80 
 70    2  80 
Eigen vectors are X1    ; X2   
 1  50   70 
Where 1 ,  2 Eigen values of A
   80 
X1T X 2  70 1  50   2   70   2  80    1  50  70
 70 
 70 2  5600  701  3500  70  1   2   9100
 70 130   9100  9100  9100  0

 sum of eigen values  1   2 


 
 Trace  50  80  130 

50. The radius of gyration of a compound pendulum about the point of suspension is 100mm. The
distance between the point of suspension and the centre of mass is 250mm. Considering the
acceleration due to gravity as 9.81 m/s2, the natural frequency (in radian/s) of the compound
pendulum is _________.
Key: 15 to 16

Exp: k  100mm  0.1m


L  250mm  0.250m
g  9.81m / sec

I  mk 2  m  0.1
2

mgL m  9.81 0.250


Wn    15.66rad / sec
m  0.1
2
I

51. Consider the differential equation 3y ''  x   27y  x   0 with initial conditions y(0) = 0 and
y'(0)  2000. The value of y at x = 1 is ________.
Key: 93 to 95
Exp: 3y ''  x   27y  x   0, y  0   0, y '  0   2000
Auxillary equation, 3m 2  27  0  m 2  9  0  m  0  3i
y c  c1 cos3x  c 2 sin 3x and y p  0
 y c  c1 cos3x  c 2 sin 3x
y  0   0  c1  0  0  c1  0
 y  c2 sin 3x
y '  3c 2 cos3x

 ICP–Intensive Classroom Program  eGATE-Live Internet Based Classes DLP  TarGATE-All India Test Series
Leaders in GATE Preparations  65+ Centers across India
© All rights reserved by Gateforum Educational Services Pvt. Ltd. No part of this booklet may be reproduced or utilized in any form without the written permission.

17
|ME| GATE-2017-PAPER-II www.gateforum.com

2000
y '  0   2000  2000  3c 2  c 2 
3
2000 2000
y  sin 3x, y 1  sin 3  94.08
3 3

52. If f(z) = (x2+ay2) + ibxy is a complex analytic function of z = x + iy, where i = 1, then
(A) a = –1, b= –1 (B) a = –1, b = 2 (C) a = 1, b = 2 (D) a = 2, b = 2
Key: (B)
Exp: Given f  z    x 2  ay2   i bxy is analytic

u  x 
  , 
x y y x
value u  x 2  ay 2 ,   bxy
u 
 2x  by
x x
u 
 2ay  bx
y y
Clearly for b = 2 and a= –1 above Cauchy-Riemann equations holds

53. A project starts with activity A and ends with activity F. The precedence relation and durations
of the activities are as per the following table:

Duration
Activity Immediate Predecessor
(days)
A - 4
B A 3
C A 7
D B 14
E C 4
F D,E 9

The minimum project completion time (in days) is ______


Key: 30 to 30
Exp: 3 D
B
3 14
1 A 2 5 F 6
4 4 9
C, 7 4 E
Minimum project completion Time = Length of longest path = 4+3+14+9=30 Days.

54. Maximize Z = 5x1+3x2


Subject to
x1+2x2  10,
x1–x2  8,
x1, x2  0.
In the starting Simplex tableau, x1 and x2 are non-basic variables and the value of Z is zero.
The value of Z in the next Simplex tableau is _________.
 ICP–Intensive Classroom Program  eGATE-Live Internet Based Classes DLP  TarGATE-All India Test Series
Leaders in GATE Preparations  65+ Centers across India
© All rights reserved by Gateforum Educational Services Pvt. Ltd. No part of this booklet may be reproduced or utilized in any form without the written permission.

18
|ME| GATE-2017-PAPER-II www.gateforum.com

x2
Key: 40 to 40
  26 2 
 26 2   0,5   , 
Exp: Zmax at  ,   3 3
 3 3

10,0  x1
1st Table at  0,0   Z  0  0,0   8,0 
2nd Table at 8,0   Z  40
  0, 8 

55. The principal stresses at a point in a critical section of a machine component are
1  60MPa, 2  5MPa and 3  40 MPa. For the material of the component, the tensile
yield strength is  y  200 MPa. According to the maximum shear stress theory, the factor of
safety is
(A) 1.67 (B) 2.00 (C) 3.60 (D) 4.00
Key: (B)
1  3 w y
Exp:   1  3 
2 2 FOS
200
60   40  
FOS
FOS  2
General Aptitude
Q. No. 1 - 5 Carry One Mark Each

1. If you choose plan P, you will have to _______ plan Q, as these two are mutually _________.
(A) forgo, exclusive (B) forget, inclusive
(C) accept, exhaustive (D) adopt, intrusive
Key: (A)

2. P looks at Q while Q looks at R. P is married, R is not. The number of people in which a


married person is looking at an unmarried person is
(A) 0 (B) 1 (C) 2 (D) Cannot be determined
Key: (B)

3. If a and b are integers and a – b is even, which of the following must always be even?
(A) ab (B) a2 + b2 + 1 (C) a2 + b + 1 (D) ab – b
Key: (D)
Exp: According to the given relation of a–b = even, there is a possibility of odd-odd (or) even-even
is equal to even. From the options, Option (D) is correct. Since, odd × odd–odd (or) even ×
even–even → is always even number.

Alternate Method:
Let a – b =2K  a  b  2K
ab  b   b  2K  b  b  b2  2Kb  b   b2  b   2Kb  b  b  1  2Kb
Even; since 2Kb always even & b (b-1) also even, if with b is odd or even.

 ICP–Intensive Classroom Program  eGATE-Live Internet Based Classes DLP  TarGATE-All India Test Series
Leaders in GATE Preparations  65+ Centers across India
© All rights reserved by Gateforum Educational Services Pvt. Ltd. No part of this booklet may be reproduced or utilized in any form without the written permission.

19
|ME| GATE-2017-PAPER-II www.gateforum.com

4. A couple has 2 children. The probability that both children are boys if the older one is a boy is
(A) 1/4 (B) 1/3 (C) 1/2 (D) 1
Key: (C)
No.of Favourablecases
Exp: Probability 
Total No.of Possible cases
1
The probability that both children are boys if the order one is a boy =
2

5. The ways in which this game can be played __________ potentially infinite.
(A) is (B) is being (C) are (D) are being
Key: (C)

Q. No. 6- 10 Carry Two Marks Each

6. “If you are looking for a history of India, or for an account of the rise and fall of the British
Raj, or for the reason of the cleaving of the subcontinent into two mutually antagonistic parts
and the effects this mutilation will have in the respective sections, and ultimately on Asia, you
will not find it in these pages; for though I have spent a lifetime in the country, I lived too near
the seat of events, and was too intimately associated with the actors, to get the perspective
needed for the impartial recording of these matters.”
Which of the following closest in meaning to „cleaving‟?
(A) Deteriorating (B) Arguing (C) Departing (D) Splitting
Key: (D)

7. There are 4 women P, Q, R, S, and 5 men V, W, X, Y, Z in a group. We are required to form


pairs each consisting of one woman and one man. P is not to be paired with Z, and Y must
necessarily be paired with someone. In how many ways can 4 such pairs be formed?
(A) 74 (B) 76 (C) 78 (D) 80
Key: (C)
Exp: If P is paired with y; they Q has 4 choices
R has 3 choices
S has 2 choices
Total 24 choices
(or)
If Q is paired with y; then P has 3 choices
R has 3 choices
S has 2 choices
Total 18 choices
(or)
If R is paired with y; then P has 3 choices
Q has 3 choices
S has 2 choices
Total 18 choices

 ICP–Intensive Classroom Program  eGATE-Live Internet Based Classes DLP  TarGATE-All India Test Series
Leaders in GATE Preparations  65+ Centers across India
© All rights reserved by Gateforum Educational Services Pvt. Ltd. No part of this booklet may be reproduced or utilized in any form without the written permission.

20
|ME| GATE-2017-PAPER-II www.gateforum.com

(or)
If S is paired with y; then P has 3 choices
Q has 3 choices
S has 2 choices
Total 18 choices
 Total number of ways = 24+18+18+18=78

8. In the graph below, the concentration of a particular pollutant in a lake is plotted over
(alternate) days of a month in winter (average temperature 10°C) and a month in summer
(average temperature 30°C).

Consider the following statements based on the data shown above:


(i) Over the given months, the difference between the maximum and the minimum pollutant
concentrations is the same in both winter and summer.
(ii) There are at last four days in the summer month such that the pollutant concentrations on
those days are within 1 ppm of the pollutant concentrations on the corresponding days in
the winter month.
Which one of the following options is correct?
(A) Only i (B) Only ii (C) Both i and ii (D) Neither i nor ii
Key: (B)
Exp: The difference between the maximum and the minimum pollutant concentrations
(i) in winter = 8-0=8 ppm,
(ii) in summer = 10.5-1.5=9 ppm
 (i) is false & (ii) is correct from the graph.

9. All people in a certain island are either „Knights‟ or „Knaves‟ and each person knows every
other person‟s identity. Knights NEVER lie, and knaves ALWAYS lie.
P says “Both of us are knights”. Q says “None of us are knaves”.
Which one of the following can be logically inferred from the above?
(A) Both P and Q are knights
(B) P is a knight; Q is a knave
(C) Both P and Q are knaves
(D) The identities of P, Q cannot be determined
Key: (D)

 ICP–Intensive Classroom Program  eGATE-Live Internet Based Classes DLP  TarGATE-All India Test Series
Leaders in GATE Preparations  65+ Centers across India
© All rights reserved by Gateforum Educational Services Pvt. Ltd. No part of this booklet may be reproduced or utilized in any form without the written permission.

21
|ME| GATE-2017-PAPER-II www.gateforum.com

10. X bullocks and Y tractors take 8 days to plough a field. If we halve the number of bullocks and
double the number of tractors, it takes 5days to plough the same field. How many days will it
take X bullocks alone to plough the field?
(A) 30 (B) 35 (C) 40 (D) 45
Key: (A)
Exp: Given Number of days required that X bullocks and Y tractors to plough a field = 8 days 
(1)  i.e, X  Y  8D  8X  8Y  1day  Number of days required that
X
bullocks and 2y tractors
2
to plough field  5   2 
 X 5 
i.e, 2  2Y  5D  2 X  10Y  1D 
 
From (1) & (2); we have
5X 11X
8X  8Y   10Y  Y 
2 4
15X
 From 1 X  Y  8D   8D  X  30days
4

 ICP–Intensive Classroom Program  eGATE-Live Internet Based Classes DLP  TarGATE-All India Test Series
Leaders in GATE Preparations  65+ Centers across India
© All rights reserved by Gateforum Educational Services Pvt. Ltd. No part of this booklet may be reproduced or utilized in any form without the written permission.

22
|ME| GATE-2017-PAPER-I www.gateforum.com

Mechanical Engineering
Q. No. 1 to 25 Carry One Mark Each

1. A motor driving a solid circular steel shaft transmits 40kW of power at 500 rpm. If the
diameter of the shaft is 40 mm, the maximum shear stress in the shaft is ________MPa.
Key: 60 to 61
Exp: P=40kW N=500rpm
16T
D= 40mm T=
d 3
P  60,000 40  60,000
T   763.94N  m
2N 2  500
16  763.94  103
  60.79MPa
   40 
3

2. Consider the following partial differential equation for u(x,y) with the constant c > 1 :
u u
c 0
y x
Solution of this equation is
(A) u  x, y   f  x  cy  (B) u  x, y   f  x  cy 
(C) u  x, y   f  cx  y  (D) u  x, y   f  cx  y 
Key: (B)
u u
Exp: Given c 0
y x
u
 f '  x  cy 
x
u
 c f '  x  cy 
x
u u
 0
x y
u  x, y   f  x  cy 

3. The following figure shows the velocity- time plot for a particle traveling along a straight line.
The distance covered by the particle from t = 0 to t= 5 s is __________m.

4
3
Velocity (m / s)
2
1

1 2 3 4 5 6
Time(s)

 ICP–Intensive Classroom Program  eGATE-Live Internet Based Classes DLP  TarGATE-All India Test Series
Leaders in GATE Preparations  65+ Centers across India
© All rights reserved by Gateforum Educational Services Pvt. Ltd. No part of this booklet may be reproduced or utilized in any form without the written permission.

1
|ME| GATE-2017-PAPER-I www.gateforum.com

Key: 10 to 10
Exp: Area under V- T wave
S  a1  a 2  a 3  a 4  a 5
1    1  4    4  2  
  11   1 1     1     2
2    2    2  
 0.5  1  2.5  6  10m

4. The damping ratio for a viscously damped spring mass system, governed by the relationship
d2 x dx
m 2 C  kx  F  t  , is given by
dt dt
c c c c
(A) (B) (C) (D)
mk 2 km km 2mk
Key: (B)
dx 2 dx c c
Exp: m 2
 c  k x  F t ;  
dt dt cc 2 km

d2 y
5. The differential equation  16y  0 for y  x  with the two boundary conditions
dx 2
dy dy
 1 and  1 has
dx x 0 dx x  
2

(A) no solution (B) exactly two solutions


(C) exactly one solution (D) infinitely many solutions
Key: (A)
d2 y dy dy
Exp:  16y  0 1  1
dx 2 dx x 0 dx x

2

 m 2  16  0
m  0  4i
yc  c1cos 4x  c 2 sin 4x and y p  0
 y  c1 cos 4x  c 2 sin 4x
y'  x   4c1 sin 4x  4c2 cos 4x
1
y'  0   1  0  4c2  1  c2 
4
 1
y '    1  0  4c 2  1  c 2 
2 4
1 1
c 2  and both not possible
4 4
Hence there is no solution

6. Metric thread of 0.8 mm pitch is to be cut on a lathe. Pitch of the lead screw is 1.5 mm. If the
spindle rotates at 1500 rpm, the speed of rotation of the lead screw (rpm) will be _________
Key: 800 to 800
 ICP–Intensive Classroom Program  eGATE-Live Internet Based Classes DLP  TarGATE-All India Test Series
Leaders in GATE Preparations  65+ Centers across India
© All rights reserved by Gateforum Educational Services Pvt. Ltd. No part of this booklet may be reproduced or utilized in any form without the written permission.

2
|ME| GATE-2017-PAPER-I www.gateforum.com

1500  0.8
Exp: Speed of rotation of lead screw   800rpm
1.5

7. The molar specific heat at constant volume of an ideal gas is equal to 2.5 times the universal
gas constant (8.314 J/mol.K). When the temperature increases by 100K, the change in molar
specific enthalpy is _______________ J/mol.
Key: 2908 to 2911
Exp: C v  2.5R v where  R v  8.314 J mol.K 
T 100K
H  ?
H  Cp T
Cp  C v  R v
[Cp  3.5R v ]
So, H  3.5  8.314 100
[H  2909.9 J mol]

8. A particle of unit mass is moving on a plane. Its trajectory, in polar coordinates, is given by
r(t) = t2,   t   t, where t is time. The kinetic energy of the particle at time t = 2 is
(A) 4 (B) 12 (C) 16 (D) 24
Key: (C)
Exp: r = t 2 ;θ = t
1
K.E = mv 2  ? at t  2 sec
2
 m 1kg

V  r(t) ˆ  dr rˆ  t 2 1(t)
ˆ  2trˆ
dt
ˆ  2t(r)
V  t 2 (t) ˆ
at t  2s
ˆ  4(r)
V  4(t) ˆ
V  16  16  32
1 1
K.E.  mv 2  1 32  16
2 2

9. The Poisson‟s ratio for a perfectly incompressible linear elastic material is


(A) 1 (B) 0.5 (C) 0 (D) infinity
Key: (B)

10. A heat pump absorbs 10 kW of heat from outside environment at 250 K while absorbing 15
kW of work. It delivers the heat to a room that must be kept warm at 300K. The Coefficient of
Performance (COP) of the heat pump is ___________.
Key: 1.66 to 1.70
 ICP–Intensive Classroom Program  eGATE-Live Internet Based Classes DLP  TarGATE-All India Test Series
Leaders in GATE Preparations  65+ Centers across India
© All rights reserved by Gateforum Educational Services Pvt. Ltd. No part of this booklet may be reproduced or utilized in any form without the written permission.

3
|ME| GATE-2017-PAPER-I www.gateforum.com

Exp:
300K

25kW  QH QH 25
COP    1.66
Wi/ p 15
H.P 15kW

10kW

250K

11. Which one of the following is NOT a rotating machine?


(A) Centrifugal pump (B) Gear pump
(C) Jet pump (D) Vane pump
Key: (C)
Exp: In the given options all the pumps have rotating machine elements except Jet pump.

12. Consider the schematic of a riveted lap joint subjected to tensile load F, as shown below. Let d
be the diameter of the rivets, and Sf be the maximum permissible tensile stress in the plates.
What should be the minimum value for the thickness of the plates to guard against tensile
failure of the plates? Assume the plates to be identical.

F F F 2F
(A) (B) (C) (D)
Sf  W  2d  Sf W Sf  W  d  Sf W
Key: (A)
F F F
Exp: Sf   Sf  ; t
A t  W  2d  Sf  w  2d 
13. Water (density =1000 kg/m3) at ambient temperature flows through a horizontal pipe of
uniform cross section at the rate of 1 kg/s. If the pressure drop across the pipe is 100 kPa, the
minimum power required to pump the water across the pipe, in watts, is _______
Key: 100 to 100
Exp: given,
w 1000 kg m3

m 1kg s
P 10kPa

So, minimum power require  m gh f
  P 
 mg 
 g 
1  100 1000
 100Watts
1000

 ICP–Intensive Classroom Program  eGATE-Live Internet Based Classes DLP  TarGATE-All India Test Series
Leaders in GATE Preparations  65+ Centers across India
© All rights reserved by Gateforum Educational Services Pvt. Ltd. No part of this booklet may be reproduced or utilized in any form without the written permission.

4
|ME| GATE-2017-PAPER-I www.gateforum.com

14. For steady flow of a viscous incompressible fluid through a circular pipe of constant diameter,
the average velocity in the fully developed region is constant. Which one of the following
statements about the average velocity in the developing region is TRUE?
(A) It increases until the flow is fully developed.
(B) It is constant and is equal to the average velocity in the fully developed region.
(C) It decreases until the flow is fully developed.
(D) It is constant but always lower than the average velocity in the fully developed region.
Key: (B)
Exp: The average velocity in pipe flow always be same either for developing flow or fully
developed flow.

15. Cylindrical pins of diameter 150.020 mm are being produced on a machine. Statistical quality
control tests show a mean of 14.995 mm and standard deviation of 0.004mm. The process
capability index Cp is
(A) 0.833 (B) 1.667 (C) 3.333 (D) 3.750
Key: (B)
USL- LSL 15.02  14.98
Exp: Cp    1.666
6σ 6  0.004

2 0 1 
16. The product of Eigen values of the matrix P is P   4 3 3
0 2 1
(A) -6 (B) 2 (C) 6 (D) -2
Key: (B)
2 0 1 
Exp: P   4 3 3
0 2 1
We know that, product of eigen values of P = determinant of P
 2  3  6   0  18   6  8  2

17. Match the processes with their characteristics.


Process Characteristics
P : Electrical Discharge machining 1. No residual stress
Q : Ultrasonic machining 2. Machining of electrically conductive
materials
R : Chemical machining 3. Machining of glass
S : Ion Beam Machining 4. Nano-machining
(A) P – 2, Q – 3, R – 1, S – 4 (B) P – 3, Q – 2, R – 1, S – 4
(C) P – 3, Q – 2, R – 4, S – 1 (D) P – 2, Q – 4, R – 3, S – 1
Key: (A)

 ICP–Intensive Classroom Program  eGATE-Live Internet Based Classes DLP  TarGATE-All India Test Series
Leaders in GATE Preparations  65+ Centers across India
© All rights reserved by Gateforum Educational Services Pvt. Ltd. No part of this booklet may be reproduced or utilized in any form without the written permission.

5
|ME| GATE-2017-PAPER-I www.gateforum.com

x 3  sin  x 
18. The Value of limx 0 is
x
(A) 0 (B) 3 (C) 1 (D) -1
Key: (D)

Exp: t
x 0
x 3  sin x
x x 0
 
 t x 2   t
 x 0 x 

sin x 
  0  1  1

19. In an arc welding process, welding speed is doubled. Assuming all other process parameters to
be constant, the cross sectional area of the weld bead will
(A) Increase by 25% (B) Increase by 50% (C) Reduce by 25% (D) Reduce by 50%
Key: (D)
Exp:  V.I  H m A.V
1
A
V
A 2 V1 V
 
A1 V2 2V
A1
A2 
2
By doubling welding speed, Area reduces by 50%

20. A six-face fair dice is rolled a large number of times. The mean value of the outcomes is
______.
Key: 3.5 to 3.5
Exp: The Probabilities corresponding to the outcomes are given below:

Face 1 2 3 4 5 6
Probability 1/6 1/6 1/6 1/6 1/6 1/6

mean  E  x    x.P  x 
1 1  6   2  16   3 16   4  16   5  16   6  16 
1 21
 1  2  3  4  5  6   3.5
6 6


21. Consider the two dimensional velocity field given by V  5  a1x  b1y  i   4  a 2 x  b2 y  j,
wher a1 , b1 , a 2 and b2 are constants. Which one of the following conditions needs to be
satisfied for the flow to be incompressible?
(A) a1  b1  0 (B) a1  b 2  0 (C) a 2  b2  0 (D) a 2  b1  0
Key: (B)

Exp: Given V   5  a1 x  b1 y  i   4  a 2 x  b 2 y  j
 u V
For Incompressible V  0; i.e.,  0
x y
a1  b 2  0

 ICP–Intensive Classroom Program  eGATE-Live Internet Based Classes DLP  TarGATE-All India Test Series
Leaders in GATE Preparations  65+ Centers across India
© All rights reserved by Gateforum Educational Services Pvt. Ltd. No part of this booklet may be reproduced or utilized in any form without the written permission.

6
|ME| GATE-2017-PAPER-I www.gateforum.com

22. Consider a beam with circular cross-section of diameter d. The ratio of the second moment of
area about the neutral axis to the section modulus of the area is.
d d
(A) (B) (C) d (D) d
2 2
Key: (A)
 4
d
I I 64 d
Exp: Z y  
y Z 
d3 2
32

23. Saturated steam at 100°C condenses on the outside of a tube. Cold fluid enters the tube at 20°
C and exists at 50°C. The value of the Log Mean Temperature Difference (LMTD) is
________°C.
Key: 63.5 to 64 S
o  i 80  50
Exp: LMTD=  100C 100C 
n  o i  n  80 50  i  50C  
LMTD  63.82C 50C  o  80C
20C 

24. In a metal forming operation when the material has just started yielding, the principal stresses
are 1  180 MPa, 2  100 MPa, 3  0. Following Von Mises criterion, the yield stress is
________ MPa.
Key: 245 to 246
Exp: As per Von-Mises criteria
 1  2    1  3    5  1   2m2
2 2 2

180  100    100  0    0  180   22m


2 2 2

m  245.76MPa

25. In the engineering stress-strain curve for mild steel, the Ultimate Tensile Strength (UTS) refers
to
(A) Yield stress (B) Proportional limit
(C) Maximum stress (D) Fracture stress.
Key: (C)

Q. No. 26 to 55 Carry Two Marks Each

 u   u 
26. A parametric curve defined by x  cos   , y  sin   in the range 0  u  1 is rotated
 2   2 
about the X – axis by 360 degrees. Area of the surface generated is.

(A) (B)  (C) 2  (D) 4 
2
Key: (C)
 ICP–Intensive Classroom Program  eGATE-Live Internet Based Classes DLP  TarGATE-All India Test Series
Leaders in GATE Preparations  65+ Centers across India
© All rights reserved by Gateforum Educational Services Pvt. Ltd. No part of this booklet may be reproduced or utilized in any form without the written permission.

7
|ME| GATE-2017-PAPER-I www.gateforum.com

 u   u 
Exp: Given x  cos   , y  sin   0    1
 2   2 
dx   u 
 sin  
d 2  2 
dy  u
 cos
dx 2 2
We know that surface area when the curve revolved about X- axis of a parametric curve is
2 2 2 2
 u     u      u  
1 1
 dx   dy 
 2 y      du  2 sin    sin      cos    du
0  du   du  0  2   2  2   2  2 
1
 u 
1
u  
2
u  1cos 2 
1
2  
 2 sin du  2   sin dx   
2
    cos  cos 0 
2

2 4 20 2    2 
0
2 
 0
 2 cos 0  1  2

27. Assume that the surface roughness profile is triangular as shown schematically in the figure. If
the peak to valley height is 20 m, The central line average surface roughness Ra (in m ) is

(A) 5 (B) 6.67 (B) 10 (D) 20


Key: (A)
R max 20
Exp: Ra    5m
4 4

28. A thin uniform rigid bar of length L and mass M is hinged at point O, located at a distance of
L
from one of its ends. The bar is further supported using springs, each of stiffness k, located
3
M
at the two ends. A particle of mass m  is fixed at one end of the bar, as shown in the
4
figure. For small rotations of the bar about O, the natural frequency of the systems is.

5k 5k 3k 3k
(A) (B) (C) (D)
M 2M 2M M
Key: (B)
Exp: Max moment of inertia of Rod.

 ICP–Intensive Classroom Program  eGATE-Live Internet Based Classes DLP  TarGATE-All India Test Series
Leaders in GATE Preparations  65+ Centers across India
© All rights reserved by Gateforum Educational Services Pvt. Ltd. No part of this booklet may be reproduced or utilized in any form without the written permission.

8
|ME| GATE-2017-PAPER-I www.gateforum.com

I0  Ic  mr 2 L
2 L
ML2  2L L  m
I0   M   3
12  3 2
ML2 ML2 ML2 0
I0    K K
12 36 9
Mass moment of inertia of particular mass
2
M  2L  ML2
I particular    
4  3  9
ML2 ML2 2(ML2 )
ITotal   
9 9 9
 M0  0
 L  L   2L  2L  
K      K      I  0
 3  3   3  3 
5L2
 2ML 2
   5L 2
 .k
9 5k
   K    0  n  
 9   9  2ML2 2M
9

29. A point mass of 100 kg is dropped onto a massless elastic bar (cross-sectional area = 100 mm2,
length = 1m, Young‟s moduls = 100 GPa) from a height H of 10mm as shown (Figure is not to
scale). If g = 10m/s2, the maximum compression of the elastic bar is _______ mm.
m  100kg
H  10mm

L  1m
Key: 1.50 to 1.52
Exp: Given that
m  100kg, g  10m sec 2 , E  100GPa
H  10mm, L  1m  100mm,
A  100mm2
From the given figure, we can say that this is case of Impact loading,
We know that, stress due to Impact load is
2
P  P   2PhE 
I.L      
A  A   AL 
P  mg  100  10  1000N

 1000   2  1000  10  1  10 
2 5
1000
I.L    
    151.7745 N mm
2

100  100   100  1000 


I.L  L 151.7745  1000
Compression       1.5177 mm
E 1  105

 ICP–Intensive Classroom Program  eGATE-Live Internet Based Classes DLP  TarGATE-All India Test Series
Leaders in GATE Preparations  65+ Centers across India
© All rights reserved by Gateforum Educational Services Pvt. Ltd. No part of this booklet may be reproduced or utilized in any form without the written permission.

9
|ME| GATE-2017-PAPER-I www.gateforum.com

30. One kg of an ideal gas (gas constant, R = 400 J/kg.K; specific heat at constant volume,
c  1000J/kg.K) at 1 bar, and 300 K is contained in a sealed rigid cylinder. During an
adiabatic process, 100kJ of work is done on the system by a stirrer. The increase in entropy of
the system is _________ J/K.
Key: 286 to 288
Exp: Given that m  1kg, R  400 kJ kgK, CV  1000 J kgK
P1  1 bar, T1  300K
Since the gas is contained in a sealed rigid cylinder, and given that adiabatic process is done to
the system, means no heat is transferred from/to the system, Q = 0
And we know from first law of thermo dynamics, Adiabatic wall, Q  0
Q  du  W
0  mCV  T2  T1   100  103  du  mC V  T2  T1  
mCV  T2  T1   100  103 still Q work
given  100 kJ
1  1000  T2  300   100  103
100  103
T2   300  400K
1000
We know that the first law of thermodynamics can be written as
Tds  mC V dT  pdV
dT P  mRT 
ds  mC V  dv  PV  mRT  P  
T T  V 
dT mRT dT dV
ds  mC V  dV  mC V  mR
T rT T V
T  V 
Entropy increase  S2  S1   mCV n  2   mR n  2 
 T1   V1 
Since the above process is a constant volume process that is V2  V1
 400 
 S2  S1   1  1000   n    287.6821 J K
 300 

31. For an inline slider-crank mechanism, the lengths of the crank and connecting rod are 3m and
4m, respectively. At the instant when the connecting rod is perpendicular to the crank, if the
velocity of the slider is 1m/s, the magnitude of angular velocity (upto 3 decimal points
accuracy) of the crank is _________ radian/s.
Key: 0.26 to 0.27
Exp: Given that velocity of slider
VB  1m / sec A
Length of crank (OA) = 3m 4mm
Length of connecting rod (AB) = 4m 3mm
From the configuration diagram  B
A O 5mm
sin      53.130o C
5

 ICP–Intensive Classroom Program  eGATE-Live Internet Based Classes DLP  TarGATE-All India Test Series
Leaders in GATE Preparations  65+ Centers across India
© All rights reserved by Gateforum Educational Services Pvt. Ltd. No part of this booklet may be reproduced or utilized in any form without the written permission.

10
|ME| GATE-2017-PAPER-I www.gateforum.com

The velocity diagram for the above configuration diagramO,c


is 1m sec
Oa b
 From velocity diagram sin 53.13o 
Ob 53.130o
 Oa  1 sin 53.130o  0.8
 VA  Oa  OA  OA  0.8
a
0.8
Angular velocity of crank  OA    0.267 rad sec
3

32. In an epicyclic gear train, shown in the figure, the outer


ring gear is fixed, while the sun gear rotates
counterclockwise at 100rpm. Let the number of teeth on
the sun, planet and outer gears to be 50, 25, and 100,
respectively. The ratio of magnitudes of angular velocity
of the planet gear to the angular velocity of the carrier
arm is _________.

Key: 3 to 3
Exp: TS = 50
TP=25
TR=100

Arm
S.No Condition of motion Gear S Gear P Gear R
Gear A
Arm is fixed gear S Ts Ts
1 0 +1  
with +1 revolution TP TR
Arm is fixed Gear S Ts Ts
2 0 +x -x x
with +x revolution TP TR
Arm with + y
3 y y y y
revolution
Ts Ts
4 Total y x+y yx y x
TP TR

NS  x  y  100 ...(1)
50
NR  y  x0
100
y  0.5x ...  2 
x  0.5x  100
100
x  66.66rpm
1.5
y  33.34 rpm
 50 
N P  33.34   66.66    99.99rpm
 25 
NP 99.99
  3  approx 
N arm 33.32

 ICP–Intensive Classroom Program  eGATE-Live Internet Based Classes DLP  TarGATE-All India Test Series
Leaders in GATE Preparations  65+ Centers across India
© All rights reserved by Gateforum Educational Services Pvt. Ltd. No part of this booklet may be reproduced or utilized in any form without the written permission.

11
|ME| GATE-2017-PAPER-I www.gateforum.com

33. Moist air is treated as an ideal gas mixture of water vapor and dry air (molecular weight of air
= 28.84 and molecular weight of water = 18). At a location, the total pressure is 100 kPa, the
temperature is 30°C and the relative humidity is 55%. Given that the saturation pressure of
water at 30°C is 4246 Pa, the mass of water vapor per kg of dry air is _____________ grams.
Key: 14.7 to 15.1
Exp: Ma  28.84
M w 18
P 100kPa, T  30C, RH  55%,
Ps  4246Pa
Pv P
  0.55  v  Pv  2335.3Pa
Ps 4246
 P   2335.3 
 0.622  v   0.622   
 P  Pv   100000  2335.3 
 0.01487 kg of vapour kg of D.A
14.87 gm of vapour kg of D.A

34. Following data refers to the jobs (P, Q, R, S) which have arrived at a machine for scheduling.
The shortest possible average flow time is ___________ days.

Job Processing Time (days)


P 15
Q 9
R 22
S 12

Key: 31 (not matching with IIT key)


Exp: For shortest avg. flow time SPT rule is used
Job Sequence Processing Time In Out Flow Time
Q 9 0 9 9
S 12 9 21 21
P 15 21 36 36
R 22 36 58 58
9  21  36  58
Min Avg. Flow time   31 days
4

35. Two models, P and Q, of a product earn profits of Rs. 100 and Rs. 80 per piece, respectively.
Production times for P and Q are 5 hours and 3 hours, respectively, while the total production
time available is 150 hours. For a total batch size of 40, to maximize profit, the number of
units of P to be produced is ____________.
Key: 15 to 15
Exp: Let x1  No. of units of P
x 2  No. of units of Q

 ICP–Intensive Classroom Program  eGATE-Live Internet Based Classes DLP  TarGATE-All India Test Series
Leaders in GATE Preparations  65+ Centers across India
© All rights reserved by Gateforum Educational Services Pvt. Ltd. No part of this booklet may be reproduced or utilized in any form without the written permission.

12
|ME| GATE-2017-PAPER-I www.gateforum.com
x2
max. z  100 x1  80x 2  0,50 
5x1  3x 2  150
x1  x 2  40  0, 40  15, 25 
Z 0,40  Rs.3200
Z15,225  Rs.3500  max.profit  0,0 
x1
 30,0   40,0 
So, for maximum profit, No. of units of P produced is 15 units.

36. Circular arc on a part profile is being machined on a vertical CNC milling machine. CNC part
program using metric units with absolute dimensions is listed below:
--------------------------------
N60 G01 X 30 Y 55 Z – 5 F 50
N70 G02 X 50 Y 35 R 20
N80 G01 Z 5
--------------------------------
The coordinates of the centre of the circular arc are :
(A) (30, 55) (B) (50, 55) (C) (50, 35) (D) (30, 35)
Key: (D)
Exp: y

 30,56 

20

 50,35 
 30,35 

x
20

Centre of circular arc is (30, 35)


37. Two black surfaces, AB and BC, of lengths 5m and 6m,
respectively, are oriented as shown. Both surfaces extend
infinitely into the third dimension. Given that view factor
F12=0.5, T1=800K, T2=600K, Tsurrounding=300K and Stefan
Boltzmann constant,   5.67 108 W /  m2 K 4  , the heat
transfer rate from Surface 2 to the surrounding environment is
____________ kW.
Key: 13.7 to 13.9
Exp: Given that two black surfaces „AB‟ and „BC‟

Length of AB = 5m, BC = 6 m A
And temperature of
Surface „1‟  TBC  T1   800o K Surface '1'
Surroundings'3'
Temperature of surface „2‟  TAB  T2  600o K 

Temperature of surroundings  T3   300o K


B C

 ICP–Intensive Classroom Program  eGATE-Live Internet Based Classes DLP  TarGATE-All India Test Series
Leaders in GATE Preparations  65+ Centers across India
© All rights reserved by Gateforum Educational Services Pvt. Ltd. No part of this booklet may be reproduced or utilized in any form without the written permission.

13
|ME| GATE-2017-PAPER-I www.gateforum.com

  5.67  108 W / m 2 K 4
F12  0.5
W.K.T. F11  F22  0, sin ce they areflat surfaces
F21  F22  F23  1
A1F12  A 2 F21
  6  0.5    5  F21  Assume unit width for surfaces 
3
 F21   0.6
5
0.6  0  F23  1  F23  1  0.6  0.4
Using resistance concept we can draw as follows
Since surfaces are black and area of surrounding is large we can write  1 
E b1  J1 , E b2  J 2 , E b3  J 3  E b2  J 2   A 2 F23   E b3  J 3 
E  E b3   T2  T3 
4 4

Q 23  b2  2 3
 1  1
  A 2 F23 1 1
 A 2 F23 
 5.67  108   6004  300 4   5  0.4
A2 F21 A1F13

 13.778kW / metre
1
 1 0 1  E b1  J1
 2 2 

38. Consider the matrix P   0 1 0 .
 1 0 1 

 2 2 
Which one of the following statements about P is INCORRECT?
(A) Determinant of P is equal to 1.
(B) P is orthogonal.
(C) Inverse of P is equal to its transpose.
(D) All Eigen values of P are real numbers
Key: ( D)
 1 0 1 
 2 2 

Exp: P  0 1 0 
 1 0 1 
 
 2 2 
1  1  1  1  1 1
P    0  0      1
2 2  2 2 2 2
 1 0 1  1 0 1 
 2    
 2 2 2  1 0 0 
P.P T  0 1 0  0 1 0   0 1 0 
 1 0 1  1 0 
1  0 0 1 
 
 2 2   2 2 

 ICP–Intensive Classroom Program  eGATE-Live Internet Based Classes DLP  TarGATE-All India Test Series
Leaders in GATE Preparations  65+ Centers across India
© All rights reserved by Gateforum Educational Services Pvt. Ltd. No part of this booklet may be reproduced or utilized in any form without the written permission.

14
|ME| GATE-2017-PAPER-I www.gateforum.com

P is an orthogonal matrix
(A) Is correct  Inverse of P is its transpose only
(B) and (C) both are correct
(D) is incorrect

39. The Pressure ratio across a gas turbine (for air, specific heat at constant pressure,
c p  1040J / kg.K and ratio of specific heats,   1.4) is 10. If the inlet temperature to the
turbine is 1200K and the isentropic efficiency is 0.9, the gas temperature at turbine exit is
______ K.
Key: 675 to 684
Exp: Cp  1040 J kg.K , r 1.4
P2 P1  10, T3 1200K
is  0.9
T
P2
 Isentropic Expansion,
r 1 3 P1
 P3  r  T3  1200 2
      10 
0.4/1.4

 P4   T4  T4
41
T4  621.5K 4
1
T3  T 1
S
iso  4

T3  T4
So, T41  T3  iso  T3  T4   1200  09 1200  621.5 
T4  679.38K

40. An initially stress-free massless elastic beam of length L and circular cross-section with
diameter d (d << L) is held fixed between two walls as shown. The beam material has Young‟s
modulus E and coefficient of thermal expansion  .

If the beam is slowly and uniformly heated, the temperature rise required to cause the beam to
buckle is proportional to
(A) d (B) d2 (C) d3 (D) d4
Key: (B)
EI
Exp: P  EA  T  
 2e
 1  I
T    
  A
 4 d
d L
 1 64
T   
   d2
4
T  d 2
 ICP–Intensive Classroom Program  eGATE-Live Internet Based Classes DLP  TarGATE-All India Test Series
Leaders in GATE Preparations  65+ Centers across India
© All rights reserved by Gateforum Educational Services Pvt. Ltd. No part of this booklet may be reproduced or utilized in any form without the written permission.

15
|ME| GATE-2017-PAPER-I www.gateforum.com

 
41.  the value of .   V is ____________
For the vector V  2yzi  3xzj  4xyk,  
Key: 0 to 0

Exp: V  2yzi  3xzj  4xyk
we know that .   V   0 for any vector V

42. A 10 mm deep cylindrical cup with diameter of 15mm is drawn from a circular blank.
Neglecting the variation in the sheet thickness, the diameter (upto 2 decimal points accuracy)
of the blank is _________ mm.
Key: 28.71 to 28.73

Exp: D  d2  4dh  152  4 10 15  28.72 mm

43. A machine element has an ultimate strength  u  of 600 N/mm2, and endurance limit   en  of
250 N/mm2. The fatigue curve for the element on log-log plot is shown below. If the element
is to be designed for a finite of 10000 cycles, the maximum amplitude of a completely
reversed operating stress is _________ N/mm2.

Key: 370 to 390 (0.8 u )


A
Exp: u  600MPa

en  250MPa B
e
N  10000 cycle
log  0.8u   log  250  log  0.8u   log   

36 34
log  480   log  250  log  480   log    103 104 106

3 1
3log  480   log  480   log  250  2log  480   log  250 
log     
3 3
max  386.19MPa

44. A sprue in a sand mould has a top diameter of 20mm and height of 200mm. The velocity of
the molten metal at the entry of the sprue is 0.5m/s. Assume acceleration due to gravity as 9.8
m/s2 and neglect all losses. If the mould is well ventilated, the velocity (upto 3 decimal points
accuracy) of the molten metal at the bottom of the sprue is ________ m/s.

Key: 2.04 to 2.07

 ICP–Intensive Classroom Program  eGATE-Live Internet Based Classes DLP  TarGATE-All India Test Series
Leaders in GATE Preparations  65+ Centers across India
© All rights reserved by Gateforum Educational Services Pvt. Ltd. No part of this booklet may be reproduced or utilized in any form without the written permission.

16
|ME| GATE-2017-PAPER-I www.gateforum.com

Exp: Apply Bernoulli‟s between (1) and (2)


P1 V12 P V2 1
  z1  2  2  z 2
 2g  2g
0.52 V22
 0.2  200 mm
2g 2g
V2  2.042 m s
2

45. Air contains 79% N2 and 21% O2 on a molar basis. Methane (CH4) is burned with 50% excess
air than required stoichiometrically. Assuming complete combustion of methane, the molar
percentage of N2 in the products is ________________
Key: 73 to 74
 79  79
Exp: Stoichiometric reaction CH4  2. O2  N2   2H2O  CO2  2   N2
 21  21
50% excess air
 79  79
CH 4  3. O 2  N 2   2H 2O  CO 2  3   N 2  O 2
 21  21
3  79
%N 2  21 100  73.83%
79
2 11 3
21

46. P(0,3), Q(0.5, 4), and R (1,5) are three points on the curve defined by f(x). Numerical
integration is carried out using both Trapezoidal rule and Simpson‟s rule within limits x = 0
and x =1 for the curve. The difference between the two results will be.
(A) 0 (B) 0.25 (C) 0.5 (D) 1
Key: (A)
Exp: Let x 0 0.5 1
y 3 4 5
Trapezoidial rule
1
0.5 0.5
 f  x  dx 
0
2 
 3  5   2  4   
2
 16  4

Simpsons rule
1
0.5 0.5
 f  x  dx 
0
3
 3  5   0  4  4   
3
 24  4

Difference  0

47. Heat is generated uniformly in a long solid cylindrical rod (diameter = 10mm) at the rate of
4×107 W/m3. The thermal conductivity of the rod material is 25W/m.K. Under steady state
conditions, the temperature difference between the centre and the surface of the rod is
_________ °C.
Key: 10 to 10

 ICP–Intensive Classroom Program  eGATE-Live Internet Based Classes DLP  TarGATE-All India Test Series
Leaders in GATE Preparations  65+ Centers across India
© All rights reserved by Gateforum Educational Services Pvt. Ltd. No part of this booklet may be reproduced or utilized in any form without the written permission.

17
|ME| GATE-2017-PAPER-I www.gateforum.com

Exp: Given that, heat is generated uniformly i.e., g  4 107 W/ m3

Diameter at the rod (d) = 10 mm


Thermal conductivity of the rod (K) = 25 W/mK

dT To
0
dr
Temperatureprofile
TS T3
g  4 107 W / m3
K  25W / m3K
d  10mm

W.K.T for steady state, with internal heat generation, the conduction equation will be,
1 d  dT  g
 r.    0
r dr  dr  k
d  dT  gr
 r  0
dr  dr  K
After Integration once
dT gr 2 dT gr  c
r.   C1    1  (1)
dr 2K dr 2K r
After second Integration
 2
gr
Tr   C1n  r   C 2  (2)
4K
dT
Substituting boundary condition of  0at r  0in eq(1)
dr
That gives C1  0
 2
gr
 Tr   C 2 , substitute at r =0, T(r) = To
4k
 To   o   C2  C2  To
 2
gr
T  r    To
4K
10
Substitute r  ro   5mm  0.005m andT  r   TS
2
The above equation will become
 2
gr
TS  o  T o
4K
4  107  0.0052
 To  Ts   10o C
4  25

 ICP–Intensive Classroom Program  eGATE-Live Internet Based Classes DLP  TarGATE-All India Test Series
Leaders in GATE Preparations  65+ Centers across India
© All rights reserved by Gateforum Educational Services Pvt. Ltd. No part of this booklet may be reproduced or utilized in any form without the written permission.

18
|ME| GATE-2017-PAPER-I www.gateforum.com

48. Two disks A and B with identical mass (m) and radius (R) are initially at rest. They roll down
from the top of identical inclined planes without slipping. Disk A has all of its mass
concentrated at the rim, while Disk B has its mass uniformly distributed. At the bottom of the
plane, the ratio of velocity of the center of disk A to the velocity of the center of disk B is.
3 3
(A) (B) (C) 1 (D) 2
4 2
Key: (A)
Exp: IA  MR 2
MR 2
IB  A,B
2
m

P.E   K.E T  K.E R A   K.E T  K.E R B
1 1 1 1 H
 MVA2  IA2  MVB2  IB2
2 2 2 2
1 1 1 1 MR 2 2
 MVA2  MR 2 2A  MVB2  B
2 2 2 2 2
VB2
VA2  VA2  VB2 
2
3 V 3
2VA2  VB2  A 
2 VB 4

49. A block of length 200mm is machined by a slab milling cutter 34mm in diameter. The depth of
cut and table feed are set at 2mm and 18mm/minute, respectively. Considering the approach
and the over travel of the cutter to be same, the minimum estimated machining time per pass is
_____________ minutes.
Key: 12 to 12
2  Dd  d 2  L 2  (34  2)  2  200
2
Exp: Milling Time   12
f 18
Where, D  Dia of cutter  mm  , d  depth of cut  mm 
L  length f  feed  mm min 

50. A horizontal bar, fixed at one end (x = 0), has a length of 1 m, and cross-sectional area of
100mm2. Its elastic modulus varies along its length as given by E(x) = 100e -x GPa, Where x is
the length coordinate (in m) along the axis of the bar. An axial tensile load of 10 kN is applied
at the free end (x=1). The axial displacement of the free end is _______ mm.
Key: 1.70 to 1.72
Exp: Given that

P  10kN  10  103 N, A  100mm 2 Stripof lengthdx


x
100  e  10 9
x
E  x   100e x Gpa   105  e x dx
106
P  10kN
P  dx 
Change in the length of small strip   x0 x 1
AE x
L  1mm

 ICP–Intensive Classroom Program  eGATE-Live Internet Based Classes DLP  TarGATE-All India Test Series
Leaders in GATE Preparations  65+ Centers across India
© All rights reserved by Gateforum Educational Services Pvt. Ltd. No part of this booklet may be reproduced or utilized in any form without the written permission.

19
|ME| GATE-2017-PAPER-I www.gateforum.com

L
Pdx
Total change in the length of the bar     
0
AE x
L L L 1
P dx P dx P P P
 x
 5  x
 5 
e x dx  5 
e x dx  e1  1
5 
0
A 100e A  10 0 e A  10 0 A 10 0 A 10
10  103
  2.7183  1 1000  1.7183mm
100  105

51. Consider steady flow of an incompressible fluid through two long and straight pipes of
diameters d1 and d2 arranged in series. Both pipes are of equal length and the flow is turbulent
in both pipes. The friction factor for turbulent flow though pipes is of the form, f  K(Re) n
where K and n are known positive constants and Re is the Reynolds number. Neglecting minor
 P 
losses, the ratio of the frictional pressure drop in pipe 1 to that in pipe 2,  1  , is given by
 P2 
 5 n  5  3 n   5 n 
d  d  d  d 
(A)  2  (B)  2  (C)  2  (D)  2 
 d1   d1   d1   d1 
Key: (A)
 f L Q2  12d 52g  f1  d 2   Re1   d 2 
5 n 5
P1  64 
Exp:   1 15  2 
    n   ......(1)  f  
P2  12d1 g  f 2 L2 Q  f 2  d1   Re2   d1   Re 

Vd   4Q 
Re  & Q  AV  d 2 V   V  2 
 4  d 
4Q d 4Q
So, Re  2  
d  d
d1 d2
5 L1  L2
1 P d n  d 
Re  From(1) 1  1 n  2 
d P2 d 2  d1 
5 n
P1  d 2  L1 L2
 
P2  d1 

52. The velocity profile inside the boundary layer for flow over a flat plate is given as
u  y
 sin   , where U is the free stream velocity and  is the local boundary layer
U 2 
*
thickness. If  * is the local displacement thickness, the value of is

2 2 2
(A) (B) 1  (C) 1+ (D) 0
  
Key: (B)
U  y 
Exp:  sin  
U  2 
displacement thickness
  y 
*   1  U U  dy  1  sin  dy
0
 2 

 ICP–Intensive Classroom Program  eGATE-Live Internet Based Classes DLP  TarGATE-All India Test Series
Leaders in GATE Preparations  65+ Centers across India
© All rights reserved by Gateforum Educational Services Pvt. Ltd. No part of this booklet may be reproduced or utilized in any form without the written permission.

20
|ME| GATE-2017-PAPER-I www.gateforum.com


cos  y 2   0
  
*

  2 
2
*    cos  2  cos 0

 * 2 
     
 
   1  2  
*


53. For a steady flow, the velocity field is V   x 2  3y  i   2xy  j. The magnitude of the
acceleration of a particle at (1, -1) is
(A) 2 (B) 1 (C) 2 5 (D) 0
Key: (C)

Exp: V   x 2  3y  i   2xy  j
U   x 2  3y
 
V  2xy

U U
a x  U   V     x 2  3y   2x    2xy  3  2x 3  6xy  6xy  2x 3
 x  y               
 
V V
a y  U   V     x 2  3y   2y    2xy  2x   2x 2 y  6y 2  4x 2 y  2x 2 y  6y 2
 x  y                 
x y
but x  1, y  1
but x  1, y  1

ax  2
   2       2  
a y  2 1  1  6  1  6  2  4
    
a  a 2x  a 2y  4  16  20

a2 5

54. Two cutting tools with tool life equations given below are being compared:
Tool 1: VT0.1=150
Tool 2: VT0.3=300
Where V is cutting speed in m/minute and T is tool life in minutes. The breakeven cutting
speed beyond which Tool 2 will have a higher tool life is ________ m/minute.
Key: 105 to 107
Exp: At Breakeven point
T1  T2
1 0.1 1 0.3
 150   300 
   
 V   V 
V  106.121 m min

 ICP–Intensive Classroom Program  eGATE-Live Internet Based Classes DLP  TarGATE-All India Test Series
Leaders in GATE Preparations  65+ Centers across India
© All rights reserved by Gateforum Educational Services Pvt. Ltd. No part of this booklet may be reproduced or utilized in any form without the written permission.

21
|ME| GATE-2017-PAPER-I www.gateforum.com

55. A rectangular region in a solid is in a state of plane strain. The (x,y) coordinates of the corners
of the under deformed rectangle are given by P(0,0), Q (4,0), S (0,3). The rectangle is
subjected to uniform strains,  xx  0.001,  yy  0.002,  xy  0.003. The deformed length of the
elongated diagonal, up to three decimal places, is _________ units.
Key: 5.013 to 5.015
Exp: Given that

 xx  0.001,  yy  0.002
rxy  0.003

Length of the diagonal  PR   42  32  5m


y
 yy
S  0,3 R  4,3 

5mm  xx
 xx rxy
x
P  0,0  Q  4,0 
3  yy
tan 1  
4
To find the diagonal (PR) strain, the direction of the plane angle from the +ve x-axis will from
„R‟ towards „P‟
  ?
Where   180  tan 1  3 / 4   216.87o
 xx   yy  xx   yy rxy
 216.87o   cos 2 
sin 2
2 2 2
0.001  0.002 0.001  0.002
sin  2  216.87 o 
0.003
  cos  2  216.87  
2 2 2
 0.0015  1.4  104  1.44  103
 2.8  103
Elongation of the diagonal = 216.87  5  2.8 103 • 5  0.014
Defined length of diagonal  5  0.014  5.014

 ICP–Intensive Classroom Program  eGATE-Live Internet Based Classes DLP  TarGATE-All India Test Series
Leaders in GATE Preparations  65+ Centers across India
© All rights reserved by Gateforum Educational Services Pvt. Ltd. No part of this booklet may be reproduced or utilized in any form without the written permission.

22
|ME| GATE-2017-PAPER-I www.gateforum.com

General Aptitude
Q. No. 1 - 5 Carry One Mark Each
1. A right – angled cone (with base radius 5cm and height 12cm), as shown in the figure below,
is rolled on the ground keeping the point P fixed until the point Q (at the base of the cone, as
shown) touches the ground again.

By what angle (in radians) about P does the cone travel?


5 5 24 10
(A) (B) (C) (D)
12 24 5 13
Key: (D)
Exp: L  52  122 13cm 
L
Circumference of base circle = length of arc QR
R P
(2r)  R (R  Slant height of the Cone  13 cm)
(2  5) 13
10 R

13
2. In a company with 100 employees, 45 earn Rs. 20,000 per month, 25 earn Rs. 30,000, 20 earn
Rs. 40,000,8 earn Rs. 60,000, and 2 earn Rs. 150,000. The median of the salaries is
(A) Rs. 20,000 (B) Rs.30,000 (C) Rs. 32,300 (D) Rs. 40,000
Key: (B)
Exp: All the values put either in ascending or descending order first.
Now number of observations equal to 100 [even]
The median of these values = Avg of two middle most observations.
50 th observation  51st observation 30,000  30,000
   30,000
2 2

3. As the two speakers became increasingly agitated, the debate became __________.
(A) lukewarm (B) poetic (C) forgiving (D) heated
Key: (D)

4. P,Q, and R talk about S‟s car collection. P states that S has at least 3 cars. Q believes that S has
less than 3 cars. R indicates that to his knowledge, S has at least one Car. Only one of P, Q and
R is right the number cars owned by S is.
(A) 0 (B) 1 (C) 3 (D) Cannot be determined
Key: (A)
Exp: P States that S has atleast 3 cars, i.e.,  3
Q believes that S has less than 3 cars, i.e.,  3
R indicates that S has atleast one car  1
P‟s and Q‟s statements are exactly opposite in nature and R‟s statement is proportional to P‟s
statement.

 ICP–Intensive Classroom Program  eGATE-Live Internet Based Classes DLP  TarGATE-All India Test Series
Leaders in GATE Preparations  65+ Centers across India
© All rights reserved by Gateforum Educational Services Pvt. Ltd. No part of this booklet may be reproduced or utilized in any form without the written permission.

23
|ME| GATE-2017-PAPER-I www.gateforum.com

From the given data, only one person statement is right as it mean that two persons statements
are wrong, i.e., P and R wrong when S has zero cars.

5. He was one of my best __________ and I felt his loss _________.


(A) friend, keenly (B) friends, keen (C) friend, keener (D) friends, keenly
Key: (D)
Q. No. 6- 10 Carry Two Marks Each

6. Two very famous sportsmen Mark and Steve happened to be brothers, and played for country K.
Mark teased James, an opponent from country E, “There is no way you are good enough to play
for your country.‟‟ James replied, “Maybe not, but at least I am the best player in my own family.”
Which one of the following can be inferred from this conversation?
(A) Mark was known to play better than James
(B) Steve was known to play better than Mark
(C) James and Steve were good friends
(D) James played better than Steve
Key: (B)

7. “Here, throughout the early 1820s, Stuart continued to fight his losing battle to allow his
sepoys to wear their caste-marks and their own choice of facial hair on parade, being again
reprimanded by the commander-in-chief. His retort that „A stronger instance than this of
European prejudice with relation to this country has never come under my observations‟ had
no effect on his superiors.”
According to this paragraph, which of the statements below is most accurate?
(A) Stuart‟s commander – in chief was moved by this demonstration of his prejudice.
(B) The Europeans were accommodating of the sepoys‟ desire to wear their caste – marks.
(C) Stuart‟s losing battle‟ refers to his inability to succeed in enabling sepoys to wear caste-
marks.
(D) The commander– in – Chief was exempt from the European preiudice that dictated how
the sepoys were to dress.
Key: (C)

8. The growth of bacteria (lactobacillus) in milk leads to curd formation. A minimum bacterial
population density of 0.8(in suitable units) is needed to form curd. In the graph below, the
population density of lactobacillus in 1 litre of milk is plotted as a function of time, at two
different temperatures, 25°C and 37°C.

Consider the following statements based on the data shown above:


 ICP–Intensive Classroom Program  eGATE-Live Internet Based Classes DLP  TarGATE-All India Test Series
Leaders in GATE Preparations  65+ Centers across India
© All rights reserved by Gateforum Educational Services Pvt. Ltd. No part of this booklet may be reproduced or utilized in any form without the written permission.

24
|ME| GATE-2017-PAPER-I www.gateforum.com

(i) The growth in bacterial population stops earlier at 37°C as compared to 25°C
(ii) The time taken for curd formation at 25°C is twice the time taken at 37°C
Which one of the following options is correct?
(A) Only i (B) only ii (C) Both i and ii (D) Neither i nor ii
Key: (A)
Exp: From the graph, Statement (i) is correct,
The time taken for curd formation @25o C= 120 min,
the time taken for curd formation @ 37o C= 80 min, hence (ii) is incorrect.

9. Let S1 be the plane figure consisting of the points (x,y) given by the inequalities x  1  2 and
y  2  3. Let S2 be the plane figure given by the inequalities x  y  2, y 1, and x  3 Let
S be the union of S1 and S2. The area of S is.
(A) 26 (B) 28 (C) 32 (D) 34
Key: (C)

10. What is the sum of the missing digits in the subtraction problem below?
5_ _ _ _
4 8 _ 8 9
1111
(A) 8 (B) 10 (C) 11 (D) Cannot be determined
Key: (D)

 ICP–Intensive Classroom Program  eGATE-Live Internet Based Classes DLP  TarGATE-All India Test Series
Leaders in GATE Preparations  65+ Centers across India
© All rights reserved by Gateforum Educational Services Pvt. Ltd. No part of this booklet may be reproduced or utilized in any form without the written permission.

25
No part of this publication may be reproduced or distributed in any form or any means, electronic, mechanical,
photocopying, or otherwise without the prior permission of the author.

GATE SOLVED PAPER


Mechanical Engineering
2015-1

Copyright © By NODIA & COMPANY

Information contained in this book has been obtained by authors, from sources believes to be reliable. However,
neither Nodia nor its authors guarantee the accuracy or completeness of any information herein, and Nodia nor its
authors shall be responsible for any error, omissions, or damages arising out of use of this information. This book
is published with the understanding that Nodia and its authors are supplying information but are not attempting
to render engineering or other professional services.

NODIA AND COMPANY


B-8, Dhanshree Tower Ist, Central Spine, Vidyadhar Nagar, Jaipur 302039
Ph : +91 - 141 - 2101150
www.nodia.co.in
email : enquiry@nodia.co.in
GATE SOLVED PAPER - ME
2015-1

General Aptitude

Q. 1 - Q. 5 Carry One Mark Each.

Q. 1 What is the adverb for the given work below?


Misogynous
(A) Misogynousness (B) Misogynity
(C) Misogynously (D) Misogynous
Sol. 1 Correct option is (C).

.i n
An adverb is a word or set of words that modifies verbs, adjectives or other

how much.
c o
adverbs. An adverb answers how, when, where, or to what extent- how often or

Example: daily, completely etc.


a .
d i
The given word is Misogynuous which is a adjective means, hating women in
particular the adverb of Misogynous is Misogynously.
Q. 2
o
Ram and Ramesh appeared in an interview for two vacancies in the same

.n
department. The probability of Ram’s selection is 16 and that of Ramesh is 18 .
What is the probability that only one of them will be selected?
(A) 47 w (B) 1

(C) 13
48
w 4
(D) 35
48
w
Correct option is (B).
48

©
Sol. 2

It is given that Ram and Ramesh appeared in an interview for two vacancies in
the same department and
Probability of Ram’s selection P ^Ramh = 1
6
Probability of Ramesh’s selection P ^Rameshh = 1
8
For the selection of only one their are two possibilities
1. Selection of Ram and Rejection of Ramesh
2. Selection of Ramesh and Rejection of Ram
Probability of only one selection is
= P 6Ram selected@ # P 6Ramesh rejected@
+ P 6Ramesh selected@ # P 6Ram Rejected@
= b 1 l;1 − b 1 lE + b 1 lb1 − 1 l
6 8 8 6
= 7 + 5 = 12 = 1
48 48 4
So, the probability of only one selection is 1 .
4

Buy Online all GATE Books: shop.nodia.co.in


*Shipping Free* *Maximum Discount*
GATE SOLVED PAPER - ME 2015-1

Q. 3 Choose the appropriate word/phrase, out of the four options given below, to
complete the following sentence:
Dhoni, as well as the other team members of Indian team, ______ present on
the occasion.
(A) were (B) was

Sol. 3
(C) has
Correct option is (B).
(D) have

.i n
Rule for subject-verb agreement for as well as
c o
a .
When the subjected is separated from the verb by such words as along with, as
well as, besides, not etc. These terms and phrases and the words which follow

the singular subject is given.


d i
these terms are not part of subject, so ignore then and use a singular verb when

o
In the given sentence, Dhoni is a singular noun and the sentence showing simple
n
.
tense sense so was will be used.
Q. 4
w
An electric bus has on board instruments that report the total electricity consumed
since the start of the trip as well as the total distance covered. During a single
w
day of operation, the bus travels on stretches M , N , O and P in that order. The

w
cumulative distances travelled and the corresponding electricity consumption are
shown in the table below.

©
Stretch Cumulative distance (km) Electricity used (kWh)
M 20 12
N 45
.i n 25
O 75
100
c o 45
57
P

a .
(A) M
d i
The stretch where the electricity consumption per km is minimum is
(B) N
(C) O

n o (D) P
Sol. 4 Correct option is (D).
.
In the table showing below electricity used per km
w
distance w
Stretch Cumulative Electricity
used
Stretch
distance
Electricity
used in that
Electricity
used per
w (total) stretch km

M
N
© 20
45
12
25
20
45 − 20 = 25
12
25 − 12 = 13
12
20
13
25
= 0.6
= 0.52
O 75 45 75 − 45 = 30 45 − 25 = 20 20
30 = 0.66
P 100 57 100 − 75 = 25 57 − 45 = 12 12
25 = 0.48

So, in the stretch P , electricity consumption per km is minimum.


Q. 5 Choose the word most similar in meaning to the given word:
Awkward
(A) Inept (B) Graceful
(C) Suitable (D) Dreadful

Buy Online all GATE Books: shop.nodia.co.in


*Shipping Free* *Maximum Discount*
GATE SOLVED PAPER - ME 2015-1

Sol. 5 Correct option is (A).


(A) Inept
Showing a lack of judgment, sense or reason; inappropriate or foolish.
Synonyms of inept: Awkward, clumsy or incompetent
(B) Graceful
having or showing grace or elegance

. i n
Synonyms of graceful: elegant, stylish, sophisticated etc
(C) Suitable

c o
a .
Right or appropriate for a particular person, purpose, or situation.
Synonyms of suitable: acceptable, satisfactory, fit etc
(D) Dreadful

d i
Causing or involving great suffering, fear, or unhappiness, extremely bad or
serious.

n o
Synonyms of Dreadful: terrible, frightful, awful etc.
.
w
w
Q. 6 - Q. 10 Carry Two Marks Each.

Q. 6 w
In the following sentence certain parts are underlined and marked P , Q and R.

©
One of the parts may contain certain error or may not be acceptable in standard
written communication. Select the part containing an error. Choose D as your
Answer : if there is no error.

answer book
R
P
. i n
The student corrected all the errors that the instructor marked on the
Q

(A) P o
(B) Q
c
(C) R

a . (D) No Error
Sol. 6 Correct option is (B).
Use of The-
d i
n
specific or particular. o
The definite article is used before singular and plural nouns when the noun is

.
The signals that the noun is definite, that it refers to a particular member of a
w
group. Here instructor is representing a group, it is not a particular or definite

w
subject so use of the before it is wrong.
Q. 7
w
Given below are two statements followed by two conclusions. Assuming these
statements to be true, decide which one logically follows.

©
Statement:
I All film stars are playback singers
II All film directors are film stars.
Conclusions:
I All film directors are playback singers.
II Some film stars are film directors.
(A) Only conclusion I follows
(B) Only conclusion II follows
(C) Neither conclusion I nor II follows
(D) Both conclusions I and II follow

Buy Online all GATE Books: shop.nodia.co.in


*Shipping Free* *Maximum Discount*
GATE SOLVED PAPER - ME 2015-1

Sol. 7 Correct option is (D).


By drawing Venn diagram of statements
Statement (1)

.i n
Statement (2)
c o
a .
d i
n o
.
Statements (1) and (2) collectively can be shown by
w
w
w
©
Both conclusions are following statements (1) and (2).
Lamenting the gradual of the arts in school curricula, a group of prominent
n
Q. 8

.i
artists wrote to the Chief Minister last year, asking him to allocate more funds to
support arts education in schools. However, no such increase has been announced
o
in this year’s Budget. The artists expressed their deep anguish at their request
c
future.
a .
not being approved, but many of them remain optimistic about funding in the

the above statements?


d i
Which of the statement (s) below is/are logically valid and can be inferred from

o
(i) The artists expected funding for the arts to increase this year.

n
.
(ii) The Chief Minister was receptive to the idea of increasing funding for the
arts.
w
(iii) The Chief Minister is a prominent artist.
w
(iv) Schools are giving less importance to arts education nowadays.

w
(A) (iii) and (iv)
(B) (i) and (iv)
©
(C) (i), (ii) and (iv)
(D) (i) and (iii)
Sol. 8 Correct option is (B).
Statements (i) and (iv) are logically valid and can be inferred from the given
statement.
Q. 9 If a2 + b2 + c2 = 1, then ab + bc + ac lies in the interval

(A) ;1, 2E (B) :- 1, 1D


3 2
(C) :- 1, 1 D (D) [2, - 4]
2

Buy Online all GATE Books: shop.nodia.co.in


*Shipping Free* *Maximum Discount*
GATE SOLVED PAPER - ME 2015-1

Sol. 9 Correct option is (B).


We know ^a + b + c h2 = o2 + b2 + c2 + 2 ^ab + bc + ca h
Given a2 + b2 + c2 = 1
So, ^a + b + c h2 = 1 + 2 ^ab + bc + ca h
Since, square is always positive quantity, so
1 + 2 ^ab + bc + ca h $ 0
. i n
ab + bc + ca $- 1
2
c o
Q. 10

a .
A tiger is 50 leaps of its own behind a deer. The tiger takes 5 leaps per minute

d i
to the deer’s 4. If the tiger and the deer cover 8 metre and 5 metre per leap
respectively, what distance in meters will the tiger have a run before it catches
the deer?
Sol. 10 Correct answer is 800.
n o
.
Current distance between tiger and deer

w = 50 tiger leaps

wSpeed of tiger = 5 leaps/mnt


= 3 # 8 = 40 m/mnt
w Speed of deer = 4 leaps/mnt

© = 4 # 5 = 20 m/mnt
Relative speed = 40 − 20 = 20 m/mnt
Time taken to catch deer = distance = 400
speed
= 20 mnt
20
. i n
o
Distance traveled by tiger before it catches the deer is
c
.
= 20 # 40 = 800 meter.

a
d i
END OF THE QUESTION PAPER

n o
.
w
w
w
©

Buy Online all GATE Books: shop.nodia.co.in


*Shipping Free* *Maximum Discount*
GATE SOLVED PAPER - ME 2015-1

Mechanical Engineering

Q. 1 - Q. 25 Carry One Mark Each.

4 7 8
Q. 1

9 6 2
.i n
If any two columns of a determinant P = 3 1 5 are interchanged, which

o
one of the following statements regarding the value of the determinant is
c
CORRECT?

a .
(A) Absolute value remains unchanged but sign will change

d i
(B) Both absolute value and sign will change
(C) Absolute value will change but sign will not change
o
(D) Both absolute value and sign will remain unchanged
n
Sol. 1 Correct option is (A). .
w
It is the fundamental property of determinant by that when any two column’s or

w
row of a determinant any two column’s or row of a determinant are interchanged
then absolute value remains unchanged but sign will change
Q. 2 w
A wheel of radius r rolls without slipping on a horizontal surface shown below.

©
If the velocity of point P is 10 m/s in the horizontal direction, the magnitude of
velocity of point Q (in m/s) is_______.

.i n
c o
a .
d i
Sol. 2 Correct answer is 20.
n o
.
The given problem is of pure rolling

w
w
w
©

So from pure rolling concept we know that the point Q will be at rest, so
VP = ωr
and for velocity of point Q will be

Buy Online all GATE Books: shop.nodia.co.in


*Shipping Free* *Maximum Discount*
GATE SOLVED PAPER - ME 2015-1

= Vp + ωr
So, VQ = 2VP
= 2 # 10
= 20 m/sec
So magnitude of velocity of point Q is 20 m/sec
Q. 3
. i n
Which one of the following types of stress-strain relationship best describes the

stress and ε = strain)?


c o
behaviour of brittle materials, such as ceramics and thermosetting plastics, (σ =

a .
d i
n o
.
w
w
w
©
Sol. 3 Correct option is (D).
. i n
We know that brittle material does not show elongation when external load
o
is applied on them and alter certain amount of load they get cracked from
c
a .
unexpected point. The amount of ultimate load and point of fracture can not be
predetermined and elongation is also very-very less so these properties are best
described by option (D).
d i
Q. 4

n
(A) control spindle speedo
The function of interpolator in a CNC machine controller is to

.
(B) coordinate feed rates of axes

w
(C) control tool rapid approach speed

w
(D) perform Miscellaneous (M ) functions (tool change, coolant control etc.)
Sol. 4
w
Correct option is (B).
Interpolator in a CNC machine controller is used for circular interpolation purpose.
©
It simple means that interpolator gives motion (or feed) to the coordinate axes
for circular interpolations.
Some of the circular interpolation codes are given as follows
GO2 = Circular interpolation, clockwise
GO3 = Circular interpolation, counter clockwise
Q. 5 Holes of diameter 25.0 ++00..040
020 mm are assembled interchangeably with the pins of
diameter 25.0 −+00..008
005
mm. The minimum clearance in the assembly will be
(A) 0.048 mm (B) 0.015 mm
(C) 0.005 mm (D) 0.008 mm

Buy Online all GATE Books: shop.nodia.co.in


*Shipping Free* *Maximum Discount*
GATE SOLVED PAPER - ME 2015-1

Sol. 5 Correct option is (B).


Clearance Fit : The clearance fit may be slide fit, easy sliding fig, running fit,
stack running fit and loose running fit.

.i n
c o
a .
Maximum clearance
Minimum clearance
d i
= upper limit of hole – lower limit of shaft
= lower limit of hole – upper limit of shaft
lower limit of hole o
= 25.0 + 0.020 mm
n
upper limit of shaft
.
= 25 + 0.005 mm
= ^25 + 0.020h − ^25 + 0.005h
Minimum clearance
w
= 0.015 mm
w
Simpson’s 13 rule is used to integrate the function f (x) = 35 x2 + 95 between x = 0
Q. 6

w
and x = 1 using the least number of equal sub-intervals. The value of the integral

Sol. 6
is_______
©
Correct answer is 2.
Numerical integration through simpson’s 13 rd rule
# 3
n

.i n
ydx = h 6^y 0 + yn h + 4 ^y1 + y 3 + y5 + ....h + 2 ^y2 + y 4 + y 6h + ...@
x

o
x 0

Given function is
F ^x h = 3 x2 + 9 between x 0 = 0 to xn + 1 .c
5 5
ia
Using the least number of equal sub-interval let take h = 1

d
4

0 1/4 1/2 3/4 2


xi = x 0 + h
y = F ^xi h 9 3

n o
+ 95 3
+ 95 27
+ 95 3
+ 95
5

. 80 20 80

#0 ydx = ^34 h ;' 95 + b 53 + 95 l1 + 4 'b 803 + 95 l + b 27


1
5

w
1

80 5 l1
+ 9 + 2 & 3 + 9 0E
20 5
1
w
#0 ydx = 121 64.2 + 15.9 + 3.9@ = 12 24 = 2

x=1
w
#x = 0 b 53 # 2 + 59 ldx = 2
Q. 7 © 0

Consider fully developed flow in a circular pipe with negligible entrance length
effects. Assuming the mass flow rate, density and friction factor to be constant,
if the length of the pipe is doubled and the diameter is halved, the head loss due
to friction will increase by a factor of
(A) 4 (B) 16
(C) 32 (D) 64
Sol. 7 Correct option is (D).
In a laminar flow through a pipe, the head loss due to friction is given by
2
hL = 8θ2 FL5
πgD

Buy Online all GATE Books: shop.nodia.co.in


*Shipping Free* *Maximum Discount*
GATE SOLVED PAPER - ME 2015-1

where hL =
head loss due to friction
D =
diameter of pipe
f =
friction factor
L =
length of pipe
hL \ L5
D
Lets take proportionality constant k . i n
hL = k L5
c o
D
hL (1) = kL5
a .
D
k ^2L h
d i
hL (2) = D 5 = 2 # 25 k L5 = 26 ^hL h(1)

n o
^2h D

.
hL (2) = 64hL (1)

w
So head loss due to friction will be increased by a factor of 64.
Q. 8
w
For an ideal gas with constant values of specific heats, for calculation of the
specific enthalpy,
w
(A) it is sufficient to know only the temperature

©
(B) both temperature and pressure are required to be known
(C) both temperature and volume are required to be known
(D) both temperature and mass are required to be known
Sol. 8 Correct option is (A).
. i n
We know that
dh = Tds + VdP
c o
From second Tds equation
a .
For an ideal gas
d i
dh = CP dT + T b−2V l dP + VdP
2T

o
PV = RT & V = RT
n
.
For const Pressure (P = cons)
P

w
Pdv = RdT

w b2T l = P
2V R

w
P

Now dh = CP dT + T b− R l dP + b RT l dP
P P
© dh = CP T
For constant valves of specific heats
dh = F ^T h only
So for calculation of specific enthalpy it is sufficient to know only the temperature.
Q. 9 A Carnot engine (CE-1) works between two temperature reservoirs A and B,
where TA = 900 K and TB = 500 K. A second Carnot engine (CE-2) works between
temperature reservoirs B and C , where TC = 300 K. In each cycle of CE-1 and
CE-2, all the heat rejected by CE-1 to reservoir B is used by CE-2. For one cycle
of operation, if the net Q absorbed by CE-1 from reservoir A is 500 KJ, the net
heat rejected to reservoir C by CE-2 (in MJ) is_______.

Buy Online all GATE Books: shop.nodia.co.in


*Shipping Free* *Maximum Discount*
GATE SOLVED PAPER - ME 2015-1

Sol. 9 Correct answer is 50.


Given data of two Carnot engines is shown in figure.

.i n
c o
a .
d i
n o
.
η carnot (1) = 1 − TB = WD1
w TA Q supplied

w Q − QA
= 1 − 500 = B
900 QA

w Q
= B −1
150
© & QB = 53.33 MJ
η carnot (2) = 1 − TC = WD2
TB Q supplied
Q − QB
= 1 − 300 = C
.i n
QC
500 QB

c o
=
53.33
−1

a .
or QC = 50 MJ

d i
So, net heat rejected to reservoir C is 50 MJ.
Q. 10

n o
A stream of moist air (mass flow rate = 10.1 kg/s) with humidity ratio of
kg
0.01 kg dry
.
air mixes with a second stream of superheated water vapour flowing

w
at 0.1 kg/s. Assuming proper and uniform mixing with no condensation, the
humidity ratio of the final stream _in kg dry
kg
air i is_______.

Sol. 10 w
Correct answer is 0.0197
w
Given problem can be drawn as shown in Figure.

Buy Online all GATE Books: shop.nodia.co.in


*Shipping Free* *Maximum Discount*
GATE SOLVED PAPER - ME 2015-1

Form mass conservation principle


m1 w1 + m2 w2 = ^m1 + m2h w 3
o 1+m o 2 w2
w 3 = m1 w
o1 + m
o2
m
^0.1h^10.1h + ^.1h^1 h
w3 =
10.1 + 0.1
w 3 = 0.0197 kg/kg dry air . i n
o
The humidity ratio of final stream is w = 0.0197 kg/kg of dry air.
c
Q. 11

a .
Consider a steel (Young’s modulus E = 200 GPa) column hinged on both sides.

buckling load (in N) is_______.


d i
Its height is 1.0 m and cross-section is 10 # 20 mm. The lowest Euler critical

Sol. 11

n o
Correct answer is 3289.868
Euler critical buckling load : A long, slender column becomes unstable when its
.
axial compressive load reaches a value called the critical buckling load. At this

w
value, the structure is in equilibrium regardless of the magnitude of the angle.

w
It is given by
2
Pcr = π EI
w l e2
Pcr = Euler’s critical buckling load
©
where
E = Young’s Modulus (E = 200 GPa)
I = Moment of Inertia
le = Equivalent length of column
. i
(Le = l for both hinged side)
n
o
Note that the least moment of inertia (I ) should be taken in order to find lowest
c
critical stress.
.
Cross section of beam = ^10 # 20h mm
a
Least moment of inertia
d i
20 # ^10h3
o
3
= bh = = I min
12 12

. n
Pcr hmin = π # 2002# 10
2

^1 h
9 ^20h^10h
# ^10 h
3
−3 4

w = 3289.868 N
12

w
So lowest Euler critical buckling load of given beam is 3289.868 N.
Q. 12 w
Air enters a diesel engine with a density of 1.0 kg/m3. The compression ratio is

©
21. At steady state, the air intake is 30 # 10-3 kg/s and the net work output is
15 kW. The mean effective pressure (kPa) is_______.
Sol. 12 Correct answer is 525.
Mean effective pressure ( Pm /mep ) : When we imagine a constant pressure such
that it gives us the some work done like the actual work done. For the same
volume covered during the actual cycle, then the imaginary constant pressure is
known as mean effective pressure.
The actual work done is given by
W = PM VS
where VS = swept volume
PM = W
VS

Buy Online all GATE Books: shop.nodia.co.in


*Shipping Free* *Maximum Discount*
GATE SOLVED PAPER - ME 2015-1

Given that net work output is


W = 15 kW
Compression ratio = total volume = 21
clearance volume
M ^kg/sech
ρ ^kg/m3h :
Swept volume = 1 − 1 D m3 / sec
21
30 # 10−3 kg/ sec
.i
:1 − 21D
1
n
o
=
1.0 kg/m3
= 0.02857 m3 / sec
. c
PM = W = 15 kW m3 /sec
VS 0.02857
i a
= 525 kPa

o
So, the mean effective pressure is 525 kPa.
d
Q. 13
.n
Match the following products with preferred manufacturing processes:
Product
w Process
P
Q
Rails
w
Engine crankshaft
(1)
(2)
Blow molding
Extrusion
R w
Aluminium channels (3) Forging

(A)
S
P-4,
© PET water bottles
Q-3, R-1, S-2
(4) Rolling

(B)
(C)
P-4,
P-2,
Q-3,
Q-4,
R-2,
R-3,
S-1
S-1
.i n
(D) P-3, Q-4, R-2, S-1
c o
Sol. 13 Correct option is (B).
a .
(1) Blow Molding

d i
Blow molding is a manufacturing process by which hollow plastic parts are formed.
Example: PET water bottles etc
(2) Extrusion
n o
.
Extrusion is process used to create objects of a fixed cross sectional profile. A

w
material is pushed or pulled through a die of the desired cross section.

w
Example: Aluminum channels etc.
(3) Forging
w
Forging is a manufacturing process involving the shaping of metal using localized
compressive forces.
©
Example: Engine crankshaft etc.
(4) Rolling
Rolling is a forming process in which metal stock is passed through one or move
pairs of rolls.
Example: Rails etc
Q. 14 Under certain cutting conditions, doubling the cutting speed reduces the tool
life to ^ 161 hth of the original. Taylor’s tool life index (n ) for this tool-workpiece
combination will be _______
Sol. 14 Correct answer is 0.25

Buy Online all GATE Books: shop.nodia.co.in


*Shipping Free* *Maximum Discount*
GATE SOLVED PAPER - ME 2015-1

Based on flank wear Taylor’s tool life equation is given as


VT n = C
where
V = Cutting speed (m/min)
T = Time (min)
n = Exponent depends
. i n
o
C = Constant based on tool and work material and cutting conditions
It is given that while doubling the cutting speed reduces the tool life to ^ 161 hth of
c
the original
a .
d i
VT n = C = ^2 Vhb T l
16
n

^16h = 2 n o
VT n = VT n 2 n
^16h
. n

w n = 0.25
So the Taylor’s tool like index ^n h. For this tool workpiece combination will be
0.25 . w
Q. 15 w
Consider a slider crank mechanism with nonzero masses and inertia. A constant

©
torque τ is applied on the crank as shown in the figure. Which of the following
plots best resembles variation of crank angle, θ versus time

. i n
c o
a .
d i
n o
.
w
w
w
©

Sol. 15 Correct option is (D).


1 - cos (x2)
Q. 16 The value of lim x " 0 is
2x 4
(A) 0 (B) 1
2
(C) 1 (D) undefined
4

Buy Online all GATE Books: shop.nodia.co.in


*Shipping Free* *Maximum Discount*
GATE SOLVED PAPER - ME 2015-1

Sol. 16 Correct option is (C).


Given function is
1 - cos ^x2h
lim
x"0 2x 4
To simplify the term put x2 = t , when x " 0 , t " 0 . So, we have
lim 1 - cos
t"0 2t 2
t

.i n
we have
c o
If we put t = 0 in the term, it becomes 00 form, so by applying L’hospital rule,

lim 0 + sin t
4t
a .
0
t"0

d i
Again 0 form, so we apply L’hospital rule to obtain

t"0

n
4o
lim cos t = cos 0 = 1
4 4
Q. 17
.
Two identical trusses support a load of 100 N as shown in the figure. The length
of each truss is 1.0 m, cross-sectional area is 200 mm2; Young’s modulus E = 200
w
GPa. The force in the truss AB (in N ) is_______

w
w
©
.i n
Sol. 17 Correct answer is 100.
c o
.
For given configuration of stress the FBD can be draws as

a
d i
n o
.
w
From Sine law, w
w FAB = FBC = 100 = k
sin 120 sin 120 sin 120
© FAB = FBC = 100 N
So, force in truss ^AB h will be 100 N.
Q. 18 Among the four normal distributions with probability density functions as shown
below, which one has the lowest variance?

Buy Online all GATE Books: shop.nodia.co.in


*Shipping Free* *Maximum Discount*
GATE SOLVED PAPER - ME 2015-1

. i n
c o
a .
d i
(A) I
n o (B) II
(C) III
. (D) IV
Sol. 18
w
Correct option is (D).

w
We know that probability density function of normal distribution is given by
F ^x h = 1 e− 2a σ k
1 x−µ
2

w σ 2π
µ = mean
−3 < x < 3

©
where
σ2 = variance
σ = standard deviation
For having lowest variance ^σ2h, standard deviation ^σ h will be lowest. From the
i n
density function f ^x h, we can say that as σ decreases F ^x h will increase, so curve
.
o
having highest peak has lowest standard deviation and variance.

c
Q. 19
z1 in degrees is
a .
Given two complex numbers Z1 = 5 + (5 3 ) i and z2 = 2 + 2i , the argument of
3
z2
(A) 0
d i (B) 30
(C) 60
n o (D) 90
Sol. 19
.
Correct option is (A)

w
Given complex number z1 , z2 are
z1 = 5 + ^5 3 h j
w z 2 = 2 + 2j
w
We have to find Argument d z1 n
3

©
Since, we know that
z2

Argument d z1 n = Argument ^z1h − Argument ^z2h


z2

Argument ^z1h = tan−1 c 5 3 m = tan−1 3 = 60c


5

Argument ^z2h = tan−1 e 22 o = tan−1 3 = 60c


3

Argument d z1 n = 60 − 60 = 0c
z2

Buy Online all GATE Books: shop.nodia.co.in


*Shipping Free* *Maximum Discount*
GATE SOLVED PAPER - ME 2015-1

Q. 20 The Blasius equation related to boundary layer theory is a


(A) third-order linear partial differential equation
(B) third-order nonlinear partial differential equation
(C) second-order nonlinear ordinary differential equation
(D) third-order nonlinear ordinary differential equation
Sol. 20 Correct option is (D).
.i n
f ml + 1 ff m = 0
c o
The Blasius equation related to boundary layer theory is

a .
It is a third order non-linear ordinary differential equation.
Q. 21
d i
A swimmer can swim 10 km in 2 hours when swimming along the flow of a river.

n o
While swimming against the flow, she takes 5 hours for the same distance. Her
speed in still water (in km/h) is _______.
Sol. 21 Correct answer is 3.5 .
w
For swimming in a flowing river, we have the following

w
w
©
Assume that
.i n
River flow velocity = x
c o
Swimmer velocity = y
Time taken in down stream direction,
a .
distance = 10 = 2
speed x+y
d i
x+y = 5
n o ...(1)

.
Time taken in upstream direction,

speedw
distance = 10 = 5
y−x

w y-x = 2 ...(2)

w
Adding equation (1) and (2),
2y = 7
© y = 3.5 km/hr
So, the velocity of swimmer in still water will be 3.5 km/hr.
Q. 22 For flow of viscous fluid over a flat plate, if the fluid temperature is the same as
the plate temperature, the thermal boundary layer is
(A) thinner than the velocity boundary layer
(B) thicker than the velocity boundary layer
(C) of the same thickness as the velocity boundary layer
(D) not formed at all

Buy Online all GATE Books: shop.nodia.co.in


*Shipping Free* *Maximum Discount*
GATE SOLVED PAPER - ME 2015-1

Sol. 22 Correct option is (D).


For the flow of viscous fluid over a flat plate if the fluid temperature is the same
as the plate temperature the thermal boundary layer is not formed at all because
boundary layer formation took place if there is some difference in fluid property
of no slip layer and remaining fluid.

i n
As here given that fluid is viscous and flowing over a flat plate. Here, definitely a

.
kinematic boundary layer will be formed but as there is no temperature difference
so no formation of thermal boundary layer.

c o
Q. 23
(A) Soderberg
a .
Which one of the following is the most conservative fatigue failure criterion?

(B) Modified Goodman


d i
(C) ASME Elliptic
(D) Gerber
n o
.
w
Correct option is (A).
Sol. 23

When a component is subjected to fluctuating stresses there is mean stress


^σ m h as well as stress amplitude ^σ a h. It has been observed that the mean stress
w
w
component has an effect on fatigue failure when it is present in combination with
an alternating component.
When the component is subjected to both component of stress ^σ m h and ^σ a h the
©
actual failure occurs at different scattered points as shown in figure.
There exist a border which decides sole region from unsole region for various

. i n
combinations of σ m and σ a and different criterion are proposed to constant the
borderline dividing safe zone and failure zone, they include Gerber line, soderberg
line, Goodman line, ASME elliptic.
Gerber Line:
c o
.
A parabolic curve jointing σ e on the ordinate to σ ut on the abscissa is called
a
Gerber line. It is written as
sa
d
sm 2i
a s e k + a s ut k = 1
Soderberg Line:
n o
.
A straight line joining σ e on the ordinate to σ yt on the abscissa is called soderberg

w
line. It is written as
sa sm
a s e k + b s yt l = 1
w
w
Goodman Line:
A straight line joining σ e on the ordinate to σ ut on the abscissa is called the
©
Goodman line. It is written as
sa sm
a s e k + a s ut k = 1
ASME Elliptic:
The ASME elliptic is written as
sa 2 sm 2
a s e k + b s yt l = 1

Buy Online all GATE Books: shop.nodia.co.in


*Shipping Free* *Maximum Discount*
GATE SOLVED PAPER - ME 2015-1

.i n
c o
a .
consider even yielding in this case.
d i
The soderberg line is a more conservative failure criterion and there is no need to

Q. 24

n o
In a linear are welding process, the heat input per unit length is inversely
proportional to
(A) welding current .
(B) welding voltage w
(C) welding speed
w
w
(D) duty cycle of the power source
Sol. 24

©
Correct option is (C).
In Linear arc welding process the heat input per unit length depends on many
factors.
(A) Welding current:

.i n
As welding current increase as heat input increases because we know that heat
input is I 2 RT so
o
(Heat input per unit length) \ ^currenth2
c
(B) Welding Voltage:
a .
2
input is VR .
d i
As welding voltage increase as heat input also increase because we know that heat

o
(Heat input per unit length) = ^voltageh2
n
(C) Welding Speed:
.
As welding speed increase as welding electrode spend less time to release heat so
w
as speed increase as heat input per unit length decrease proportionaly

w
(Heat input per unit length) \ 1
^welding speedh
w
(D) Duty cycle:

©
Duty cycle is defined as the % of time for which arc is on without overheating
element of welding machine so
(Heat input per unit length) \ (duty cycle)
Q. 25 Consider a stepped shaft subjected to a twisting moment applied at B as shown
in the figure. Assume shear modulus, G = 77 GPa. The angle of twist at C (in
degrees) is _______

Buy Online all GATE Books: shop.nodia.co.in


*Shipping Free* *Maximum Discount*
GATE SOLVED PAPER - ME 2015-1

Sol. 25 Correct answer is 0.23685


The given problem of twisting moment is shown in figure below.

. i n
c o
a .
As there is no twisting moment after point B so BC portion of shaft will not feel
any stress so angle of twist at B will be equal to angle of twist at C .
No we know that
θ = TL d i
n oGJ
where T =
.
torque (N-m) = 100 N-m
L
G
=
= w
Length of shaft = 500 mm = 0.5 m
shear modulus = 77 GPa = 77 # 109 Pa
w
J =Polar moment of Inertia
w = pd =
4 p ^20h4
32 #
10−12
© θ =
32
100 # 0.5 # 32
77 # 109 # π ^20h4 # 10−12
= 0.4133 radian
θ = 0.4133 radian
. i n
π
c o
= 0.4133 # 180 = .23685 degree

.
So the angle of twist at C is 0.23685 degree.
a
d i
Q. 26 – Q. 55 Carry Two Marks Each.
n o
Q. 26
.
A 10 mm diameter electrical conductor is covered by an insulation of 2 mm

w
thickness. The conductivity of the insulation is 0.08 W/m-K and the convection

w
coefficient at the insulation surface is 10 W/m2-K. Addition of further insulation
of the same material will

w
(A) increase heat loss continuously
(B) decrease heat loss continuously
©
(C) increase heat loss to a maximum and then decrease heat loss
(D) decrease heat loss to a minimum and then increase heat loss
Sol. 26 Correct option is (C).
Maximum heat transfer role take place at the critical radius of insulation
For a cylindrical shape it is given by
Rc = kin
h
where Rc = radius of critical insulation
kin = thermal conductivity of insulation
h = heat transfer coefficient of surrounding

Buy Online all GATE Books: shop.nodia.co.in


*Shipping Free* *Maximum Discount*
GATE SOLVED PAPER - ME 2015-1

Rc = 0.08 = 0.008 m = 8 cm
10

.i n
c o
a .
d i
So here we can see that heat transfer will increase upto 8 cm mean max heat
o
transfer will took place at 3 cm of insulation after that heat transfer will start
n
decreasing.
.
Q. 27
w
A machine element is subjected to the following bi-axial state of stress; σ y = 20
MPa; τ xy = 40 MPa. If the shear strength of the material is 100 MPa, the factor
w
of safety as per Tresca’s maximum shear stress theory is
(A) 1.0
(C) 2.5 w (B) 2.0
(D) 3.3
Sol. 27 ©
Correct option is (B).
For a machine element subjected to bi-axial state of stress
σ x = 80 MPa
σ y = 20 MPa .i n
σ xy = 40 MPa
c o
maximum shear stress will be given by

a
τ max = ! a x y k + ^τ xy h2
.
σ σ 2
2
d i
b
o2 l ^ h
τ max = ! 80 − 20 2 + 40 2

. n
τ max = ! 50 MPa
Factor of safety =
w shear strength of Material
Maximum shear stress
w = 100 = 2
w 50
So factor of safety according to Tresca’s maximum shear stress theory is 2.
Q. 28

(A) 425
©
The probability of obtaining at least two ‘‘SIX’’ in throwing a fair dice 4 time is
(B) 19
432 144
(C) 13 (D) 125
144 432
Sol. 28 Correct option is (B).
Probability of obtaining atleast two ‘six’ P ^x $ 2h in throwing a fair dice 4 times
is written as = 1 − probability of obtaining less than 2 six in Four throws
P ^x $ 2h = 1 − P ^x < 2h
= 1 − 6P ^x = 0h + P ^x = 1h@

Buy Online all GATE Books: shop.nodia.co.in


*Shipping Free* *Maximum Discount*
GATE SOLVED PAPER - ME 2015-1

Probability of getting six-in one chance = 16 from Binomial distribution of


probability
P ^x = 0h = 4C 0 b 1 l b 5 l
0 4

6 6
P ^x = 1h = 4C1 b 1 l b 5 l
1 3

6 6

. i n
P ^x $ 2h = 1 − ;4C 0 b 1 l b 5 l + 4C1 b 1 lb 5 l E
0 4 3

4
6 6

c o
= 1 − ;b 5 l + 4 b 1 lb 5 l E
3
6 6

a .
= 1 − 5 4 65 + 4@
6 6

i
3

o
= 1−5 #
6
d
3
4
9

. n 6
= 1 − 1125 = 1296 − 1126
1296 1296
w = 171 = 19
w 1296 144
19

Q. 29
w
So probability of getting atleast two six is 144 .
A horizontal plate has been joined to a vertical post using four rivets arranged
©
as shown in figure. The magnitude of the load on the worst loaded rivet (in N)
is_______

. i n
c o
a .
d i
Sol. 29

n o
Correct answer is 1839.84 .
For the given configuration of eccentric loading two types of shear force will work
. Is = direct shear load
w
Which will be equal for all rivets and given by

w Pl = P = P1l= P2l= P3l= P4l


4

w = 400 = 100 N
4
© II = Secondary shear stress
Which will occur due to torque generated by eccentric loading. FBD of given
configuration is shown in figure

Buy Online all GATE Books: shop.nodia.co.in


*Shipping Free* *Maximum Discount*
GATE SOLVED PAPER - ME 2015-1

By balancing the torque,


P1m r1 + P2m r2 + P3m r3 + P4m r4 = Pe ...(1)

.i n
r1 = 200 2 mm = 0.02 2 m
c o
r1 = r2 = r3 = r4
a .
But, we can say that
Pm \ r
d i
So,
From equations (1) and (2),
n o
P1m = kr1 ; P2m= kr2 ; ....... ...(2)

. Per1
P1m = 2
w
r 1 + r 2 + r 32 + r 42
2

w = 400 # 0.5 #20.02 2


^0.02 2 h # 4
w = 1767.7669 N

©
Similarly P2m, P3m, P4m can be found.
Now, we can see from FBD that angle between these two shear stress is minimum
in (1). So, we get
Presultant =
. n
^P1lh2 + ^P1mh2 + 2P1lP1m cos 45c
i
^100h2 + ^1767.7669h + 2 ^100h^1767.7669h cos 45c
2
=
= 1839.8368 N
c o
.
So, the worst loaded rivet will have load of 1839.8368 N.
a
Q. 30

d i
Temperature of nitrogen in a vessel of volume 2 m3 is 288 K. A U-tube manometer
connected to the vessel shows a reading of 70 cm of mercury (level higher in

n o
the end open to atmosphere). The universal gas constant is 8317 J/kmol-K,
atmospheric pressure is 1.01325 bar, acceleration due to gravity is 9.81 m/s2 and
.
density of mercury is 13600 kg/m3. The mass of nitrogen (in kg) in the vessel
is_______.
w
Sol. 30
w
Correct answer is 4.55
For a vessel of nitrogen connected to U-tube manometer
w
Reading of U-tube manometer = 70 cm Hg

©
We know that
P = ρgh
where ρ = density of Hg = 13600 kg/m3
g = gravitational acceleration = 9.81 m/ sec2
h = Hight of Hg column
P = ^13600h^9.81h^0.7h
= 93391.2 Pa
Now, absolute pressure is
= Gauge pressure + Atmospheric pressure
= 93391.2 + 101325 = 194716.2 Pa

Buy Online all GATE Books: shop.nodia.co.in


*Shipping Free* *Maximum Discount*
GATE SOLVED PAPER - ME 2015-1

Now, applying ideal gas equation, we have


PV = mRT
where P = absolute pressure
V = Volume = 2m3
= mass of substance
m

.
R = Characteristic gas const = Rui n
R = 8314
28
For nitrogen
c om

T = temperature
a .
So, m = 194716
8314
28 #
d i
.2 # 2 = 4.55 kg
288
Hence, the mass of N2 gas in vessel is 4.55 kg.
Q. 31
n o
The solidification time of a casting is proportional to ^VA h2 , where V is the volume
.
of the casting and A is the total casting surface area losing heat. Two cubes of

w
same material and size are cast using sand casting process. The top face of one of
the cubes is completely insulated. The ratio of the solidification time for the cube
w
with top face insulated to that of the other cube is
(A) 25
36 w (B) 36
25

©
(C) 1 (D) 6
5
Sol. 31 Correct option is (B).
It is given that solidification time of casting ^ts h
ts \ bV l
2

. i n
where
A
V = Volume of casting
c o
.
A = surface area of casting losing heat
a
waiting to loose.
d i
For a cube of side ‘a ’ volume of cube is a3 which represent total heat present or

n o
Surface area of First cube which can loose heat from all surface = 6a2 .
Surface area of second cube which can loose heat from 5 surfaces only = 5a2 .
.
Ratio of solidification time for cube with top face insulated to that of the other
cube is
w _ 5a i
w a3 2

= ts insulated = a 2 = b 6 l = 36
ts unisulated _ i
2 2

w 5 25
3
2
6a

Match the following pairs:


©
Q. 32

Equation Physical Interpretation


(P) d # Vv = 0 (I) Incompressible continuity equation
(Q) d.Vv = 0 (II) Steady flow
(R) DVv = 0 (III) Irrotational flow
Dt
(S) 2Vv = 0 (IV) Zero acceleration of fluid particle
2t

Buy Online all GATE Books: shop.nodia.co.in


*Shipping Free* *Maximum Discount*
GATE SOLVED PAPER - ME 2015-1

(A) P-IV, Q-I, R-II, S-III


(B) P-IV, Q-III, R-I, S-II
(C) P-III, Q-I, R-IV, S-II
(D) P-III, Q-I, R-II, S-IV
Sol. 32 Correct option is (C).
(P) d # Vv = 0
.i n
d # Vv = 2
i j
2
k
2

c o
2x
u
2y
v w
2z

a .
2y 2z i
= it 2w − 2v + tj −2w + 2u + kt 2v − 2u
d 2x 2z 2x 2y

d # Vv = 0
n o
= it wx + tj wy + kt wz
So, if
Then, wx = wy = wz = 0 .
w
It represents that flow is irrotational.
(Q) d : Vv = 0
w
w
Generalized 3-D continuity equation is given by
^ h ^ h ^ h
2 eu + 2 ev + 2 ew + 2ρ = 0
2x
© 2y 2z 2t
For e = constant (Incompressible flow), we get
2u + 2v + 2w + 0 = 0
2x 2y 2z
d : Vv = 0 .i n
It represents incompressible continuity equation.
v
c o
(R) TV = 0
Tt
a .
d i
Total derivative of velocity vector with respect to time is zero then it makes sure
that acceleration of fluid particle is zero. Total derivative is used when we studies

v
n o
about particular particle only. So, it represents zero acceleration of fluid particle.
(S) 2V = 0
2t .
w
Partial derivative of velocity vector with respect to time is zero then it gives that

w
velocity of flow individual in each direction is constant and their is no variation
in fluid flow pattern. So, it represents steady flow.
Q. 33 w
Steam enters a well insulated turbine and expands isentropically throughout. At

©
an intermediate pressure, 20 percent of the mass is extracted for process heating
and the remaining steam expands isentropically to 9 kPa.
Inlet to turbine P = 14 MPa, T = 560° C, h = 3486 kJ/kg,
s = 6.6 kJ/(kg.K)
Intermediate stage h = 27776 kJ/kg
Exit of turbine P = 9 kPa, h f = 174 kJ/kg, hg = 2574 kJ/kg,
s f = 0.6 kJ/(kg.K), sg = 8.1 kJ/(kg.K)
If the flow rate of steam entering the turbine is 100 kg/s, then the work output
(in MW) is_______.
Sol. 33 Correct answer is 125.56

Buy Online all GATE Books: shop.nodia.co.in


*Shipping Free* *Maximum Discount*
GATE SOLVED PAPER - ME 2015-1

The process is shown in T-S diagram below.

. i n
c o
a .
d i
n o
It is given that steam expands isentropically. So, we have
.
S1 = S 3
w
6.6 = S f + x 6Sg − S f @
6.6 = 0.6 + x 68.1 − 0.6@
w
w x = 0.8
Now, for enthalpy of point 3, we can use

© h 3 = h f + x hg − h f
= 174 + 0.8 62574 − 174@
= 2094 ks/kg
Hence, total work out from the turbine will be
o 6h1 − h2@ + 61 − 0.2@m . i
o 6h2 − h 3@
n
wT = m
o
= 100 6− 2776 + 3486@ + 0.8 # 100 6− 2094 + 2776@
c
wT = 125.56 MW
a .
d i
The work output from turbine is 125.56 MW.
Water ( ρ = 1000 kg/m3) flows through a venturimeter with inlet diameter 80 mm
o
Q. 34

and throat diameter 40 mm. The inlet and throat gauge pressures are measured to

. n
be 400 kPa and 130 kPa respectively. Assuming the venturimeter to be horizontal
and neglecting friction, the inlet velocity (in m/s) is_______.
Sol. 34 w
Correct answer is 6.
w
The flow through given venturimeter is shown in figure

w
©

by applying energy equation and continuity equation of flow


P i2 + V i2 = P t2 + V t2 ...(1)
ρg 2g 2g 2 g
π ^di h2
Vi ; E = Vt : πd t D
2
and
4 4

Buy Online all GATE Books: shop.nodia.co.in


*Shipping Free* *Maximum Discount*
GATE SOLVED PAPER - ME 2015-1

Vi 680@2 = Vt 640@2
4Vi = Vt ...(2)
From equations (1) and (2), we get
400 # 103 + V i2 = 30 # 103 + ^4Vi h
2

103 ^10h 2 ^10h 103 ^10h 2 ^10h


2
400 + V i = 130 + 8V i2
2
.i n
270 = ^8 − 0.5h V i2
c o
Vi = 6 m/sec

a .
So the inlet velocity of venturimeter is 6 m/sec.
Q. 35
d i
For a canteen, the actual demand for disposable cups was 500 units in January

n o
and 600 units in February. The forecast for the month of January was 400 units.
The forecast for the month of March considering smoothing coefficient as 0.75 is
_______
.
Sol. 35 Correct answer is 569.
w
w
Forecasting value of next month from exponential smoothing method is given by
Ft = Ft − 1 + αρ t − 1
w = Ft − 1 + α 6Dt − 1 − Ft − 1@
where
© Ft = Forecasting demand for next month
Ft - 1 = Forecasting demand of previous month
Dt - 1 = Actual demand of previous month
α = exponential smoothing constant
.i n
The calculations are shown in the form of table

Actual Demand 6D@ Forcasting 6F @ c o


Month

a . Error e = D − F
Jan
Feb
500
600
400

d i
400 + 0.75 # 100
500 − 400 = 100
600 − 475 = 125

n o
= 475
475 + ^0.75 # 125h
March –
.
w = 568.75

w
So, the forecast value for the month of march will be 568.75 units or 569 units

Q. 36
(approx.).
w
Consider a spatial curve in three-dimensional space given in parametric form by
©x (t) cos t , y (t) = sin t , z (t) = 2 t , 0 # t # π
π 2
The length of the curve is_______
Sol. 36 Correct answer is 1.862
The length of arc x = φ ^ t h and y = ψ ^ t h between t = t1 and t = t2 is given by

b dt l b dt l
t
dx 2 + dy 2 − dt
#
2
l =
t 1

Given that
x ^ t h = cos t & dx =− sin t
dt

Buy Online all GATE Books: shop.nodia.co.in


*Shipping Free* *Maximum Discount*
GATE SOLVED PAPER - ME 2015-1

y ^ t h = sin t &
dy
= cos t
dt
z ^ t h = 2 t & dz = 2 0#t#π
p dt p 2
So the length of curve
l #
0
π/2

. i n
^− sin t h2 + ^cos t h2 + b π2 l dt
2

l = #
π/2
1 + 42 dt
c o
.
0 π
1 + 42 t 0π/2
l =
π
i a
=

o
l = 1.86209
p 2d
1 + 42 a p k

. n
The length of curve is 1.86209
Q. 37
w
Considering massless rigid rod and small oscillations, the natural frequency (in

w
rad/s) of vibration of the system show in the figure is

w
©
. i n
(A) 400
1
c o
(B) 400
2
(C) 400
3
(D)
a .
400
4
Sol. 37 Correct option is (D).
d i
For the given configuration of massless rigid rod. For a small oscillation the toque
o
equation about hing is written as
n
.
w
w
w
© 6k r q@r + I qp = 0
I = M 62r @2 = 4r2 M
qp + c kr2 m q = 0
2

4r M
Comparing from standard equation, we have
θp + ω2 θ = 0
ω = k = 400 rad/ sec
4M 4

Buy Online all GATE Books: shop.nodia.co.in


*Shipping Free* *Maximum Discount*
GATE SOLVED PAPER - ME 2015-1

Q. 38 A triangular facet in a CAD model has vertices: P1(0, 0, 0); P2(1, 1, 0) and P3(1,
1, 1). The area of the facet is
(A) 0.500 (B) 0.707
(C) 1.414 (D) 1.732
Sol. 38 Correct option is (B).
Area of triangular is given by
= 1 (base) (height) .i n
2
In vector form & 1 6av : bv@ c o
and
2
a .
v
where av and b are adjacent vector

d i
n o
.
w
w
w
© av = ^1 − 0h it + ^1 − 0htj + ^1 − 0h kt
= it + tj + kt
bv = ^1 − 0h it + ^1 − 0htj + ^0 − 0h kt
= it + tj
.i n
Area of triangle = 1 6av : bv@
2
c o
.
= 1 6^i + j + k h : ^i + j h@ = 1 12 + 12
a
=
2
2 = 1 = 0.7071
d i 2

n
2
o
Area of triangle is 0.7071 unit square.
.
In a slab rolling operation, the maximum thickness reduction (Th max ) is given by
w
Q. 39

Th max = µ2 R , where R is the radius of the roll and µ is the coefficient of friction

w
between the roll and the sheet. If µ = 0.1, the maximum angle subtended by the
deformation zone at the centre of the roll (bite angle in degree) is_______.
Sol. 39 w
Correct answer is 5.7319

©
Reduction in thickness in rolling operation is given by
TH = D ^1 − cos αh
where α is bite angle
For maximum reduction the bite angle will be maximum.
So
TH max = µ2 R = D 61 − cos α@
µ2 R = 2R 61 − cos α@
^0.1h2 = 2 61 − cos α@
α = 5.7319c
So the maximum angle subtended by the deformation zone at the centre of roll
is 5.7319c.

Buy Online all GATE Books: shop.nodia.co.in


*Shipping Free* *Maximum Discount*
GATE SOLVED PAPER - ME 2015-1

Q. 40 Fine the solution of d2 y


dx2
= y which passes through the origin and the point ^In2, 34 h,

(A) y = 1 ex − e−x (B) y = 1 (ex + e−x )


2 2
(C) y = 1 (ex − e−x ) (D) y = 1 ex + e−x
2 2
Sol. 40 Correct option is (C)
Given differential equation d2 y
. i n
= y is a higher degree differential equation.
We can write it as
dx2

c o
d2 y
-y = 0
dx2
^D - 1h y = 0 a .
i
2

o d
m2 - 1 = 0 is auxiliary equation of given differential eq.
m = ! 1 are roots of auxiliary equation

. n
Solution contents only complimentary function so for real and distinct root of
auxiliary differential equation C.F. is given by
w y = C1 e M x + C 2 e M x
1 2

w y = C1 ex + C 2 e−x

w
It is said that it passes through (0, 0) and ( ln 2, 34 )
So

© 0 = C1 e 0 + C 2 e 0 & C1 + C 2 = 0
3 = C e ln 2 + C e− ln 2
1 2
...(1)

From eq. (1) and (2)


1
2C1 + C2 =
2
3
4
. i n ...(2)

C1 = 1 , C2 =− 1
c o
2
y = 1 e+x − 1 e−x
a .
2
So

or
2

d i 2
y = 1 ^ex − e−x h
Q. 41
n o2
For the truss shown in figure, the magnitude of the force in member PR and the
.
support reaction at R are respectively

w
w
w
©

(A) 122.47 kN and 50 kN


(B) 70.71 kN and 100 kN
(C) 70.71 kN and 50 kN
(D) 81.65 kN and 100 kN

Buy Online all GATE Books: shop.nodia.co.in


*Shipping Free* *Maximum Discount*
GATE SOLVED PAPER - ME 2015-1

Sol. 41 Correct option is (C).


For the given truss FBD is shown in Figure

.i n
c o
a .
d i
n o
.
w
At point Q truss is hinged so taking moment about point Q
w
ΣMQ = 0
^100 cos 60h # 4 = RR # 4 & RR = 50 N
w
Now force in member PR by making balance of horizontal forces at point P
© 100 cos 60 = FPR cos 45
FPR = 50 2 = 70.71 N
Q. 42

.i n
A ball of mass 0.1 kg, initially at rest, is dropped from height of 1 m. Ball hits the
ground and bounces off the ground Upon impact with the ground, the velocity
o
reduces by 20%. The height (in m) to which the ball will rise is_______
c
Sol. 42 Correct answer is 0.64
a .
velocity will be
d i
When a ball of mass 0.1 kg initially at rest, is dropped from a height of 1 m. The

V =

n o2gh
V = 2 # 9.01 # 1
.
V = 4.4294 m/ sec
w
It is said that upon impact with ground, the velocity reduces by 20%. So, Vl will
be
w Vl = 0.8 # 4.4294
w = 3.5435 m/sec

©
The height reached by ball with velocity 3.5435 m/sec is
h =V
2

2g
^3.5435h2
= = 0.64 m
2 # 9.81
So height of ball reached after impact will be 0.64 m.
Q. 43 A DC welding power source has a linear voltage-current (V-I) characteristic with
open circuit voltage of 80 V and a short circuit current of 300 A. for maximum
arc power, the current (in Amperes) should be set as_______.
Sol. 43 Correct answer is 150.

Buy Online all GATE Books: shop.nodia.co.in


*Shipping Free* *Maximum Discount*
GATE SOLVED PAPER - ME 2015-1

The relationship in open circuit voltage (OCV) and short circuit current (SCC)
is given by
VP + IP = 1 ...(1)
OCV SCC
OCV and SCC is characteristics of welding machine whereas VP and IP is of

value of OCV and SCC, we get


. i n
power supply and can be varied. Multiplying equation (1) by IP and substituting

VP IP + I P2 = I
80 300 P

c o
We know that VP IP is power (P ), so
a .
P + I P2 = I
80 300
d i
P

P = 80 bIP − I P l
2

n o 300
...(2)

.
To maximize the power,

w dP = 0 and d 2 P < 0
dIP dI P2
w
So, from equation (2), we get

:
w 300 D
dP = 80 1 − 2IP = 0
dI

© IP = 150 amp
d 2 P =− 80 # 2 < 0
dI P2 300

Q. 44
. n
So, for maximum arc power, the current should be set as 150 ampere.
i
A well insulated rigid container of volume 1 m3 contains 1.0 kg of an ideal gas [
o
C p = 1000 J/(kg.K) and Cv = 800 J/(kg.K)] at a pressure of 105 Pa. A stirrer
c
a .
is rotated at constant rpm in the container for 1000 rotations and the applied
torque is 100 N-m. The final temperature of the gas (in K) is _______
Sol. 44 Correct answer is 1132.62
d i
The container is well insulated and rigid so the work given by stirrer will completely
o
convert into heat. So, we have
n
.
work = torque # angular displacement

w = 100 # 1000 # 2π

w = 628.3185 kJ.
Heat given to gas = mCv TT
w 628318.5 = ^1 h^800h TT

© TT = 785.39 K
Initial temp = ?
We know PV = mRT
T = PV
mR
= 105 # 1 = 347.22 K
^1 h^288h
Final temperature = Initial temperature + TT
= 347.22 + 785.39 = 1132.62 K
So, the final temperature of gas is 1132.62 K

Buy Online all GATE Books: shop.nodia.co.in


*Shipping Free* *Maximum Discount*
GATE SOLVED PAPER - ME 2015-1

Q. 45 A pinion with radius r1 , and inertia I1 is driving a gear with radius r2 and inertia
I2 . Torque τ 1 is applied on pinion. The following are free body diagrams of pinion
and gear showing important forces (F1 and F2 ) of interaction. Which of the
following hold true?

.i n
c o
a .
d i
n o
(A) F1 ! F2 ; τ 1 = I1 θp1; F2 = I2 r12 θp1
r2
.
(B) F1 = F2 ; τ 1 = :I1 + I2 a r1 k D θp1 ; F2 = I2 r12 θp1
2

w r2 r2

w
(C) F1 = F2 ; τ 1 = I1 θp1 ; F2 = I2 1 θp2
r2

w
(D) F1 ! F2 ; τ 1 = :I1 + I2 a r1 k D θp1; F2 = I2 1 θp2
r2
2
r2
Sol. 45
©
Correct option is (B).
We know that system of pinion and gear have no translative movement. So, we
get
ΣFexternal = 0
.i n
F1 - F2 = 0
F1 = F2
c o ...(1)
For Pinion
a .
d i
n o
.
w
w
Equilibrium equation

For Gear
w τ 1 - F1 r1 = I1 θp1 ...(2)

© F2 r2 = I2 θp2
p
F2 = I2 θ 2 ...(3)
r2

p
F1 = F2 = I2 θ 2
r2

Buy Online all GATE Books: shop.nodia.co.in


*Shipping Free* *Maximum Discount*
GATE SOLVED PAPER - ME 2015-1

θp \ 1
r
r1 θp1 = r2 θp2
qp2 = r1
a r2 k (4)
qp1
Form equations (1) and (2), and (3), we obtain

τ 1 - ;I2 θ 2 E r1 = I1 θp1
p
. i n
r2
o
τ 1 = I2 ^qp2ha r1 k + I1 qp
c
a .
p
r2

d i
τ 1 = >I1 + I2 f qp2 pa r1 kHqp1
q 1 r2
Now, for
n o p
F2 = I2 θ 2
r2

.
From equation (4), we have

w θp2 = a r1 k θp1
r2
w F2 = I2 θp1 r12
w r2
F2 = I2 b r12 l θp1

Q. 46
© r2
A cantilever beam with flexural rigidity of 200 Nm2 is loaded a shown in the
figure. The deflection (in mm) at the tip of the beam is _______.

. i n
c o
a .
d i
Sol. 46 Correct answer is 0.2604

n o
For the given configuration of cantilever loaded beam at mid point the deflection
.
at the point of load will be affected by load, further it will be deflected by the

w
slope of loaded beam only. So, we have
Deflection at end point = deflection at the point of load + slope # distance
w
Deflection at the point of load
w = wL
3

3EI
©
where W = Load = 500 N
L = length = 0.05 m
EI = Flexural rigidity = 200 Nm2
500 # ^0.05h3
δ at po int load = = 0.10416 mm
3 # 200
Deflection due to slope
= slope # distance
2
= wL # 0.05
2EI
500 # ^0.05h2
= 0.05 = 0.1562 mm
2 # 200 #

Buy Online all GATE Books: shop.nodia.co.in


*Shipping Free* *Maximum Discount*
GATE SOLVED PAPER - ME 2015-1

Total deflection at end point is


= 0.10416 + 0.1562
= 0.26041 mm
So, total deflection is 0.2604 mm.
In the assembly shown below, the part dimensions are:
n
Q. 47

L1 = 22.0! 0.01 mm,


.i
L2 = L 3 = 10.0! 0.005 mm

c o
for assembly condition would be:
a .
Assuming the normal distribution of part dimensions, the dimension L 4 in mm

d i
n o
.
w
w
w
©
(A) 2.0! 0.008
(C) 2.0! 0.016
(B) 2.0! 0.012
(D) 2.0! 0.020
Sol. 47 Correct option is (D).
For dimensions of L 4 , we deduce
.i n
Maximum dimension of L 4
o
= ^maxmum of L1h − ^minimum of L2 & L 3h
c
.
= ^22.0 + 0.01h − 2 ^10.0 − 0.005h
a
= 2.02 mm
Minimum dimension of L 4 d i
n o
= ^minimum of L 4h − ^maximum of L 2 & L 3h
= ^22.0 − 0.01h − 2 ^10.0 + 0.005h
.
w
= 1.98 mm

w
Total tolerance = 2.02 − 1.98 = 0.04
For bilateral dimension system, it can be expressed as ! 0.02 . So, the dimension
w
of L 4 can be expressed in bilateral system as 2.0 ! 0.02 mm .
Q. 48
©
A mobile phone has a small motor with an eccentric mass used for vibrator mode.
The location of the eccentric mass on motor with respect to center of gravity
(CG) of the mobile and the rest of the dimensions of the mobile phone are shown.
The mobile is kept on a flat horizontal surface.

Buy Online all GATE Books: shop.nodia.co.in


*Shipping Free* *Maximum Discount*
GATE SOLVED PAPER - ME 2015-1

Given in addition that the eccentric mass = 2 grams, eccentricty = 2.19 mm,
mass of the mobile = 90 grams, g = 9.81 m/s2. Uniform speed of the motor in
RPM for which the mobile will get just lifted off the ground at the end Q is
approximately
(A) 3000 (B) 3500
(C) 4000
i n
(D) 4500

.
Sol. 48 Correct option is (B).

c o
A precision instrument package (m = 1 kg) needs to be mounted on a surface
.
Q. 49
vibrating at 60 Hz. It is desired that only 5% of the base surface vibration

i a
amplitude be transmitted to the instrument. Assuming that the isolation is

o d
designed with its natural frequency significantly lesser than 60 Hz, so that the
effect of damping may be ignored. The stiffness (in N/m) of the required mounting

Sol. 49
pad is _______
.
Correct answer is 6767.53n
w
For the given precision instrument package, it is desired that only 5% of basis

w
surface vibration amplitude be transmitted to the instrument. So, we get
Transitivity ^ ε h = 1
w 1 −a w k
wn
2
= 0.05

©
where ω = Angular velocity
ω n = Natural angular velocity
We know that
ω = 2πf
. i n
where
So, o
f = Vibrational frequency = 60 Hz
ω = 2π ^60h = 120π = 376.99 rad/sec
c
Now, we have
a .
ε= 1

d
n
i
1 − ^ ww h2
= ! 0.05

1 - a w k = ! 20
wn
2

n o
.
w 2
a w n k = 21/ − 19
w
w 2
a w n k =− 19 Not possible
w
So,
w w 2
a w n k = 21

© ωn = ω
21
376 .99 = 82.265 rad/sec
ωn =
21
Now, we know that
ωn = k
m
where k = stiffness
k = ω n2 m = ^82.265h2 ^1 h
k = 6767.53 N/m
The stiffness of the required mounting pad is 6767.53 N/m.

Buy Online all GATE Books: shop.nodia.co.in


*Shipping Free* *Maximum Discount*
GATE SOLVED PAPER - ME 2015-1

Q. 50 Following data refers to the activities of a project, where node 1 refers to the start
and node 5 refers to the end of the project

Activity Duration (days)


1-2 2
2-3
4-3
1
3
.i n
1-4 3
c o
2-5 3

a .
3-5
4-5
2
4
d i
o
The critical path (CP) in the network is
n
(A) 1-2-3-5
. (B) 1-4-3-5
(C) 1-2-3-4-5
w (D) 1-4-5
Sol. 50
w
Correct option is (B).
The network diagram for given activities is drawn below.
w
©
.i n
c o
Possible path are
Path (1): ^1 - 2 - 5h
a .
Path (2): – ^1 - 2 - 3 - 5h
Path (3): – ^1 - 4 - 3 - 5h d i
Path (4): – ^1 - 3 - 5h
n o
Length o paths are
.
w
Path (1): = 2 + 3 = 5 days

w
Path (2): = 2 + 1 + 2 = 5 days

w
Path (3): = 3 + 3 + 2 = 8 days
Path (4): = 3 + 4 = 7 days

Q. 51
©
So, maximum time consumed is in path (3). Hence, path (3) is critical path.
For flow through a pipe of radius R, the velocity and temperature distribution
are as follows:
u (r, x) = C1
and T (r, x) = C2 81 − ^ Rr h3B , where C1 and C2 are constants
The bulk mean temperature is given by
R
Tm = 2
um R 2 # u (r, x) T (r, x) rdr
0
with Um being the mean velocity of flow. The value of Tm is

(A) 0.5C2 (B) 0.5 C2


Um
(C) 0.6 C2 (D) 0.6C2
Um

Buy Online all GATE Books: shop.nodia.co.in


*Shipping Free* *Maximum Discount*
GATE SOLVED PAPER - ME 2015-1

Sol. 51 Correct option is (C).


From given relations
# 6u^r, x h@6T^r, x h@rdr
R
Tm = 2
um R 2 0

6C1@=C2 d1 − d R G
r n n rdr
R 3
2
#
Tm = 2
um R 0
Tm = 2C1 C22
R
dr − 3 n dr
# r4
. i n
um R 0 R
Tm = 2C1 C22 <r − r 2 F c o
.
2 5 R

um R 2 5R 0
i
Tm = 2C1 C22 ;R − R E
2 2
a
um R 2

o d 5
Tm = 2C1 C22 b 3 R2 l = 0.6C1 C2

. n um R 10
It is given that um is mean velocity of flow
um

i.e.
w
u ^r, x h = C1

w Tm = 0.6C1 C2 = 0.6C 2
C1
Q. 52 w
Consider an ant crawling along the curve (x − 2) 2 + y2 = 4 , where x and y are

©
in meters. The ant starts at the point (4, 0) and moves counter-clockwise with a
speed of 1.57 meters per second. The time taken by the ant to reach the point (2,
2) is (in seconds) _______
Sol. 52 Correct answer is 2.
. i n
The given curve is a circle with center (2, 0) and radius of 2 units

c o
a .
d i
n o
.
w
w
Distance covered by ant from (4, 0) to (2, 2) along the given path is 1
4 of

w
circumference. So, length traveled is
= 1 # 62p ^2 h@ = p units
©
and time taken by ant is
4

= distance
speed
= π = 2 sec
1.57
Hence, the time taken by ant to reach the point (2, 2) is 2 seconds.
Q. 53 Air ( ρ = 1.2 kg/m3 and kinematic viscosity, v = 2 # 10−5 m2/s) with a velocity
of 2 m/s flows over the top surface of a flat plate of length 2.5m. If the average
value of friction coefficient is C f = 1.Re
328
x
, the total drag force (in N) per unit width
of the plate is _______

Buy Online all GATE Books: shop.nodia.co.in


*Shipping Free* *Maximum Discount*
GATE SOLVED PAPER - ME 2015-1

Sol. 53 Correct answer is 0.01593


We know that total drop force of the plate is given by
F0 = 1 CF PAV2
2
where CF = density of flowing fluid coefficient
e = density of flowing fluid = 1.2 kg/m3
A = Area of contact = ^2.5 # unil.widthh = 2.5
.i n
V = velocity of flow = 2 m/sec
c o
i.e.
a .
Now, to find the flow property (i.e. CF ), first we need to know Reynold’s Number,

x
µ µ 2 # 10.5 i
Re = evx = vx = 2 # 2.5 = 2.5 # 105
d
o
Re < 5 # 105 i.e. laminar flow
n
.
So, for laminar flow, we have
CF = 1.328
w
Re

w
CF = 1.328
2.5 # 10 5
= 2.656 # 10−3

w 1
FD = # 2.656 # 10−3 # 1.2 # 2.5 # 22 = 0.01593 N
2
©
So, total drag force per unit width of the plate is 0.01593 N.
Q. 54 The velocity field of an incompressible flow is given by

c 3 =− 4 . The value of b2 is _______


.i n
V = (a1 x + a2 y + a 3 z) i + (b1 x + b2 y + b 3 z) j + (c1 x + c2 y + c 3 z) k , where a1 = 2 and

Sol. 54 Correct answer is 2.


c o
2 u + 2v + 2 w = 0
a .
Continuity equation for incompressible fluid flow in 3.0 is given by

2x 2y 2z
d i
u = a1 x + a2 y + a 3 t or 2u = a1

n o 2x

.
v b2 x + a2 y + b 3 z or 2v = b2
2y
w
w = c1 x + c2 y + c 2 z or 2w = c 3
So, we get w 2z

2y 2y 2z
w
2u + 2v + 2w = a + b + c = 0
1 2 3

© 2 − 4 + b2 = 0
b2 = 2
The value of b2 is 2.
Q. 55 An orthogonal turning operation is carried out under the following conditions:
rake angle = 5° , spindle rotational speed = 400 rpm; axial feed = 0.4 m/min and
radial depth of cut = 5 mm. The chip thickness tc , is found to be 3 mm. The shear
angle (in degrees) in this turning process is _______
Sol. 55 Correct answer is 18.88

Buy Online all GATE Books: shop.nodia.co.in


*Shipping Free* *Maximum Discount*
GATE SOLVED PAPER - ME 2015-1

Shear angel ^φh in orthogonal turning process is given by


φ = tan−1 a r cos a k
1 − r sin a
where
r = chip thickness ratio = t/tc
t = uncut chip thickness

.
tc = cut chip thickness = 3 mm i n
α = rake angle of tool = 5c
c o
feed per revolution
a .
We know that in orthogonal turning operation uncut chip thickness is given by

d i
Axial feed = 0.4 M/min
Spindal rotation speed = 400 rpm

n o
Feed (in mm/rev) = 0.4 # 1000 = 1 mm/rev
. 400

w
Uncut chip thickness = 1 mm
Now, we get
w r = t = 1 = 0.333

So,
w tc 3
φ = tan−1 b 0.333 cos 5c l = tan−1 ^0.342h
© 1 − 0.333 sin 5c
= 18.88c

END OF THE QUESTION PAPER


. i n
c o
a .
d i
n o
.
w
w
w
©

Buy Online all GATE Books: shop.nodia.co.in


*Shipping Free* *Maximum Discount*
GATE SOLVED PAPER - ME 2015-1

ANSWER KEY
General Aptitude
1 2 3 4 5 6 7 8 9 10
(C) (B) (B) (D) (A) (B) (D) (B) (B) (800)

Mechanical Engineering .i n
1 2 3 4 5 6
c o 7 8 9 10
(A) (20) (D) (B) (B)
a .
(2) (D) (A) (50) (0.0197)
11 12
(3289.868) (525)
13
(B)
14
(0.25)
15
(D) d i 16
(C)
17 18
(100) (D)
19
(A)
20
(D)
21 22 23 24
n o 25 26 27 28 29 30
(3.5) (D) (A) (C)
. (0.23685) (C) (B) (B) (1839.84) (4.55)
31 32 33 34
w 35 36 37 38 39 40
(B) (C) (125.56)
w (6) (569) (1.862) (D) (B) (5.7319) (C)
41
(C)
42
(0.64)
43
(150) w 44
(1132.62)
45
(B)
46
(0.2604)
47
(D)
48 49
(B) (6767.53)
50
(B)
51
(C)
52
(2)
© 53
(0.01593)
54
(2)
55
(18.88)

.i n
c o
a .
d i
n o
.
w
w
w
©

Buy Online all GATE Books: shop.nodia.co.in


*Shipping Free* *Maximum Discount*
No part of this publication may be reproduced or distributed in any form or any means, electronic, mechanical,
photocopying, or otherwise without the prior permission of the author.

GATE SOLVED PAPER


Mechanical Engineering
2015-2

Copyright © By NODIA & COMPANY

Information contained in this book has been obtained by authors, from sources believes to be reliable. However,
neither Nodia nor its authors guarantee the accuracy or completeness of any information herein, and Nodia nor its
authors shall be responsible for any error, omissions, or damages arising out of use of this information. This book
is published with the understanding that Nodia and its authors are supplying information but are not attempting
to render engineering or other professional services.

NODIA AND COMPANY


B-8, Dhanshree Tower Ist, Central Spine, Vidyadhar Nagar, Jaipur 302039
Ph : +91 - 141 - 2101150
www.nodia.co.in
email : enquiry@nodia.co.in
GATE SOLVED PAPER - ME
2015-2

General Aptitude

Q. 1 - Q. 5 Carry one mark each.


Q. 1 Find the missing sequence in the letter series below:
A, CD, GHI, ?, UVWXY
(A) LMN (B) MNO
(C) MNOP (D) NOPQ

Q. 2
Let us _______.
.i n
Choose the correct verb to fill in the blank below:

(A) Introvert
o
(B) Alternate

c
(C) Atheist

a . (D) Altruist

Q. 3
following sentence?
d i
Choose the most appropriate word from the options given below to complete the

o
If the athlete had wanted to come first in the race, he_______ several hours

.n
every day.
(A) Should practice (B) Should have practiced
(C) Practised
w (D) Should be practicing

Q. 4
w
Choose the most suitable one word substitute for the following expression

w
Connotation of a road or way
(A) Pertinacious (B) Viaticum

Q. 5
©
(C) Clandestine (D) Ravenous

If x > y > I , which of the following must be true?


(i) In x > In y
(ii) ex > ey
(iii) yx > xy
(iv) cos x > cos y
(A) (i) and (ii) (B) (i) and (iiii)
(C) (iii) and (iv) (D) (ii) and (iv)

Q. 6 - Q. 10 Carry two marks each.


Q. 6 From a circular sheet of paper of radius 30 cm, a sector of 10% area is removed. If
the remaining part is used to make a conical surface, then the ratio of the radius
and height of the cone is______.

Buy Online all GATE Books: shop.nodia.co.in


*Shipping Free* *Maximum Discount*
GATE SOLVED PAPER - ME 2015-2

Q. 7 In the following question, the first and the last sentence of the passage in order
and numbered 1 and 6. The rest of the passage is split into 4 parts and numbered
as 2, 3, 4 and 5. These 4 parts are not arranged in proper order. Read the
sentences and arrange them in a logical sequence to make a passage and choose
the correct sequence from the given options.
1. One Diwali, te family rises early in the morning

.i n
2. The whole family, including the young and the old enjoy doing this

c o
3. Children let off fireworks later in the night with their friends
4. At sunset, the lamps are lit and the family performs various rituals

a .
5. Father, mother and children visit relatives and exchange gifts and sweets

(A) 2, 5, 3, 4 d i
6. Houses look so pretty with lighted lamps all around.
(B) 5, 2, 4, 2
(C) 3, 5, 4, 2
n o (D) 4, 5, 2, 3

.
Ms X will be in Bagdogra from 01/05/2014 to 20/05/2014 and from 22/05/2014
w
Q. 8
to 31/05/2014. On the morning of 21/05/2014, she will reach Kochi via Mumbai

w
Which one of the statements below is logically valid and can be inferred from the
above sentences?
w
(A) Ms. X will be in Kochi for one day, only in May

©
(B) Ms. X will be in Kochi for only one day in May
(C) Ms. X will be only in Kochi for one day in May
(D) Only Ms. X will be in Kochi for one day in May

Q. 9
.i n
log tan 1° + log tan 2° + ..... + log tan 89° is_______
(A) 1
c o(B) 1
2
(C) 0
a . (D) - 1

Q. 10

d i
Ram and Shyam shared a secret and promised to each other that it would remain
between them. Ram expressed himself in one of the following ways as given in the
choices below.
n o
.
Identify the correct way as per standard English.
(A) It would remain between you and me
w
(B) It would remain between I and you
w
(C) It would remain between you and I

w
(D) It would remain with me

© END OF THE QUESTION PAPER

Buy Online all GATE Books: shop.nodia.co.in


*Shipping Free* *Maximum Discount*
GATE SOLVED PAPER - ME 2015-2

Mechanical Engineering

Q. 1 - Q. 25 Carry one mark each.


Q. 1

. i n
The uniaxial yield stress of a material is 300 MPa. According to von Mises
criterion, the shear yield stress (in MPa) of the material is_______
Q. 2
is
c o
The primary mechanism of material removal in electrochemical machining (ECM)

(A) Chemical corrosion


a .
(B) Etching
(C) Ionic dissolution
d i
(D) Spark erosion
n o
Q. 3 .
Curl of vector V (x, y, z) = 2x2 i + 3z2 j + y3 k at x = y = z = 1 is
(A) - 3i
w (B) 3i
(C) 3i - 4j
w (D) 3i - 6k

Q. 4
w
A small ball of mass 1 kg moving with a velocity of 12 m/s undergoes a direct
central impact with a stationary ball of mass 2 kg. The impact is perfectly elastic.

Q. 5
©
The speed (in m/s) of 2 kg mass ball after the impact will be_______
A road is subjected to a unit-axial load within linear elastic limit. When the

. i n
change in the stress is 200 MPa, the change in the strain is 0.001. If the Poisson’s
ratio of the rod is 0.3, the modulus of rigidity (in GPa) is_______
Q. 6
(A) holds because the flow is steady
c o
Within a boundary layer for a steady incompressible flow, the Bernoulli equation

a .
(B) holds because the flow is incompressible

d i
(C) holds because the flow is transitional
(D) does not holds because the flow is frictional

n o
.
The atomic packing factor for a material with body centered cubic structure
Q. 7
is_______
Q. 8 w
If a foam insulation is added to a 4 cm outer diameter pipe as shown in the figure,
w
the critical radius of insulation (in cm) is_______

w
©

Buy Online all GATE Books: shop.nodia.co.in


*Shipping Free* *Maximum Discount*
GATE SOLVED PAPER - ME 2015-2

Q. 9 During the development of a product an entirely new process plan is made based
on design logic, examination of geometry and tolerance information. This type of
process planning is known as
(A) Retrieval
(B) Generative
(C) Variant
(D) Group technology based
.i n
Q. 10
c o
Annual demand of a product is 50000 units and the ordering cost is Rs. 7000 per
.
order considering the basic economic order quantity model, the economic order
a
holding cost (in Rs.) is _______
d i
quantity is 10000 units. When the annual inventory cost is minimized, the annual

Q. 11

n o
Sales data of a product is given in the following table:
Month
.
January February March April May
Number of unit sold
w 10 11 16 19 25

w
Regarding forecast for the month of June, which one of the following statements
is TRUE?
w
(A) Moving average will forecast a higher value compared to regression

©
(B) Higher the value of order N, the greater will be the forecast value by moving
average
(C) Exponential smoothing will forecast a higher value compared to regression

i n
(D) Regression will forecast a higher value compared to moving average
.
Q. 12

c o
The Vander Waals equation of state is ^p + h (v − b) = RT , where p is pressure,
a
v2
v is specific volume, T is temperature and R is characteristic gas constant. The
SI unit of a is
a .
(A) J/kg, K
(B) m3/kg
d i
(C) m5/kg-s2

n o
(D) Pa/kg
.
Q. 13

TRUE? w
Which of the following statements regarding a Rankine cycle with reheating are

w
(i) increase in average temperature of heat addition

w
(ii) reduction in thermal efficiency

©
(iii) drier steam at the turbine exit
(A) only (i) and (ii) are correct
(B) only (ii) and (iii) are correct
(C) only (i) and (iii) are correct
(D) (i), (ii) and (iii) are correct

Q. 14 In a spring-mass system, the mass is m and the spring constant is k . The critical
damping coefficient of the system is 0.1 kg/s. In another spring-mass system, the
mass is 2 m and the spring constant is 8K. The critical damping coefficient (in
kg/s) of this system is_______
Q. 15 The COP of a cannot heat pump operating between 6°C and 37°C is______

Buy Online all GATE Books: shop.nodia.co.in


*Shipping Free* *Maximum Discount*
GATE SOLVED PAPER - ME 2015-2

Q. 16 The number of degrees of freedom of the planetary gear train shown in the figure
is

. i n
c o
a .
d i
(A) 0
n o (B) 1
(C) 2
. (D) 3

Q. 17
w
A rope-brake dynamometer attached to the crank shaft of an I.C. engine measures

w
a brake power of 10kW when the speed of rotation of the shaft is 400 rad/s. The
shaft torque (in N-m) sensed by the dynamometer is _______
Q. 18 w
At least one eigen value of a singular matrix is
©
(A) Positive
(C) Negative
(B) Zero
(D) Imaginary

Q. 19

. i n
If the fluid velocity for a potential flow is given by V (x, y) = u (x, y) i + v (x, y) j
with usual notations, then the slope of the potential line at (x, y) is
(A) v
u
c o
(B) - u
v
2
(C) v 2
a . (D) u
u
i
v
Correct option is (B)
d
Sol. 19

Q. 20

n o
Which one of the following statements is TRUE?
(A) The ‘GO’ gage controls the upper limit of a hole
.
(B) The ‘NO’ gage controls the lower
w
(C) The ‘GO’ gage controls the lower limit of a hole

w
(D) The ‘NO GO’ gage controls the lower limit of a hole

Q. 21
w
There vendors were asked to supply a very high precision component. The
respective probabilities of their meeting the strict design specifications are 0.8,
©
0.7 and 0.5. Each vendor supplies one component. The probability that out of
total three components supplied by the vendors, at least one will meet the design
specification is_______
Q. 22 The Laplace transform of e i5t where i = − 1 , is
(A) s2 - 5i
s - 25
(B) s2 + 5i
s + 25
(C) s2 + 5i
s − 25
(D) s2 − 5i
s + 25

Buy Online all GATE Books: shop.nodia.co.in


*Shipping Free* *Maximum Discount*
GATE SOLVED PAPER - ME 2015-2

Q. 23 A gas is stored in a cylindrical tank of inner radius 7 m and wall thickness 50


mm. The gage pressure of the gas is 2MPa. The maximum shear stress (in MPa)
in the wall is
(A) 35 (B) 70
(C) 140 (D) 280

Q. 24

.i n
In the laminar flow of air ( Pr = 0.7 ) over a heated plate if δ and δT denote,
respectively, the hydrodynamic and thermal boundary layer thickness, then
(A) d = dT (B) d > dT
c o
(C) d < dT
.
(D) δ = 0 but δT ! 0

a
Q. 25 At x = 0 , the function f (x) = x has
(A) A minimum
d i
(B) A maximum
n o
(C) A point of inflexion
.
w
(D) Neither a maximum nor minimum

w
w
Q. 26 - Q. 55 Carry two marks each.

The total emissive power of a surface is 500 W/m2 at a temperature T1 and 1200
©
Q. 26
W/m2 at a temperature T2 . Where the temperatures are in Kelvin. Assuming the
emissivity of the surface to be constant, the ratio of the temperatures TT is
1
2

(A) 0.308
(C) 0.803
.i n
(B) 0.416
(D) 0.874

Q. 27

c o
A hallow shaft of 1m length is designed to transmit a power of 30 kW at 700 rpm.

a .
The maximum permissible angle of twist in the shaft is 1°. The inner diameter of
the shaft is 0.7 times the outer diameter. The modulus of rigidity is 80 GPa. The
i
outside diameter (in mm) of the shaft is_______
d
Q. 28

n o
In a Rankine cycle, the enthalpies at turbine entry and outlet are 3159 kJ/kg and
2187 kJ/kg, respectively. If the specific pump work is 2 kJ/kg the specific steam
.
consumption (in kg/kW-h) of the cycle based on net output is_______
Q. 29
w
A single point cutting tool with 0° rake angle is used in an orthogonal machining

w
process. At a cutting speed of 180 m/min, the thrust fore is 490 N. If the coefficient
of friction between the tool and the chip is 0.7, then the power consumption (in
w
kW) for the machining operation is _______
Q. 30
©
The chance of a student passing an exam is 20%. The chance of a student passing
the exam and getting above 90% marks in it is 5%. Given that a student passes
the examination, the probability that the student gets above 90% marks is
(A) 1
18
(B) 1
4
(C) 2
9
(D) 5
18

Buy Online all GATE Books: shop.nodia.co.in


*Shipping Free* *Maximum Discount*
GATE SOLVED PAPER - ME 2015-2

Q. 31 A manufacturer has the following data regarding a product:


Fixed cost per month = Rs. 50000
Variable cost per unit = Rs. 200
Selling price per unit = Rs. 300
Production capacity = 1500 units per month

profit (in Rs.) is _______


. i n
If the production is carried out at 80% of the rated capacity, that the monthly

Q. 32

c o
The head loss for a laminar incompressible flow through a horizontal circular

a .
pipe is h1 . Pipe length and fluid remaining the same, if the average flow velocity
doubles and the pipe diameter reduces to half its previous value, the head loss is
h2 . The ratio hh is
(A) 1
2
1

d i (B) 4
(C) 8
n o (D) 16

Q. 33
.
A cube and a sphere made of cat iron (each of volume 1000 cm3) were cast

w
under identical conditions. The time taken for solidifying the cube was 4s. The
solidification time (in s) for the sphere is_______
w
One kg of air (R = 287 J/kg.K) undergoes an irreversible process between
w
Q. 34
equilibrium state 1 (20°C, 0.9 m3) and equilibrium state 2 (20°C, 0.6m3). The

Q. 35
©
change in entropy S2 - S1 (in J/kg.K) is_______
In a plane stress condition, the components of stress at point are σ x = 20 MPa,
σ y = 80 MPa and τ xy = 40 MPa. The maximum shear stress (in MPa) at the
point is
(A) 20
.
(B) 25i n
(C) 50
o
(D) 100
c
Q. 36

a .
Work is done on an adiabatic system due to which its velocity changes from 10

d i
m/s to 20 m/s, elevation increases by 20 m and temperature increases by 1 K.
The mass of the system is 10 kg. CV = 100 J/kg.K and gravitational acceleration

n o
is 10 m/s2. If there is no change in any other component of the energy of the
system, the magnitude of total work done (in kJ) on the system is_______
Q. 37 .
A hollow shaft do = 2di where do and di are the outer and inner diameters
w
respectively needs to transmit 20 kW power at 3000 RPM. If the maximum

(A) 11.29 mm w
permissible shear stress is 30 MPa, do is
(B) 22.58 mm
w
(C) 33.87 mm (D) 45.16 mm

Q. 38
©
A cantilever beam OP is connected to another beam PQ with a pin joint as
shown in the figure. A load of 10 kN is applied at the mid-point of PQ . The
magnitude of bending moment (in kN-m) at fixed end O is \

(A) 2.5 (B) 5


(C) 10 (D) 25

Buy Online all GATE Books: shop.nodia.co.in


*Shipping Free* *Maximum Discount*
GATE SOLVED PAPER - ME 2015-2

Q. 39 The flow stress (in MPa) of a material is given by


σ = 500ε0.1
Where ε is true strain. The Young’s modulus of elasticity of the material is 200
GPa. A block of thickness 100 mm made of this material is compressed to 95 mm
thickness and then the load is removed. The final dimension of the block (in mm)

Q. 40
is _______

.i n
The initial velocity of an object is 40 m/s. The acceleration a of the object is
given by the following expression: a = 0.1 V
c o
after 3 seconds will be_______
a .
Where V is the instantaneous velocity of the object. The velocity of the object

Q. 41

d i
A balanced counter flow heat exchanger has a surface area of 20m2 and overall
heat transfer coefficient of 20W/m2-K Air (CP = 1000 J/kg-K) entering at 0.4
o
kg/s and 280 K is to be preheated by the air leaving the system at 0.4 kg/s and
n
(A) 290 .
300 K. The outlet temperature (in K) of the preheated air is
(B) 300
(C) 320 w (D) 350

Q. 42 w
The values of function f (x) at 5 discrete point are given below:
x w 0 0.1 0.2 0.3 0.4
f (x)
© 0 10 40 90
0.4
160

Using Trapezoidal role with step size of 0.1, the value of # f (x) dx is_______
Q. 43
.i n 0
In a two-stage wire drawing operation, the fractional reduction (ratio of change

c o
in cross-sectional area to initial cross-sectional area) in the first stage is 0.4. The
fractional reduction in the second stage is 0.3. The overall fractional reduction is
(A) 0.24
a .
(B) 0.58
(C) 0.60

d i (D) 1.00

A single-degree. Freedom spring-mass system is subjected to a sinusoidal force of


o
Q. 44
10 N amplitude and frequency ω along the axis of the spring. The stiffness of the

. n
spring is 150 N/m, damping factor is 0.2 and the undamped natural frequency is
10ω . At steady state, the amplitude of vibration (in m) is approximately
(A) 0.05 w
(B) 0.07
w
(C) 0.70
(D) 0.90
w
Q. 45
©
For a fully developed laminar flow of water (dynamic viscosity 0.001 Pa-s) through
a pipe of radius 5 cm. The axial pressure gradient is - 10 Pa/m. The magnitude
of axial velocity (in m/s) at a radial location of 0.2 cm is_______
Q. 46 In a certain slider-crank mechanism, lengths of crank and connecting rod are
equal. If the crank rotates with a uniform angular seed of 14 rad/s and the crank
length is 300 mm, the maximum acceleration of the slider (in m/s2) is______

Q. 47 The surface integral ## π1 (9xi - 3yj). ndS over the sphere given by x
s
2
+ y2 + z2 = 9
is_______

Buy Online all GATE Books: shop.nodia.co.in


*Shipping Free* *Maximum Discount*
GATE SOLVED PAPER - ME 2015-2

Q. 48 A cantilever beam with square cross-section of 6 mm side is subjected to a load


of 2kN normal to the top surface as shown in the figure. The young’s modulus of
slope. (in radian) at Q (20 mm from the fixed end) is_______

. i n
c o
Q. 49

a .
A cylindrical uranium fuel rod of radius 5 mm in a nuclear reactor is generating

d i
heat at the rate of 4 # 107 W/m3. The rod is cooled by a liquid (convective
heat transfer coefficient 1000 W/m2.K) at 25°C. At steady state, the surface

(A) 308
n o
temperature (in K) of the rod is
(B) 398
(C) 418 . (D) 448

Q. 50 w
For the same values of peak pressure, peak temperature and heat rejection, the
w
correct order of efficiencies for Otto, Dual and Diesel cycles is

w
(A) h Otto > h Dual > h Diesel
(C) h Dual > h Diesel > h Otto
(B) h Diesel > h Dual > h Otto
(D) h Diesel > h Otto > h Dual

Q. 51 ©
During a TIG welding process, the are current and are voltage were 50 A and 60
V, respectively, when in the welding speed was 150 mm/mi. In another process.

. i n
the TIG welding is carried out at a welding speed of 120 mm/min at the same
are voltage and heat input to the material so that weld quality remains the same.

(A) 40.00
c o
The welding current (in A) for this process is
(B) 44.72
(C) 55.90
a . (D) 62.25

Q. 52

dy d i
Consider the following differential equation:

dt
n o
=− 5y ; initial condition : y = 2 at t = 0

(A) - 5e-10 .
The value of y at t = 3 is
(B) 2e-10
(C) 2e-15 w (D) - 15e2

Q. 53 w
For the truss shown in the figure, the magnitude of the force (in kN) in the
w
member SR is

(A) 10 (B) 14.14


(C) 20 (D) 28.28

Buy Online all GATE Books: shop.nodia.co.in


*Shipping Free* *Maximum Discount*
GATE SOLVED PAPER - ME 2015-2

Q. 54 A project consists of 7 activities. The network along with the time durations (in
days) for various activities is shown in the figure.

.i n
c o
.
The minimum time (in days) for completion of the project is_______

a
Q. 55

d i
A resistance-capacitance relaxation circuit is used in an electrical discharge
machining process. The discharge voltage is 100 V. At a spark cycle time of µs ,

is
n o
the average power input required is 1 kW. The capacitance (in µ F) in the circuit

(A) 2.5
.
(B) 5.0
w
(C) 7.5
(D) 10.0 w
w
© END OF THE QUESTION PAPER

.i n
c o
a .
d i
n o
.
w
w
w
©

Buy Online all GATE Books: shop.nodia.co.in


*Shipping Free* *Maximum Discount*
GATE SOLVED PAPER - ME 2015-2

ANSWER KEY
General Aptitude
1 2 3 4 5 6 7 8 9 10
(C) (B) (B) (B) (A) (1.9 to
2.2)
. n
(B)
i
(B) (C) (A)

Mechanical Engineering
c o
1 2 3 4 5
a . 6 7 8 9 10
(171 to
175)
(C) (A) (7.8 to
8.2)
(76 to
78)
d i (D) (0.66 to (4.9 to
0.69) 5.1)
(B) (34000
to

n o 36000)
11 12 13
.
14 15 16 17 18 19 20
(D) (C) (C)
w
(0.38 to (9.8 to
0.42) 10.2)
(C) (24 to
26)
(B) (B) (C)

21 22 23
w 24 25 26 27 28 29 30
(0.96 to
0.98)
(B)
w(C) (C) (A) (C) (43 to
45)
(3.6 to
3.8)
(2.0 to
2.2)
(B)

31
(68000
©
32
(C)
33 34
(6.0 to (-117 to
35
(C)
36
(4.5)
37
(B)
38
(C)
39
(95.14
40
(29.5 to
to
72000)
6.3) -115)

. i n to
95.20)
29.7)

41
(B)
42
(21.8 to
43
(B)
44
(B)
45
(6.2 to
46
(115 to
c o 47 48
(214 to (0.15 to
49
(B)
50
(B)
22.2) 6.3) 120)
a . 218) 0.17)
51
(A)
52
(C)
53
(C)
54
(39 to
55
(B)
d i
40)
n o
.
w
w
w
©

Buy Online all GATE Books: shop.nodia.co.in


*Shipping Free* *Maximum Discount*
No part of this publication may be reproduced or distributed in any form or any means, electronic, mechanical,
photocopying, or otherwise without the prior permission of the author.

GATE SOLVED PAPER


Mechanical Engineering
2015-3

Copyright © By NODIA & COMPANY

Information contained in this book has been obtained by authors, from sources believes to be reliable. However,
neither Nodia nor its authors guarantee the accuracy or completeness of any information herein, and Nodia nor its
authors shall be responsible for any error, omissions, or damages arising out of use of this information. This book
is published with the understanding that Nodia and its authors are supplying information but are not attempting
to render engineering or other professional services.

NODIA AND COMPANY


B-8, Dhanshree Tower Ist, Central Spine, Vidyadhar Nagar, Jaipur 302039
Ph : +91 - 141 - 2101150
www.nodia.co.in
email : enquiry@nodia.co.in
GATE SOLVED PAPER - ME
2015-3

General Aptitude

Q. 1 - Q. 5 Carry one mark each.


Q. 1 Fill in the blank with the correct idiom/phrase.
That boy from the town was a _______ in the sleepy village.
(A) dog out of herd
(B) sheep from the heap
(C) fish out of water
(D) bird from the flock
.i n
Sol. 1 Correct option is (C).
c o
(A) dog out of herd
It is not an idiom/phrase.
a .
(B) sheep out of water
It is not an idiom/phrase. d i
(C) fish out of water o
.n
You feel like a fish out of water if you are surrounded by people who are different

w
to you and its making you feel a little uncomfortable
(D) bird from the flock
w
It is not an idiom/phrase.
Q. 2 w
Five teams have to compete in a league, with every team playing every other

©
team exactly once, before going to the next round. How many matches will have
to be held to complete the league round of matches?
(A) 20 (B) 10
(C) 8 (D) 5
Sol. 2 Correct option is (B).
Lets assume 5 teams are (A), (B), (C), (D) and (E)
From team (A), playing with (B), (C), (D) and (E) team, there are 4 matches.
From team (B), it will paly with (C), (D) and (E) team because with (A) it is
already played so there are 3 matches
As from team (C), 2 matches
As from team (D), 1 match
Total match played in the league
are 4 + 3 + 2 + 1 = 10 matches
ALTERNATIVE METtOD
Numbers of matches = number of ways to select two teams
= 5c2 = 10

Buy Online all GATE Books: shop.nodia.co.in


*Shipping Free* *Maximum Discount*
GATE SOLVED PAPER - ME 2015-3

Q. 3 Choose the statement where underlined word is used correctly.


(A) When the teacher eludes to different authors, he is being elusive.
(B) When the thief keeps eluding the police, he is being elusive.
(C) Matters that are difficult to understand, identify of remember are allusive.
(D) Mirages can be allusive, but a better way to express them is illusory.
Sol. 3 Correct option is (B).
.i n
Elusive [adjective]
Difficult to find, catch, achieve
c o
.
Example- Success will become ever more elusive.
a
difficult to remember

d i
Example- The elusive thought he had moment before.
Elude

n o
escape from or avoid (a danger, enemy)
.
typically in a skilfull or cunning way

w
Example- when thief keeps eluding the police, he is being elusive. (option B)
Allusive (Adjective)
w
Using or cantoning suggestion rather than explicit mention

w
[Allusive reference to the body]
Example- The heading in almost my daily newspaper usually include general

Q. 4
©
allusive phrases a well as idioms.
Choose the appropriate word/phrase, out of the four options given below, to
complete the following sentence:
Apparent lifelessness _______ dormant life.
.i n
(A) harbours
(C) supports
c o
(B) leads to
(D) affects
Correct option is (A).
a .
i
Sol. 4
(A) harbours (verb)

o
Meaning- Gives none or shelter to d
(B) leads to (noun)
. n
Example- woodlands that once harboured a colony of red deer.

w
Meaning- The initiative in an action

w
Example- Britain is now taking the lead in environmental policies
(C) supports (verb)

w
Meaning- Give assistance to, especially financially

©
Example- Poor childers of school supports by government.
(D) affects (verb)
Meaning- Cause to happen; bring about
Example- The prime minister effected many policy changes.
Conclusion: In the blank space a verb has to be there to complete the meaning of
sentence so ‘Harbours’ gives a meaning full sentence.
Q. 5 Tanya is older than Eric.
Cliff is older than Tanya.
Eric is older than Cliff.
If the first two statements are true, then the third statement is:
(A) True (B) False
(C) Uncertain (D) Data insufficient

Buy Online all GATE Books: shop.nodia.co.in


*Shipping Free* *Maximum Discount*
GATE SOLVED PAPER - ME 2015-3

Sol. 5 Correct option is (B).


From first two statements, we conclude that Cliff is oldest and Eric is youngest.
So, given thirst statement that, “Eric is older than cliff” is wrong because Eric is
the youngest among three.

Q. 6 - Q. 10 Carry two marks each.


. i n
Q. 6

c o
In the given figure angle Q is a right angle, PS:QS = 3: 1, RT:QT = 5: 2 and

in cm2 is_______.
a .
PU:UR = 1: 1. If area of triangle QTS is 20 cm2, then the area of triangle PQR

d i
n o
.
w
w
Sol. 6
w
Correct answer is 280.
The given triangle is
©
. i n
c o
a .
Assume that SQ is x , so

d i
PS : QS = 3 : 1, i.e. PS = 3x

n o
Again, we assume that QT is 2y . So,
RT : QT = 5 : 2 , i.e. RT = 5y
Area of TQTS .
w
= 1 # (QS) # (QT)
2
w = 1 # (x) # (2y) = xy = 20 cm2
w
Area of TPQR
2

© = 1 # (PQ) # (QR)
2
= 1 # (PS + QS) # (RT + QT)
2
= 1 # (3x + x) # (5y + 2y) = 14xy
2
Area of TPQR
= 14 # 20 = 280 cm2
Q. 7 Given below are two statements followed by two conclusions. Assuming these
statements to be true, decide which one logically follows.
Statements:

Buy Online all GATE Books: shop.nodia.co.in


*Shipping Free* *Maximum Discount*
GATE SOLVED PAPER - ME 2015-3

I. No manager is a leader.
II. All leaders are executives.
Conclusions:
I. No manager is an executive.
II. No executive is a manager.
(A) Only conclusion I follows
(B) Only conclusion II follows .i n
(C) Neither conclusion I nor II follows
c o
(D) Both conclusions I and II follow
a .
Sol. 7 Correct option is (D).
Statement 1- No manager is a leader d i
n o
.
w
w
As statement representing both are independent.
Statement 2 - All leaders are executives
w
©
.i n
Conclusions: Their may be a chance that some or all managers can be executives

vein diagram.
c o
and also it may be possible that none of manager can be executive as showing in

a .
d i
Possible conclusion

n o
.
w
Possible conclusion
w
w
©
Possible conclusion

So both conclusions follow statement and .


Q. 8 Select the appropriate option in place of underlined part of the sentence.
Increased productivity necessary reflects greater efforts made by the employees.
(A) Increase in productivity necessary
(B) Increase productivity is necessary
(C) Increase in productivity necessarily
(D) No improvement required

Buy Online all GATE Books: shop.nodia.co.in


*Shipping Free* *Maximum Discount*
GATE SOLVED PAPER - ME 2015-3

Sol. 8 Correct option is (C).


The correct statement will be
Increase in productivity necessarily reflects greater efforts made by the employees.
Explanation
Increase will come in the first form as given statement must be in present tense
as it showing the fact or generalization

. i n
Necessarily will be used instead of necessary because here it as a adverb and in

o
the meaning of as a necessary result; inevitably.

c
Q. 9

a .
A coin is tossed thrice. Let X be the event that head occurs in each of the first
two tosses. Let Y be the event that a tail occurs on the third toss. Let Z be the

d i
event that two tails occur in three tosses.
Based on the above information, which one of the following statements is TRUE?
o
(A) X and Y are not independent
n
.
(B) Y and Z are dependent

w
(C) Y and Z are independent
(D) X and Z are independent
w
Sol. 9

w
Correct option is (D).
For a coin tossed 3 times

©
Let X be the event that head occurs in each of the first two tosses
So set X = {HHT, HHH}
Now let y be the event that a tail occurs on the third toss

set Y = {HHT}
. i n
Statement implies first two must be head for occurring tail in third toss

it implies event Y is dependent of event X .


c o
So
a .
Now let Z be the event that two tails occurs in the three bosses
set Z = {TTH, TTT, THT, HTT}

common occurrence. d i
Now we can see the event X and event Z are independent as their set has none

Q. 10
n o
Right triangle PQR is to be constructed in the xy-plane so that the right angle is
.
at P and line PR is parallel to the x-axis. The x and y coordinates of P, Q and

w
R are to integers that satisfy the inequalities: - 4 # x # 5 and 6 # y # 16 . How
many different triangles could be constructed with these properties?
(A) 110 w (B) 1,100

w
(C) 9,900 (D) 10,000
Sol. 10
©
Correct option is (C).

END OF THE QUESTION PAPER

Buy Online all GATE Books: shop.nodia.co.in


*Shipping Free* *Maximum Discount*
GATE SOLVED PAPER - ME 2015-3

Mechanical Engineering

Q. 1 - Q. 25 Carry one mark each.


2
# x ln x dx
Q. 1 Using a unit step size, the value of integral
is_______.
1

.i n by trapezoidal rule

Sol. 1 Correct answer is 0.6931


2
# x ln x dx c o
Integral
1
.
trapezoidal rule using step size 1

a
According to trapezoidal rule
x
#
1

2
i
y dx = h "^y 0 + yn h + 2 ^y1 + y2 + y 3 + ....yn − 1h,
d
x 0

o
y 0 = F (x 0) = F (1) = (1) (ln 1) = 0
n
.
y1 = F (x1) = F (1 + 1) = 2 (ln 2) = 2 ln 2
2
w
x ln x dx = 1 [0 + 2 ln 2] = ln 2 = 0.6931
# 2
Q. 2
1

w
In the figure, link 2 rotates with constant angular velocity ω 2 . A slider link 3

w
moves outwards with a constant relative velocity VQ/P , where Q is a point on
slider 3 and P is a point on link 2. The magnitude and direction of Coriolis
©
component of acceleration is given by

.i n
c o
a .
d i
n o
(A) 2 ω 2 VQ/P ; direction of VQ/P rotated by 90° in the direction of ω 2
.
(B) ω 2 VQ/P ; direction of VQ/P rotated by 90° in the direction of ω 2
w
(C) 2 ω 2 VQ/P ; direction of VQ/P rotated by 90° opposite to the direction of ω 2

w
(D) ω 2 VQ/P ; direction of VQ/P rotated by 90° opposite to the direction of ω 2
Sol. 2
w
Correct option is (A).
Coriolis component of acceleration associated with slider when slider is sliding on
©
the rotating object
ac = 2VW
where V = sliding velocity of slider
ω = angular velocity of body [slotted bar]
on which slider is sliding
As per Figure given in question

Buy Online all GATE Books: shop.nodia.co.in


*Shipping Free* *Maximum Discount*
GATE SOLVED PAPER - ME 2015-3

. i n
c o
v
a .
aC = 2VQ/p ω 2 = magnitude of coriolis acceleration

sense by 90°. d i
For direction of coriolis acceleration by taking sense of ωv and rotating Vv in the

Q. 3
n o
The ratio of momentum diffusivity (ν ) to thermal diffusivity (α ), is called
.
(A) Prandtl number

w
(B) Nusselt number

w
(C) Biot number
(D) Lewis number
Sol. 3
w
Correct option is (A).
©
Prandtl number (Pr)- It is the only dimensionless no. which is a property of
fluid defined as the ratio between kinematic viscosity of fluid and its thermal
diffusivity.
Pr = ν =
α i
thermal diffusivity
n
kinematic vis cos ity µCP
. =
K
= f (T)
where,

c o
CP = specific heat at coast pressure
K = thermal conductivity
a .
d i
Prandtl number signifies relative magnitude of diffusion rate of momentum and
heat energy to the respective boundary layer.

n o
Nusselt number (Nu)- The nusselt number represent the enhancement of heat
transfer through a fluid layer as a result of convective relative to conduction
.
across the same fluid layer
w q
Nu = conv = hLC
where,
w q cond KF

w h = convective heat transfer coefficient


LC = characteristic length
© KF = thermal conductivity of fluid.
Biot number (Bi)- This ratio determines whether or not the temperature inside
a body will vary significantly in space, while the body heats or cools over time,
from a thermal gradient opplied to its surface
Biot Number is defined as
Bi = hLC
Kb
int ernal conduction resis tan ce offered by body
=
external (surface) convective resis tan ce
hLC = LC /KA
Kb 1/hA

Buy Online all GATE Books: shop.nodia.co.in


*Shipping Free* *Maximum Discount*
GATE SOLVED PAPER - ME 2015-3

where,
h = film coefficient or heat transfer coefficient
LC = characteristic length, which is commonly defined as the
V
volume of the body devided by surface area of body LC = body
A surface
Kb = thermal conductivity of body

.i n
Lewis Number (Le)- Lewis number is a dimensionless number defined as the
ratio of thermal diffusivity to mass diffusivity.

c o
transfer by convection
a .
It is used to character fluid flow where there is simultaneous heat and mass

Le = α
D
d i
where,

n o
α = thermal diffusivity
.
D = mass diffusivity
Q. 4 w
Let φ be an arbitrary smooth real valued scalar function and Vy be an arbitrary
w
smooth vector valued function in a three-dimensional space. Which one of the

w
following is an identity?
(A) Curl (fVv) = d (f DivVv) (B) Div Vv = 0

Sol. 4
©
(C) Div Curl Vy = 0
Correct option is (C).
(D) Div (fVy) = f DivVy

Q. 5 If P (X) = 14 , P (Y) = 1
3 and P (X + Y) = 1
12

.i n
, the value of P ^ YX h is

(A) 1
4
c o
(B) 4
25
(C) 1
3
a . (D) 29
50
Sol. 5 Correct option is (C).
Given that d i
o
P (X) = 1 , P (Y) = 1
n
4
. 3

P (X + Y) = 1 , P b X l = ?
Y
12 w
w
The conditional property of Y , given X is denoted by
P b X l and is given by
Y
w Pb X l =
Y P (Y + X)
© P (X)

P b X l = 121 = 1
Y 1

4
3
Q. 6 Three parallel pipes connected at the two ends have flow-rates Q1 , Q2 and Q 3
respectively and the corresponding frictional head losses are hL1 , hL2 and hL3
respectively. The correct expressions for total flow rate (Q ) and frictional head
loss across the two ends (hL ) are
(A) Q = Q1 + Q2 + Q 3 ; hL = hL1 + hL2 + hL3
(B) Q = Q1 + Q2 + Q 3 ; hL = hL1 = hL2 = hL3
(C) Q = Q1 = Q2 = Q 3 ; hL = hL1 + hL2 + hL3
(D) Q = Q1 = Q2 = Q 3 ; hL = hL1 = hL2 = hL3

Buy Online all GATE Books: shop.nodia.co.in


*Shipping Free* *Maximum Discount*
GATE SOLVED PAPER - ME 2015-3

Sol. 6 Correct option is (B).


For a pipe that branches out into two (or more) parallel pipe and then rejoins
at a junction downstream, the total flow rate is the sum of the flow rotes in the
individual pipes. The pressure drop (or head loss) in each individual pipe connected
in parallel must be same since TP = Pin − Pout and the junction pressure Pin and
Pout are the same for all the individual pipes.
Hence for three pipe connection
. i n
c o
a .
d i
n o
Q total = Q1 + Q2 + Q 3
.
hL1 = Pout − Pin

w
hL2 = Pout − Pin = hL3
Hence
w
hL = hL1 + hL2 + h 3 and Q total = Q1 + Q2 + Q 3
Q. 7
w
An air-standard Diesel cycle consists of the following processes:
1-2: Air is compressed isentropically.
©
2-3: Heat is added at constant pressure.
3-4: Air expands isentropically to the original volume.
4-1: ;Heat is rejected at constant volume.

. i n
If γ and T denote the specific heat ratio and temperature, respectively, the
efficiency of the cycle is
(A) 1 - T4 - T1 o
(B) 1 - T4 - T1
c
T3 - T2
γ (T4 - T1)
a . γ (T3 - T2)
T4 - T1

Sol. 7
(C) 1 -
T3 - T2
Correct option is (B). d i (D) 1 -
(γ - 1) (T3 - T2)

o
For an air-standard diesel cycle
n
.
w
w
w
©

Efficiency (η ) = work done


heat sup plied
work done = (− h 4 + h 3) − (h2 − h1) = (h 3 − h2) + (h1 − h 4)
heat supplied = h 3 − h2
(h − h2) + (h1 − h 4)
η = 3 = 1 − h 4 − h1
(h 3 − h2) h3 − h2

Buy Online all GATE Books: shop.nodia.co.in


*Shipping Free* *Maximum Discount*
GATE SOLVED PAPER - ME 2015-3

= 1 − 1 :T3 − T2 D
CV [T4 − T1] T4 − T1
η = 1−
CP [T3 − T2] γ
Q. 8 Which of the following statements are TRUE for damped vibrations?
P. For a system having critical damping, the value of damping ratio is unity and
system does not undergo a vibratory motion.

i n
Q. Logarithmic decrement method is used to determine the amount of damping
in a physical system.
.
o
R. In case of damping due to dry friction between moving surfaces resisting force
c
S.
a .
of constant magnitude acts opposite to the relative motion.
For the case of viscous damping, drag force is directly proportional to the
square of relative velocity.
(A) P and Q only d i
(B) P and S only
(C) P, Q and R only
n o (D) Q and S only
Sol. 8 Correct option is (C). .
w
(S) Damped system of vibration

w
w
©
.i n
damping force α velocity
Fdamping = cxo
c o
where
a .
s = spring constant
c = damping coefficient
d i
equation of damped system

n o
xp + (23wn) xo + w n2 x = 0
.
where
(P) For w
z = c = damping factor/damping ratio
2w n m

w
z > 1 over damped system
4 No vibrations
w
z = 1 critically damped system
z < 1 under damped system
(Q) ©

Buy Online all GATE Books: shop.nodia.co.in


*Shipping Free* *Maximum Discount*
GATE SOLVED PAPER - ME 2015-3

decremental ratio = x 0 = x1 = e3w td


n

x1 x 2
logarithmic decrement (δ ) = 1 n e3w td = 3wn td
n

δ = 3w n 2p = 2pz
1 − 32 (wn) 1 − z2

i n
(R) Friction between moving surfaces is proportional to opplied normal force
.
Fr α - N
Fr = µN
c o
.
µ = kinematic friction coefficient between moving surfaces
a
d i
Fr = µN (opposite direction)
Resistance force is constant in magnitude and acting opposite to the relative
motion
n o
Q. 9
.
In a machining operation, if the generatrix and directrix both are straight lines,

(A) cylindrical w
the surface obtained is
(B) helical
(C) plane w (D) surface of revolution
Sol. 9
w
Correct option is (C).

©
Generatrix- In geometry, generotrix or generator is a point, curve or surface that,
when moved generates a new shape
Directrix- The path of generatrix motion is called directrix

Generatrix . n
Table showing generatrix and directrix of various curves
i Directrix
Cylindrical surfaces Straight line
c o Fixed curve
Helicel surfaces Curve
a . Straight line
Plane
Surface of revolution d i
Straight line
Plane curve
Straight line
Fixed curve

n o
Q. 10
.
Saturated vapor is condensed to saturated liquid in a condenser. The heat capacity
ratio is Cr = CC . The effectiveness (ε ) of the condenser is
w
min
max

1 − exp [− NTU (1 + Cr )] 1 - exp [- NTU (1 - Cr )]


(A)
w 1 + Cr
(B)
1 - Cr exp [- NTU (1 - Cr )]
(C) NTU
w
1 + NTU
(D) 1 - exp (- NTU)
Sol. 10
©
Correct option is (D).
Effectiveness for condensers and evaporators

Buy Online all GATE Books: shop.nodia.co.in


*Shipping Free* *Maximum Discount*
GATE SOLVED PAPER - ME 2015-3

‘C ’ represent heat capacity


Heat capacity ratio(R) = C min
C max
C max $ 3 for condenser and evaporators
R = C min . 0
3
For parallel heat exchanger
.i n
o
−NTU (1 + R)
Effectiveness = 1 − e

For counter heat exchanger


1+R

. c
−NTU (1 − R)
Effectiveness = 1 − e −NTU (1 − R)
i a
For R = 0 ,
1−Re

o d
.n
! parrallel =! counter = 1 − e−NTU
In the notation (a/b/c) : (d/e/f) for summarizing the characteristics of queueing
w
Q. 11
situation, the letters ‘b’ and ‘d’ stand respectively for

w
(A) service time distribution and queue discipline

w
(B) number of servers and size of calling source
(C) number of servers and queue discipline

Sol. 11
©
(D) service time distribution and maximum number allowed in system
Correct option is (A).

i n
Representation of queueing model is given by kendall lee notation
The general form of notation is
.
(a/b/c) : (d/e/f)
c o
where,

a .
a : Probability distribution for arrival pattern
i
b : Probability distribution for service pattern
d
n o
c : No. of server within system
d : Service rule or service pattern
.
e : Size and capacity of system
w
f : Size or capacity of calling population
Q. 12
w
A cylindrical tank with closed ends is filled with compressed air at a pressure of

w
500 kPa. The inner radius of the tank is 2 m and it has wall thickness of 10 mm.
The magnitude of maximum in-plane shear stress (in MPa) is_______.
Sol. 12
©
Correct answer is 25.
For a thin cylinder

longitudinal stress for thin cylinder is given by (σ L) = Pri


2t

Buy Online all GATE Books: shop.nodia.co.in


*Shipping Free* *Maximum Discount*
GATE SOLVED PAPER - ME 2015-3

500 # 103 # 2 = 50 mPa


2 # 10 # 163
Hoop stress for thin cylinder is given by ( σ h ) = σ $ ri
t
500 # 103 # 2 = 100 mPa
10 # 10-3

diagram
. n
Maximum in-plane shear stress is given by ^ s -2 s h as shown in Mohr’s circle
i
1 2

c o
a .
d i
n o
.
w
τ max = s h − s L
w 2

w τ max = 100 − 50 = 25 mPa


2

Q. 13
©
So maximum in plane-shear stress in given cylinder is 25 mPa.
Which two of the following joining processes are autogeneous?
(i) Diffusion welding
(ii) Electroslag welding
. i n
(iii) Tungsten inert gas welding
(iv) Friction welding
c o
(A) (i) and (iv)
a . (B) (ii) and (iii)
(C) (ii) and (iv)
Correct option is (A). d i (D) (i) and (iii)

o
Sol. 13

In autogeneous welding no filler matorial is used and joint take place in solid state
only
. n
w
Classification of autogeneous welding process

w
w
©

Q. 14 In full mould (cavity-less) casting process, the pattern is made of


(A) Expanded polystyrene (B) Wax
(C) Epoxy (D) Plaster of Paris

Buy Online all GATE Books: shop.nodia.co.in


*Shipping Free* *Maximum Discount*
GATE SOLVED PAPER - ME 2015-3

Sol. 14 Correct option is (A).


Full moulding is used to produce complex shape of object which are made of low
melting point materials [upto 1000°C]
In full moulding, pattern is prepared by plastic material like polystyrene, poly
urothene foam, pvc, epoxy, rubber etc.
Q. 15
values of tensions T1 and T2 are respectively
.i n
A weight of 500 N is supported by two metallic ropes as shown in the figure. The

c o
a .
d i
n o
.
(A) 433 N and 250 N w (B) 250 N and 433 N

w
(C) 353.5 N and 250 N (D) 250 N and 353.5 N
Sol. 15
w
Correct option is (A).
For given configuration of weight hanging by two metallic ropes by using Lom’s
theorem©
.i n
c o
a .
d i
o
By Lom’s theorem T1 = T2 = T2 = const
n
For given configuration .
sin α sin β sin γ

T1w = T2 = 500
wsin 120° sin 150° sin 90°
T1 = 500 sin 120° = 433 N
w T2 = 500 sin 150° = 250 N
Q. 16 ©
For the same material and the mass, which of the following configurations of
flywheel will have maximum mass moment of inertia about the axis of rotation
OOl passing through the centre of gravity.

Buy Online all GATE Books: shop.nodia.co.in


*Shipping Free* *Maximum Discount*
GATE SOLVED PAPER - ME 2015-3

. i n
c o
a .
Sol. 16 Correct option is (B).
For a flywheel
d i
o
TE = Iw cs2

where
TE = . n
Fluctuation in energy
w =
w
Rotation speed
cs
w =
=
Sufficient of fluctuation of speed

w I Mass moment of inertia


I = MR2

©
For some mass and some material, I will be more whose mass is more concentrated
on outer periphery of wheel so from given configurations rimmed flywheel will
have maximum mass moment of inertia.
Q. 17 Couette flow is characterized by
. i n
(A) steady, incompressible, laminar flow through a straight circular pipe

c o
(B) fully developed turbulent flow through a straight circular pipe
.
(C) steady, incompressible, laminar flow between two fixed parallel plates
a
d
moving with a constant velocityi
(D) steady, incompressible, laminar flow between one fixed plate and the other

Sol. 17 Correct option is (D).


n o
Couette flow-
.
In fluid dynamics, couette flow is the laminar flow of a viscous fluid in the space
w
between two parallel plates, one of which is moving relative to the other.

w
The flow is driven by virtue of viscous drag force acting on the fluid and the

w
applied pressure gradient parallel to the plates

The value of lim b


2 sin x + x cos x l
Q. 18
− sin x is _______.
x"0

Sol. 18 Correct answer is - 0.3333

Buy Online all GATE Books: shop.nodia.co.in


*Shipping Free* *Maximum Discount*
GATE SOLVED PAPER - ME 2015-3

The value of lim b − sin x


l=?
x " 0 2 sin x + x cos x

It is 00 form so applying L' hospital rule


= lim a − cos x
k
x " 0 2 cos x + cos x − x sin x

= lim a − cos x
k
by putting x = 0
x " 0 3 cos x − x sin x

.i n
=
− cos (0)
c o
= − 1 =− 0.3333
a .
3 cos (0) − (0) sin (0)

Q. 19
3

d i
A drill is positioned at point P and it has to proceed to point Q. The coordinates

program will be
n o
of point Q in the incremental system of defining position of a point in CNC part

.
w
w
w
©
(A) (3, 12)
.i n
(B) (5, 7)
(C) (7, 12)
o
(D) (4, 7)

c
Sol. 19 Correct option is (D).

a .
Incremental system of defining position of point in CNC programming

d i
n o
.
w
w
w
©
Incremental coordinate of point Q with respect to P will be given by
x = x 2 − x1 = 7 − 3 = 4
y = y2 − y1 = 12 − 5 = 7
Incremental coordinate of Q with respect to point P in CNC programming is (4,7)
Q. 20 A rigid container of volume 0.5 m3 contains 1.0 kg of water at 120°C ( ν f = 0.00106
m3/kg, ν g = 0.8908 m3/kg). The state of water is
(A) compressed liquid
(B) saturated liquid
(C) a mixture of saturated liquid and saturated vapor
(D) superheated vapor

Buy Online all GATE Books: shop.nodia.co.in


*Shipping Free* *Maximum Discount*
GATE SOLVED PAPER - ME 2015-3

Sol. 20 Correct option is (C).


Volume of water in container = 0.5 m3
Specific volume of water in container (ν) = 0.5 ms/kg
Properties of water at that temperature
ν f = 0.00106 m3/kg [saturated liquid]

i n
ν g = 0.8908 m3/kg [saturated vapour]
.
Specific volume of water inside contain is less then specific volume of saturated
o
vapour and more then specific volume of saturated liquid hence water inside
c
.
container is present in the mixture form of saturated vapour and saturated liquid.

a
Q. 21

d i
The thermodynamic cycle shown in figure (T - s diagram) indicates

n o
.
w
w
w
©
(A) reversed Carnot cycle
(C) vapor compression cycle
(B) reversed Brayton cycle
(D) vapor absorption cycle
Sol. 21 Correct option is (B).
(A) Reversed carnot cycle
. i n
c o
a .
d i
n o
.
w
w
w
(B) Reversed brayton cycle

Buy Online all GATE Books: shop.nodia.co.in


*Shipping Free* *Maximum Discount*
GATE SOLVED PAPER - ME 2015-3

(C) and (D) vapour compression cycle and vapour absorption cycle

.i n
c o
a .
d i
Q. 22
n o
The lowest eigenvalue of the 2 # 2 matrix = G is _______.
4 2

Sol. 22 Correct answer is 2. . 1 3

Given matrix is A = > H


4 2
w
1 3
w
The roots of characteristic equation [A − λI] = 0 are known as eigen value of
matrix A w 4−l 2
© [A - λI] = >
1 3 − lH
=0

(4 - l)(3 - l) - 2 = 0
12 − (3l + 4l) + l2 − 2 = 0
l2 − 7l + 10 = 0 .i n
l 1, l 2 =
c o
− (− 7) ! (− 7) 2 − 4 (1) (10)
2 (1)
a .
l 1, l 2 = 7 ! 49 − 40
2
d i
2 2
n o
l 1, l 2 = 7 ! 9 = 7 ! 3 = 5, 2

.
The lowest Eigen value of matrix A is 2.
Q. 23
w
For the given fluctuating fatigue load, the values of stress amplitude and stress
ratio are respectively
w
w
©

(A) 100 MPa and 5


(B) 250 MPa and 5
(C) 100 MPa and 0.20
(D) 250 MPa and 0.20

Buy Online all GATE Books: shop.nodia.co.in


*Shipping Free* *Maximum Discount*
GATE SOLVED PAPER - ME 2015-3

Sol. 23 Correct option is (A)


For given fluctuating load, stress amplitude is
= s max − s min = 250 − 50 = 200 = 100 MPa
2 2 2
and stress ratio is
= s max = 250 = 5
s min 50
. i n
Q. 24

c o
A gear train is made up of five spur gears as shown in the figure. Gear 2 is driver
and gear 6 is driven member. N2, N3, N4, N5 and N6 represent number of teeth on
.
gears 2, 3, 4, 5 and 6 respectively. The gear (s) which act (s) as idler (s) is/are
a
d i
n o
.
w
w
w
(A) Only 3 (B) Only 4

Sol. 24
©
(C) Only 5
Correct option is (C).
(D) Both 3 and 5

In a simple gear train, all odd numbered gears move in one direction and all even

i n
numbered gears in opposite direction. Intermediate gears have no effect on the

.
speed ratio and therefore, they are known as Idlers.
o
For the given configuration of gear train gear 5 is idle gear because it has no effect

c
a .
on speed ratio between gear 4 and 6 except direction manipulation.
The strain hardening exponent n of stainless steel SS 304 with distinct yield and
i
Q. 25

UTS values undergoing plastic deformation is


(A) n < 0
(B) n = 0 o d
(C) 0 < n < 1
. n
(D) n = 1
w
Sol. 25
w
Correct option is (C).

w
Strain hardening exponent (n ): The strain hardening exponent (n ) is a material
constant which is used in calculation for stress and strain behavior in work

©
hardening
σ = kε n
where
σ = applied stress on the material
ε = strain
k = strength coefficient
The value of strain hardening constant lies between 0 and 1
n = 0 means material is perfectly plastic solid
n = 1 means material is perfectly elastic solid
For stainless steel ss 304 the value of strain hardening exponent is 0.44

Buy Online all GATE Books: shop.nodia.co.in


*Shipping Free* *Maximum Discount*
GATE SOLVED PAPER - ME 2015-3

Q. 26 - Q. 55 Carry two marks each.


Q. 26 Orthogonal turning of a mild steel tube with a tool of rake angle 10° is carried
out at a feed of 0.14 mm/rev. If the thickness of the chip produced is 0.28 mm,
the values of shear angle and shear strain will be respectively
(A) 28°20l and 2.19 (B) 22°20l and 3.53
(C) 24°30l and 4.19
i n
(D) 37°20l and 5.19
.
Sol. 26 Correct option is (A).

c o
Given data for a orthogonal turning of mild steel is
Rake angle (α ) = 10°
a .
d i
Feed = 0.14 mm/rev = uncut chip thickness (t)
[as turning is orthogonal]
o
chip thickness (tc ) = 0.28 mm
n
.
We know that shear angle (φ ) is given by

w
φ = tan−1 a r cos a k
1 − r sin a
wr = chip thickness ratio = t = 0.14 = 0.5
w φ = tan−1 =
1 − 0.5 (sin 10)G
0.5 (cos 10)
tc 0.28

= 28.33°
©
Now for shear strain we know that
ε = tan (φ − α) + cot φ

.i n
ε = tan [28.33 − 10] + cot 28.33
ε = 2.1861

c o
So given conditions of orthogonal tuning shear angle and shear strain will be
respectively 28.33° and 2.1861
a .
Q. 27

d i
The number of degrees of freedom of the linkage shown in the figure is

n o
.
w
w
(A) - 3
(B) 0
w
(C) 1
(D) 2
©
Sol. 27 Correct option is (C).

Buy Online all GATE Books: shop.nodia.co.in


*Shipping Free* *Maximum Discount*
GATE SOLVED PAPER - ME 2015-3

F = 3 (l − 1) − 2j − h
where,
F = degree of freedom = ?
l = number of links = 6
j = number of lower pairs = 7
h = number of higher pairs = 0
. i n
F = 3 [6 − 1] − 2 (7) − 0
F = 15 − 14 = 1
c o
.
Given linkage has 1 degree of freedom.
a
Q. 28

d i
A Prandtl tube (Pitot-static tube with C = 1) is used to measure the velocity
of water. The differential manometer reading is 10 mm of liquid column with
o
a relative density of 10. Assuming g = 9.8 m/s2, the velocity of water (in m/s)
n
is_______.
.
Sol. 28
w
Correct answer is 1.3281
For prandtl [pitot static tube]
w
velocity of flow = Cv 2g a em − 1k x when em > e
where, w e

© Cv = coefficient of velocity = 1
g = gravitational acceleration = 9.8 m/sec2
em = density of manometric fluid (em /e = 1)
e =
x =
density of flowing fluid
. i n
manometer reading = 10 mm = 10 # 10−3 m
o
V = (1) 2 # 9.8 b 10 − 1l (10 # 10−3)
c
V =
a . 1
2 # 9.8 # 9 # 10 # 10−3 = 1.3281 m/sec
Q. 29
d i
The dimensions of a cylindrical side riser (height = diameter) for a 25 # 15 # 5

n o
cm steel casting are to be determined. For the tabulated shape factor values given
below, the diameter of the riser (in cm) is_______.
.
Shape factor
w
Riser volume/Casting volume
2
1.0
4
0.70
6
0.55
8
0.50
10
0.40
12
0.35
Sol. 29 w
Correct answer is 10.5 to 10.7
Q. 30 w
Which of the following statements are TRUE, when the cavitation parameter

©
σ = 0?
(i) the local pressure is reduced to vapor pressure
(ii) cavitation starts
(iii) boiling of liquid starts
(iv) cavitation stops
(A) (i), (ii) and (iv)
(B) only (ii) and (iii)
(C) only (i) and (iii)
(D) (i), (ii) and (iii)

Buy Online all GATE Books: shop.nodia.co.in


*Shipping Free* *Maximum Discount*
GATE SOLVED PAPER - ME 2015-3

Sol. 30 Correct option is (D).


Cavitation parameter (Ca ) -
Cavitation parameter is a dimensionless number used to express the relationship
between the difference of a local absolute pressure from the vapour pressure and
the kinetic energy per volume and is used to characterize the potential of flow to
cavitate
It is defined as
.i n
Ca = P1 − P2v
2 eV
c o
where,
a .
P = density of fluid
P = Local pressure
d i
o
Pv = vapour pressure of the fluid

n
.
V = characteristic velocity of the flow
when the cavitation parameter falls to zero
w
(i) P = Pv ; the local pressure is reduced to vapour pressure

w
(ii) Cavitation just prons to start

w
(iii) Boiling of liquid just prons to start

Q. 31
©
One side of a wall is maintained at 400 K and the other at 300 K. The rate of heat
transfer through the wall is 1000 W and the surrounding temperature is 25°C.
Assuming no generation of heat within the wall, the irreversibility (in W) due to

Sol. 31
heat transfer the wall is_______
Correct answer is 248.33
.i n
o
For the given data we have to find irreversibility = ?
c
a .
d i
n o
.
w
w
w
We know that for a irreversible process
dθ < 0
#
©# T
dθ + (Tso) = 0
gen
T
1000 − 1000 + (Tso) = 0
gen
400 300
(Tso) gen = 1000 # 100 = 10
400 # 300 12

Irreversibility = T $ (Tso) gen = [25 + 273] :12 D


10

= 248.33 watt

Buy Online all GATE Books: shop.nodia.co.in


*Shipping Free* *Maximum Discount*
GATE SOLVED PAPER - ME 2015-3

Q. 32 The torque (in N-m) exerted on the crank shaft of a two stroke engine can be
described as T = 10000 + 1000 sin 2q − 1200 cos 2q , where θ is the crank angle as
measured from inner dead center position. Assuming the resisting torque to be
constant, the power (in kW) developed by the engine at 100 rpm is_______
Sol. 32 Correct answer is 104.719

i n
The torque exerted on the crank shaft of a two stroke engine is given by
.
T = 10000 + 1000 sin 2q − 1200 cos 2q (N-m)
o
cycle of sin 2θ and cos 2θ is π so Tmean is 10,000 N-m
c
Now,
Power developed by engine = = T $ W
a .
60
d i
W = 2pN = 2p # 100
60
o
P = 10, 000 # :2π # 100 D # 1 kW
n
.
P = 104.719 kW
60 1000

w
So power developed by given two stroke given is 104.719 kW
Q. 33
w
For the overhanging beam shown in figure, the magnitude of maximum bending

w
moment (in kN-m) is _______

©
. i n
Sol. 33 Correct answer is 40.

c o
For given overhanging beam considering equilibrium we get
ΣM A = 0
a .
d i
(10 # 4) # 2 - 20 # 6 - RB = 0
RB = 40 kN
o
RA + RB = 10 # 4 + 20
n
.
RA = 40 + 20 − 40 = 20 kN

w
Now consider any cross-section XX which is at a distance ‘X ’ from left end A
and as shown in figure
w
w
©
In the region o < x < 4M
Shear force (VX ) = R 4 − 10X = 20 − 10x
Bending moment (MX ) = RA $ x − 10x a 2 k
x

= 20x − 5x2
at x = 0 , VX = 20 , MX = 0
at x = 4 , VX =− 20 , MX = 0

Buy Online all GATE Books: shop.nodia.co.in


*Shipping Free* *Maximum Discount*
GATE SOLVED PAPER - ME 2015-3

In the rigion 4 m < x < 6M


shear force (VX ) = RA − 10 # 4 = 20 − 40 =− 20
bending moment (MX ) = RA $ x − (10 # 4) (x − 2)
= 20x − 40x + 80 = 80 − 20x
at x = 4 , VX =− 20 , MX = 0
at x = 6 , VX =− 20 , MX =− 40 *
.i n
So, maximum bending moment is (- 40 ) acting at x = 6 from left end (A).
Negative sign show direction of moment.
c o
Q. 34

a .
Ratio of solidification time of a cylindrical casting (height = radius) to that of

Sol. 34 Correct answer is 0.5625


d i
a cubic casting of side two times the height of cylindrical casting is _______

n o
Solidification time of casting is given by

ts = K b SA l
. V 2
where,
w
V = volume of casting
w
SA = surface area of casting
w
K = solidification factor (sec/m2)

©
here volume represent amount of heat content and surface area represent amount
of heat transfer
Now for cylindrical casting (height = radius)
Vol =
SA
p 2
4d H
i n p 2
4 (2H) H
2 6 p4 d @ + (pd # H) 2 6 p4 (2H) @ + p (2H) H
2 =
. 2

= pH 2 = H
3

c o
4pH 4
For cubical casting (side = 2 # height)
a .
Vol = a3 = a = 2H = H
6a 2 6 6 d i 3
SA

n o
solidification time of cylindrical casting (t )
= s cylinder
.
solidification time of cubical casting (ts) cubical
K ^ SA hcyl. ^ H4 h2
w (ts) cyl.
=
V 2

K ^ SA
= H 2
hcub ^ 3 h
w (ts) cub. V 2

w = 9 = 0.5625
16

©
So, the ratio of solidification time of cylindrical casting to cubical casting will be
0.5625
Q. 35 In a rolling operation using rolls of diameter 500 mm, if a 25 mm thick plate
cannot be reduced to less than 20 mm in one pass, the coefficient of friction
between the roll and the plate is _______
Sol. 35 Correct answer is 0.1414
For a rolling operation
Rolls diameter = 500 mm
Plate thickness = 25 mm
Rolled plate thickness = 20 mm [min. possible]

Buy Online all GATE Books: shop.nodia.co.in


*Shipping Free* *Maximum Discount*
GATE SOLVED PAPER - ME 2015-3

Maximum draft possible


= plate thickness - min. possible rolled palte thickness
TH @max = maximum draft = 25 − 20 = 5 mm
we know that maximum draft in rolling is given by = µ2 R
where
µ = Friction coefficient
. i n
R = Roll’s diameter
TH @max = µ2 R
c o
5 = µ2 : 2 D
500
a .
µ2 = 0.02
d i
o
µ = 0.1414

n
Q. 36
.
For ball bearings, the fatigue life L measured in number of revolutions and the

w
1
radial load F are related by FL3 = K , where K is a constant. It withstands a
radial load of 2 kN for a life of 540 million revolutions. The load (in kN) for a life
w
of one million revolutions is _______
Sol. 36
w
Correct answer is 16.2865
For a ball bearing it is given that
©
where,
F L1/3 = K

F = radial load
. i n
L = fatigue life measured in number of revolutions

K = constant
c o
a .
For a radial load of 2 kN for a life of 540 million revolutions
K = [2 # 1000] [540 # 106] 1/3

d i
For a line of one million revolution radial load is = ?
K = (2 # 1000) (540 # 106) 1/3
o
= (F) (1 # 106) 1/3
n
.
2000 # (540) 1/3 = F

wF = 16, 286.50 N = 16.2865 kN


Q. 37
w
For the linear programming problem:
Maximize
w
Subject to
Z = 3X1 + 2X2

© − 2X1 + 3X2 # 9
X1 - 5X2 $- 20
X1, X2 $ 0
The above problem has
(A) unbounded solution (B) infeasible solution
(C) alternative optimum solution (D) degenerate solution
Sol. 37 Correct option is (A).
For the linear programming problem, given constraints are
Constraint 1:
− 2X1 + 3X2 # 9

Buy Online all GATE Books: shop.nodia.co.in


*Shipping Free* *Maximum Discount*
GATE SOLVED PAPER - ME 2015-3

or X + X2 # 1
− 92 3
Constraint 2:
X1 - 5X2 $- 20
or 5X2 - X1 $ 20
or X1 - 5X2 # 20
X1 + X 2 # 1
.i n
or
20 − 4

c o
Constraint 3:
X1, X2 $ 0
a .
d i
n o
.
w
w
w
©
.i n
In this condition highest value of objective function goes upto infinite and it

c o
is simply means that common feasible reason is not bounded by limit on the
constraints. It is known as unbounded solution.
Q. 38

a .
The annual requirement of rivets at a ship manufacturing company is 2000 kg.

d i
The rivets are supplied in units of 1 kg costing Rs. 25 each. If it costs Rs. 100 to
place an order and the annual cost of carrying one unit is 9% of its purchase cost,

o
the cycle length of the order (in days) will be_______

n
Sol. 38 Correct answer is 76.94
.
Given data for a ship manufacturing company is
w
Annual requirement of rivets (D ) = 2000 kg
w
Cost of one unit (1 kg) = Rs.25 each

wOrdering cost (Co ) = Rs.100/order


Carrying cost (Ch ) = 9% of its purchase cost/unit/year
©
Cycle length of order in days = ?
Now, we know that economic ordering quantity (Q *) is given by
Q * = 2Dco
Ch
= 2 # 2000 # 100 = 421.637 units
0.09 # 25
Number of orders = 2000 = 4.7434 orders/year
421.637
= 4.7434 order/365 days
= 76.94 days/order
So, time between consecutive order will be 76.94 day i.e. cycle length of order.

Buy Online all GATE Books: shop.nodia.co.in


*Shipping Free* *Maximum Discount*
GATE SOLVED PAPER - ME 2015-3

Q. 39 Steam enters a turbine at 30 bar, 300°C (u = 2750 kJ/kg, h = 2993 kJ/kg) and
exits the turbine as saturated liquid at 15 kPa (u = 225 kJ/kg, h = 226 kJ/kg).
Heat loss to the surrounding is 50 kJ/kg of steam flowing through the turbine.
Neglecting changes in kinetic energy and potential energy, the work output of the
turbine (in kJ/kg of steam) is _______
Sol. 39 Correct answer is 2717.
. i n
From given data schematic diagram is drawn below

c o
a .
d i
n o
.
w
w
w
©
From energy conservation principle
Heat input - Heat output = Work output + Heat loss

. i n
[Neglecting changes in kinetic energy and potential energy]
2993 - 226 = WT + 50

c o
WT = 2717 kJ/kg of steam
.
So work output of turbine will be 2717 kJ/kg steam.
a
Q. 40

d i
Figure shows a single degree of freedom system. The system consists of a massless
rigid bar OP hinged at O and a mass m at end P. The natural frequency of
o
vibration of the system is
n
.
w
w
w
©
(A) fn = 1

k
4m
(B) fn = 1 k
2p 2p
(C) fn = 1 k
2π m
(D) fn = 1 2k
2π m
Sol. 40 Correct option is (A).
For a single degree of freedom system shown in figure by deviating small tilit by
angle θ

Buy Online all GATE Books: shop.nodia.co.in


*Shipping Free* *Maximum Discount*
GATE SOLVED PAPER - ME 2015-3

.i n
c o
a .
Writing torque equation for given system
d i
Iqo + (0) qo + K a 2 k a = 0
x
where
n o
. I = moment of inertia = M (2a) 2 = 4Ma2
w
(2a) θ = x

w
(4Ma2) qp + K b 2 l a $ q = 0
2a

w 2
qp + Ka 2 q = 0
© 4Ma
qp + b 4M l q = 0
K

Comparing from standard equation qp + W2 q = 0


W = K
.i n
4M

c o
frequency of vibration = W = 1
2p 2p 4M
a . K

Q. 41

d i
A solid sphere 1 of radius r is placed inside a hollow, closed hemispherical surface
2 of radius ‘ 4r ’. The shape factor F2-1 is

n o
.
w
w
(A) 1
w (B) 1
12
(C) 2
© 2
(D)12
Sol. 41 Correct option is (A).
Shape factor: The shape factor may be defined as the fraction of radiative energy
that is diffused from one surface element and strikes the other surface directly
with no intervening reflections
F1 - 2 = Fraction of energy leaving surface 1 and reaching surface 2
Reciprocity theorem of shape factor
A1 F12 = A2 F2 − 1

Buy Online all GATE Books: shop.nodia.co.in


*Shipping Free* *Maximum Discount*
GATE SOLVED PAPER - ME 2015-3

Now, the given configuration is

. i n
c o
a .
so F1 - 2 will be 1
d i
We know that energy leaving from surface (1) will reach at surface (2) completely

n o
From reciprocity theorem,
F21 = A1 (F1 − 2)
. A2

wF21 = 4pR2 (1) = 1


3p (4R) 2 12
w
Shape factor F2 - 1 is 1
w 12
Air in a room is at 35°C and 60% relative humidity (RH ). The pressure in the
©
Q. 42
room is 0.1 MPa. The saturation pressure of water at 35°C is 5.63 kPa. The
humidity ratio of the air (in gram/kg of dry air) is _______
Sol. 42 Correct answer is 21.74

. i n
It is given that relative humidity of air in room of 35°C temperature is 60%
Relative humidity = Pv = 0.6
Pv
c o
where,
.
s

Pv = vapour pressure in room


i a
Pv = saturation pressure of water at room temp.
d
s

Saturation pressure of water at 35°C is 5.63 kPa.


o
So, vapour pressure of water in room is
n
.Pv = 0.6 # Pv s

w
So, humidity ratio (W)
= 0.6 # 5.63 = 3.378 kPa

w = 0.622 Pv = 0.622 Pv
w
total pressure (P ) of room is
Pa P − Pv

© = 0.1 MPa
= 100 kPa
humidity ratio (W ) = 0.622 b 100 − 3.378 l
3.378

= 0.2174 kg/kg of dry air


= 21.74 gram/kg of dry air
So, humidity ratio of air inside room is 21.74 gram per kg of dry air.

Buy Online all GATE Books: shop.nodia.co.in


*Shipping Free* *Maximum Discount*
GATE SOLVED PAPER - ME 2015-3

3
Q. 43 Laplace transform of the function f (t) is given by F (s) = L {f (t)} = # f (t) e
0
−st
dt.
Laplace transform of the function shown below is given by

.i n
c o
(A) 1 - e
s
-2s
.
(B) 1 - e
a 2s
-s

(C) 2 - 2e
s
-s
i
(D) 1 - 2e
d s
-s

Sol. 43 Correct option is (C).


n o
. #
Laplace transformation of the function f (t) is given by

w
3
F (s) = L {f (t)} = f (t) e−st dt
0

w
F (s) = #
3
f (t) e−st dt = #
1
f (t) e−st dt +
3
# f (t) e −st
dt

w
0 0 1

it is given that *
f (t) = 2 0#x#1

So,
© f (t) = 0 x>1

n
1 3
# (2) e−st dt + # 0 (e −st
F (s) =
0 1

= 2 e−st dt = 2 b e l = 2 (1 − e−s)
.i ) dt

o
1 −st 1
# −s 0 s
0

F (s) = 2 − 2e
−s
.c
Q. 44
s
ia
Refrigerant vapor enters into the compressor of a standard vapor compression

o d
cycle at - 10° C (h = 402 kJ/kg) and leaves the compressor at 50°C (h = 432
kJ/kg). It leaves the condenser at 30°C (h = 237 kJ/kg). The COP of the cycle
is_______
. n
Sol. 44 Correct answer is 5.5
w
For given standard vapour compression refrigeration cycle schematic diagram is
w
drawn below as per given data.

w
©

Buy Online all GATE Books: shop.nodia.co.in


*Shipping Free* *Maximum Discount*
GATE SOLVED PAPER - ME 2015-3

For process (3-4),


constant enthalpy process so h 3 = h 4
Refrigeration effect (RE ) = h1 − h 4 = 402 − 237 = 165 kJ/kg
work done of compressor = h2 − h1 = 432 − 402 = 30 kJ/kg
Refrigeration effect 165
coefficient of performance =

coefficient of performance = 5.5


work done
. i n =
30

Q. 45
c o
In a CNC milling operation, the tool has to machine the circular are form point
.
(20, 20) to (10, 10) at sequence number 5 of the CNC part program. If the
a
d i
center of the arc is at (20, 10) and the machine has incremental mode of defining
position coordinates, the correct tool path command is

n o
(A) N05 G90 G01 X-10 Y-10 R10
(B) N05 G91 G03 X-10 Y-10 R10
.
(C) N05 G90 G03 X20 Y20 R10
w
(D) N05 G91 G02 X20 Y20 R10
Sol. 45
w
Correct option is (B).

w
In a CNC milling operation we have to travel in a circular are as shown in figure

©
. i n
c o
a .
d i
n o
Incremental co-ordinates arc [(10 - 20 ), (10 - 20 )]
. = (− 10, − 10)
w
center of arc = (20, 10)

w
radius of arc =− 10

w
so the correct tool path command will be
N 05 G91 G03 X-10 Y-10 R10

©
where,
N05 = Represents sequence number 5 of CNC part program
G91 = Represents incremental co-ordinate system
G03 = Represents circular interpolation in CCW direction
X-10 Y-10 = Represents incremental co-ordinates of path
R10 = Represents radius of circular arc

Buy Online all GATE Books: shop.nodia.co.in


*Shipping Free* *Maximum Discount*
GATE SOLVED PAPER - ME 2015-3

Q. 46 The value of moment of inertia of the section shown in the figure about the axis-
XX is

.i n
c o
a .
d i
(A) 8.5050 # 106 mm4
n o (B) 6.8850 # 106 mm4
(C) 7.7625 # 106 mm4 . (D) 8.5725 # 106 mm4
Correct option is (B). w
w
Sol. 46

Moment of inertia of the section shown in figure about axis XX is = ?

w
©
.i n
c o
a .
d i
n o
This section can be rearranged as shown in figure below.
.
w
w
w
©

Moment of inertia of section (1) = moment of inertia of section (2)


- moment of inertia of section (3)
(about XX axis)

Buy Online all GATE Books: shop.nodia.co.in


*Shipping Free* *Maximum Discount*
GATE SOLVED PAPER - ME 2015-3

Moment of inertia of section (2) about XX axis is


2 (60) (120) 3
= bh = = 8.64 # 106 mm4
12 12
where
b = bridth
h = height

. i n
Moment of inertia of section (3) about XX axis is

= 2 # : 12 + AR D
bh2 2

c o [by parallel axis theorm]


where
A = area of section
a .
d i
R = distance from XX axis to section axis

n o
= 2 #: #
30 303 + (30 30)(30) 2D
12 #

. = 2 # 30 4 :12 + 1D = 1.75 # 106 mm4


1

w
w
Moment of inertia of section (1) about XX axis is
= 8.64 # 106 − 1.75 # 106
w = 6.885 # 106 mm4
Q. 47
©
A mixture of ideal gases has the following composition by mass:
N2 O2 CO 2
60% 30% 10%

. i n
If the universal gas constant is 8317 J/kmoI-K, the characteristic gas constant of
the mixture (in J/kg-K) is_______
Correct answer is 274 to 276.
c o
.
Sol. 47

Q. 48

i a
A cantilever bracket is bolted to a column using three M12 # 1.75 bolts P, Q
and R. The value of maximum shear stress developed in the bolt P (in MPa)
is_______
o d
. n
w
w
w
©

Buy Online all GATE Books: shop.nodia.co.in


*Shipping Free* *Maximum Discount*
GATE SOLVED PAPER - ME 2015-3

Sol. 48 Correct answer is 332.5454


Individual forces due to eccentric loading on cantilever beam are shown is figure

.i n
c o
a .
d i
n o
.
P 2' = direct shear stress on P bolt due to external load

w
P 2" = secondary shear stress on P bolt due to external load
Direct shear stress is
w = P 1' = P 2' = P 3' = P = 9 kN = 3 kN
w
Secondary shear stress by torque balance is
3 3

but
©P 2" # 30 + P 3" + 30 = 9 # 10 # 250
P 2" = P 3" (by symmetry)

Forces on bolt P
3

30 # 2
.i n
P 211 = 9 # 10 # 250 = 37.5 kN

c o
a .
d i
Ptotal =
n
P 2' 2 + P 2"2 = o
32 + 37.52 = 37.61 kN
.
Maximum shear stress = max imum force
w area

Area of bolt = pd = p # 12 # 12 = 113.097 mm2


2

w 4 4

w 37.61 # 103 = 332.5454 MPa


τ max =
113.097 # 10−6
©
So, maximum shear stress on bolt P having eccentric load is 332.5454 MPa
4 + 3i − i
Q. 49 For a given matrix P = = G , where i = − 1 , the inverse of matrix P
is i 4 − 3 i
4 − 3i i i 4 − 3i
(A) 1 = G (B) 1 =
24 − i 4 + 3i 25 4 + 3i − i G

4 + 3i −i 4 + 3i −i
(C) 1 = G (D) 1 = G
24 i 4 − 3i 25 i 4 − 3i

Buy Online all GATE Books: shop.nodia.co.in


*Shipping Free* *Maximum Discount*
GATE SOLVED PAPER - ME 2015-3

Sol. 49 Correct option is (A).

For a given matrix P = > H


4 + 3i − i
i 4 − 3i
where,
i = − 1 , the inverse of matrix P = ?
we know that P-1 =
Adj [P]
[P]
. i n
o
Adj [P] = transpose of cofactor matrix of P
c
co-factor matrix of P = > .
+ (4 − 3i)

a
− (i)
− (− i) + (4 + 3i)
H

d i
transpose of cofactor matrix P = > H
+ 4 − 3i +i

n o −i + 4 + 3i

.
[P] = >
4 + 3i − i
H = (4 + 3i) (4 − 3i) − (− i)(i)
w i 4 − 3i

w = 16 + 9 − 1 = 24

= 1> H
w P-1 =
AdjP
[P]
4 − 3i i
24 − i 4 + 3i
Q. 50
©
A brick wall ^k = 0.9 mW.K h of thickness 0.18 m separates the warm air in a room from
the cold ambient air. On a particular winter day, the outside air temperature is
- 5° C and the room needs to be maintained at 27°C. The heat transfer coefficient

air inside the room, the heat loss, in _ W


2

m i
i
, is
n
associated with outside air is 20 mWK . Neglecting the convective resistance of the
.
o
2

(A) 88 (B) 110


(C) 128
. c (D) 160
Sol. 50 Correct option is (C).
i a
d
For given data schematic diagram shown below.

o
. n
w
w
w
©
Equivalent thermal resistance of wall
1 = 1 + x + 1
UA h1 A kA h 0 A
1 = 0.18 + 1 = 0.25
UA 0.9A 20A A
U = 1 = 4 watt/m2-k
0.25
Heat transferred = uTT = 4 # 32 = 128 w/m2
So, heat loss per unit area is 128 w/m2

Buy Online all GATE Books: shop.nodia.co.in


*Shipping Free* *Maximum Discount*
GATE SOLVED PAPER - ME 2015-3

Q. 51 Figure shows a wheel rotating about O2 . Two points A and B located along the
radius of wheel have speeds of 80 m/s and 140 m/s respectively. The distance
between the points A and B is 300 mm. The diameter of the wheel (in mm)
is_______

.i n
c o
a .
d i
n o
Sol. 51

.
Correct answer is 1390 to 1410.
Q. 52
w
A shaft of length 90 mm has a tapered portion of length 55 mm. The diameter of
the taper is 80 mm at one end and 65 mm at the other. If the taper is made by
w
tailstock set over method, the taper angle and the set over respectively are

w
(A) 15°32l and 12.16 mm
(B) 18°32l and 15.66 mm
©
(C) 11°22l and 10.26 mm
(D) 10°32l and 14.46 mm
Sol. 52 Correct option is (A).
.i n
Given taper schematically shown in figure is mode by tailstock set over method,
topper angle and set over = ?
c o
a .
d i
n o
.
w
tan α = D − d = 80 − 65
w 2l 2 (55)

w α = half toper angle = 7.765°


By offsetting the tailstock the axis of rotation of the job is inclined by the half

©
angle of taper so the taper angle is 15.53°
No for tailstock offset (h ) is given by
L (D − d)
h = = L tan α
2l
= 90 # tan 7.765
h = 12.2727 mm
Diameter of wheel is = 2RB = 2 [x + 300]
= 2 [400 + 300] mm
= 1400 mm
Diameter of given wheel is 1400 mm

Buy Online all GATE Books: shop.nodia.co.in


*Shipping Free* *Maximum Discount*
GATE SOLVED PAPER - ME 2015-3

Q. 53 The value of # [(3x − 8y ) dx + (4y − 6xy) dy], (where C


C
2
is the boundary of the

region bounded by x = 0 , y = 0 and x + y = 1) is _______


Sol. 53 Correct answer is 1.60 to 1.70
Q. 54

i n
Newton-Raphson method is used to find the roots of the equation,
x3 + 2x2 + 3x − 1 = 0 . If the initial guess is x 0 = 1, then the value of x after 2nd
.
Sol. 54
iteration is _______
Correct answer is 0.29 to 0.31
c o
Q. 55
a .
A bullet spins as the shot is fired from a gun. For this purpose, two helical slots

engage in each of the slots.


d i
as shown in the figure are cut in the barrel. Projections A and B on the bullet

n o
.
w
w
Helical slots are such that one turn of helix is completed over a distance of 0.5 m.
If velocity of bullet when it exits the barrel is 20 m/s, its spinning speed in rad/s
w
is _______
Sol. 55
©
Correct answer is 251.3274
From given statement of question we can conclude that bullet will rotate by one
rotation when it travels 0.5 m in the barrel.

i n
It is also given that exit velocity of bullet is 20 m/sec, so time take to travel 0.5

.
m in gun barrel will be time to make rotate bullet by one rotation
So spinning speed of bullet is

c o
= 2π
T
a .
20
d i
= 20.p5 = 2p # 20
0.5

n o
= 80π = 251.3274 rad/sec
Spinning speed of bullet in gun barrel will be 251.3274 radian/sec.
.
w
w END OF THE QUESTION PAPER

w
©

Buy Online all GATE Books: shop.nodia.co.in


*Shipping Free* *Maximum Discount*
GATE SOLVED PAPER - ME 2015-3

ANSWER KEY
General Aptitude
1 2 3 4 5 6 7 8 9 10
(C) (B) (B) (A) (B) (280) (C) (C) (B) (C)

.i n
1 2 3 4
Mechanical Engineering
5 6
c o7 8 9 10
(0.6931) (A) (A) (C) (C) (B)
a . (B) (C) (C) (D)
11
(A)
12
(25)
13
(A)
14
(A)
15
(A) d i16
(B)
17
(D)
18
(–0.3333)
19
(D)
20
(C)
21 22 23 24
n o
25 26 27 28 29 30
(B) (2) (C) (C)
. (C) (A) (C) (1.3281) (10.5 (D)

w to
10.7)
31 32 33 w
34 35 36 37 38 39 40

w
(248.33) (104.719) (40) (0.5625) (0.1414) (16.2865) (A) (76.94) (2717) (A)
41
(A)
42
(21.74) ©43
(C)
44
(5.5)
45
(B)
46
(B)
47
(274
48
(332.54)
49
(A)
50
(C)
to 276
51 52 53 54 55
.i n
(1390 to
1410)
(A) (1.60 (0.29 to (251.32)
to 0.31)
c o
1.70)

a .
d i
n o
.
w
w
w
©

Buy Online all GATE Books: shop.nodia.co.in


*Shipping Free* *Maximum Discount*
Downloaded From : www.EasyEngineering.net

**Note: Other Websites/ Blogs Owners Please do not Copy (or) Republish
this Materials without Legal Permission of the Publishers.

**Disclimers : EasyEngineering not the original publisher of this Book/Material


on net. This e-book/Material has been collected from other sources of net.
Downloaded From : www.EasyEngineering.net
Downloaded From : www.EasyEngineering.net

ME-GATE 2018

Section-I: General Ability

1. Seven machines take 7 minutes to make 7 identical toys. At the same rate, how many minutes
would it take for 100 machines to make 100 toys?
(A) 1 (B) 7 (C) 100 (D) 700
Key: (B)
7  7 100  T2
Sol: 
7 100
 T2  7 minutes

2. "Her ______ should not be confused with miserliness; she is ever willing to assist those in need."
The word that best fills the blank in the above sentence is:
(A) cleanliness (B) punctuality (C) frugality (D) greatness
Key: (C)
Sol: The sentence explains that the person spoken of is not miserly, since she is quite prepared to be
generous. So for the sentence to make sense, the word filling the blank has to be something that is
consistent with generosity and yet might, by those without a full understanding of her behaviour,
be mistaken for miserliness. The words “frugality” and thrift fulfill this requirement and yield two
sentences that are alike in meaning. Thus the correct answer is frugality.

3. A number consists of two digits. The sum of the digits is 9. If 45 is subtracted from the number, its
digits are interchanged. What is the number?
(A) 63 (B) 72 (C) 81 (D) 90
Key: (B)
Sol: Let number be „xy‟
Given x  y  9 ... i 
& xy  45  yx

 10x  y   45  10y  x
 9x  9y  45
xy5 ... ii 

solving  i  &  ii   x  7, y  2
Required number  72

© All rights reserved by Thinkcell Learning Solutions Pvt. Ltd. No part of this booklet may be reproduced or utilized in any form without the written permission.

Downloaded From : www.EasyEngineering.net


Downloaded From : www.EasyEngineering.net

ME-GATE 2018

4. Going by the ______ that many hands make light work, the school _______ involved all the
students in the task."
(A) principle, principal (B) principal, principle
(C) principle, principle (D) principal, principal
Key: (A)

5. A rectangle becomes a square when its length and breadth are reduced by 10 m and 5 m,
respectively. During this process, the rectangle loses 650 m2 of area. What is the area of the original
rectangle in square meters?
(A) 1125 (B) 2250 (C) 2924 (D) 4500
Key: (B)
Sol:

x 5 x

x  10 x

Area of rectangle = (x+10)(x+5) = x2+15x+50 …(1)


Area of square = x2 … (2)
By the given condition,
15x  50  650
 15x  600  x  40.
 Area of rectangle   40   15  40  50  2250( from(1))
2

6. Given that a and b are integers and a+ a2b3 is odd, which one of the following statements is correct?
(A) a and b are both odd (B) a and bare both even
(C) a is even and b is odd (D) a is odd and b is even
Key: (D)
Sol: By verifying options
Let us consider a=1, b=2
a  a 2 b3  1  12  23  9 i.e.,odd
Only this condition is valid.

© All rights reserved by Thinkcell Learning Solutions Pvt. Ltd. No part of this booklet may be reproduced or utilized in any form without the written permission.

Downloaded From : www.EasyEngineering.net


Downloaded From : www.EasyEngineering.net

ME-GATE 2018

7. Consider the following three statements:


(i) Some roses are red
(ii) All red flowers fade quickly.
(iii) Some roses fade quickly
Which of the following statements can be logically inferred from the above statements?
(A) If (i) is true and (ii) is false, then (iii) is false.
(B) If (i) is true and (ii) is false, then (iii) is true.
(C) If (i) and (ii) are true, then (iii) is true.
(D) If (i) and (ii) are false, then (iii) is false.
Key: (C)
Sol:
Fadedquickly
Roses
Red

8. Which of the following functions describe the graph shown in the below figure.

y
3
2
1
-3 -2 -1 0 1 2 3x
-1
-2
-3

(A) y  x  1  2 (B) y  x  1  1 (C) y  x  1  1 (D) y  x  1  1


Key: (B)
Sol: Given graph passing through X-axis at x= –2, 2
From option (B), y=0 at x= –2, 2
Alternate Method:
Step 1
y x
0

© All rights reserved by Thinkcell Learning Solutions Pvt. Ltd. No part of this booklet may be reproduced or utilized in any form without the written permission.

Downloaded From : www.EasyEngineering.net


Downloaded From : www.EasyEngineering.net

ME-GATE 2018

Step  2 
y 1  x
 or  1
y  x 1

Step  3 1
y  x 1

Step  4 
y 1  x 1
0
 or 
y  x 1 1

9. From the time the front of a train enters a platform, it takes 25 seconds for the back of the train to
leave the platform, while travelling at a constant speed of 54 km/h. At the same speed, it takes 14
seconds to pass a man running at 9 km/h in the same directions as the train. What is the length of
the train and that of the platform in meters, respectively?
(A) 210 and 140 (B) 162.5 and 187.5
(C) 245 and 130 (D) 175 and 200
Key: (D)
Sol: Let Train length  x
Platform length  y

x  y 25   54 
5
  375
 18 
  
Distance  Time  Speed

x
also,  14
5
 54  9 
18

Relative speed
 x  175 and y  200

10. For integers a, b and c, what would be the minimum and maximum values respectively of a + b + c
if log a  lob b  log c  0?

(A) –3 and 3 (B) –1 and 1 (C) –1 and 3 (D) 1 and 3

© All rights reserved by Thinkcell Learning Solutions Pvt. Ltd. No part of this booklet may be reproduced or utilized in any form without the written permission.

Downloaded From : www.EasyEngineering.net


Downloaded From : www.EasyEngineering.net

ME-GATE 2018

Key: (A)
Sol: Given log a  log b  log c  0
 log e a b c  0
 a b c  1  a  1, b  1,c  1 ( a,b,c are integers)
Minimum value  1  1  1  3
Maximum value  1  1  1  3

Section-I: Mechanical Engineering

1. An ideal gas undergoes a process from state 1 (T 1 = 300 K, p1 = 100 kPa) to state 2 (T2 = 600K,
p2 = 500 kPa). The specific heats of the ideal gas are: cp = 1 kJ/kg-K and cv = 0.7 kJ/kgK. The
change in specific entropy of the ideal gas from state 1 to state 2 (in kJ/kg-K) is _____ (correct to
two decimal places).
Key: (0.21)
Sol: Given that,
T1  300K, P1  100kPa
T2  600K , P2  500kPa
Cp  1kJ/kg-k
Cv  0.7kJ/kg-k,
R  Cp  Cv  1  0.7  0.3kJ/kg-k
Changein Entropy  s 2  s1  per unit mass
T  P 
 Cp n  2   R n  2 
 T1   P1 
 600   500 
1 n   0.3 n  
 300   100 
s 2  s1  0.21kJ/kg-K

2. In a linearly hardening plastic material, the true stress beyond initial yielding
(A) increases linearly with the true strain
(B) decreases linearly with the true strain
(C) first increases linearly and then decreases linearly with the true strain
(D) remain constant
Key: (A)

© All rights reserved by Thinkcell Learning Solutions Pvt. Ltd. No part of this booklet may be reproduced or utilized in any form without the written permission.

Downloaded From : www.EasyEngineering.net


Downloaded From : www.EasyEngineering.net

ME-GATE 2018

3. Using the Taylor's tool life equation with exponent n = 0.5, if the cutting speed is reduced by 50%,
the ratio of new tool life to original tool life is
(A) 4 (B) 2 (C) 1 (D) 0.5
Key: (A)
Sol: VT n  C
V1T1n  V2 T2n
1
T2  V1  n
 
T1  V2 
1
T2  V1  0.5
   V2  0.5V1 
T1  0.5V1 
T2  4T1

4. The height (in mm) for a 125 mm sine bar to measure a taper of 27o32' on a flat work piece is ____
(correct to three decimal places).
Key: (57.77)
Sol:   2732'  27.53
h
sin  

h  sin    25sin 27.53  57.77mm

5. A six-faced fair dice is rolled five times. The probability (in %) of obtaining "ONE" at least four
times is
(A) 33.3 (B) 3.33 (C) 0.33 (D) 0.0033
Key: (C)
1
Sol: n  5, p  obtaining 'ONE ' 
6
1 5
 q 1 
6 6
p  x  4  ?
By Binomial distribution,
p  x  4   p  x  4   p  x  5
4 5 4 5 5 5
1 5 1 5
 5 C 4      5 C5    
6 6 6 6
26 26
 5  5 100  in%  0.33
6 6

© All rights reserved by Thinkcell Learning Solutions Pvt. Ltd. No part of this booklet may be reproduced or utilized in any form without the written permission.

Downloaded From : www.EasyEngineering.net


Downloaded From : www.EasyEngineering.net

ME-GATE 2018

6. The time series forecasting method that gives equal weightage to each of the m most recent
observations is
(A) Moving average method (B) Exponential smoothing with linear trend
(C) Triple Exponential smoothing (D) Kalman Filter
Key (A)
Sol ∵ in Moving Average Method
Dn  Dn 1  Dn  2  ...
Fn 1 
n
1
So, weightage of all past data is
n
i.e., equal weightage provided to all past data.

 4 1 1
7. The rank of the matrix  1 1 1 is
 7 3 1 

(A) 1 (B) 2 (C) 3 (D) 4


Key: (B)

 4 1 1
Sol: Let A   1 1 1
 7 3 1 
A  4  1  3  1 1 7    3  7   0
4 1
consider a 2  2 sub matrix,  4  1 5  0
1 1
 Rank of A  2

8. Interpolator in a CNC machine


(A) controls spindle speed (B) coordinates axes movements
(C) operates tool changer (D) commands canned cycle
Key: (B)

9. Four red balls, four green balls and four blue balls are put in a box. Three balls are pulled out of the box
at random one after another without replacement. The probability that all the three balls are red is
(A) 1/72 (B) 1/55 (C) 1/36 (D) 1/27

© All rights reserved by Thinkcell Learning Solutions Pvt. Ltd. No part of this booklet may be reproduced or utilized in any form without the written permission.

Downloaded From : www.EasyEngineering.net


Downloaded From : www.EasyEngineering.net

ME-GATE 2018

Key: (B)
Sol: 4-Red; 4-Green; 4-Blue
4 3 2 1
Required probability    
12 11 10 55

10. A grinding ratio of 200 implies that the


(A) grinding wheel wears 200 times the volume of the material removed.
(B) grinding wheel wears 0.005 times the volume of the material removed
(C) aspect ratio of abrasive particles used in the grinding wheel is 200
(D) ratio of volume of abrasive particle to that of grinding wheel is 200
Key: (B)
MRR
Sol: Grinding ratio 
wheel wear
1
200 
wheel wear
1
wheel wear   0.005
200

11. Which one of the following statements is correct for a superheated vapour?
(A) Its pressure is less than the saturation pressure at a given temperature.
(B) Its temperature is less than the saturation temperature at a given pressure.
(C) Its volume is less than the volume of the saturated vapour at a given temperature.
(D) Its enthalpy is less than enthalpy of the saturated vapour at a given pressure.
Key: (A)
Sol: At a given temperature the pressure of superheated vapour is less than the saturation pressure.
For example let us take100oC, the saturation pressure corresponding to this temperature is
1.01325bar. If the saturation pressure decreases saturation temperature decreases and vice-versa.
From this we can easily say that if the superheated temperature has to be 100oC then saturation
temperature should be less than 100oC, therefore corresponding saturation pressure will be less.
Hence we can say that for same temperature the pressure of superheated vapour is less than the
saturation pressure.

12. If 1 and 3 are the algebraically largest and smallest principal stresses respectively, the value of
the maximum shear stress is

1  3 1  3  1  3 1  3
(A) (B) (C) (D)
2 2 2 2

© All rights reserved by Thinkcell Learning Solutions Pvt. Ltd. No part of this booklet may be reproduced or utilized in any form without the written permission.

Downloaded From : www.EasyEngineering.net


Downloaded From : www.EasyEngineering.net

ME-GATE 2018

Key: (B)
Sol: Diameter of Mohr circle = 1  3 and

1  3
 max  Radius 
2

13. The number of atoms per unit cell and the number of slip systems, respectively, for a face-centered
cubic (FCC) crystal are
(A) 3, 3 (B) 3, 12 (C) 4, 12 (D) 4, 48
Key: (C)

14. A four bar mechanism is made up of links of length 100, 200, 300 and 350 mm. If the 350 mm link
is fixed, the number of links that can rotate fully is ______.
Key: (1) 200
Sol: s  p  q
100  350  200  300 100 300
Hence it is crank rocker
(100) link will rotate -1.
350
So, link whose length is 100 mm will rotate fully. So, only one link will rotate fully.

15. For an Oldham coupling used between two shafts, which among the following statements are
correct?
I. Torsional load is transferred along shaft axis.
II. A velocity ratio of 1:2 between shafts is obtained without using gears.
III. Bending load is transferred transverse to shaft axis.
IV. Rotation is transferred along shaft axis.
(A) I and III (B) I and IV (C) II and III (D) II and IV
Key: (B)

16. A flat plate of width L = 1 m is pushed down with a velocity U = 0.01 m/s towards a wall resulting
in the drainage of the fluid between the plate and the wall as shown in the figure.
L
U Plate
u avg d  0.1m u avg

Wall
© All rights reserved by Thinkcell Learning Solutions Pvt. Ltd. No part of this booklet may be reproduced or utilized in any form without the written permission.

10

Downloaded From : www.EasyEngineering.net


Downloaded From : www.EasyEngineering.net

ME-GATE 2018

Assume two-dimensional incompressible flow and that the plate remains parallel to the wall. The
average velocity, uavg of the fluid (in m/s) draining out at the instant shown in the figure is ___
(correct to three decimal places).
Key: (0.05)
Sol: Let b = length of plate perpendicular to paper
Volume drain out per sec = L.b.U
So, L.b.U  2b.u avg .d
L.U 1  0.01
u avg  
2d 2  0.1
u avg  0.05m/s

17. According to the Mean Value Theorem, for a continuous function f(x) in the interval [a, b], there
b
exists a value  in this interval such that  f (x)dx 
a

(A) f    (b a) (B) f (b) (  a) (C) f (a)(b  ) (D) 0

Key: (A)
y
Sol: By using mean value theorem for definite integrals,
b
f  a, b  s.t  f  x  dx   b  a  f    f x
a d
f   c
 in between a & b s.t

Area of rectangle abcd = Area under the curve f(x) with x-axis
x
b
a  b
i.e.,  b  a  f      f  x  dx
a

18. A steel column of rectangular section (15 mm × 10 mm) and length 1.5 m is simply supported at
both ends. Assuming modulus of elasticity, E = 200GPa for steel, the critical axial load (in kN) is
___ (correct to two decimal places)
Key: (1.097)
Sol: e  1500mm, b  15mm,d  10mm, E  200GPa
2 EI 15103
PE  2
; I
e 12
15103
2  200 103 
 12  1.097kN
1500 
2

© All rights reserved by Thinkcell Learning Solutions Pvt. Ltd. No part of this booklet may be reproduced or utilized in any form without the written permission.

11

Downloaded From : www.EasyEngineering.net


Downloaded From : www.EasyEngineering.net

ME-GATE 2018

19. If the wire diameter of a compressive helical spring is increased by 2%, the change in spring
stiffness (in %) is ___ (correct to two decimal places.)
Key: (8.243)
Sol: d 2  1.02d1
Gd 4
k  k  d4
8WD3
4
k1  d1 
   k 2  1.082
k 2  1.02d1 
k  k1  1.082  1 
% change in stiffness  2   100%  8.243%
k1  1 

20. For a two-dimensional incompressible flow field given by u  A xiˆ  yjˆ , where A > 0,  
which one of the following statements is FALSE?
(A) It satisfies continuity equation
(B) It is unidirectional when x  0 and y  ,
(C) Its streamlines are given by x = y.
(D) It is irrotational
Key: (C)
Sol: ∵ Equation of streamline
dy v  y
 
dx u x
dy dx
 y    x  ny  nx  nc
xy  C. So, option C is false.

21. The equation of motion for a spring-mass system excited by a harmonic force is
mx + Kx = Fcos(  t),
where M is the mass, K is the spring stiffness, F is the force amplitude and  is the angular
frequency of excitation. Resonance occurs when  is equal to
M 1 K K K
(A) (B) (C) 2 (D)
K 2 M M M
Key: (D)

Sol: Resonance will occur when, r  1.


K
i.e., when   n 
M

© All rights reserved by Thinkcell Learning Solutions Pvt. Ltd. No part of this booklet may be reproduced or utilized in any form without the written permission.

12

Downloaded From : www.EasyEngineering.net


Downloaded From : www.EasyEngineering.net

ME-GATE 2018

22. A bar of uniform cross section and weighing 100 N is held horizontally using two massless and
inextensible strings S1 and S2 as shown in the figure.

Rigid support

T1=? T2  ?
S1 S2 Bar

L/2 L/2
The tensions in the strings are
(A) T1 = 100 N and T2 = 0 N (B) T1 = 0 N and T2 = 100 N
(C) T1 = 75 N and T2 = 25 N (D) T1 = 25 N and T2 = 75 N
Key: (B)
Sol: M A  0
T1 T2  ?
L L
T2  100 
2 2
T2  100 L/2 L/2
T1  T2  100 T1 T2
T1  0N, T2  100N

A
L/2
100N

23. F (z) is a function of the complex variable z = x + i y given by


F(z) = iz + k Re(z) + i Im(z)
For what value of k will F (z) satisfy the Cauchy-Riemann equations?
(A) 0 (B) 1 (C) –1 (D) y
Key: (B)

Sol: F  z   iz  k Re  z   iI m  z 
 i  x  iy   kx  iy
 ix  y  kx  iy
  kx  y   i  x  y 
 u  iv  say 

u v u v
Applying C-R equations, i.e.,  &   k 1
x y y x

© All rights reserved by Thinkcell Learning Solutions Pvt. Ltd. No part of this booklet may be reproduced or utilized in any form without the written permission.

13

Downloaded From : www.EasyEngineering.net


Downloaded From : www.EasyEngineering.net

ME-GATE 2018

24. The type of weld represented by the shaded region in the figure is

(A) groove (B) spot (C) fillet (D) plug


Key: (C)

25. For a Pelton wheel with a given water jet velocity, the maximum output power from the Pelton
wheel is obtained when the ratio of the bucket speed to the water jet speed is _____ (correct to two
decimal places).
Key: (0.5)
Sol: ∵ Efficiency of Pelton wheel is maximum,
u
when  0.5
v
Where u = velocity of blade
v = velocity of jet (Absolute) at inlet.

26. Let X1, X2 be two independent normal random variables with means μ1, μ2 and standard deviations
1 ,  2 respectively. Consider Y = X1 – X2; μ1 = μ2 = 1, 1  1, 2  2, Then,

(A) Y is normal distributed with mean 0 and variance 1


(B) Y is normally distributed with mean 0 and variance 5
(C) Y has mean 0 and variance 5, but is NOT normally distributed
(D) Y has mean 0 and variance 1, but is NOT normally distributed
Key: (B)
Sol: Sum or difference of normal random variables is again a normal random variable
Mean of ' y'  E  y   E  x1  x 2   E  x1   E  x 2 
 1  2
 1 1  0
Variance of ' y'  v  y   v  x1  x 2   v  x1   v  x 2 
12  22
1  4  5

© All rights reserved by Thinkcell Learning Solutions Pvt. Ltd. No part of this booklet may be reproduced or utilized in any form without the written permission.

14

Downloaded From : www.EasyEngineering.net


Downloaded From : www.EasyEngineering.net

ME-GATE 2018

27. A slider crank mechanism is shown in the figure. At some instant, the crank angle is 45° and a force
of 40 N is acting towards the left on the slider. The length of the crank is 30 mm and the connecting
rod is 70 mm. Ignoring the effect of gravity, friction and inertial forces, the magnitude of the
crankshaft torque (in Nm) needed to keep the mechanism in equilibrium is ___ (correct to two
decimal places).

40 N

Key: (1.117)
Sol: Fp  40N
30sin 45  70sin  30 70
sin   0.303   17.6 o

45 40 N
Fp
Torque  sin       r
cos 
40  sin 17.6  45 
  0.03  1.117N  mts
cos17.6

28. The value of integral

 r.n dS
S

over the closed surface S bounding a volume V, where r  xiˆ  y j  zkˆ is the position vector and n
is the normal to the surface S, is
(A) V (B) 2 V (C) 3 V (D) 4 V
Key: (C)
Sol: By Gauss divergence theorem,

 r.n ds   div r dv
s v

 r  xiˆ  yj  zkˆ 
  3dv  3V  
v  div r 111  3

29. A plane slab of thickness L and thermal conductivity k is heated with a fluid on one side (P), and
the other side (Q) is maintained at a constant temperature, TQ of 25°C, as shown in the figure. The
fluid is at 45°C and the surface heat transfer coefficient, h, is 10 W/m2K. The steady state
temperature, TP (in °C) of the side which is exposed to the fluid is ____ (correct to two decimal
places).

© All rights reserved by Thinkcell Learning Solutions Pvt. Ltd. No part of this booklet may be reproduced or utilized in any form without the written permission.

15

Downloaded From : www.EasyEngineering.net


Downloaded From : www.EasyEngineering.net

ME-GATE 2018

TP

h  10 W / m2 K TQ  25C
T  45C
k  2.5W / mK

L  20cm

Key: (33.89)
Sol: For the steady state heat transfer,
T  Tp TP  TQ
Q 
 1   L  TP
   
 hA   kA 
45  Tp T  25
Q  P h  10 W / m2 K
 1   0.2  TQ  25C
    T  45C
k  2.5W / mk
 10  A   2.5 A 
2.5  Tp  25 
10  45  Tp  
0.2 L  20cm
  90  2Tp    2.5  Tp  62.5
 Tp  33.89C, So answer is33.89

30. F(s) is the Laplace transform of the function


f (t)  2t 2 e  t

F(1) is ____ (correct to two decimal places).


Key: (0.5)

Sol: L  2t 2 e t   2L  t 2 e  t 
2  2! 2 
 L  t   3  3 & L e f  t   F  s  a   
 at
 2 2

 s  1  
3
s s
4
  Fs
 s  1
3

4
 F 1   0.5
8

© All rights reserved by Thinkcell Learning Solutions Pvt. Ltd. No part of this booklet may be reproduced or utilized in any form without the written permission.

16

Downloaded From : www.EasyEngineering.net


Downloaded From : www.EasyEngineering.net

ME-GATE 2018

31. The true stress (  ) - true strain (  ) diagram of a strain
(MPa)
hardening material is shown in figure. First, there is A
500
loading up to point A, i.e. up to stress of 500 MPa and
strain of 0.5. Then from point A, there is unloading up to
point B, i.e., to stress of 100 MPa, Given that the
100 B
Young's modulus E = 200GPa, the natural strain at point
B (εB) ___ (correct to two decimal places). 
B 0.5

Exp: (0.498) 
(MPa)
AC A
Sol: E  slope  500
BC
400 106
200 109   BC  2 103  0.002
BC
B
So, B  0.5  0.002  0.498 100 C

B 0.5 

32. The percentage scrap in a sheet metal blanking operation of a continuous strip of sheet metal as
shown in the figure is___ (correct to two decimal places).

D/5 D/5 D/5


D/5 D/5
Feed D D

D/5 D/5 D/5

Key: (53.27)
Sol: Area of rectangle A B
6D 7D 42 2
ABCD =   D  1.68D2
5 5 25
D 7D
D  2 
and 5 5
 2
Area of blank = D  0.785D2
4 D C
1.68D  0.785D
2 2
So, % scrap = 100  53.27%
1.68D2 D 6D
D 
5 5

© All rights reserved by Thinkcell Learning Solutions Pvt. Ltd. No part of this booklet may be reproduced or utilized in any form without the written permission.

17

Downloaded From : www.EasyEngineering.net


Downloaded From : www.EasyEngineering.net

ME-GATE 2018

33. An epicyclic gear train is shown in the figure below. The number of teeth on the gears A, B and D
are 20, 30 and 20 respectively. Gear C has 80 teeth on the inner surface and 100 teeth on the outer
surface. If the carrier arm AB is fixed and the sun gear A rotates at 300 rpm in the clockwise
direction, then the rpm of D in the clockwise direction is
(A) 240 C
B
(B) –240
A D
(C) 375

(D) –375
Key: (C)
Sol: N A  300rpm; TA  20; TB  30; Tcin  80
Tcext  100
N A TA 300  20
NB    200
TB 30
N B TB 200  30
NC    75
TC 80
N C TC 75100
ND    375 rpm
TD 20

34. The minimum value of 3x + 5y such that:


3x + 5y  15
4x + 9y  8
13x + 2y  2
x  0, y  0
is _______.
Key: (0)
Sol: ∵ All main constraints are of  type and

Non-negativity constraints are of  type.

So, feasible region contain origin (0, 0) as corner point.


So, at origin (0, 0)
zmin  3 0  5 0  0

© All rights reserved by Thinkcell Learning Solutions Pvt. Ltd. No part of this booklet may be reproduced or utilized in any form without the written permission.

18

Downloaded From : www.EasyEngineering.net


Downloaded From : www.EasyEngineering.net

ME-GATE 2018

35. Block P of mass 2 kg slides down the surface and has a speed 20 m/s at the lowest point, Q, where
the local radius of curvature is 2 m as shown in the figure. Assuming g = 10 m/s2, the normal force
(in N) at Q is ____ (correct to two decimal places).
P

Q
Key: (420)
mv2
Sol: R Q  mg 
R
 2  202 
  2 10      20  400  420N
 2 

36. A solid block of 2.0 kg mass slides steadily at a velocity V along a vertical wall as shown in the
figure below. A thin oil film of thickness h = 0.15 mm provides lubrication between the block and
the wall. The surface area of the face of the block in contact with the oil film is 0.04 m2. The
velocity distribution within the oil film gap is linear as shown in the figure. Take dynamic viscosity
of oil as 7 × 10–3 Pa-s and acceleration due to gravity as 10 m/s2. Neglect weight of the oil. The
terminal velocity V (in m/s) of the block is ___ (correct to one decimal place).

h  0.15mm

m kg
m=2.0 2.0kg
v V

A  0.04m2
Impermeable
Key: (10.714) wall
Sol: F  .A
 du  V
F     .A  F  . .A
 dy  h
V
 2 10  7 103   0.04
0.15 103
 V  10.714m/s

© All rights reserved by Thinkcell Learning Solutions Pvt. Ltd. No part of this booklet may be reproduced or utilized in any form without the written permission.

19

Downloaded From : www.EasyEngineering.net


Downloaded From : www.EasyEngineering.net

ME-GATE 2018

37. A machine of mass m = 200 kg is supported on two mounts, each of stiffness k = 10 kN/m. The
machine is subjected to an external force (in N) F(t) = 50 cos5t. Assuming only vertical translatory
motion, the magnitude of the dynamic force (in N) transmitted from each mount to the ground is
___ (correct to two decimal places).
F(t)

k k

Key: (33.33)
Sol: k  10kN/m, m=200kg
F=50cos5t,   5rad/sec
F=50
20000
n   10rad / sec
200
0

1   2r 
2
F
 T 
F
1  r    2r 
2 2 2

FT 1
 2
50 5
1  
 10 
FT 1 200
  FT   66.66
50  3 3
 
4
66.66
Forceon each mount   33.33
2

38. A bar is compressed to half of its original length. The magnitude of true strain produced in the
deformed bar is ____ (correct to two decimal places).
Key: (0.693)
 
Sol:   n  f    n0.5
 i
  0.693
But magnitude of true strain = 0.693

© All rights reserved by Thinkcell Learning Solutions Pvt. Ltd. No part of this booklet may be reproduced or utilized in any form without the written permission.

20

Downloaded From : www.EasyEngineering.net


Downloaded From : www.EasyEngineering.net

ME-GATE 2018

39. A carpenter glues a pair of cylindrical wooden logs by bonding their end faces at an angle of  =
30° as shown in the figure.

Log1 Log 2
Interface  axis
4MPa 4MPa

  30
The glue used at the interface fails if
Criterion 1: the maximum normal stress exceeds 2.5MPa.
Criterion 2: the maximum normal stress exceeds 1.5MPa.
Assume that the interface fails before the logs fail. When a uniform tensile stress of 4 MPa is
applied, the interface
(A) fails only because of criterion 1
(B) fails only because of criterion 2
(C) fails because of both criterion 1 and 2
(D) does not fail
Key: (C)
P
Sol:   30o     4MPa
A
P
  cos 2 
A
 4cos 2 30  3MPa  2.5MPa
P 4
  sin 2    sin 601.732 1.5MPa
2A 2
Fails by both Criterion 1 and 2

40. In a Lagrangian system, the position of a fluid particle in a flow is described as x = x0e-kt and
Y = y0ekt where t is the time while x0, y0, and k are constants. The flow is
(A) unsteady and one-dimensional (B) steady and two-dimensional
(C) steady and one-dimensional (D) unsteady and two-dimensional
Key: (B)
dx
Sol: u
dt
u  kx o e kt  u  kx
dy
&v   v  ky
dt

© All rights reserved by Thinkcell Learning Solutions Pvt. Ltd. No part of this booklet may be reproduced or utilized in any form without the written permission.

21

Downloaded From : www.EasyEngineering.net


Downloaded From : www.EasyEngineering.net

ME-GATE 2018

So, V  kxi  kyj


i.e,V  f  t 
So, steady flow & V  f  x, y . So, 2D flow.

41. Processing times (including setup times) and due dates for six jobs waiting to be processed at a
work centre are given in the table. The average tardiness (in days) using shortest processing time
rule is ___ (correct to two decimal places).

Job Processing time (days) Due date (days)

A 3 8

B 7 16

C 4 4

D 9 18

E 5 17

F 13 19

Key: (6.33)
,

Sol: By SPT rule

Job Sequence P.T Machining D.D Tardiness

In Out

A 3 0 3 8 0

C 4 3 7 4 3

E 5 7 12 17 0

B 7 12 19 16 3

D 9 19 28 18 10

F 13 28 41 19 22

Total Tardiness = 38
38
Average Tardiness   6.33days
6

© All rights reserved by Thinkcell Learning Solutions Pvt. Ltd. No part of this booklet may be reproduced or utilized in any form without the written permission.

22

Downloaded From : www.EasyEngineering.net


Downloaded From : www.EasyEngineering.net

ME-GATE 2018

42. Steam flows through a nozzle at mass flow rate of m = 0.1 kg/s with a heat loss of 5 kW. The
enthalpies at inlet and exit are 2500 kJ/kg and 2350 kJ/kg, respectively. Assuming negligible
velocity at inlet (C1  0), the velocity (C2) of steam (in m/s) at the nozzle exit is ___ (correct to two
decimal places)
Q  5kW

h1  2500 kJ / kg h 2  2350kJ / kg
m  0.1kg / s
C1  0 C2

Key: (447.21)
Sol: m  0.1kg/sec, Q  5kW  Heat Rejected 
h1  2500kJ/kg, h 2  2350kJ/kg
C1  0m/sec,C2  ?
Q  5kW

h1  2500 kJ / kg h 2  2350kJ / kg
m  0.1kg / s
C1  0 C2  ?

Applying steady flow energy equation

 C2   C2 
m  h1  1   Q  m  h 2  2   W
 2000   2000 
W  0, since no work done, C1  0
 C2 
 0.1 2500   5  0.1 2350  2 
 2000 
 C 
2
C2
245  0.1 2350  2   2450  2350  2
 2000  2000
 C2  447.21m/sec

© All rights reserved by Thinkcell Learning Solutions Pvt. Ltd. No part of this booklet may be reproduced or utilized in any form without the written permission.

23

Downloaded From : www.EasyEngineering.net


Downloaded From : www.EasyEngineering.net

ME-GATE 2018

43. A point mass is shot vertically up from ground level with a velocity of 4 m/s at time, t = 0. It loses
20% of its impact velocity after each collision with the ground. Assuming that the acceleration due
to gravity is 10 m/s2 and that air resistance is negligible, the mass stops bouncing and comes to
complete rest on the ground after a total time (in seconds) of
(A) 1 (B) 2 (C) 4 (D) 
Key: (C)
Sol: V1  4m / sec, V2  0.8 4  3.2m / sec

2V1
V1  V  t1 
g
2V2
V2  0.8V  t 2 
g
2V
V3  0.64V  t 3  3
g
T  t1  t 2  ...................
 2V 2V 2V 
  1  2  3  .......
 g g g 
2
  4 1 0.8  0.64  .....
10
2
  4 1 0.8   0.8  ..... 
2

10  
1 0.8
 0.8   4  4sec
1  0.8 0.2

44. A tank of volume 0.05 m3 contains a mixture of saturated water and saturated steam at 200°C. The
mass of the liquid present is 8 kg. The entropy (in kJ/kgK) of the mixture is _____ (correct to two
decimal places)
Property data for saturated steam and water are:
At 200°c, Psat = 1.5538 MPa
vf = 0.001157 m3/kg, vg = 0.12736 m3/kg
sfg = 4.1014 kJ/kgK, sf = 2.3309 kJ/kgK
Key: (2.49)
Sol: V  0.05m3 ,Tsat  200C, m L  8kg
At 200C, Psat  1.5538MPa
vf  0.001157m3 /kg, v g  0.12736m 3 /kg
sfg  4.1014kJ/kgK, s f  2.3309kJ/kgK

© All rights reserved by Thinkcell Learning Solutions Pvt. Ltd. No part of this booklet may be reproduced or utilized in any form without the written permission.

24

Downloaded From : www.EasyEngineering.net


Downloaded From : www.EasyEngineering.net

ME-GATE 2018

Given container has both water vapor & liquid


i.e., it is in wet-vapour region,
In that case entropy of mixture is s  sf  xsfg

mv
Where x=dryness fraction=
m v  mL

vf  liquid   m L vf   8  0.001157 
vf  liquid   0.01
v vapour  v  vf  liquid   0.05  0.01  0.04m3
v vapour 0.04
mv    0.31kg
vg 0.12736
0.31
x  0.04
 0.31  8
 s  2.3309  0.04  4.1014   2.49 kJ/kgK

45. A tank open at the top with a water level of 1 m, as shown in the figure, has a hole at a height of 0.5
m. A free jet leaves horizontally from the smooth hole. The distance X (in m) where the jet strikes
the floor is

Free Jet
1m

0.5m

(A) 0.5 (B) 1.0 (C) 2.0 (D) 4.0


Key: (B)

Sol: u x  2  9.81 0.5  3.13


uy  0
1 1
sy  u y t  a y t 2  t 
2 g
1 1
s x  u x  a x t 2  s x  3.13 (here a x  0)
2 g
s x  1m

© All rights reserved by Thinkcell Learning Solutions Pvt. Ltd. No part of this booklet may be reproduced or utilized in any form without the written permission.

25

Downloaded From : www.EasyEngineering.net


Downloaded From : www.EasyEngineering.net

ME-GATE 2018

46. A self-aligning ball bearing has a basic dynamic load rating (C10, for 106 revolutions) of 35 kN. If
the equivalent radial load on the bearing is 45kN, the expected life (in 106 revolutions) is
(A) below 0.5 (B) 0.5 to 0.8 (C) 0.8 to 1.0 (D) above 1.0
Key: (A)
Sol: Pe  45kN; C  35kN
3
 C   35 
3

L10        0.47  0.5


 Pe   45 

47. A simply supported beam of width 100 mm, height 200 mm and length 4 m is carrying a uniformly
distributed load of intensity 10kN/m. The maximum bending stress (in MPa) in the beam is __
(correct to one decimal place)
10kN / m

4m

Key: (30)
M wL2 10  42
Sol:  y, M    20kN-m
I 8 8
100  2003
I=  66.67 106
12
200
y  100;
2
20 106  200 
    30MPa
66.67 106  2 

300
48. The schematic of an external drum rotating 200
clockwise engaging with a short shoe is shown in
the figure. The shoe is mounted at point Y on a rigid
lever XYZ hinged at point X. A force F = 100 N is X
applied at the free end of the lever as shown. Given 300
F
that the coefficient of friction between the shoe and Y
the drum is 0.3, the braking torque (in Nm) applied Z
on the drum is ____ (correct to two decimal places). 100

Key: (8.18)
Sol: M x  0 (All dimensions are in mm)

© All rights reserved by Thinkcell Learning Solutions Pvt. Ltd. No part of this booklet may be reproduced or utilized in any form without the written permission.

26

Downloaded From : www.EasyEngineering.net


Downloaded From : www.EasyEngineering.net

ME-GATE 2018

 R N  200    R N  300    F 300   0


 R N  200    0.3 R N  300   100  300   0
RN 
100 300   272.72 N
 200   0.3 300  
T  R N  Radius  8.18 N  mts

49. The state of stress at a point, for a body in plane stress, is shown in the figure below. If the
minimum principal stress is 10 kPa, then the normal stress y (in kPa) is
y

xy  50 kPa

x  100kPa

(A) 9.45 (B) 18.88 (C) 37.78 (D) 75.50


Key: (C)

 x  y   x  y 
2

Sol: 2       Txy
2

 2   2 

100   y   100  y 
2

10       50
2

 2   2 
2
 100   y    100   y 
2

10        50
2

  2   2 
 100   y   100  y   100   y 
2 2

100  2 10         2500


 2   2   2 
4 100  y
100  1000  10 y   2500
4
90 y  2500 1000  100  y  37.77 MPa

50. An electrochemical machining (ECM) is to be used to cut a through hole into a 12 mm thick
aluminium plate. The hole has a rectangular cross-section, 10 mm × 30 mm. The ECM operation
will be accomplished in 2 minutes, with efficiency of 90%. Assuming specific removal rate for
aluminium as 3.44 × 10–2 mm3/ (A s), the current (in A) required is ____ (correct to two decimal
places).

© All rights reserved by Thinkcell Learning Solutions Pvt. Ltd. No part of this booklet may be reproduced or utilized in any form without the written permission.

27

Downloaded From : www.EasyEngineering.net


Downloaded From : www.EasyEngineering.net

ME-GATE 2018

Key: (968.99)
Sol: Volume to be removed = 10  30 12  3600 mm 3
3600
Energy required =  116279 As
3.44 102  0.9
116279
Current   968.99A
120

51. An engine working on air standard Otto cycle is supplied with air at 0.1 MPa and 35 oC. The
compression ratio is 8. The heat supplied is 500kJ/kg. Property data for air: cp = 1.005 kJ/kgK, cv =
0.718 kJ/kg K, R = 0.287 kJ/kg K. The maximum temperature (in K) of the cycle is _____ (correct
to one decimal place).
Key: (1403.98)
Sol: P  0.1MPa,T1  35C, rc  8

Qsup plied  500kJ/kg 3 3


VC
SC
Cp  1.005kJ/kgK, 2 4
T P
CV  0.718kJ/kgK, VC
4 2
1 SC
R  0.287kJ/kgK 1
S
T1  35  273  308K V
1
T2  V1 
   rc 
T1  V2 
1.4 1
 T2   3088  707.60K
Qsup plied  Cv  T3  T2 
500   0.718 T3  707.60 
T3  1403.98K

52. An orthogonal cutting operation is being carried out in which uncut thickness is 0.010 mm, cutting
speed is 130 m/min, rake angle is 15° and width of cut is 6 mm. It is observed that the chip
thickness is 0.015 mm, the cutting force is 60 N and the thrust force is 25 N. The ratio of friction
energy to total energy is ____ (correct to two decimal places).
Key: (0.4406)
Sol: t1  0.01mm; v  130m / min;   15
b  6mm; t c  0.015mm,
cutting force  FH  60N
Thrust force  FV  25N

© All rights reserved by Thinkcell Learning Solutions Pvt. Ltd. No part of this booklet may be reproduced or utilized in any form without the written permission.

28

Downloaded From : www.EasyEngineering.net


Downloaded From : www.EasyEngineering.net

ME-GATE 2018

R  602  252  65N


FH
F  R sin   39.66N

F 25
tan       v   22.61 Fv

FH 60 R F

  37.61
Friction energy F t1 N
 .  0.4406
Totalenergy FH t c

53. An explicit forward Euler method is used to numerically integrate the differential equation
dy
y
dt
using a time step of 0.1. With the initial condition y (0) = 1, t he value of y (1) computed by this
method is ___ (correct to two decimal places).
Key: (2.59)
Sol: By Euler method,
y n 1  y n  nf  t n , y n 
where given f  t, y   y
 y n 1  y n  hy n
 y n 1  1  h  y n
y1  y  0.1  1  0.1 1  1.1  y 0  1
y 2  y  0.2   1.11.1
y3  y  0.3  1.1 1.11.1
:
y10  y 1  1.1
10
2.59

54. The maximum reduction in cross-sectional area per pass (R) of a cold wire drawing process is
R = 1 – e–(n + 1)
where n represents the strain hardening coefficient. For the case of a perfectly plastic material, R is
(A) 0.865 (B) 0.826 (C) 0.777 (D) 0.632
Key: (D)
Sol: ∵ for perfectly plastic material, n=0
So, R  1  e1  R  0.632

© All rights reserved by Thinkcell Learning Solutions Pvt. Ltd. No part of this booklet may be reproduced or utilized in any form without the written permission.

29

Downloaded From : www.EasyEngineering.net


Downloaded From : www.EasyEngineering.net

ME-GATE 2018

55. A sprinkler shown in the figure rotates about its hinge point in a horizontal plane due to water flow
discharged through its two exit nozzles.

Q/2 Q/2

10 cm 20 cm

The total flow rate Q through the sprinkler is 1 litre/sec and the cross-sectional area of each exit
nozzle is 1 cm2. Assuming equal flow rate through both arms and a frictionless hinge, the steady
state angular speed of rotation (rad/s) of the sprinkler is ________ (correct to two decimal places).
Key: (10)
Q 1103
Sol: u1  u 2    5m/s u1 u2
2A 2 104
Now Absolute velocity
v1  u1  r1 1  2
v2  u 2  r2  0.1  r1 r2  0.2m
external Torque  0 
m1 v1 r1  m2 v2 r2  0 ( m1  m2 )
   10rad / s

© All rights reserved by Thinkcell Learning Solutions Pvt. Ltd. No part of this booklet may be reproduced or utilized in any form without the written permission.

30

Downloaded From : www.EasyEngineering.net


Downloaded From : www.EasyEngineering.net

ME-GATE 2018

© All rights reserved by Thinkcell Learning Solutions Pvt. Ltd. No part of this booklet may be reproduced or utilized in any form without the written permission.

31

Downloaded From : www.EasyEngineering.net


Downloaded From : www.EasyEngineering.net

**Note: Other Websites/ Blogs Owners Please do not Copy (or) Republish
this Materials without Legal Permission of the Publishers.

**Disclimers : EasyEngineering not the original publisher of this Book/Material


on net. This e-book/Material has been collected from other sources of net.
Downloaded From : www.EasyEngineering.net
Downloaded From : www.EasyEngineering.net

ME-GATE 2018

Section-I: General Ability

1. The perimeters of a circle, a square and an equilateral triangle are equal. Which one of the
following statements is true?
(A) The circle has the largest area (B) The square has the largest area
(C) The equilateral triangle has the largest area. (D) All the three shapes have the same area.
Key: (A)
Sol: Let, side of equilateral triangle as 'a'.
Then perimeter of equilateral triangle = 3a.
We know that, perimeter of circle = 2r
Perimeter of square = 4x; [where x is side of square]
3a
Given, 3a  2r 3a  4x  x 
3a 4
r 3a
2  sideof square 
3a 4
 Radiusof circle  r  3a
2  x
4

 9a 2  9a 2
 Area of circle  r 2   2   0.716a 2
 4  4
3a 3a 9a 2
∴ Area of square = x  x    0.563a 2
4 4 16
3a 2
∴ Area of equilateral triangle =  0.433a 2
4
∴ The Circle has the largest area.

2. Find the missing group of letters in the following series:


BC, FGH, LMNO, ________
(A) UVWXY (B) TUVWX (C) STUVW (D) RSTUV
Key: (B)
3 4 5
Sol: BC FGH LMNO TUVWX

3. “The judge‟s standing in the legal community, though shaken by false allegations of wrongdoing,
remained _______.”
The world that best fills the blank in the above sentence is
(A) Undiminished (B) damaged (C) illegal (D) uncertain
Key: (A)

© All rights reserved by Thinkcell Learning Solutions Pvt. Ltd. No part of this booklet may be reproduced or utilized in any form without the written permission.

Downloaded From : www.EasyEngineering.net


Downloaded From : www.EasyEngineering.net

ME-GATE 2018

1 1 1
4. The value of the expression   is ______
1  log u vw 1  log v wu 1  log w uv
(A) –1 (B) 0 (C) 1 (D) 3
Key: (C)
1 1 1
Sol:  
log vw log wu log uv
1 1 1
log u log v log w
log u log v log w
  
log u  log vw log v  log wu log w  log uv
log u log v log w log  uvw 
    1
log  uvw  log  uvw  log  uvw  log  uvw 

5. “The dress ______ her so well that they all immediately ______ her on her appearance.”
The words that best fill the blanks in the above sentence are
(A) complemented, complemented (B) complimented, complemented
(C) complimented, complimented (D) complemented, complimented
Key: (D)

6. Forty students watched films A, B and C over a week. Each student watched either only one film
or all three. Thirteen students watched film A, sixteen students watched film B and nineteen
students watched film C. How many students watched all three films?
(A) 0 (B) 2 (C) 4 (D) 8
Key: (C)
Sol: Given, Total no. of students who watched films A, B and C over a week = n  s   40.
Also given that, each student watched either only one film or all three.
  
i.e.,n A  B  C  n B  C  A  n  C  A  B  0; 
Where  A, B, C are complementatry events of A, B,C respectively 

Given, n  A   13;n  B  16;n  C  19


Assume that no. of students who watch all three films = n  A  B  C  x


Given, n A  B  C  13  x  n S  40

n  B  C  A   16  x
A 13 B16
n  C  B  A   19  x 13  x 0 16  x
x
n  s   13  x  16  x  19  x  x 0 0

 40  48  2x  2x  48  40 19  x
8 C 19
 2x  8  x   4
2
∴ No. of students who watched all the three films = x=4
© All rights reserved by Thinkcell Learning Solutions Pvt. Ltd. No part of this booklet may be reproduced or utilized in any form without the written permission.

Downloaded From : www.EasyEngineering.net


Downloaded From : www.EasyEngineering.net

ME-GATE 2018

7. A house has a number which needs to be identified. The following three statements are given that
can help in identifying the house number.
i. If the house number is a multiple of 3, then it is a number from 50 to 59.
ii. If the house number is NOT a multiple of 4, then it is a number from 60 to 69.
iii. If the house number is NOT a multiple of 6, then it is a number from 70 to 79.
What is the house number?
(A) 54 (B) 65 (C) 66 (D) 76
Key: (D)
Sol: From Statement-i, we have
if house no. is multiple of 3, then house no. 50,59
From Statement-ii, we have
if house no. is not a multiple of 4, then house no. 60,69
From Statement-iii, we have
if house no. is not a multiple of 6, then house no 70,79
∴ The required house number, is 76 among {54, 65, 66, 76};
Since (i) 76 is not multiple of 3, so house no. 50,59
(ii) 76 is not multiple of 6, so house no. 70, 79
(iii) 76 is multiple of 4, so house no. 60, 69
∴ The required house no. is 76.

8. A contract is to be completed in 52 days and 125 identical robots were employed, each operational
for 7 hours a day. After 39 days, five-seventh of the work was completed. How many additional
robots would be required to complete the work on time, if each robot is now operational for 8
hours a day?
(A) 50 (B) 89 (C) 146 (D) 175
Key: (0)
Sol: 1Robot 1hr 1 work
7 hrs  7 work
 125 Robots  7 hrs / day  125 7
125Robots  7 hrs / day  52days 125  7 52  45,500  Total work
Given, After 39 days, 5/7th of work was completed.
5
i.e., 45,500   32,500.  Actually,in 39days 34125work has to becompleted 
7
Remaining work  45,500  32,500  13,000 work
After 39 days, each robot is working 8hrs/day (given)
13days  8hrs/day  125 robots can work 13,000 work.
i.e.Addtional robots not required, if each robot work8hrs/dayon13days.

© All rights reserved by Thinkcell Learning Solutions Pvt. Ltd. No part of this booklet may be reproduced or utilized in any form without the written permission.

Downloaded From : www.EasyEngineering.net


Downloaded From : www.EasyEngineering.net

ME-GATE 2018

9. An unbiased coin is tossed six times in a row and four different such trials are conducted. One trial
implies six tosses of the coin. If H stands for head and T stands for tail, the following are the
observations from the four trials:
(1) HTHTHT (2) TTHHHT (3) HTTHHT (4) HHHT __ __.
Which statement describing the last two coin tosses of the fourth trial has the highest probability
of being correct?
(A) Two T will occur (B) One H and one T will occur
(C) Two H will occur (D) One H will be followed by one T
Key: (B)
Sol: In this, we are talking about 4th trial, ie., nothing but one trial
One trial → Tossing a coin six times (or) six coins tossed at a time
6

1  P  H T H T H T          
1 1 1 1 1 1 1
2 2 2 2 2 2 2
6

 2   P T T H H H T   
1

2

 3  P  H T T H H T   
1

2
 4  H H H T _ _

Option(A) : Option(B) : Option(C) : Option(D) :


HHHTTT HHHTHT HHHTHH HHHTTH
 Required probability  or   Required probability  Required probability
6 6 6
1 HHHTTH 1 1
     
2  Required probability 2 2
6 6 6
1 1 1
      2 
2 2 2

10. A wire would enclose an area of 1936 m2, if it is bent into a square. The wire is cut into two
pieces. The longer piece is thrice as long as the shorter piece. The long and the short pieces are
bent into a square and a circle, respectively. Which of the following choices is closest to the sum
of the areas enclosed by the two pieces in square meters?
(A) 1096 (B) 1111 (C) 1243 (D) 2486
Key: (C)
Given that, a wire bent over square has area of 1936m2
Let us assume that, the side of square as 'x'.
Then x 2 1936  x  44meters.

© All rights reserved by Thinkcell Learning Solutions Pvt. Ltd. No part of this booklet may be reproduced or utilized in any form without the written permission.

Downloaded From : www.EasyEngineering.net


Downloaded From : www.EasyEngineering.net

ME-GATE 2018

∴ Length of wire= perimeter of square = 4x 176m


Again given a  b  176 such that a  3b
 3b  b  17b  4b 17b
 b  44  a  132  a  3b 
Given 'a' is bent over square and 'b' bent over circle.
132
i.e,. 4x '  a  132  x '   33  x '  length of thesideof square 
4
 x '  33
Area of square  33 33 1089 ...1
 2r  b  'b'bent over circle
44
r   b  44  r  7
2
 Area of circle   72 154   ... 2 
 From 1  &  2   sumof area 's 1089 154 1243m 2

Section-II: Mechanical Engineering

1. The Fourier cosine series for an even function f(x) is given by



f  x   a 0   a n cos  nx 
n 1

The value of the coefficient a2 for the function f  x   cos  x  in 0,  is


2

(A) –0.5 (B) 0.0 (C) 0.5 (D) 1.0


Key: (C)
Exp: We have, Fourier expression for even function:
a 
cos nx
f  x   0  an ; if f(x) is defined over the range –L to L. [i.e., period 2L]
2 n 1 L
1  cos 2x
But given f  x   cos 2 x 
2
1 1 a
  cos 2x  0  a 2 .cos 2x
2 2 2
 a 0  1; a 2  1 ; a1  0
2

2. Select the correct statement for 50% reaction stage in a steam turbine.
(A) The rotor blade is symmetric.
(B) The stator blade is symmetric.
(C) The absolute inlet flow angle is equal to absolute exit flow angle.
(D) The absolute exit flow angle is equal to inlet angle of rotor blade.

© All rights reserved by Thinkcell Learning Solutions Pvt. Ltd. No part of this booklet may be reproduced or utilized in any form without the written permission.

Downloaded From : www.EasyEngineering.net


Downloaded From : www.EasyEngineering.net

ME-GATE 2018

Key: (D)
Exp: For 50% reaction turbine   ,   

3. During solidification of a pure molten metal, the grains in the casting near the mould wall are
(A) coarse and randomly oriented (B) fine and randomly oriented
(C) fine and ordered (D) coarse and ordered
Key: (B)
Exp: Cooling at wall is faster so, grain size is fine & randomly oriented

4. The peak wavelength of radiation emitted by a black body at a temperature of 2000 K is 1.45 m.
If the peak wavelength of emitted radiation changes to 2.90 m, then the temperature (in K) of the
black body is
(A) 500 (B) 1000 (C) 4000 (D) 8000
Key: (B)
Exp: 1  Tmax 1   2  Tmax 2
1.45  2000  2.90   Tmax 2
  Tmax 2  1000K

5. Metal removal in electric discharge machining takes place through


(A) ion displacement (B) melting and vaporization
(C) corrosive reaction (D) plastic shear
Key: (B)
Exp: In EDM, electric spark is used to melt the metal & vaporization of metal takes place

6. The preferred option for holding an odd-shaped work piece in a centre lathe is
(A) Live and dead centres (B) three jaw chuck
(C) lathe dog (D) four jaw chuck
Key: (D)

7. The arrival of customers over fixed time intervals in a bank follow a Poisson distribution with an
average of 30 customers / hour. The probability that the time between successive customer arrival
is between 1 and 3 minutes is _____ (Correct to two decimal places).
Key: (0.3834)
Exp:   30 / hr
  0.5 / min
 P t   1  et
P 1  1  e0.51  0.3934
P  3  1  e0.53  0.7768
So, P 1  T  3  0.7768  0.3934  0.3834

© All rights reserved by Thinkcell Learning Solutions Pvt. Ltd. No part of this booklet may be reproduced or utilized in any form without the written permission.

Downloaded From : www.EasyEngineering.net


Downloaded From : www.EasyEngineering.net

ME-GATE 2018

8. For an ideal gas with constant properties undergoing a quasi-static process, which one of the
following represents the change of entropy  s  from state 1 to 2?
T  P  T  V 
(A) s  Cp ln  2   R ln  2  (B) s  Cv ln  2   Cp ln  2 
 T1   P1   T1   V1 
T  P  T   V1 
(C) s  Cp ln  2   Cv ln  2  (D) s  Cv ln  2   R ln  V 
 T1   P1   T1   2
Key: (A)
Exp: For an ideal gas undergoing a quasi static process,
T  P 
Change in entropy, s  Cp n  2   R n  2 
 T1   P1 
Proof:- dQ  dU  pdV  CV dT  pdV
dQ dT P
s   CV  dV
T T T
dT R
 CV  dV  PV  RT   'v' is specific volume
T V
On Integration,
T  V 
s2  s1  CV n  2   R n  2  ... i 
 T1   V1 
We know that
P1V1 P2 V2

T1 T2
V2 P1 T2
   ... 2 
V1 P2 T1
Substituting (2) in (1)
T  P T 
s 2  s1  CV n  2   R n  1  2 
T
 1  P2 T1 
T  P  T 
 CV n  2   R n  1   R n  2 
 T1   P2   T1 
T  P 
 n  2   Cv  R   R  1 
 T1   P2 
T  P   Cp  C v  R 
 s 2  s1  Cp n  2   R n  2   
 T1   P1  Cp  Cv  R 

© All rights reserved by Thinkcell Learning Solutions Pvt. Ltd. No part of this booklet may be reproduced or utilized in any form without the written permission.

Downloaded From : www.EasyEngineering.net


Downloaded From : www.EasyEngineering.net

ME-GATE 2018

9. In a single degree of freedom under damped spring-mass-damper system


as shown in the figure, an additional damper is added in parallel such that M
the system still remains underdamped. Which one of the following
statements is ALWAYS true?
(A) Transmissibility will increase K C
(B) Transmissibility will decrease
(C) Time period of free oscillations will increase.
(D) Time period of free oscillations will decrease.
Key: (C)
Exp: d  1  2
If additionaldamper used, then  increases and d decrease
2
Td 
d
so, Td increases

10. The divergence of the vector field u  ex  cos y ˆi  sin y ˆj is


(B) e cos y  e sin y
x x x x
(A) 0 (C) 2e cos y (D) 2e sin y
Key: (C)
Exp: We have; Div u  .u
  ˆ  ˆ   x
  ˆi  j  k  . e cos yiˆ  e x sin yjˆ 
  x y z 
 x 
 e cos y   e x sin y 
x y
 e x cos y  e x cos y  2e x cos y
 Div u  2e x cos y
11. Match the following products with the suitable manufacturing process
Product Manufacturing Process
P Toothpaste tube 1 Centrifugal casting
Q Metallic pipes 2 Blow moulding
R Plastic bottles 3 Rolling
S Threaded bolts 4 Impact extrusion

(A) P-4, Q-3, R-1, S-2 (B) P-2, Q-1, R-3, S-4
(C) P-4, Q-1, R-2, S-3 (D) P-1, Q-3, R-4, S-2
Key: (C)

12. A hollow circular shaft of inner radius 10mm, outer radius 20mm and length 1m is to be used as a
torsional spring. If the shear modulus of the material of the shaft is 150 GPa, the torsional stiffness
of the shaft (in kN-m/rad) is _______ (correct to two decimal places).

© All rights reserved by Thinkcell Learning Solutions Pvt. Ltd. No part of this booklet may be reproduced or utilized in any form without the written permission.

Downloaded From : www.EasyEngineering.net


Downloaded From : www.EasyEngineering.net

ME-GATE 2018

Key: (35.34)
Exp: Ri =10mm. Di=20mm
R0=20mm, D0=40mm
L=1mts =1000mm
G=150GPa

150  103   404  204 
T GJ 32
 
 1000 
 35.34 kN  m / rad
dy
13. If y is the solution of the differential equation y3  x 3  0, y  0   1, the value of y(-1) is
dx
(A) -2 (B) -1 (C) 0 (D) 1
Key: (C)
Exp: Given D.E
dy
y3  x3  0
dx
 y3dy  x 3dx  0  var iable  separable D.E
  y3dy   x 3dx  c
y4 x 4
   c  1
4 4
Given y  0   1; i.e., y  1 at x  0
1
From 1 ; c 
4
x 4 y4 1
 From 1 ;  
4 4 4
 x 4  y4  1
 y4  1  x 4
 y  4 1  x4
 y  1  4 1   1  0
4

14. An engine operates on the reversible cycle 650


as shown in the figure. p
The work output from the engine (in kJ/cycle)  kPa 
is ________ (correct to two decimal places).
400

2 2.5
V  m3 

© All rights reserved by Thinkcell Learning Solutions Pvt. Ltd. No part of this booklet may be reproduced or utilized in any form without the written permission.

10

Downloaded From : www.EasyEngineering.net


Downloaded From : www.EasyEngineering.net

ME-GATE 2018

Key: (62.5)
1
Exp: Work done = Area   0.50  250 650
2
 62.5kN  m

400

m3
2 2.5

15. Pre-tensioning of a bolted joint is used to


(A) strain harden the bolt head (B) decrease stiffness of the bolted joint
(C) increase stiffness of the bolted joint (D) prevent yielding of the thread root
Key: (C)
Exp: Pre-tensioning increases the stiffness of the bolts.

16. A ball is dropped from rest from a height of 1 m in a frictionless tube as shown in the figure. If the
tube profile is approximated by two straight lines (ignoring the curved portion), the total distance
travelled (in m) by the ball is ______ (correct to two decimal places).

1.0 m

45

Key: (2.414)
Exp: Ball will start moving from point B to point A & then reach at point C.
Since ignoring friction,
i.e, potential energy at point B and C will be same.

B C
 
B C

1.0 m
1m 1m

45
A D A

1
 1  AC, AC 
sin 45
 1  2  2.414mts

© All rights reserved by Thinkcell Learning Solutions Pvt. Ltd. No part of this booklet may be reproduced or utilized in any form without the written permission.

11

Downloaded From : www.EasyEngineering.net


Downloaded From : www.EasyEngineering.net

ME-GATE 2018

17. The viscous laminar flow of air over a flat plate results in the formation of a boundary layer. The
boundary layer thickness at the end of the plate of length L is  L . When the plate length is
increased to twice its original length. The percentage change in laminar boundary layer thickness
at the end of the plate (with respect to L ) is _______ (correct to two decimal places).
Key: (41.42)
Exp: For laminar boundary layer
Where   x
 =Boundary Layer thickness of location „x‟ from leading edge
2 2L
So, 
1 L
2  1.41421
2  1
So, % change   100
1
1.41421  1
  100  41.42%
1

18. The minimum axial compressive load, P, required to initiate buckling for a pinned-pinned slender
column with bending stiffness EI and length L is
2 EI 2 EI 32 EI 42 EI
(A) P  (B) P  (C) P  (D) P 
4L2 L2 4L2 L2
Key: (B)
2 EI
Exp: PE 
L2e
for both ends pinned,
Le  L
2 EI
PE 
L2

19. Consider a function u which depends on position x and time t. The partial differential equation
u  2 u

t x 2
is known as the
(A) Wave equation (B) Heat equation
(B) Laplace‟s equation (D) Elasticity equation
Key: (B)
du  2 u
Exp: Clearly;  is known as the heat equation
dt x 2

© All rights reserved by Thinkcell Learning Solutions Pvt. Ltd. No part of this booklet may be reproduced or utilized in any form without the written permission.

12

Downloaded From : www.EasyEngineering.net


Downloaded From : www.EasyEngineering.net

ME-GATE 2018

20. Feed rate in slab milling operation is equal to


(A) rotation per minute (rpm)
(B) product of rpm and number of teeth in the cutter
(C) product of rpm, feed per tooth and number of teeth in the cutter
(D) product of rpm, feed per tooth and number of teeth in contact
Key: (C)
Exp: feed rate in milling,
fm  ft  Z  N
f t  feed per tooth
Z  No. of teeth
N  rpm

21. Denoting L as liquid and M as solid in a phase-diagram with the subscripts representing different
phases, a eutectoid reaction is described by
(A) M1  M 2  M3 (B) L1  M1  M2
(C) L1  M1  M2 (D) M1  M 2  M3
Key: (A)
Exp: At Eutectoid point one solid converts into the another solid
M1  M 2  M3
M = Solid
i.e.     Fe3C

1 2 3
22. If A  0 4 5  then det(A-1) is _______ (correct to two decimal places).
0 0 1 
Key: (0.25)
Exp: Given,
1 2 3
A  0 4 5
0 0 1
Clearly; A is upper triangular matrix
1
det  A1   A1  ( From the properties of determinant)
A
1 1
 A 1    0.25
1 4  1 4
( The determinant of upper triangular matrix is the product of diagonal elements).

© All rights reserved by Thinkcell Learning Solutions Pvt. Ltd. No part of this booklet may be reproduced or utilized in any form without the written permission.

13

Downloaded From : www.EasyEngineering.net


Downloaded From : www.EasyEngineering.net

ME-GATE 2018

23. A local tyre distributor expects to sell approximately 9600 steel belted radial tyres next year.
Annual carrying cost is Rs. 16 per tyre and ordering cost is Rs. 75. The economic order quantity
of the tyres is
(A) 64 (B) 212 (C) 300 (D) 1200
Key: (C)
Exp: D = 9600units/year
C = Rs.16/unit/year
F = Rs. 75/order
2DF
EOQ 
C
2  9600  75

16
 300 units

24. Fatigue life of a material for a fully reversed loading condition is estimated from
a  1100N 0.15
Where a is the stress amplitude in MPa and N is the failure life in cycles. The maximum
allowable stress amplitude (in MPa) for a life of 1×105cycles under the same loading condition is
______ (correct to two decimal places).
Key: (195.61)
Exp: For completely reversed loading,
max  a
a  1100N 0.15
a  1100  1 105 
0.15

 195.61MPa

25. A frictionless gear train is shown in the figure. The leftmost 12-teeth gear is given a torque of
100N-m. The output torque from the 60-teeth gear on the right in N-m is
48 Teeth
12 Teeth
T  100 N  m

60 Teeth
12 Teeth
(A) 5 (B) 20 (C) 500 (D) 2000

© All rights reserved by Thinkcell Learning Solutions Pvt. Ltd. No part of this booklet may be reproduced or utilized in any form without the written permission.

14

Downloaded From : www.EasyEngineering.net


Downloaded From : www.EasyEngineering.net

ME-GATE 2018

Key: (D)
T1 T2
Exp: 
Z1 Z 2
100  48
T2   400N  m
12
T2  T3  400N  m
T3 T4

Z3 Z4
400 T4

12 60
400  60
T4   2000N  m
12

26. A circular hole of 25 mm diameter and depth of 20mm is machined by EDM process. The material
removal rate (in mm3/min) is expressed as
4×104IT-1.23
Where I=300 A and the melting point of the material, T=1600°C. The time (in minutes) for
machining this hole is _________ (correct to two decimal places)
Key: (7.143)
Exp: MRR  4  104 IT –1.23
 4  104  300  16001.23
 1374.4mm3 / min
 2
Volume to be removed  DL
4

 252  20  9817.4mm3

4
9817.4
Time required   7.143min
1374.4

27. Following data correspond to an orthogonal turning of a 100mm diameter rod on a lathe. Rake
angle: +15°; Uncut chip thickness: 0.5 mm; nominal chip thickness after the cut:1.25 mm. The
shear angle (in degrees) for this process is ________ (correct to two decimal places).
Key: (23.31)
Exp: d=100mm
  15, t1  0.5mm
t c  1.25mm
t1
r  0.4
tc

© All rights reserved by Thinkcell Learning Solutions Pvt. Ltd. No part of this booklet may be reproduced or utilized in any form without the written permission.

15

Downloaded From : www.EasyEngineering.net


Downloaded From : www.EasyEngineering.net

ME-GATE 2018

r cos  0.4  cos15


tan    tan  
1  r sin  1  0.4  sin15
tan   0.4309   23.31

28. A rigid rod of length 1 m is resting at an angle   45 as shown in the figure. The end P is
dragged with a velocity of U=5 m/s to the right. At the instant shown, the magnitude of the
velocity V (in m/s) of point Q as it moves along the wall without losing contact is
y

U  5m / s
  45 P
x

(A) 5 (B) 6 (C) 8 (D) 10


Key: (A)
Exp: VP   IP  
Q cos 45
VQ   IQ   IP  IQ  cos 45 I

VP  VQ  5m/sec
sin 45

U  5m / s
45
P

29. The true stress (in MPa) versus true stain relationship for a metal is given by
  10200.4
The cross-sectional area at the start of a test (when the stress and strain values are equal to zero) is
100mm2. The cross – sectional area at the time of necking (in mm2) is _________ (correct to two
decimal places)
Key: (67.032)
Exp:   1020 0.4
  K n Ai  100mm 2
at necking n  0.4
   Ai 
 n    n A 
f

 i   f
A  100
0.4  n  i   e0.4 
 Af  Af
100
Af  0.4
 A f  67.032mm2
e

© All rights reserved by Thinkcell Learning Solutions Pvt. Ltd. No part of this booklet may be reproduced or utilized in any form without the written permission.

16

Downloaded From : www.EasyEngineering.net


Downloaded From : www.EasyEngineering.net

ME-GATE 2018

30. A bar of circular cross section is clamped at ends P and Q as shown in the figure. A torsional
moment T=150 Nm is applied at a distance of 100mm from end P. The torsional reactions (T P,TQ)
in Nm at the ends P and Q respectively are
P T
Q

100 200

(All dimensions are in mm)

(A) (50, 100) (B) (75, 75) (C) (100, 50) (D) (120, 30)
Key: (C)
Exp: T  150N  m
T
TP  TQ  150N  m
1  2
TP 1 TQ 2 1  2

GJ GJ
100 200
TP 100   TQ  200 
TP  2TQ  TQ  0.5TP
TP  TQ  150
TP  0.5TP  150
TP  100N  m, TQ  50N  m
 T ,T   100,50
P Q

31. Air flows at the rate of 1.5m3/s through a horizontal pipe with a gradually reducing cross-section
as shown in the figure. The two cross-sections of the pipe have diameters of 400mm and 200 mm.
Take the air density as 1.2 kg/m3 and assume inviscid incompressible flow. The change in
pressure (P2-P1) (in kPa) between sections 1 and 2 is
200mm
(1)
Air Flow 400mm (2)

1.5 m3 / s

(A) –1.28 (B) 2.56 (C) -2.13 (D) 1.28

© All rights reserved by Thinkcell Learning Solutions Pvt. Ltd. No part of this booklet may be reproduced or utilized in any form without the written permission.

17

Downloaded From : www.EasyEngineering.net


Downloaded From : www.EasyEngineering.net

ME-GATE 2018

Key: (A)
Exp: Q=1.5m2/s
Apply Bernoulli‟s equation b/w (1) & (2)
P1 V12 P V2 Q 1.5  4
  Z1  2  2  Z2 V ; V1   11.936m / s
 2g  2g   0.4 
2
A1
P1  P2 V22  V12 1.5  4
 V2   47.746m / s
g    0.2 
2
2g
 47.7462  11.9362 
So, P1  P2  1.2   
 2 
P1  P2  1.28kPa
or P2  P1  1.28kPa

32. A frictionless circular piston of area 10-2 m2 and mass g  10m / s 2


patm
100kg sinks into a cylindrical container of the same area
filled with water of density 1000 kg/m3 as shown in the 100kg
figure. The container has a hole of area 10-3 m2 at the
bottom that is open to the atmosphere. Assuming there is
no leakage from the edges of the piston and considering 0.5m

water to be incompressible, the magnitude of the piston


velocity (in m/s) at the instant shown is ______ (correct
to three decimal places).
Key: (1.456)
Exp: A1V1  A 2 V2
102 V1  103 V2
V2  10V1
1
100  10
P1  2
N / m2
10
P2  0 0.5m
Apply Bernoulli‟s equation between (1) & (2)
P1 V12 P V2 2
  Z1  2  2  Z2
 2g  2g
105 V12 100V12
  0.5  0  0
103  10 2  10 2  10
99V12
 10  0.5
20
V1  1.456m/s

33. A 0.2 m thick infinite black plate having a thermal conductivity of 3.96 W/m-K is exposed to two
infinite black surfaces at 300 K and 400 K as shown in the figure. At steady state, the surface

© All rights reserved by Thinkcell Learning Solutions Pvt. Ltd. No part of this booklet may be reproduced or utilized in any form without the written permission.

18

Downloaded From : www.EasyEngineering.net


Downloaded From : www.EasyEngineering.net

ME-GATE 2018

temperature of the plate facing the cold side is 350K. The value of Stefan-Boltzmann constant,  ,
is 5.67×10-8 W/m2 K4. Assuming 1-D heat conduction, the magnitude of heat flux through the
plate (in W/m2) is _______ (correct to two decimal places).
0.2m

400K
vacuum vacuum
300K

0.2m

Key: (391.58)
Exp: 1-D study state & Black Surfaces    1
Heat flux(q)    T14  T24 
Vacuum Vacuum

 5.67  108 3504  3004 
 391.58 W
m2
T2  300K T1  350K 400K

34. A steel wire is drawn from an initial diameter (di) of 10 mm to a final diameter (df) of 7.5mm. The
half cone angle    of the die is 5° and the coefficient of friction    between the die and the
wire is 0.1. The average of the initial and final yield stress   Y  avg  is 350 MPa. The equation
for drawing stress f ,(in MPa) is given as:

1    df  
2  cot 

f   Y avg 1   1    
  cot     d i  
The drawing stress (in MPa) required to carry out this operation is _______ (correct to two
decimal places).
Key: (316.24)
Exp: di  10mm,   5, y  350MPa
d f  7.5mm   0.1
1    dt  
2  cot 

f    y  1   1    
avg
  cot     d i  
   7.5  
20.1cot5
 1
 350 1   1      316.24MPa
 0.1cot 5    10  

© All rights reserved by Thinkcell Learning Solutions Pvt. Ltd. No part of this booklet may be reproduced or utilized in any form without the written permission.

19

Downloaded From : www.EasyEngineering.net


Downloaded From : www.EasyEngineering.net

ME-GATE 2018

35. The arc lengths of a directed graph of a project are as shown in the figure. The shortest path length
from node 1 to node 6 is ____________
4
2 4
2
2
2
1 1 1 6

4 4
3
Key: (7) 3 5
Exp: Shortest Path is
(1) – (2) – (5) – (4) – (6)
& length of shortest path = 2+2+1+2 = 7

36. The problem of maximizing z=x1-x2 subject to constraints x1  x 2  10, x1  0, x 2  0 and


x 2  5 has
(A) no solution (B) one solution
(C) two solutions (D) more than two solutions
Key: (B)
Exp: slope of objective function is not equal to slope of any one constraint so, unique optional
solution

37. Ambient air is at a pressure of 100kPa, dry bulb temperature of 30°C and 60% relative humidity.
The saturation pressure of water at 300 C is 4.24 kPa. The specific humidity of air (in g/kg of dry
air) is ___________ (correct to two decimal places).
Key: (16.24)
Exp: Pt  100kPa,   60%
t db  30C, Pvs  4.424kPa
Pv
  Pv   Pvs
Pvs
 0.6  4.24  2.544kPa
Pv
Specific humidity  w   0.622
Pt  Pv
2.544
 0.622 
100  2.544
 0.016236 kgw.v
kgd.a
 16.24 gmw.v
kgd.a

© All rights reserved by Thinkcell Learning Solutions Pvt. Ltd. No part of this booklet may be reproduced or utilized in any form without the written permission.

20

Downloaded From : www.EasyEngineering.net


Downloaded From : www.EasyEngineering.net

ME-GATE 2018

38. A standard vapor compression refrigeration cycle operating with a condensing temperature of
35°C and an evaporating temperature of -10°C develops 15 kW of cooling. The p-h diagram
shows the enthalpies at various states. If the isentropic efficiency of the compressor is 0.75, the
magnitude of compressor power (in kW) is _____ (correct to two decimal places).

35C
650

p
 kPa 
10C
400

250 400 475


h  kJ / kg 

Key: (10)
Exp: R .E  15k W h1  400kJ / kg
isent C  0.75 h 2  475kJ / kg
h 4  250kJ / kg 3 35C 2
650
h1  h 4 400  250 150
C.O.P    2 p
h 2  h1 475  400 75  kPa 
R.E 10C
But C.O.P  400 1
Win 4

15
Win   7.5kW 250 400 475
2
Isotropic efficiency of compressor is given, actual compressor power required
W 7.5
 in   10kW
isent C 0.75

39. For sand-casting a steel rectangular plate with dimensions 80mm×120mm×20mm, a cylindrical
riser has to be designed. The height of the riser is equal to its diameter. The total solidification
time for the casting is 2 minutes. In Chvorinov‟s law for the estimation of the total solidification
time, exponent is to be taken as 2. For a solidification time of 3 minutes in the riser, the diameter
(in mm) of the riser is _______ (correct to two decimal places).
Key: (51.84)

Exp: Given, t c  2min A r  d 2  d 2
2
3
t r  3min A r  d 2
2

© All rights reserved by Thinkcell Learning Solutions Pvt. Ltd. No part of this booklet may be reproduced or utilized in any form without the written permission.

21

Downloaded From : www.EasyEngineering.net


Downloaded From : www.EasyEngineering.net

ME-GATE 2018

Vc  80  120  20  192000mm3
 3
Vr  d
4
Ac  2  80 120   120  20    20  80    27200mm 2
2
t c  Vc A r 
 
t r  A c Vr 
2
 3 2
d 
2  192000 2
 
3  27200  3 
d
 4 
2
2  42.352 
  d  51.84mm
3  d 

40. A welding operation is being performed with voltage =30V and current =100A. The cross
sectional area of the weld bead is 20mm2. The work-piece and filler are of titanium for which the
specific energy of melting is 14 J/mm3. Assuming a thermal efficiency of the welding process
70% the welding speed (in mm/s) is ______ (correct to two decimal places).
Key: (7.5)
Exp: Hm .A.V  .V.I
14×20×V=0.7×30×100
V=7.5m/s

41. For a position vector r  xiˆ  yjˆ  zkˆ the norm of the vector can be defined as | r | x 2  y2  z2 .
Given a function   ln | r |, its gradient  is
r r r
(A) r (B) (C) (D)
|r| r.r | r |3
Key: (C)
Exp: Given r  xiˆ  yjˆ  zkˆ

| r | x 2  y 2  z 2
Also given;   n | r |
 n  x 2  y 2  z 2 

n  x 2  y2  z 2 
1

2
gradient of   
 1 
 î  n  x 2  y2  z 2 
x  2 

© All rights reserved by Thinkcell Learning Solutions Pvt. Ltd. No part of this booklet may be reproduced or utilized in any form without the written permission.

22

Downloaded From : www.EasyEngineering.net


Downloaded From : www.EasyEngineering.net

ME-GATE 2018

1 1  
 î  2 2x
2  x  y2  z 2 
 x 
 î  2 2 
x  y  z 
2

 x  ˆ y  ˆ z 
 ˆi  2 2 
 j 2 2
k 2 2
x  y  z  x  y  z  x  y  z 
2 2 2

xiˆ  yjˆ  zkˆ r


 2 
x y z 2 2
r.r

42. A bar is subjected to a combination of a steady load of 60kN and a load fluctuating between -
10kN and 90 kN. The corrected endurance limit of the bar is 150 MPa, the yield strength of the
material is 480 MPa and the ultimate strength of the material is 600MPa. The bar cross-section is
square with side a. If the factor of safety is 2, the value of a (in mm), according to the modified
Goodman‟s criterion, is _____ (correct to two decimal places).
Key: (31.62)
Exp: Pstatic  60kN, yt  480MPa
Pmax  90kN, ut  600MPa
Pmin  10kN, e  150MPa,F.S  2
Considering Static varying load
Pmax  90  60  150kN
Pmin  60  10  50kN
150  50
Pmean   100kN
2
150  50
PV   50kN
2
P
mean  mean
a2
P
 v  V2
a
Good man equation
mean  v 1
 
ut e F.S
100  103 50 103
a2 2 1
  a 
600 150 2
2
a =1000
a=31.62mm

© All rights reserved by Thinkcell Learning Solutions Pvt. Ltd. No part of this booklet may be reproduced or utilized in any form without the written permission.

23

Downloaded From : www.EasyEngineering.net


Downloaded From : www.EasyEngineering.net

ME-GATE 2018

43. A bimetallic cylindrical bar of cross sectional area 1 m2 is made by bonding steel (Young‟s
modulus =210 GPa) and Aluminium (Young‟s modulus =70GPa) as shown in the figure. To
maintain tensile axial strain of magnitude 10-6 in Steel bar and compressive axial strain of
magnitude 10-6 in Aluminium bar, the magnitude of the required force P (in kN) along the
indicated direction is
L/2 L/2

clamped end P clamped end


Steel Aluminium
perfectly bonded interface

(A) 70 (B) 140 (C) 210 (D) 280


Key: (D)
Exp: al st  106
st al
Pst Pal

Ast .Est A al .E al
210
Pst  Pal  Pst  3Pal
70
st  3al
st st st
 210  103 106  0.21MPa
al  70  103  106  0.07
P   st  al  A
  0.21  0.07  106  280kN

44. A vehicle powered by a spark ignition engine follows air standard Otto cycle    1.4 . The
engine generates 70kW while consuming 10.3kg/hr of fuel. The calorific value of fuel is
44,000kJ/kg. The compression ratio is _____ (correct to two decimal places).
Key: (7.61)
Exp: γ = 1.4, W=70kW
mf = 10.3kg/h
C.V=44000KJ/kg
rc =?
For Otto cycle
1
 1
 rc 
1

© All rights reserved by Thinkcell Learning Solutions Pvt. Ltd. No part of this booklet may be reproduced or utilized in any form without the written permission.

24

Downloaded From : www.EasyEngineering.net


Downloaded From : www.EasyEngineering.net

ME-GATE 2018

W.D
But  
H.S
W.D 1
 1
 rc 
1
H.S
70 1
1
 rc 
1.4 1
10.3
 44000
3600
rc  7.61

45. Steam in the condenser of a thermal power plant is to be condensed at a temperature of 300 C with
cooling water which enters the tubes of the condenser at 14°C and exits at 22°C. The total surface
area of the tubes is 50 m2, and the overall heat transfer coefficient is 2000 W/m2 K. The heat
transfer (in MW) to the condenser is ______ (correct to two decimal places).
Key: (1.15) T
Exp: t h1  t h2  30C

t c2  22C

t c1  14C
L

1  t h1  t c1  30  14  16C A  50m
2

  t  t  30  22  8C U  2000 W
2 h2 c2
m2 K
1  2
LMTD  m  
 
n 1 
 2 
16  8
  11.54C
 16 
n 
8
 Heat Transfer  Q   UAm
 2000  50  11.54  1154156W  1.15MW

46. Air is held inside a non insulated cylinder using a piston (mass M=25kg and area A=100 cm2) and
stoppers (of negligible area), as shown in the figure. The initial pressure Pi and temperature Ti of
air inside the cylinder are 200kPa and 400°C, respectively. The ambient pressure P and
temperature T are 100kPa and 27°C, respectively. The temperature of the air inside the cylinder
(°C) at which the piston will begin to move is ________ (correct to two decimal places).

© All rights reserved by Thinkcell Learning Solutions Pvt. Ltd. No part of this booklet may be reproduced or utilized in any form without the written permission.

25

Downloaded From : www.EasyEngineering.net


Downloaded From : www.EasyEngineering.net

ME-GATE 2018

P  100kPa
T  27C

PISTON M  25kg
A  100cm2
AIR
g  10m / s 2
Pi  200kPa
Ti  400C

Key: (147.63)
Exp: P  100kPa
T  27C

Piston M  25kg
A  100cm 2
stops
Air
g  10m / s 2
Pi  200kPa
Ti  400C

Total pressure on the piston = P  pressure due to weight of piston


 mg 
 P   
 A 
 25  10  103 
 100     125kPa
 100  104 
Internal pressure of air =200kPa
The pressure at which the piston can move is 125kPa
Let T2 the temperature corresponding to P2=125KPa
PV P V
 From 1 1  2 2
T1 T2
200 125
  volume of air constant 
673 T2
T2  420.625K
 147.625C  147.63C

47. In a cam-follower, the follower rises by h as the cam rotates by  (radians) at constant angular
velocity  (radians/s). The follower is uniformly accelerating during the first half of the rise

© All rights reserved by Thinkcell Learning Solutions Pvt. Ltd. No part of this booklet may be reproduced or utilized in any form without the written permission.

26

Downloaded From : www.EasyEngineering.net


Downloaded From : www.EasyEngineering.net

ME-GATE 2018

period and it is uniformly decelerating in the latter half of the rise period. Assuming that the
magnitudes of the acceleration and deceleration are same, the maximum velocity of the follower is
4h 2h
(A) (B) h (C) (D) 2h
 
Key: (C)
Exp: S=h,   
 
t 
 

V  u  at  V  a  h
2
h h t 
1  
S  ut  at 2 2 2 2 2
2
2
h 1 t
 a  
2 2 2
2
h 1   
 a  
2 2  2 
42 h
a
2
 42 h 
V a  2 
2  2
2h
V 

48. A thin-walled cylindrical can with rigid end caps has a mean radius R=100 mm and a wall
thickness of t=5 mm. The can is pressurized and an additional tensile stress of 50MPa is imposed
along the axial direction as shown in the figure. Assume that the state of stress in the wall is
uniform along its length. If the magnitudes of axial and circumferential components of stress in
the can are equal, the pressure (in MPa) inside the can is __________(correct to two decimal
places).

end cap t end cap

Pr essurized can
50MPa 50MPa
R

Key: (5)
Exp: t=5mm
R=100m  D  200mm
Additional   50MPa

© All rights reserved by Thinkcell Learning Solutions Pvt. Ltd. No part of this booklet may be reproduced or utilized in any form without the written permission.

27

Downloaded From : www.EasyEngineering.net


Downloaded From : www.EasyEngineering.net

ME-GATE 2018

 t  L  50
PD PD
  50
2t 4t
PD
 50
4t
50  4  5
P  5MPa
200

49. In a rigid body in plane motion, the point R is accelerating with respect to point P at 10180
m/s2. If the instantaneous acceleration of point Q is zero, the acceleration (in m/s2) of point R is

20
y

16 R
Q x

(A) 8233 (B) 10255 (C) 10217 (D) 8217


Key: (D)
Exp: a PR  a R  a P , PR  20
10   PR   2

10 1
2  
20 2
a QR   QR  2

2
 1 
 16     8m / sec
2

 2
120
PQR  sin  
20
1
 
  sin 0.6  36.86.
180  36.86  216.86  217
8217

50. A force of 100N is applied to the centre of a circular disc, of


mass 10 kg and radius 1m, resting on a floor as shown in the 100N
figure. If the disc rolls without slipping on the floor, the linear
acceleration (in m/s2) of the centre of the disc is ______
(correct to two decimal places).

© All rights reserved by Thinkcell Learning Solutions Pvt. Ltd. No part of this booklet may be reproduced or utilized in any form without the written permission.

28

Downloaded From : www.EasyEngineering.net


Downloaded From : www.EasyEngineering.net

ME-GATE 2018

Key: (6.66)
Exp: r =1m
Fx  ma  100  F   ma
100  F   10  a
100  10a   F 100N
Torque, I  F  r
1 2   
 2 mr    100  10a  1
1 F
2

 2  10  1    100  10a

5  100  10  15  100


  6.66rad / sec2
a  r  6.66 1  6.66m / sec2

51. A test is conducted on a one-fifth scale model of a Francis turbine under a head of 2m and
volumetric flow rate of 1m3/s at 450 rpm. Take the water density and the acceleration due to
gravity as 103 kg/m3 and 10 m/s2, respectively. Assume no losses both in model and prototype
turbines. The power (in MW) of a full sized turbine while working under a head of 30 m is
___________ (correct to two decimal places).
Key: (29.025)
D1 Dm 1
Exp:  
D2 Dp 5
(1)  2
H1  2m H2  30m
Q1  1m /s, N1  450rpm P2  ?
3

H  N 2 D2
2 2
H 2  N 2   D2 
 
H1  N1   D1 
2
30  N 2  N
   52  2  0.774
2  N1  N1
3
Q2  N 2  D 2 

Q1  N1  
 D1 
Q2
 0.774  53
1
Q2  96.75  m3 / s
P2  gQ2 H2  1000 10  96.75  30  29.025MW

© All rights reserved by Thinkcell Learning Solutions Pvt. Ltd. No part of this booklet may be reproduced or utilized in any form without the written permission.

29

Downloaded From : www.EasyEngineering.net


Downloaded From : www.EasyEngineering.net

ME-GATE 2018

52. Given the ordinary differential equation


d2 y dy
  6y  0
dx 2 dx
dy
With y  0   0 and  0   1, the value of y(1) is _____ (correct to two decimal places).
dx
Key: (1.47)
Exp: Given D.E
d2 y dy
  6y  0
dx 2 dx
  D2  D  6  y  0
The Auxiliary equation is m 2  m  6  0
 m 2  3m  2m  6  0
 m  m  3  2  m  3  0
  m  3  m  2   0
 m  2, 3.  Roots are real and distinct 
The solution is
y  C1e 3x  C2 e 2x ...  i 
given y  0   0
from  i   0  C1  C2 ...  ii 
dy
from  i    3C1e3x  2C2 e2x
dx
dy
  1 at x  0
dx
1  3C1  2C2   iii 
 1  3C1  2  C1   From  ii  
 C1   1  C2  1
5 5  C2  C1 
1 3x 1 2x
 The required solution is y  e  e
5 5
1 3 1 2
 y 1  e  e
5 5
 y 1   e2  e3 
1
5
 
 y 1  1.47

© All rights reserved by Thinkcell Learning Solutions Pvt. Ltd. No part of this booklet may be reproduced or utilized in any form without the written permission.

30

Downloaded From : www.EasyEngineering.net


Downloaded From : www.EasyEngineering.net

ME-GATE 2018

53. Let z be a complex variable. For a counter-clockwise integration around a unit circle C centred at
origin,
1
C 5z  4 dz  Ai,
the value of A is
(A) 2/5 (B) 1/2 (C) 2 (D) 4/5
Key: (A)
Exp: Singular point is z = 4/5, which lies inside the unit circle C: |z|=1
By Cauchy‟s integral formula:
1 1 1
C 5z  4 dz  C 5 z  4 dz  5  2i 1
5  
2
 i ... i 
5
1
But given  dz  Ai
C 5z  4

2
 i  Ai  from  i  
5
A2
5

54. Let X1 and X2 be two independent exponentially distributed random variables with means 0.5 and
0.25, respectively. Then Y=min (X1, X2) is
(A) exponentially distributed with mean 1/6
(B) exponentially distributed with mean 2
(C) normally distributed with mean 3/4
(D) normally distributed with mean 1/6
Key: (A)
Exp: We know that, if X1 and X2 are independent and exponential R. V‟s with parameters 1 and 2
then X = min(X1, X2) is exponential R.V with parameter   1   2 .

Given E  X1   0.5 E  X 2   0.25


1 1
 parameter of X1   X1    X2  4
0.5 0.25
  X1  2
 parameter of Y   X1   X2  2  4  6
1
Mean of Y  E  Y  
6

© All rights reserved by Thinkcell Learning Solutions Pvt. Ltd. No part of this booklet may be reproduced or utilized in any form without the written permission.

31

Downloaded From : www.EasyEngineering.net


Downloaded From : www.EasyEngineering.net

ME-GATE 2018

55. Taylor‟s tool life equation is used to estimate the life of a batch of identical HSS twist drills by
drilling through holes at constant feed in 20mm thick mild steel plates. In test 1, a drill lasted 300
holes at 150 rpm while in test 2, another drill lasted 200 holes at 300 rpm. The maximum number
of holes that can be made by another drill from the above batch at 200 rpm is _______ (correct to
two decimal places).
Key: (254)
(1)  2
Exp: T1  300holes T2  200holes
N1  150rpm N 2  300rpm
At N3=200rpm, T3=?
VT n  C  V1T1n  V2 T2n
n n
N1  T2  150  200 
    
N 2  T1  300  300 
n
2
0.5     0.67n  0.5
3
log 0.67  n  n  1.73
0.5

V1T1n  V3T3n
1/n 1/1.73
 T3   N1   T3   150 
 T    N    300    200 
 1  3  
T3  254holes

© All rights reserved by Thinkcell Learning Solutions Pvt. Ltd. No part of this booklet may be reproduced or utilized in any form without the written permission.

32

Downloaded From : www.EasyEngineering.net


Downloaded From : www.EasyEngineering.net

ME-GATE 2018

© All rights reserved by Thinkcell Learning Solutions Pvt. Ltd. No part of this booklet may be reproduced or utilized in any form without the written permission.

33

Downloaded From : www.EasyEngineering.net


|ME-2019-Forenoon|

General Aptitude
Q. No. 1 - 5 Carry One Mark Each

1. The minister avoided any mention of the issue of women’s reservation in the private sector. He
was accused of _____ the issue.
(A) belting (B) skirting (C) tying (D) collaring
Key: (B)

2. _____ I permitted him to leave, I wouldn’t have had any problem with him being
absent _____ I?
(A) Had, would (B) Have, wouldn’t
(C) Have, would (D) Had, wouldn’t
Key: (A)

3. A worker noticed that the hour hand on the factory clock had moved by 225 degrees during her
stay at the factory. For how long did she stay in the factory?
(A) 3.75 hours (B) 7.5 hours
(C) 4 hours and 15mins (D) 8.5 hours
Key: (B)

4. John Thomas, an ______ writer, passed away in 2018.


(A) imminent (B) prominent (C) dominant (D) eminent
Key: (D)

5. The sum and product of two integers are 26 and 165 respectively. The difference between these
two integers is ________.
(A) 3 (B) 6 (C) 2 (D) 4
Key: (D)

© All rights reserved by Thinkcell Learning Solutions Pvt. Ltd. No part of this booklet may be reproduced or utilized in any form without the written permission.

1
|ME-2019-Forenoon|

Q. No. 6 - 10 Carry Two Marks Each

6. A person divided an amount of Rs. 100,000 into two parts and invested in two different
schemes. In one he got 10% profit and in the other he got 12%. If the profit percentages are
interchanged with these investments he would have got Rs. 120 less. Find the ratio between his
investments in the two schemes.
(A) 37:63 (B) 9:16 (C) 11:14 (D) 47:53
Key: (D)

7. Under a certain legal system, prisoners are allowed to make one statement. If their statement
turns out to be true then they are hanged. If the statement turns out to be false then they are shot.
One prisoner made a statement and the judge had no option but to set him free. Which one of
the following could be that statement?
(A) I will be shot (B) I committed the crime
(C) I did not commit the crime (D) You committed the crime
Key: (A)

8. A firm hires employees at five different skill levels P, Q, R, S, T. The shares of employment at
these skill levels of total employment in 2010 is given in the pie chart as shown. There were a
total of 600 employees in 2010 and the total employment increased by 15% from 2010 to 2016.
The total employment at skill levels P, Q and R remained unchanged during this period. If the
employment at skill level S increased by 40% from 2010 to 2016, how many employees were
there at skill level T in 2016?
Percentage share of skills in 2010

T
S 5 P
25 20

Q
R
25
25

(A) 30 (B) 72 (C) 35 (D) 60


Key: (D)

© All rights reserved by Thinkcell Learning Solutions Pvt. Ltd. No part of this booklet may be reproduced or utilized in any form without the written permission.

2
|ME-2019-Forenoon|

9. M and N had four children P, Q, R and S. Of them, only P and R were married. They had
children X and Y respectively. If Y is a legitimate child of W, which one of the following
statement is necessarily FALSE?
(A) M is the grandmother of Y (B) W is the wife of R
(C) W is the wife of P (D) R is the father of Y
Key: (B)

10. Congo was named by Europeans. Congo’s dictator Mobuto later changed the name of the
country and the river to Zaire with the objective of Africanising names of persons and spaces.
However, the name Zaire was a Portuguese alteration of Nzadi o Nzere, a local African term
meaning ‘River that swallows Rivers’. Zaire was the Portuguese name for the Congo river in the
16th and 17 centuries. Which one of the following statements can be inferred from the
paragraph above?
(A) The term Nzadi o Nzere was of Portuguese origin
(B) As a dictator Mobuto ordered the Portuguese to alter the name of the river to Zaire
(C) Mobuto’s desire to Africanise names was prevented by the Portuguese
(D) Mobuto was not entirely successful in Africanising the name of his country
Key: (D)

© All rights reserved by Thinkcell Learning Solutions Pvt. Ltd. No part of this booklet may be reproduced or utilized in any form without the written permission.

3
|ME-2019-Forenoon|

Mechanical Engineering
Q. No. 1 to 25 Carry One Mark Each

1. Consider the stress-strain curve for an ideal elastic-plastic strain hardening metal as shown in
the figure. The metal was loaded in uniaxial tension starting from O. Upon loading, the stress-
strain curve passes through initial yield point at P, and then strain hardens to point Q, where the
loading was stopped. From point Q, the specimen was unloaded to point R, where the stress is
zero. If the same specimen is reloaded in tension from point R, the value of stress at which the
material yields again is _______MPa.

210 Q

P
180
Stress (MPa)

O R Strain
Key: (210)

2. The length, width and thickness of a steel sample are 400 mm, 410 mm, 40 mm and 20 mm,
respectively. Its thickness needs to be uniformly reduced by 2 mm in a single pass by using
horizontal slab milling. The milling cutter (diameter: 100 mm, width: 50 mm) has 20 teeth and
rotates at 1200 rpm. The feed per tooth is 0.05 mm. The feed direction is along the length of the
sample. If the over-travel distance is the same as the approach distance, the approach distance
and time taken to complete the required machining task are
(A) 14mm, 21.4 s (B) 21 mm, 39.4 s (C) 21 mm, 28.9s (D) 14mm, 18.4 s
Key: (A)

3. As per common design practice, the three types of hydraulic turbines, in descending order of
flow rate, are
(A) Francis, Kaplan, Pleton (B) Kaplan, Francis, Pelton
(C) Pelton, Kaplan, Francis (D) Pelton, Francis, Kaplan
Key: (B)

© All rights reserved by Thinkcell Learning Solutions Pvt. Ltd. No part of this booklet may be reproduced or utilized in any form without the written permission.

4
|ME-2019-Forenoon|

4. The table presents the demand of a product. By simple three-months moving average method,
the demand-forecast of the product for the month of September is
Month Demand
January 450
February 440
March 460
April 510
May 520
June 495
July 475
August 560

(A) 490 (B) 536.67 (C) 510 (D) 530


Key: (C)

5. The lengths of a large stock of titanium rods follow a normal distribution with a mean    of

440 mm and a standard deviation    of 1 mm. What is the percentage of rods whose lengths

lie between 438 mm and 441 mm?


(A) 86.64% (B) 68.4% (C) 99.75% (D) 81.85%
Key: (D)

6. During a non-flow thermodynamic process (1-2) executed by a perfect gas, the heat interaction
is equal to the work interaction  Q12  W12  when the process is

(A) Isentropic (B) Isothermal (C) Polytropic (D) Adiabatic


Key: (B)


4
7. Evaluation of x 3dx using a 2-equal-segment trapezoidal rule gives a value of __________.
2

Key: (63)

© All rights reserved by Thinkcell Learning Solutions Pvt. Ltd. No part of this booklet may be reproduced or utilized in any form without the written permission.

5
|ME-2019-Forenoon|

8. The natural frequencies corresponding to the spring-mass systems I and II are I and II ,

I
respectively. The ratio is
II
k k

SYSTEM  I

k
SYSTEM  II

1 1
(A) (B) 4 (C) 2 (D)
2 4
Key: (D)

9. A solid cube of side 1 m is kept at a room temperature of 32° C. The coefficient of linear
thermal expansion of the cube material is 1×10–5/°C and the bulk modulus is 200 GPa. If the
cube is constrained all around and heated uniformly to 42°C, then the magnitude of volumetric
(mean) stress (in MPa) induced due to heating is ________.
Key: (60)

10. For a hydro dynamically and thermally fully developed laminar flow through a circular pipe of
constant cross-section, the Nusselt number at constant wall heat flux (Nuq) and that at constant
wall temperature (NuT) are related as

Nu q  Nu T (B) Nu q   Nu T  (C) Nu q  Nu T (D) Nu q  Nu T


2
(A)

Key: (D)

© All rights reserved by Thinkcell Learning Solutions Pvt. Ltd. No part of this booklet may be reproduced or utilized in any form without the written permission.

6
|ME-2019-Forenoon|

11. A flat-faced follower is driven using a circular eccentric cam rotating at a constant angular
velocity . At time t = 0, the vertical position of the follower is y(0) = 0, and the system is in
the configuration shown below

yt

The vertical position of the follower face, y(t) is given by


(A) e 1  cos 2t  (B) esin t (C) esin 2t (D) e 1  cos t 

Key: (D)

12. In a casting process, a vertical channel through which molten metal and flows downward from
pouring basin to runner for reaching the mold cavity is called
(A) sprue (B) pin hole (C) riser (D) blister
Key: (A)

13. Air of mass 1 kg, initially at 300K and 10 bar, is allowed to expand isothermally till it reaches a
pressure of 1 bar. Assuming air as an ideal gas with gas constant of 0.287 kJ/kg.K, the change in
entropy of air (in kJ/kg.K, round off to two decimal places) is __________.
Key: (0.66)

© All rights reserved by Thinkcell Learning Solutions Pvt. Ltd. No part of this booklet may be reproduced or utilized in any form without the written permission.

7
|ME-2019-Forenoon|

14. A block of mass 10 kg rests on a horizontal floor. The acceleration due to gravity is 9.81 m/s 2.
The coefficient of static friction between the floor and the block is 0.2.

10N
10kg

A horizontal force of 10 N is applied on the block as shown in the figure. The magnitude of
force of friction (in N) on the block is _____.
Key: (10)

1 1 0 
15. Consider the matrix P   0 1 1 
 0 0 1 

The number of distinct eigenvalues of P is


(A) 0 (B) 1 (C) 3 (D) 2
Key: (B)

16. During a high cycle fatigue test, a metallic specimen is subjected to cyclic loading with a mean
stress of +140 MPa, and a minimum stress of –70 MPa. The R-ratio (minimum stress to
maximum stress) for this cycle loading is _____ (round off to one decimal place).
Key: (0.2)

17. A slender rod of length L, diameter d (L >> d) and thermal conductivity k1 is joined with
another rod of identical dimensions, but of thermal conductivity k2, to form a composite
cylindrical rod of length 2L. The heat transfer in radial direction and contact resistance are
negligible. The effective thermal conductivity of the composite rod is
2k1k 2 k1k 2
(A) k1  k 2 (B) k1 k 2 (C) (D)
k1  k 2 k1  k 2
Key: (C)

© All rights reserved by Thinkcell Learning Solutions Pvt. Ltd. No part of this booklet may be reproduced or utilized in any form without the written permission.

8
|ME-2019-Forenoon|

18. Consider an ideal vapor compression refrigeration cycle. If the throttling process is replaced by
an isentropic expansion process, keeping all the other processes unchanged, which one of the
following statements is true for the modified cycle?
(A) Coefficient of performance is the same as that of the original cycle
(B) Coefficient of performance is lower than that of the original cycle
(C) Refrigerating effect is lower than that of the original cycle
(D) Coefficient of performance is higher than that of the original cycle
Key: (D)

19. The position vector OP of point P(20, 10) is rotated anti-clockwise in X-Y plane by an angle
  30 such that point P occupies position Q, as shown in the figure. The coordinates (x, y) of
Q are
Y
Q

P

X
O

(A) (13.40, 22.32) (B) (12.32, 18.66) (C) (22.32, 8.26) (D) (18.66, 12.32)
Key: (B)

20. A cylindrical rod of diameter 10 mm and length 1.0 m fixed at one end. The other end is twisted
by angle of 10° by applying a torque. If the maximum shear strain in the rod is p×10 –3, then p is
equal to _____ (round off to two decimal places).
Key: (0.8726)

21. Which one of the following welding methods provides the highest heat flux (W/mm2)?
(A) Plasma are welding (B) Tungsten inert gas welding
(C) Oxy-acetylene gas welding (D) Laser beam welding
Key: (D)

© All rights reserved by Thinkcell Learning Solutions Pvt. Ltd. No part of this booklet may be reproduced or utilized in any form without the written permission.

9
|ME-2019-Forenoon|

4 
22. Water flows through a pipe with a velocity given by V    x  y  ˆj m s, where ĵ is the unit
t 
vector in the y direction, t(>0) is in seconds, and x and y are in meters. The magnitude of total
acceleration at the point (x, y) = (1, 1) at t = 2s is ___________ m/s2.
Key: (3)

23. A parabola x  y 2 with 0  x  1 is shown in the figure. The volume of the solid of rotation
obtained by rotating the shaded area by 360° around the x-axis is

x  y2

O 1 x

 
(A) (B) (C) 2 (D) 
4 2
Key: (B)

24. A spur gear with 20° full depth teeth is transmitting 20 kW at 200 rad/s. The pitch circle
diameter of the gear is 100mm. The magnitude of the force applied on the gear in the radial
direction is
(A) 1.39 kN (B) 2.78 kN (C) 0.36 kN (D) 0.73 kN
Key: (D)

dy
25. For the equation  7x 2 y  0, if y  0  3 7, then the value of y(1) is
dx
7 7 3 3 7 3 3 3 7 7 3 7
(A) e (B) e (C) e (D) e
3 7 7 3
Key: (B)

© All rights reserved by Thinkcell Learning Solutions Pvt. Ltd. No part of this booklet may be reproduced or utilized in any form without the written permission.

10
|ME-2019-Forenoon|

Q. No. 26 to 55 Carry Two Marks Each

26. A cube of side 100 mm is placed at the bottom of an empty container on one of its faces. The
density of the material of the cube is 800 kg/ m3. Liquid of density 1000 kg/m3 is now poured
into the container. The minimum height to which the liquid needs to be poured into the
container for the cube to just lift up is _____ mm.
Key: (80)

27. A project consists of six activities. The immediate predecessor of each activity and the
estimated duration is also provided in the table below:
Activity Immediate predecessor Estimated duration (weeks)
P - 5
Q - 1
R Q 2
S P, R 4
T P 6
U S,T 3

If all activities other than S take the estimated amount of time, the maximum duration (in
weeks) of the activity S without delaying the completion of the project is ____________.
Key: (6)

28. Consider an elastic straight beam of length L  10m, with square cross-section of side a=5
mm, and Young’s modulus E = 200 GPa. This straight beam was bent in such a way that the
two ends meet, to form a circle of mean radius R. Assuming that Euler-Bernoulli beam theory is
applicable to this bending problem, the maximum tensile bending stress in the bent beam is
___________ MPa.

L R

Ends of the beam

Key: (100)

© All rights reserved by Thinkcell Learning Solutions Pvt. Ltd. No part of this booklet may be reproduced or utilized in any form without the written permission.

11
|ME-2019-Forenoon|

29. A truss is composed of members AB, BC, CD, AD and BD, as shown in the figure. A vertical
load of 10 kN is applied at point D. The magnitude of force (in kN) in the member BC is
________. 10kN
D

45 45
A C
B

Key: (5)

30. A gas is heated in a duct as it flows over a resistance heater. Consider a 101 kW electric heating
system. The gas enters the heating section of the duct at 100 kPa and 27°C with a volume flow
rate of 15m3/s. If heat is lost from the gas in the duct to the surroundings at a rate of 51kW, the
exit temperature of the gas is
(Assume constant pressure, ideal gas, negligible change in kinetic and potential energies and
constant specific heat; Cp = 1 kJ/kg.K; R = 0.5 kJ/kg.K).
(A) 53°C (B) 32°C (C) 37°C (D) 76°C
Key: (B)

31. A harmonic function is analytic if it satisfies the Laplace equation. If u  x, y   2x 2  2y2  4xy

is a harmonic function, then its conjugate harmonic function v  x, y  is

(A)  4xy  2y 2  2x 2  constant (B) 4xy  2x 2  2y 2  constant

(C) 2x 2  2y 2  xy  constant (D) 4y 2  4xy  constant


Key: (B)

© All rights reserved by Thinkcell Learning Solutions Pvt. Ltd. No part of this booklet may be reproduced or utilized in any form without the written permission.

12
|ME-2019-Forenoon|

32. A uniform thin disk of mass 1 kg and radius 0.1 m is kept on a surface as shown in the figure. A
spring of stiffness k1 = 400 N/m is connected to the disk center A and another spring of stiffness
k2 = 100 N/m is connected at point B just above point A on the circumference of the disk.
Initially, both the springs are unstretched. Assume pure rolling of the disk. For small
disturbance from the equilibrium, the natural frequency of vibration of the system is ______
rad/s (round off to one decimal place).

k2
B

k1
A

Key: (400)

33. In ASA system, the side cutting and end cutting edge angles of a sharp turning tool are 45° and
10°, respectively. The feed during cylindrical turning is 0.1 mm/rev. The center line average
surface roughness (in m, round off to one decimal place) of the generated surface is _______.
Key: (3.747)

34. Consider a prismatic straight beam of length L  m, pinned at the two ends as shown in the
figure.

The beam has a square cross-section of side p = 6mm. The Young’s modulus E = 200 GPa, and
the coefficient of thermal expansion   3 106 K1. The minimum temperature rise required to
cause Euler buckling of the beam is ___________K.
Key: (1)

© All rights reserved by Thinkcell Learning Solutions Pvt. Ltd. No part of this booklet may be reproduced or utilized in any form without the written permission.

13
|ME-2019-Forenoon|

35. The set of equations


x  y  z 1
ax  ay  3z  5
5x  3y  az  6
has infinite solutions, if a =
(A) 4 (B) –4 (C) –3 (D) 3
Key: (A)

36. In a UTM experiment, a sample of length 100 mm, was loaded in tension until failure. The
failure load was 40 kN. The displacement, measured using the cross-head motion, at failure,
was 15 mm. The compliance of the UTM is constant and is given by 5 × 10–8 m/N. The strain at
failure in the sample is ___________%.
Key: (2)

37. A plane-strain compression (forging) of a block is shown in the figure. The strain in the z-
direction is zero. The yield strength (Sy) in uniaxial tension/compression of the material of the
block is 300 MPa and it follows the Tresca (maximum shear stress) criterion. Assume that the
entire block has started yielding. At a point where x  40 MPa (compressive) and xy  0, the

stress component y is

y
Moving platen

O x

Fixed platen

(A) 260 MPa (tensile) (B) 340 MPa (compressive)


(C) 260 MPa (compressive) (D) 340 MPa (tensile)
Key: (A)

© All rights reserved by Thinkcell Learning Solutions Pvt. Ltd. No part of this booklet may be reproduced or utilized in any form without the written permission.

14
|ME-2019-Forenoon|

38. Match the following sand mold casting defects with their respective causes.

Defect Cause
(P) Blow hole 1. Poor collapsibility
(Q) Misrun 2. Mold erosion
(R) Hot tearing 3. Poor permeability
(S) Wash 4. Insufficient fluidity
Codes:
(A) P-3, Q-4, R-2, S-1 (B) P-4, Q-3, R-1, S-2
(C) P-2, Q-4, R-1, S-3 (D) P-3, Q-4, R-1, S-2
Key: (D)

39. A steam power cycle with regeneration as shown below on the T-s diagram employs a single
open feedwater heater for efficiency improvement. The fluids mix with each other in an open
feedwater heater. The turbine is isentropic and the input (bleed) to the feedwater heater from the
turbine is at state 2 as shown in the figure. Process 3-4 occurs in the condenser. The pump work
is negligible. The input to the boiler is at state 5.
The following information is available from the steam tables:

State 1 2 3 4 5 6

Enthalpy (kJ/kg) 3350 2800 2300 175 700 1000

1
6

T
5 2
4
3

The mass flow rate of steam bled from the turbine as a percentage of the total mass flow rate at
the inlet to the turbine at state 1 is _______.
Key: (20)

© All rights reserved by Thinkcell Learning Solutions Pvt. Ltd. No part of this booklet may be reproduced or utilized in any form without the written permission.

15
|ME-2019-Forenoon|

40. The rotor of turbojet engine of an aircraft has a mass 180 kg and polar moment of inertia 10
kg.m2 about the rotor axis. The rotor rotates at a constant speed of 1100 rad/s in the clockwise
direction when viewed from the front of the aircraft. The aircraft while flying at a speed of 800
km per hour takes a turn with a radius of 1.5 km to the left. The gyroscopic moment exerted by
the rotor on the aircraft structure and the direction of motion of the nose when the aircraft
turns, are
(A) 1629.6 N.m and the nose goes up (B) 1629.6 N.m and the nose goes down
(C) 162.9 N.m and the nose goes down (D) 162.9 N.m and the nose goes up
Key: (A)

41. The wall of a constant diameter pipe of length 1 m is heated uniformly with flux q" by
wrapping a heater coil around it. The flow at the inlet to the pipe is hydrodynamically fully
developed. The fluid is incompressible and the flow is assumed to be laminar and steady all
through the pipe. The bulk temperature of the fluid is equal to 0°C at the inlet and 50°C at the
exit. The wall temperatures are measured at three locations, P, Q and R, as shown in the figure.
The flow thermally develops after some distance from the inlet. The following measurements
are made:

Point P Q R
Wall Temp (°C) 50 80 90

0.2m 0.2m 0.2m 0.2m 0.2m

P Q R
0C 50C

Constant wall flux

Among the locations P, Q and R, the flow is thermally developed at:


(A) P and Q only (B) P, Q and R (C) R only (D) Q and R only
Key: (D)

© All rights reserved by Thinkcell Learning Solutions Pvt. Ltd. No part of this booklet may be reproduced or utilized in any form without the written permission.

16
|ME-2019-Forenoon|

42. At a critical point in a component, the state of stress is given as xx  100 MPa,
yy  220 MPa, xy  yx  80 MPa and all other stress components are zero. The yield
strength of the material is 468 MPa. The factor of safety on the basis of maximum shear stress
theory is _________ (round off to one decimal place).
Key: (1.8)

43. A gas turbine with air as the working fluid has an isentropic efficiency of 0.70 when operating
at a pressure ratio of 3. Now, the pressure ratio of the turbine is increased to 5, while
maintaining the same inlet conditions. Assume air as a perfect gas with specific heat ratio
  1.4. If the specific work output remains the same for both the cases, the isentropic
efficiency of the turbine at the pressure ratio of 5 is _____ (round off to two decimal places).
Key: (51.15)

44. The value of the following definite integral is ________ (round off to three decimal places)
e

x
1
n x  dx

Key: (2.097)

45. Taylor’s tool life equation is given by VTn = C, where V is in m/min and T is in min. In a
turning operation, two tools X and Y are used. For tool X, n = 0.3 and C = 60 and for tool Y,
n = 0.6 and C = 90. Both the tools will have the same tool life for the cutting speed (in m/min,
round off to one decimal place) of _____.
Key: (40.5)

46. In a four bar planar mechanism shown in the figure, AB = 5 cm, AD = 4 cm and DC = 2 cm. In
the configuration shown, both AB and DC are perpendicular to AD. The bar AB rotates with an
angular velocity of 10 rad/s. The magnitude of angular velocity (in rad/s) of bar DC at this
instant is
B

10 rad s C

A D

(A) 25 (B) 15 (C) 10 (D) 0


Key: (A)

© All rights reserved by Thinkcell Learning Solutions Pvt. Ltd. No part of this booklet may be reproduced or utilized in any form without the written permission.

17
|ME-2019-Forenoon|

47. If one mole of H2 gas occupies a rigid container with a capacity of 1000 liters and the
temperature is raised from 27°C to 37°C, the change in pressure of the contained gas (round off
to two decimal places), assuming ideal gas behavior, is _____ Pa. (R = 8.314 J/mol.K).
Key: (83.14)

48. Three slabs are joined together as shown in the figure. There is no thermal contact resistance at
the interfaces. The center slab experience a non-uniform internal heat generation with an
average value equal to 10000 Wm–3, while the left and right slabs have no internal heat
generation. All slabs have thickness equal to 1 m and thermal conductivity of each slab is equal
to 5 Wm–1 K–1. The two extreme faces are exposed to fluid with heat transfer coefficient 100
Wm–2K–1 and bulk temperature 30°C as shown. The heat transfer in the slabs is assumed to be
one dimensional and steady, and all properties are constant. If the left extreme face temperature
T1 is measured to be 100°C, the right extreme faced temperature T2 is _____ °C.

Left extreme
face T1  100C T2
1m 1m 1m

100 W m2 .K 100 W m2 .K
30C 30C

Key: (100)

49. Five jobs (J1, J2, J3, J4 and J5) need to be processed in a factory. Each job can be assigned to any
of the five different machines (M1, M2, M3, M4 and M5). The time duration taken (in minutes)
by the machines for each of the jobs, are given in the table. However, each job is assigned to a
specific machine in such a way that the total processing time is minimum. The total processing
time is _____ minutes.
M1 M2 M3 M4 M5
J1 40 30 50 50 58
J2 269 38 60 26 38
J3 40 34 28 24 30
J4 28 40 40 32 48
J5 28 32 38 22 44
Key: (148)

© All rights reserved by Thinkcell Learning Solutions Pvt. Ltd. No part of this booklet may be reproduced or utilized in any form without the written permission.

18
|ME-2019-Forenoon|

50. In orthogonal turning of a cylindrical tube of wall thickness 5mm, the axial and the tangential
cutting forces were measured at 1259 N and 1601 N, respectively. The measured chip thickness
after machining was found to be 0.3 mm. The rake angel was 10° and the axial feed was 100
mm/min. The rotational speed of the spindle was 1000 rpm. Assuming the material to be
perfectly plastic and Merchant’s first solution, the shear strength of the martial is closest to
(A) 722 MPa (B) 875 MPa (C) 200 MPa (D) 920 MPa
Key: (A)

51. A single block brake with a short shoe and torque capacity of 250 N-m is shown. The
cylindrical brake drum rotates anticlockwise at 100 rpm and the coefficient of friction is 0.25.
1.5a a
P

a 4

The value of a, in mm (round off to one decimal place), such that the maximum actuating force
P is 2000 N, is _____.
Key: (212.5)

52. A circular shaft having diameter 65.000.01


0.05 mm is manufactured by turning process. A 50 m

thick coating of TiN is deposited on the shaft Allowed variation in TiN film thickness is 5m.
The minimum hole diameter (in mm) to just provide clearance fit is
(A) 65.12 (B) 64.95 (C) 65.01 (D) 65.10
Key: (A)

53. Two immiscible, incompressible, viscous fluids having same densities but different viscosities
are contained between two infinite horizontal parallel plates, 2 m apart as shown below. The
bottom plate is fixed and the upper plate moves to the right with a constant velocity of 3 m/s.
With the assumptions of Newtonian fluid, steady, and fully developed laminar flow with zero
pressure gradient in all directions, the momentum equations simplify to
d2u
 0.
dy2

© All rights reserved by Thinkcell Learning Solutions Pvt. Ltd. No part of this booklet may be reproduced or utilized in any form without the written permission.

19
|ME-2019-Forenoon|

If the dynamic viscosity of the lower fluid,  2 , is twice that of the upper fluid, 1 , then the
velocity at the interface (round off to two decimal places) is ___________ m/s.

y 1
2  21
2m
2 1m

x
Key: (1.5)

54. A car having weight W is moving in the direction as shown in the figure. The centre of gravity
(CG) of the car is located at height h from the ground, midway between the front and rear
wheels. The distance between the front and rear wheels is . The acceleration of the car is a,
and acceleration due to gravity is g. The reactions on the front wheels (Rf) and rear wheels (Rr)
are given by a
Direction
of motion
CG

h W

l
Rr Rf

W Wh 
(A) Rf  Rr    a
2 g 
W Wh  W Wh 
(B) Rf     a; R r    a
2 g   2 g  
W Wh 
(C) Rf  Rr    a
2 g 
W Wh  W Wh 
(D) Rf     a; R r    a
2 g   2 g  
Key: (D)

55. The variable x takes a value between 0 and 10 with uniform probability distribution. The
variable y takes a value between 0 and 20 with uniform probability distribution. The probability
of the sum of variables (x + y) being greater than 20 is
(A) 0.33 (B) 0.50 (C) 0.25 (D) 0
Key: (C)

© All rights reserved by Thinkcell Learning Solutions Pvt. Ltd. No part of this booklet may be reproduced or utilized in any form without the written permission.

20
Visit : www.Civildatas.com

|ME| GATE-2016-PAPER-01 www.Civildatas.com

General Aptitude
Q. No. 1 – 5 Carry One Mark Each

1. Which of the following is CORRECT with respect to grammar and usage?


Mount Everest is .
(A) the highest peak in the world
(B) highest peak in the world
(C) one of highest peak in the world
(D) one of the highest peak in the world
Key: (A)

2. The policeman asked the victim of a theft, “What did you ?”


(A) loose (B) lose (C) loss (D) louse
Key: (B)

3. Despite the new medicine‟s____________in treating diabetes, it is not __________ widely.


(A) effectiveness --- prescribed (B) availability --- used
(C) prescription -- available (D) acceptance --- proscribed
Key: (A)

4. In a huge pile of apples and oranges, both ripe and unripe mixed together, 15% are unripe fruits. Of the
unripe fruits, 45% are apples. Of the ripe ones, 66% are oranges. If the pile contains a total of 5692000
fruits, how many of them are apples?
(A) 2029198 (B) 2467482 (C) 2789080 (D) 3577422
Key: (A)
Exp: 5692000  Total fruits 

15% unripe 85% ripe

853800 4838200

45% 55% 34% 66%

apples oranges apples oranges

384210 469590 1644988 3193212


Total number of apples = 384210 + 1644988 = 2029198

 ICP–Intensive Classroom Program  eGATE-Live Internet Based Classes DLP  TarGATE-All India Test Series
Leaders in GATE Preparations  65+ Centers across India
© All rights reserved by Gateforum Educational Services Pvt. Ltd. No part of this booklet may be reproduced or utilized in any form without the written permission.

1
11/11
1/14
Visit : 7EC
www.Civildatas.com
Visit : www.Civildatas.com

|ME| GATE-2016-PAPER-01 www.Civildatas.com

5. Michael lives 10 km away from where I live. Ahmed lives 5 km away and Susan lives 7 km away from
where I live. Arun is farther away than Ahmed but closer than Susan from where I live. From the
information provided here, what is one possible distance (in km) at which I live from Arun‟s place?
(A) 3.00 (B) 4.99 (C) 6.02 (D) 7.01
Key: (C)

Q. No. 6 – 10 Carry Two Marks Each

6. A person moving through a tuberculosis prone zone has a 50% probability of becoming infected.
However, only 30% of infected people develop the disease. What percentage of people moving
through a tuberculosis prone zone remains infected but does not show symptoms of disease?
(A) 15 (B) 33 (C) 35 (D) 37
Key: (C)
Exp:

0.3 develop disease

0.5 Infected
0.7
not develop
0.5 not Infected
P(a person infected but does not show symptoms)  0.50  0.70  0.35
The percentage is 35%

7. In a world filled with uncertainty, he was glad to have many good friends. He had always assisted them
in times of need and was confident that they would reciprocate. However, the events of the last week
proved him wrong.
Which of the following inference(s) is/are logically valid and can be inferred from the above passage?
(i) His friends were always asking him to help them.
(ii) He felt that when in need of help, his friends would let him down.
(iii) He was sure that his friends would help him when in need.
(iv) His friends did not help him last week.
(A) (i) and (ii) (B) (iii) and (iv) (C) (iii) only (D) (iv) only
Key: (B)

8. Leela is older than her cousin Pavithra. Pavithra‟s brother Shiva is older than Leela. When Pavithra and
Shiva are visiting Leela, all three like to play chess. Pavithra wins more often than Leela does.
Which one of the following statements must be TRUE based on the above?
(A) When Shiva plays chess with Leela and Pavithra, he often loses.
(B) Leela is the oldest of the three.
(C) Shiva is a better chess player than Pavithra.
(D) Pavithra is the youngest of the three.
Key: (D)
 ICP–Intensive Classroom Program  eGATE-Live Internet Based Classes DLP  TarGATE-All India Test Series
Leaders in GATE Preparations  65+ Centers across India
© All rights reserved by Gateforum Educational Services Pvt. Ltd. No part of this booklet may be reproduced or utilized in any form without the written permission.

2
22/12
2/14
Visit : 7EC
www.Civildatas.com
Visit : www.Civildatas.com

|ME| GATE-2016-PAPER-01 www.Civildatas.com

1 1 1
9. If q  a  and r  b  and s  C  , the value of abc is_____ .
r s q
(A) ( r q s ) −1 (B) 0 (C) 1 (D) r+q+s
Key: (C)
1 1 1
Exp: q a  , r  b  , sc 
r s q
q a  r, r b  s, sc  q
r  q a   sc   sac
a

s  r b   sac   s abc  abc  1


b

10. P, Q, R and S are working on a project. Q can finish the task in 25 days, working alone for 12 hours
a day. R can finish the task in 50 days, working alone for 12 hours per day. Q worked 12 hours a day
but took sick leave in the beginning for two days. R worked 18 hours a day on all days. What is the ratio
of work done by Q and R after 7 days from the start of the project?
(A) 10:11 (B) 11:10 (C) 20:21 (D) 21:20
Key: (C)
1
Exp: Q's one hour work 
25 12
1
R 's one hour work 
50 12
Since Q has taken 2 days sick leave, he has worked only 5 days on the end of seventh day.
1
Work completed by Q on 7th day= (5  12)
25  12
1
Work completed by R on 7th day= (7  18)
50  12
5  12 7  18 20
Ratio of their work    20 : 21
25  12 50  12 21
Mechanical Engineering

Q. No. 1 – 25 Carry One Mark Each

1. The solution to the system of equations


2 5
   x  2 
      is
   y  30
 4 3
(A) 6, 2 (B) −6, 2 (C) −6, −2 (D) 6, −2

Key: (D)
 2 5  6   2 
Exp: By verification method;      
 4 3  2 30

 ICP–Intensive Classroom Program  eGATE-Live Internet Based Classes DLP  TarGATE-All India Test Series
Leaders in GATE Preparations  65+ Centers across India
© All rights reserved by Gateforum Educational Services Pvt. Ltd. No part of this booklet may be reproduced or utilized in any form without the written permission.

3
33/13
3/14
Visit : 7EC
www.Civildatas.com
Visit : www.Civildatas.com

|ME| GATE-2016-PAPER-01 www.Civildatas.com

2. If f(t) is a function defined for all t ≥ 0, its Laplace transform F(s) is defined as

(A)  est f (t)dt  est f (t)dt

(B)
0 0

 
(C) 0
eist f (t)dt (D)  0
eist f (t)dt
Key: (B)
Exp: Definition of Laplace transform of f  t  v t  0.

3. f(z)=u(x,y)+iv(x,y) is an analytic function of complex variable z=x+iy where i  1. If


u(x,y)=2xy, then v(x,y) may be expressed as
(A) –x2 + y 2 + constant (B) x 2 − y 2 + constant
(C) x 2 + y 2 + constant (D) − (x 2 + y 2 ) + constant
Key: (A)
Exp: Given u  x, y   2xy
By total derivative definition; we have
v dv
dv  dx  dy
x y
u u
 dx  dy  by C  R equations … (1)
y x
u u
 u  x, y   2xy   2y;  2x
x y
From (1) dv  2xdx  2ydy … (2)

 x 2   y2 
 V  2    2    constant  equation 2 is exact D.E 
 2   2
 V   x 2  y 2  constant
4. Consider a Poisson distribution for the tossing of a biased coin. The mean for this distribution is µ. The
standard deviation for this distribution is given by
(A)  (B)  2 (C)  (D) 1/ 
Key: (A)
Exp: Given mean of a poisson distribution for the tossing of a biased coin is .
We know that Mean  Variance  

 Standard deviation  variance   .


5. Solve the equation x = 10 cos (x) using the Newton-Raphson method. The initial guess is

x   / 4 . The value of the predicted root after the first iteration, up to second decimal, is
Key: 1.56

By Newton-Raphson method; the iterative formula for finding approximate root at  n  1 iteration is
th
Exp:

f  xn 
x n 1  x n  ; where x = 0, 1, 2 …….
f  xn 
 ICP–Intensive Classroom Program  eGATE-Live Internet Based Classes DLP  TarGATE-All India Test Series
Leaders in GATE Preparations  65+ Centers across India
© All rights reserved by Gateforum Educational Services Pvt. Ltd. No part of this booklet may be reproduced or utilized in any form without the written permission.

4
44/14
4/14
Visit : 7EC
www.Civildatas.com
Visit : www.Civildatas.com

|ME| GATE-2016-PAPER-01 www.Civildatas.com

Putting n = 0; then

f  x0 
x1  x 0  … (1)
f  x0 

    10
Let f  x   x  10cos x  f  x 0   f     .
4 4 2

 f   x   1  10sin x

 10
From (1);  f   x 0   f     1 
4 2

  10 

 4 2   1.56
 x1    
4  1  10 
 2 

6. A rigid ball of weight 100 N is suspended with the help of a string. The ball is pulled by a
horizontal force F such that the string makes an angle of 30o with the vertical. The magnitude of force
F (in N) is .

Key: 57.735
Exp: Applying Lami‟s theorem
T 30
T 100 F 120
 
sin90 sin 90  30 sin 180  30
   F
150
90
F  57.735 N

100
7. A point mass M is released from rest and slides down a spherical bowl (of radius R) from a height H as
shown in the figure below. The surface of the bowl is smooth (no friction). The velocity of the mass at
the bottom of the bowl is

(A) gH (B) 2gR (C) 2gH (D) 0

 ICP–Intensive Classroom Program  eGATE-Live Internet Based Classes DLP  TarGATE-All India Test Series
Leaders in GATE Preparations  65+ Centers across India
© All rights reserved by Gateforum Educational Services Pvt. Ltd. No part of this booklet may be reproduced or utilized in any form without the written permission.

5
55/15
5/14
Visit : 7EC
www.Civildatas.com
Visit : www.Civildatas.com

|ME| GATE-2016-PAPER-01 www.Civildatas.com

Key: (C)
Exp: Since there is no friction. Therefore there will be no loss of energy of system. Hence energy
remains conserved.
P.E1+K.E1 = P.E2+K.E2
1 1
mgH  m  0   mg  0  mVb2
2

2 2
Vb  2gH

8. The cross sections of two hollow bars made of the same material are concentric circles as shown in the
figure. It is given that r3 > r1 and r4 > r2 , and that the areas of the cross-sections are the same. J1 and J2
are the torsional rigidities of the bars on the left and right, respectively. The ratio J2/J1 is

r3
r1

r2 r4

(A) > 1 (B) < 0.5 (C) =1 (D) between 0.5 and 1
Key: (A)
Exp:
r3
r1

r2 r4

Smaller Ring 1 Bigger Ring  2 

Given A1 = A2
  r22  r12     r42  r32 

r22  r12  r42  r32 ________(1)


We know , Torsional Rigidity = J= Shear Modulus× Polar moment of Inertia
 4 4
J2 2
G
r4  r3 J2 
r42  r32 
r32  r42 r32  r42      
    2 2  2 2  2 2
J1 
G  r24  r14 J1

r2  r1 r1  r2
 r1  r2      
2
But, r4  r2 and r3  r1
J2
 1
J1

 ICP–Intensive Classroom Program  eGATE-Live Internet Based Classes DLP  TarGATE-All India Test Series
Leaders in GATE Preparations  65+ Centers across India
© All rights reserved by Gateforum Educational Services Pvt. Ltd. No part of this booklet may be reproduced or utilized in any form without the written permission.

6
66/16
6/14
Visit : 7EC
www.Civildatas.com
Visit : www.Civildatas.com

|ME| GATE-2016-PAPER-01 www.Civildatas.com

9. A cantilever beam having square cross-section of side a is subjected to an end load. If a is increased by
19%, the tip deflection decreases approximately by
(A) 19% (B) 29% (C) 41% (D) 50%
Key: (D)
p3 1 1
Exp:      4
3EI I a
1 a 24
 where, a 2  1.19a,
2 a14
1 1
  1.19   2   0.51
4

2 1.19 4
So, deflection decrease by 50%

10. A car is moving on a curved horizontal road of radius 100 m with a speed of 20 m/s. The rotating masses
of the engine have an angular speed of 100 rad/s in clockwise direction when viewed from the front of
the car. The combined moment of inertia of the rotating masses is 10 kg-m2. The magnitude of the
gyroscopic moment (in N-m) is .
Key: 200
Exp: Given: Spin velocity ( s )= 100 rad/sec
Moment of Inertia (MOI) = 10 kg-m2
V  20 m sec

Precision Angular Velocity  p  


linear speed
R R  100 m

 p
20
  0.2 rad sec
100
Gyroscopic moment = MOI s × p
 10  100  0.2
 200 Nm
11. A single degree of freedom spring mass system with viscous damping has a spring constant of 10
kN/m. The system is excited by a sinusoidal force of amplitude 100 N. If the damping factor (ratio) is
0.25, the amplitude of steady state oscillation at resonance is mm.
Key: 20
Exp: Given: Spring constant (k) = 10 kN/m = 10,000 N/m
Magnitude of force (F0) = 100N
Damping factor    0.25
Forcing frequency (ω) = Natural frequency (ωn)
F 100
Static deflection of spring  0  4  102 m
k 10
 10 mm

 ICP–Intensive Classroom Program  eGATE-Live Internet Based Classes DLP  TarGATE-All India Test Series
Leaders in GATE Preparations  65+ Centers across India
© All rights reserved by Gateforum Educational Services Pvt. Ltd. No part of this booklet may be reproduced or utilized in any form without the written permission.

7
77/17
7/14
Visit : 7EC
www.Civildatas.com
Visit : www.Civildatas.com

|ME| GATE-2016-PAPER-01 www.Civildatas.com

Staticdeflection
Dynamic deflection 
   2   
2

1       2 
  n    n 
10

1  1  2 2
  2  0.25  1
2

10
  20 mm
2  0.25
12. The spring constant of a helical compression spring DOES NOT depend on
(A) coil diameter
(B) material strength
(C) number of active turns
(D) wire diameter
Key: (B)
Exp: For Helical compression spring.
64 WR 3n
Deflection,  
Gd4
W W Gd 4
Stiffness, or spring constant   
 64 WR 3 n 64R 3 n
Gd 4
From the above formula we can say that spring constant depends on coil diameter (D), wire diameter (d),
No. of active turns (n) and modulus of rigidity (G) and is independent of material strength.

13. The instantaneous stream-wise velocity of a turbulent flow is given as follows:


u(x, y, z, t) = u (x, y, z)  u  ( x, y, z, t)
The time-average of the fluctuating velocity u  ( x, y, z, t) is
(A) u  / 2 (B) u / 2 (C) zero (D) u / 2
Key: (C) u'
Exp:
u

time  t 
Given u  u  u ' _____(1)
T
1
T 0
where u  u dt T = period after which pattern will repeat

 ICP–Intensive Classroom Program  eGATE-Live Internet Based Classes DLP  TarGATE-All India Test Series
Leaders in GATE Preparations  65+ Centers across India
© All rights reserved by Gateforum Educational Services Pvt. Ltd. No part of this booklet may be reproduced or utilized in any form without the written permission.

8
88/18
8/14
Visit : 7EC
www.Civildatas.com
Visit : www.Civildatas.com

|ME| GATE-2016-PAPER-01 www.Civildatas.com

Rewriting equation (1)


u '  u u
taking average of fluctuating component
1T 
 
T T T T
1 1 1 1
T 0 T 0 T 0 T 0
u'  u 'dt  u  u dt  u dt  u dt  u  u   dt 
T 0 
 uu
u'  0

14. For a floating body, buoyant force acts at the


(A) centroid of the floating body
(B) center of gravity of the body
(C) centroid of the fluid vertically below the body
(D) centroid of the displaced fluid
Key: (D)
15. A plastic sleeve of outer radius r0 = 1 mm covers a wire (radius r = 0.5 mm) carrying electric
current. Thermal conductivity of the plastic is 0.15 W/m-K. The heat transfer coefficient on the outer
surface of the sleeve exposed to air is 25 W/m2-K. Due to the addition of the plastic cover, the heat
transfer from the wire to the ambient will
(A) increase
(B) remain the same
(C) decrease
(D) be zero
Key: (A)
Exp: Given
r0  1mm
r  0.5mm
K  0.15W/mK
h  25W/m2 K
0.15
Critical radius  k/h   6mm
25
So radius of wire is less than critical radius, addition of plastic sleeve will increase the heat transfer.

16. Which of the following statements are TRUE with respect to heat and work?
(i) They are boundary phenomena
(ii) They are exact differentials
(iii) They are path functions
(A) both (i) and (ii) (B) both (i) and (iii) (C) both (ii) and (iii) (D) only (iii)
Key: (B)

 ICP–Intensive Classroom Program  eGATE-Live Internet Based Classes DLP  TarGATE-All India Test Series
Leaders in GATE Preparations  65+ Centers across India
© All rights reserved by Gateforum Educational Services Pvt. Ltd. No part of this booklet may be reproduced or utilized in any form without the written permission.

9
99/19
9/14
Visit : 7EC
www.Civildatas.com
Visit : www.Civildatas.com

|ME| GATE-2016-PAPER-01 www.Civildatas.com

17. Propane (C3H8) is burned in an oxygen atmosphere with 10% deficit oxygen with respect to the
stoichiometric requirement. Assuming no hydrocarbons in the products, the volume percentage of CO
in the products is
Key: 14.286
Exp: C3H8  5(0.9)O2  5(3.76)0.9N 2  aCO  bCO 2  4H 2O 16.92N 2
Carbon balance : 3  a  b
Qxygen balance : 9  a  2b  4
6  b  4
a  1, b  2
1
100  14.286%
7
18. Consider two hydraulic turbines having identical specific speed and effective head at the inlet. If the
speed ratio (N1/N2) of the two turbines is 2, then the respective power ratio (P1/P2) is
____.
Key: 0.25
Exp: Given: Specific speed of Turbine “1” (NS1) = Specific Speed of Turbine “2” (NS2)
Effective Head at inlet of Turbine “1” (H1) = Effective head at inlet of turbine “2” (H2)
and N1/N2 = 2
N P
Specific Speed of Turbine  NS  
H5 4
NS1 = NS2
N1 P1 N 2 P2

H15 4 H 25 4
2
P1  N 2   1  1
2

       0.25
P2  N1   2  4
19. The INCORRECT statement about regeneration in vapor power cycle is that
(A) it increases the irreversibility by adding the liquid with higher energy content to the steam
generator
(B) heat is exchanged between the expanding fluid in the turbine and the compressed fluid before heat
addition
(C) the principle is similar to the principle of Stirling gas cycle
(D) it is practically implemented by providing feed water heaters
Key: (A)
20. The “Jominy test” is used to find
(A) Young‟s modulus (B) hardenability
(C) yield strength (D) thermal conductivity
Key: (B)

 ICP–Intensive Classroom Program  eGATE-Live Internet Based Classes DLP  TarGATE-All India Test Series
Leaders in GATE Preparations  65+ Centers across India
© All rights reserved by Gateforum Educational Services Pvt. Ltd. No part of this booklet may be reproduced or utilized in any form without the written permission.

10
1010/
11010
Visit : /14
www.Civildatas.com
Visit : www.Civildatas.com

|ME| GATE-2016-PAPER-01 www.Civildatas.com

21. Under optimal conditions of the process the temperatures experienced by a copper work piece in fusion
welding, brazing and soldering are such that
(A) Twelding > Tsoldering> Tbrazing (B) Tsoldering > Twelding > Tbrazing
(C) Tbrazing >Twelding > Tsoldering (D) Twelding > Tbrazing > Tsoldering
Key: (D)

22. The part of a gating system which regulates the rate of pouring of molten metal is
(A) pouring basin (B) runner (C) choke (D) ingate
Key: (C)

23. The non-traditional machining process that essentially requires vacuum is


(A) electron beam machining (B) electro chemical machining
(C) electro chemical discharge machining (D) electro discharge machining
Key: (A)
Exp: Electron beam machining requires vacuum, to avoid deflection of electrons

24. In an orthogonal cutting process the tool used has rake angle of zero degree. The measured cutting force
and thrust force are 500 N and 250 N, respectively. The coefficient of friction between the tool and
the chip is
Key: 0.5
Ft
Exp: Tan      
FC
250
Tan    0  
500
Tan  1  0.5
2

25. Match the following:


P. Feeler gauge I. Radius of an object
Q. Fillet gauge II. Diameter within limits by comparison
R. Snap gauge III. Clearance or gap between components
S. Cylindrical plug gauge IV. Inside diameter of straight hole

(A) P–III, Q–I, R–II, S–IV (B) P–III, Q–II, R–I, S–IV
(C) P–IV, Q–II, R–I, S–III (D) P–IV, Q–I, R–II, S–III
Key: (A)
Exp: Feeler gauge: Clearance or gap between components
Fillet gauge: Radius of an object
Snap gauge: Diameter within limits by comparison
Cylindrical plug gauge: Inside diameter of straight hole.
 ICP–Intensive Classroom Program  eGATE-Live Internet Based Classes DLP  TarGATE-All India Test Series
Leaders in GATE Preparations  65+ Centers across India
© All rights reserved by Gateforum Educational Services Pvt. Ltd. No part of this booklet may be reproduced or utilized in any form without the written permission.

11
1111/
11111
Visit : /14
www.Civildatas.com
Visit : www.Civildatas.com

|ME| GATE-2016-PAPER-01 www.Civildatas.com

Q. No. 26 – 55 carry Two Marks Each

26. Consider the function f (x)  2x 3  3x 2 i n the domain [−1, 2]. The global minimum of f(x) is
_______
Key: -5
Exp: Given that, f  x   2x3  3x 2
 f   x   0  6x 2  6x  0
 x 2  x  0  x  x  1  0
 x  0; x  1
are Stationary points.
 f   x   12x  6
f   0  6  0
 f(x) has maximum at x = 0.
f  1  12 1  6  6  0
 f(x) has minimum at x = 1.
 f 1  2  3  1  local minimum value

But f  1  2  3  5
 Global minimum of f(x) = -5

27. If y=f(x)satisfies the boundary value problem y  9y  0, y(0)  0, y( / 2)  2, , then y( / 4) is
_________
Key: -1
Exp: Given D.E is y  9y  0

  D2  9 y  0

The A.E is D2  9  0
 D2  9  D  3i
 y  c1 cos3x  c2 sin 3x … (1)
Given
y   / 2  2
y 0  0 and
i.e; x  0, y  0 i.e; x   / 2; y  2
From(1); 0  C1 From(1); 2  0  C2 (1)
 C2   2
 From (1); y   2 sin3x

   3   1 
 y     2 sin     2    1
4  4  2

 y    1
4
 ICP–Intensive Classroom Program  eGATE-Live Internet Based Classes DLP  TarGATE-All India Test Series
Leaders in GATE Preparations  65+ Centers across India
© All rights reserved by Gateforum Educational Services Pvt. Ltd. No part of this booklet may be reproduced or utilized in any form without the written permission.

12
1212/
11212
Visit : /14
www.Civildatas.com
Visit : www.Civildatas.com

|ME| GATE-2016-PAPER-01 www.Civildatas.com

28. The value of the integral


 sin x
  x 2  2x  2
dx

evaluated using contour integration and the residue theorem is


(A) –  sin 1 / e (B)  cos 1 / e (C) sin(1)/e (D) cos(1)/e
Key: (A)
Exp: We know that sin x is the imaginary part of eix
eiz
∴ We consider the function f  z  
z  2z  2
2

Now, the poles of f(z) are given by z2  2z  2  0


2  4  4  2  2  4
z 
2 2
2  i2
  1 i
2
But Z  1  i is the only pole (simple) lie in the upper half of the Z-plane.
eiz
∴ Resf  z   Lt z   1  i  .

z 1i z 1i   z   1 i   z   1  i 
eiz ei 1i  ei 1 ei
 Lt   
z1i z  1  i  1  i  1  i 2i 2ie

eiz  ei  ei


Thus e z2  2z  2 dz  2 i  
 2ie  e
Equating imaginary parts on both sides we get
sin z    sin 1   sin 1
z
e
2
 2z  2
dz 
e

e

29. Gauss-Seidel method is used to solve the following equations (as per the given order):
x1  2x 2  3x 3  5
2x1  3x 2  x 3  1
3x1  2x 2  x 3  3
Assuming initial guess as x1  x 2  x 3  0, the value of x3 after the first iteration is ____
Key: -6
Exp: x11  0  0  5 ... 1

2x11  3x 21  0  1 ...(2)


3x11  2x 21  x 31  3 ...  3

∴ From equation (1) x11  5

 ICP–Intensive Classroom Program  eGATE-Live Internet Based Classes DLP  TarGATE-All India Test Series
Leaders in GATE Preparations  65+ Centers across India
© All rights reserved by Gateforum Educational Services Pvt. Ltd. No part of this booklet may be reproduced or utilized in any form without the written permission.

13
1313/
11313
Visit : /14
www.Civildatas.com
Visit : www.Civildatas.com

|ME| GATE-2016-PAPER-01 www.Civildatas.com

From equation (2), 2x11  3x 21  1


 3x 21 1  2x11
 1  2  5
 3x 21  9
9
 x 21   3  x 21  3
3
From equation (3), x 31  3  3x11  2x 21
 3  3  5   2  3
 3  15  6  6
 x 31  6
∴ After the first iteration, the value of x3 is -6.
30. A block of mass m rests on an inclined plane and is attached by a string to the wall as shown in the
figure. The coefficient of static friction between the plane and the block is 0.25. The string can withstand
a maximum force of 20 N. The maximum value of the mass (m) for which the string will not break and
the block will be in static equilibrium is kg.
Take cos   0.8and sin   0.6.

Acceleration due to gravity g = 10 m/s2

Key: 5 R
Exp: F  R   mg cos 
T
 0.25  m 10  0.8 m F
 2m
mgsin 
For equilibrium, 

T  F  mgsin 
 20  2m  m  10  0.6  m  5kg
mg mgsin 

 ICP–Intensive Classroom Program  eGATE-Live Internet Based Classes DLP  TarGATE-All India Test Series
Leaders in GATE Preparations  65+ Centers across India
© All rights reserved by Gateforum Educational Services Pvt. Ltd. No part of this booklet may be reproduced or utilized in any form without the written permission.

14
1414/
11414
Visit : /14
www.Civildatas.com
Visit : www.Civildatas.com

|ME| GATE-2016-PAPER-01 www.Civildatas.com

31. A two-member truss PQR is supporting a load W. The axial forces in members PQ and QR are
respectively

(A) 2W tensile and 3W compressive

(B) 3W tensile and 2W compressive

(C) 3W compressive and 2W tensile


(D) 2Wcompressive and 3W tensile
Key: (B)
Exp: F.B.D of point Q
Fx  0
FQR sin 60
 FPQ  FQR sin 60  0 ...... 1 Q
FPQ
Fy  0 30
 FQR cos60  W  0 .... 2  60
FQR cos 60
W
 FQR   FQR  2W  compressive  FQR
cos60
From equation (1) W
FPQ  2Wsin 60  0

3
 FPQ  2W   3W  Tensile 
2

32. A horizontal bar with a constant cross-section is subjected to loading as shown in the figure. The
Young‟s moduli for the sections AB and BC are 3E and E, respectively.

For the deflection at C to be zero, the ratio P/F is


Key: 4 A B B C
Exp:

FP 3E FP F E F
F.B.D

 
Since, net deflection at C is zero
 ICP–Intensive Classroom Program  eGATE-Live Internet Based Classes DLP  TarGATE-All India Test Series
Leaders in GATE Preparations  65+ Centers across India
© All rights reserved by Gateforum Educational Services Pvt. Ltd. No part of this booklet may be reproduced or utilized in any form without the written permission.

15
1515/
11515
Visit : /14
www.Civildatas.com
Visit : www.Civildatas.com

|ME| GATE-2016-PAPER-01 www.Civildatas.com

 AB  BC  0


 F  P    F
0
A.3E AE
FP
 F  0
3
 4F  P  0
P
 4
F
33. The figure shows cross-section of a beam subjected to bending. The area moment of inertia

(in mm4) of this cross-section about its base is _________.

Key: 1873 - 1879


34. A simply-supported beam of length 3L is subjected to the loading shown in the figure.

It is given that P = 1 N, L = 1 m and Young‟s modulus E = 200 GPa. The cross-section is a square with
dimension 10 mm × 10 mm. The bending stress (in Pa) at the point A located at the top surface of the
beam at a distance of 1.5L from the left end is
(Indicate compressive stress by a negative sign and tensile stress by a positive sign.)
Key: 0 P P
Exp: A L L L B
A
RA RB

Taking moment about B


MB  0  R A  3L  P  2L  PL  0  R A   P 3

 ICP–Intensive Classroom Program  eGATE-Live Internet Based Classes DLP  TarGATE-All India Test Series
Leaders in GATE Preparations  65+ Centers across India
© All rights reserved by Gateforum Educational Services Pvt. Ltd. No part of this booklet may be reproduced or utilized in any form without the written permission.

16
1616/
11616
Visit : /14
www.Civildatas.com
Visit : www.Civildatas.com

|ME| GATE-2016-PAPER-01 www.Civildatas.com

Fy  0  R B  R A  0  R B  P 3

Taking moment about A


M A  0

R A  1.5L  0.5PL  M A  Assuming MA anticlockwise 


P
   1.5L  0.5PL  M A
3
 MA  0

M b
we know,   b  Bendingstress   0 since, M A  0
I y
35. A slider crank mechanism with crank radius 200 mm and connecting rod length 800 mm is shown. The
crank is rotating at 600 rpm in the counterclockwise direction. In the configuration shown, the crank
makes an angle of 90o with the sliding direction of the slider, and a force of 5 kN is acting on the slider.
Neglecting the inertia forces, the turning moment on the crank (in kN-m) is _____

Key: 1
Exp: T cos   5
Moment about crank shaft  M   T  r sin  90   
  T cos   r T
 5  0.2 T

 1kN m
5
Alternate method:
H
  
M  5  0.2 sin  90   sin 180 
 2 
 1kN m
36. In the gear train shown, gear 3 is carried on arm 5. Gear 3 meshes with gear 2 and gear 4. The number of
teeth on gear 2, 3, and 4 are 60, 20, and 100, respectively. If gear 2 is fixed and gear 4 rotates with an
angular velocity of 100 rpm in the counterclockwise direction, the angular speed of arm 5 (in rpm) is

(A) 166.7 counterclockwise (B) 166.7 clockwise


(C) 62.5 counterclockwise (D) 62.5 clockwise
Key: (C)

 ICP–Intensive Classroom Program  eGATE-Live Internet Based Classes DLP  TarGATE-All India Test Series
Leaders in GATE Preparations  65+ Centers across India
© All rights reserved by Gateforum Educational Services Pvt. Ltd. No part of this booklet may be reproduced or utilized in any form without the written permission.

17
1717/
11717
Visit : /14
www.Civildatas.com
Visit : www.Civildatas.com

|ME| GATE-2016-PAPER-01 www.Civildatas.com

Exp:
Arm Gears
2 3 4
All gear locked to
60 60
Arm & 2 is given x 0 x x x
20 100
Rotations in C.C.W
Arm also given y yx 60 60
y y x y x
Rotations in C.C.W 20 100
Given: x  y 0 ...(1)
3
y  x 100 ...(2)
5
3 5  100
y  y  100  y   62.5 counter clockwise
5 8
37. A solid disc with radius a is connected to a spring at a point d above the center of the disc. The other
end of the spring is fixed to the vertical wall. The disc is free to roll without slipping on the ground. The
mass of the disc is M and the spring constant is K. The polar moment of inertia for the disc about its
centre is J  Ma 2 / 2.

The natural frequency of this system in rad/s is given by

2K(a  d)2 2K 2K(a  d)2 K(a  d)2


(A) (B) (C) (D)
3Ma 2 3M Ma 2 Ma 2
Key: (A)
Exp: Apply D'Alembert Principle:
 k  a  d    a  d    I cm  Ma 2  
 0 cm

k a  d
2

 0
 Ma 2 
  Ma 2  x
 2 
2k  a  d 
2

n 
3Ma 2 k a  d 
cm

 ICP–Intensive Classroom Program  eGATE-Live Internet Based Classes DLP  TarGATE-All India Test Series
Leaders in GATE Preparations  65+ Centers across India
© All rights reserved by Gateforum Educational Services Pvt. Ltd. No part of this booklet may be reproduced or utilized in any form without the written permission.

18
1818/
11818
Visit : /14
www.Civildatas.com
Visit : www.Civildatas.com

|ME| GATE-2016-PAPER-01 www.Civildatas.com

38. The principal stresses at a point inside a solid object are 1  100 MPa, 2  100 MPa and 3  0
MPa. The yield strength of the material is 200 MPa. The factor of safety calculated using Tresca
(maximum shear stress) theory is nT and the factor of safety calculated using von Mises (maximum
distortional energy) theory is nV. Which one of the following relations is TRUE?

(A) n T   3 / 2 n v

(B) n T   3n v

(C) n T  n v

(D) n v   3n T

Key: (C)

  1  3   2  3   3  1  
Exp: max  max  ,  ,     50 Mpa
 2   2   2  

max 
S yt / 2
 50 
 200 / 2    2
T T
T

  1  2 2   2  3 2   3  1 2   Syt 2
   
 2   v 

 12  22  1 2   Syt / v 


2

1  2  100 & Syt  200


2
 200 
100      v  2
2

 v 
 T  v

39. An inverted U-tube manometer is used to measure the pressure difference between two pipes A and B, as
shown in the figure. Pipe A is carrying oil (specific gravity = 0.8) and pipe B is carrying water. The
densities of air and water are 1.16 kg/m3 and 1000 kg/m3, respectively. The pressure difference between
pipes A and B is kPa.

Acceleration due to gravity g = 10 m/s2.

 ICP–Intensive Classroom Program  eGATE-Live Internet Based Classes DLP  TarGATE-All India Test Series
Leaders in GATE Preparations  65+ Centers across India
© All rights reserved by Gateforum Educational Services Pvt. Ltd. No part of this booklet may be reproduced or utilized in any form without the written permission.

19
1919/
11919
Visit : /14
www.Civildatas.com
Visit : www.Civildatas.com

|ME| GATE-2016-PAPER-01 www.Civildatas.com

Key: -2.199

Exp: PA  oil gh1  air gh2  PB water g  h1  h2  h3 


PA  PB  0.8 103  10  0.2 1.16  10  0.08 103 10  0.2  0.08  0.1
 1600  0.928  3800
  2199.072 Pa
PA  PB   2.199 kPa

40. Oil (kinematic viscosity, Voil  1.0 × 10−5 m2/s) flows through a pipe of 0.5 m diameter with a
velocity of 10 m/s. Water (kinematic viscosity, vw= 0.89 × 10−6 m2/s) is flowing through a model pipe
of diameter 20 mm. For satisfying the dynamic similarity, the velocity of water (in m/s) is _____ .

Key: 22.25
Prototype
Exp:
oil  1.0 105 m2 /s
V0 =10m/s oil doil  0.5m

model
 w  0.89 106 m2 /s
Vw = ? water
d w  0.02m

There is no free surface inside pipe flow, therefore only Reynold numbers are equal in both the cases.

 R e oil   R e water  
VD   VD 
  
  mod el    prototype
Vw  0.02 10  0.5
  Vw  22.25m/sec
0.89 106 1.0 105

41. A steady laminar boundary layer is formed over a flat plate as shown in the figure. The free stream
velocity of the fluid is Uo. The velocity profile at the inlet a-b is uniform, while that at a
  y   y 2 
downstream location c-d is given by u  U0  2       .
       

 bd , leaving through the horizontal section b-d to that entering


The ratio of the mass flow rate, m
through the vertical section a-b is .
Key: 0.33
 ICP–Intensive Classroom Program  eGATE-Live Internet Based Classes DLP  TarGATE-All India Test Series
Leaders in GATE Preparations  65+ Centers across India
© All rights reserved by Gateforum Educational Services Pvt. Ltd. No part of this booklet may be reproduced or utilized in any form without the written permission.

20
2020/
12020
Visit : /14
www.Civildatas.com
Visit : www.Civildatas.com

|ME| GATE-2016-PAPER-01 www.Civildatas.com

Exp:
This is steady state process, so time derivative will be zero. Choose b-a-c-d-b as Control Volume (CV)
   
  V.n  dA  0

cs 

where C.S  stands for control surface &



n is unit vector perpendicular to Area of flow which is always taken outside of side of cross-
sectional Area.
Let fluid is incompressible, ρ= constant

               
a  d b
  V.n  dA    V.n  dA    V.n  dA    V.n dA  0
b  a  c  d 

     y 2  y 2 
  
  U0 b.dy    0.n  dA   U 0 2       bdy  m
 bd  0
0 a  c        

  
 U 0 b  0  V b     m bd  0
 3
2 
 U 0 b  bV0   m bd  0
3
 1
m bd   U0 b  
3

m bd
 0.33
 U 0 b
42. A steel ball of 10 mm diameter at 1000 K is required to be cooled to 350 K by immersing it in a water
environment at 300 K. The convective heat transfer coefficient is 1000 W/m2-K. Thermal
conductivity of steel is 40 W/m-K. The time constant for the cooling process τ is 16 s. The time
required (in s) to reach the final temperature is _____
Key: 42.22
Exp: Given
d = 10 mm = 0.01 m
W W
t i  1000K;t  350K;t   300K; k  40 ;h  1000 2 ; th  16s
mK mK
t  t 
 e th
ti  t
 t  t   
ln    th
 ti  t 
 350  300  
ln    16
 1000  300 
  42.22s

 ICP–Intensive Classroom Program  eGATE-Live Internet Based Classes DLP  TarGATE-All India Test Series
Leaders in GATE Preparations  65+ Centers across India
© All rights reserved by Gateforum Educational Services Pvt. Ltd. No part of this booklet may be reproduced or utilized in any form without the written permission.

21
2121/
12121
Visit : /14
www.Civildatas.com
Visit : www.Civildatas.com

|ME| GATE-2016-PAPER-01 www.Civildatas.com

43. An infinitely long furnace of 0.5 m × 0.4 m cross-section is shown in the figure below. Consider all
surfaces of the furnace to be black. The top and bottom walls are maintained at temperature T1 = T3
= 927oC while the side walls are at temperature T2 = T4 = 527 oC. The view factor, F1-2 is 0.26. The
net radiation heat loss or gain on side 1 is W/m.

Stefan-Boltzmann constant = 5.67 × 10−8 W/m2-K4

Key: 24530.688
44. A fluid (Prandtl number, Pr = 1) at 500 K flows over a flat plate of 1.5 m length, maintained at
300 K. The velocity of the fluid is 10 m/s. Assuming kinematic viscosity, ν = 30 × 10−6 m2/s, the
thermal boundary layer thickness (in mm) at 0.5 m from the leading edge is .
Key: 6.124
Exp: Pr = 1
ux 10  0.5
Re x  
 30  106
 166666.67

 1.67  105
Hydrodynamic boundary layer thickness
5x 5  0.5
h x  
Re x 1.67  105

 6.124  103 m
If Pr = 1
h x  Tx  6.124  103 m

 6.124 mm
 Thermal boundary layer thickness = 6.124 mm.
45. For water at 25oC, dps / dTs  0.189kPa / K (ps is the saturation pressure in kPa and Ts is the saturation
temperature in K) and the specific volume of dry saturated vapour is 43.38 m3/kg. Assume that the
specific volume of liquid is negligible in comparison with that of vapour. Using the Clausius- Clapeyron
equation, an estimate of the enthalpy of evaporation of water at 25oC (in kJ/kg) is ______.
Key: 2443.24

 ICP–Intensive Classroom Program  eGATE-Live Internet Based Classes DLP  TarGATE-All India Test Series
Leaders in GATE Preparations  65+ Centers across India
© All rights reserved by Gateforum Educational Services Pvt. Ltd. No part of this booklet may be reproduced or utilized in any form without the written permission.

22
2222/
12222
Visit : /14
www.Civildatas.com
Visit : www.Civildatas.com

|ME| GATE-2016-PAPER-01 www.Civildatas.com

dPs h fg h fg
Exp:   0.189 
dTs Ts   g   f   25  273 43.38  0 
 h fg  2443.248kJ kg

46. An ideal gas undergoes a reversible process in which the pressure varies linearly with volume. The
conditions at the start (subscript 1) and at the end (subscript 2) of the process with usual notation are:
p1  100kPa, V1  0.2m3 and p 2  200 kPa, V2  0.1m3 and the gas constant, R = 0.275 kJ/kg-K. The
magnitude of the work required for the process (in kJ) is .
Key: 15
Exp: Pressure varies linearly with volume.
P = a + bv
P1  a  bv1

 100  a  b  0.2      (1)

P2  a  bv2

 200  a  b  0.1      (2)

Solving (1) & (2)

100
100  0.1b  b   1000
0.1
b  1000

Substituting in any of the equations to get „a‟.


 100  a   1000  0.2   a  300
2 2
W   pdv    a  bv  dv
1 1
2
 v 2  v12 
   300  1000v  dv  300  v2  v1   1000  2 
1  2 
  0.12   0.2 2 
 300  0.1  0.2   1000    30   15  15kJ
 2 
 
 Magnitude of work required is 15 kJ.
47. In a steam power plant operating on an ideal Rankine cycle, superheated steam enters the turbine at 3
MPa and 350oC. The condenser pressure is 75 kPa. The thermal efficiency of the cycle is
________ percent.
Given data:
For saturated liquid, at P = 75 kPa, h f  384.39kJ / kg, vf  0.001037m3 / kg, sf  1.213kJ / kg  K
At 75 kPa, hfg = 2278.6 kJ/kg, sfg = 6.2434 kJ/kg-K

At P = 3 MPa and T = 350oC (superheated steam), H  3115.3kJ / kg, s  6.7428kJ / kg  K


 ICP–Intensive Classroom Program  eGATE-Live Internet Based Classes DLP  TarGATE-All India Test Series
Leaders in GATE Preparations  65+ Centers across India
© All rights reserved by Gateforum Educational Services Pvt. Ltd. No part of this booklet may be reproduced or utilized in any form without the written permission.

23
2323/
12323
Visit : /14
www.Civildatas.com
Visit : www.Civildatas.com

|ME| GATE-2016-PAPER-01 www.Civildatas.com

Key: 25.99%
Exp: 1

3 2

Given:- P1  P4  3MPa, T1  350o C  350  273  623K


h1  3115.3kJ / kg, S1  6.7428kJ / kgK
P2  P3  75kPa.
h f2  h f3  384.39kJ / kg, h fg2  2278.6kJ / kg
sf2  sf3  1.213kJ / kgK, s fg2  6.2434kJ / kgK
vf3  0.001037m3 / kg
S1  S2
 S1  Sf2  x 2Sfg2
6.7428  1.213  x 2  6.2434
x 2  0.886
h 2  h f2  x 2 h fg2  384.39  0.886  2278.6
 2403.2296kJ / kg
 2403.23
Turbine work, WT  h1  h 2  3115.3  2403.23  712.07kJ / kg
Pump work, Wp  vf3  P4  P3   0.001037(3000  75)
 3.03k J / kg
h 4  h 3  Wp  h f3  Wp  384.39  3.03
 387.42kJ / kg
Heat supplied to boiler
 h1  h 4  3115.3  387.42
 2727.88kJ / kg
Net work done  WT  Wp  712.07  3.03  709.04kJ / kg
Net work done 709.04
Thermal efficiency      100  25.99%
Heat supplied 2727.88

 ICP–Intensive Classroom Program  eGATE-Live Internet Based Classes DLP  TarGATE-All India Test Series
Leaders in GATE Preparations  65+ Centers across India
© All rights reserved by Gateforum Educational Services Pvt. Ltd. No part of this booklet may be reproduced or utilized in any form without the written permission.

24
2424/
12424
Visit : /14
www.Civildatas.com
Visit : www.Civildatas.com

|ME| GATE-2016-PAPER-01 www.Civildatas.com

48. A hypothetical engineering stress-strain curve shown in the figure has three straight lines PQ, QR, RS
with coordinates P(0,0), Q(0.2,100), R(0.6,140) and S(0.8,130). 'Q' is the yield point, 'R' is the UTS
point and 'S' the fracture point.

The toughness of the material (in MJ/m3) is ________ .


Key: 0.85
Exp:

R  0.6,140 
160
140 4
3
2
S  0.8,130 
120
100
Q  0.2,100 
Stress in

80 1
MPa

60
40
20
0
P  0,0 0.2 0.4 0.6 0.8 1

Strainin %

Toughness of material
Total area – [Area of 1 + Area of 2 + Area of 3 + Area of 4]

 140 

0.8   1
    100 
100   2  0.2
100
 40 
0.2
100
1

  40 
2
0.4
100
1
  10 
2
0.2 
100   
 1.12   0.1  0.08  0.08  0.01  0.85MJ m 3

 ICP–Intensive Classroom Program  eGATE-Live Internet Based Classes DLP  TarGATE-All India Test Series
Leaders in GATE Preparations  65+ Centers across India
© All rights reserved by Gateforum Educational Services Pvt. Ltd. No part of this booklet may be reproduced or utilized in any form without the written permission.

25
2525/
12525
Visit : /14
www.Civildatas.com
Visit : www.Civildatas.com

|ME| GATE-2016-PAPER-01 www.Civildatas.com

49. Heat is removed from a molten metal of mass 2 kg at a constant rate of 10 kW till it is completely
solidified. The cooling curve is shown in the figure.

Assuming uniform temperature throughout the volume of the metal during solidification, the latent heat
of fusion of the metal (in kJ/kg) is .
Key: (50)
Exp: Given 20sec
m  2 kg; Q  10 kcal 10s
873k
Since heat is removed at constant rate 873k
10kw is removed per second.
So, latest heat = mL  10 kW  10sec  10 sec is time requiredfor phasechange
2 L  100 kJ
L  50kJ / kg

The tool life equation for HSS tool is VT f d  Constant. The tool life (T) of 30 min is obtained
0.14 0.7 0.4
50.
using the following cutting conditions:
V = 45 m/min, f = 0.35 mm, d = 2.0 mm
If speed (V), feed (f) and depth of cut (d) are increased individually by 25%, the tool life (in min) is
(A) 0.15 (B) 1.06 (C) 22.50 (D) 30.0
Key: (B)
Exp: VT 0.14 f 0.7 d 0.4  constant

V1  45m/min; f1  0.35mm; d 1  2mm


T1  30min; V2  1.25V1;
f 2  1.25f1; d 1 1.25d1;
T2  ?

 45  T10.14 f10.7 d10.4  1.25  45  T20.14 1.250.7 f10.7  1.250.4 d10.4


 T10.14 1.25 1.250.7 1.250.4  T20.14
 30 
0.14

 T2   1.055 1.06
1.252.1 / 0.14

 ICP–Intensive Classroom Program  eGATE-Live Internet Based Classes DLP  TarGATE-All India Test Series
Leaders in GATE Preparations  65+ Centers across India
© All rights reserved by Gateforum Educational Services Pvt. Ltd. No part of this booklet may be reproduced or utilized in any form without the written permission.

26
2626/
12626
Visit : /14
www.Civildatas.com
Visit : www.Civildatas.com

|ME| GATE-2016-PAPER-01 www.Civildatas.com

51. A cylindrical job with diameter of 200 mm and height of 100 mm is to be cast using modulus
method of riser design. Assume that the bottom surface of cylindrical riser does not contribute as cooling
surface. If the diameter of the riser is equal to its height, then the height of the riser (in mm) is
(A) 150 (B) 200 (C) 100 (D) 125
Key: (A)
Exp: dc  200 mm dr  h r C  Casting
h c  100 mm hr  ? R  Rises

M r  1.2 MC

V V
   1.2  
 S r  S C
 2  2
d r hr dc h c
4  1.2 4
 
d r h r  d r2 d c h c  d c2 x 2
4 4
dr 2 h r 1.2d c2 h c
 
4d r h r  d r2 4d c h c  2d c2
1.2   200   100
2
h 3r
   d v  h r 
4h 2r  h 2r 4  200  100  2  2002
hr 200 10
  1.2 
5 4 100  2  200
200 100
 h r  1.2  5   125 1.2  150
4  200
52. A 300 mm thick slab is being cold rolled using roll of 600 mm diameter. If the coefficient of
friction is 0.08, the maximum possible reduction (in mm) is .
Key: 1.92
Exp:  h max   2 R

  0.08  300
2

 1.92 mm

53. The figure below represents a triangle PQR with initial coordinates of the vertices as P(1,3),
Q(4,5) and R(5,3.5). The triangle is rotated in the X-Y plane about the vertex P by angle θ in clockwise
direction. If sin θ = 0.6 and cos θ = 0.8, the new coordinates of the vertex Q are

(A) (4.6, 2.8) (B) (3.2, 4.6) (C) (7.9, 5.5) (D) (5.5, 7.9)
 ICP–Intensive Classroom Program  eGATE-Live Internet Based Classes DLP  TarGATE-All India Test Series
Leaders in GATE Preparations  65+ Centers across India
© All rights reserved by Gateforum Educational Services Pvt. Ltd. No part of this booklet may be reproduced or utilized in any form without the written permission.

27
2727/
12727
Visit : /14
www.Civildatas.com
Visit : www.Civildatas.com

|ME| GATE-2016-PAPER-01 www.Civildatas.com

Key: (A)
Exp: y

Q  4,5

P 1,3

x
0

Rotate PQ by  in clockwise direction


y Q  xo , y0 

Q'  x, y 

P 1,3, 

x
0
Rotation of point Q about point P in anticlockwise is given as

x n  x p   x o  x p  cos    yo  yp  sin  ...(1)

yn  yp   x o  x p  sin    yo  yp  cos  ...(2)

For clockwise rotation  will be   . So the desired equation will be

x n  x p   x o  x p  cos    yo  yp  sin  ...(3)

1   4  1  0.8   5  3  0.6
 1 2.4 1.2  4.6

yn  yp   yo  yp  sin    yo  yp  cos  ...(4)

 3   4  1  0.6   5  3  0.8
 3  1.8  1.6

yn  2.8

54. The annual demand for an item is 10,000 units. The unit cost is Rs. 100 and inventory carrying charges
are 14.4% of the unit cost per annum. The cost of one procurement is Rs. 2000. The time between two
consecutive orders to meet the above demand is ______ month(s).
Key: 2

 ICP–Intensive Classroom Program  eGATE-Live Internet Based Classes DLP  TarGATE-All India Test Series
Leaders in GATE Preparations  65+ Centers across India
© All rights reserved by Gateforum Educational Services Pvt. Ltd. No part of this booklet may be reproduced or utilized in any form without the written permission.

28
2828/
12828
Visit : /14
www.Civildatas.com
Visit : www.Civildatas.com

|ME| GATE-2016-PAPER-01 www.Civildatas.com

Exp: D= 10,000, Cu = Rs 100, Ch= 0.144×Cu, Co=Rs 2,000.


2DC0 2  10,000  2000 Q*
Q*    1666.67 units
Ch 0.144  100

Q * 1666.67
We know T    0.1667 years = 2 months
D 10,000
T

55. Maximize Z=15X1 + 20X2


subject to
12X1+ 4X2 ≥ 36
12X1 − 6X2 ≤ 24
X1, X2 ≥ 0
The above linear programming problem has
(A) infeasible solution (B) unbounded solution
(C) alternative optimum solutions (D) degenerate solution
Key: (B)
Exp: Max Z  15x1  20x 2
Subject to
12x1  4x 2  36
12x1  6x 2  24
x1 , x 2  0
Since, there is no limitation of boundary for the feasible region therefore, the LPP has unbounded
solution.

 ICP–Intensive Classroom Program  eGATE-Live Internet Based Classes DLP  TarGATE-All India Test Series
Leaders in GATE Preparations  65+ Centers across India
© All rights reserved by Gateforum Educational Services Pvt. Ltd. No part of this booklet may be reproduced or utilized in any form without the written permission.

29
2929/
12929
Visit : /14
www.Civildatas.com
Visit : www.Civildatas.com

|ME| GATE-2016-PAPER-02 www.Civildatas.com

General Aptitude
Q. No. 1 – 5 Carry One Mark Each

1. The volume of a sphere of diameter 1 unit is than the volume of a cube of side 1 unit.
(A) least (B) less (C) lesser (D) low
Key: (B)

2. The unruly crowd demanded that the accused be without trial.


(A) hanged (B) hanging (C) hankering (D) hung
Key: (A)

3. Choose the statement(s) where the underlined word is used correctly:


(i) A prone is a dried plum.
(ii) He was lying prone on the floor.
(iii) People who eat a lot of fat are prone to heart disease.
(A) (i) and (iii) only (B) (iii) only (C) (i) and (ii) only (D) (ii) and (iii) only
Key: (D)

4. Fact: If it rains, then the field is wet.


Read the following statements:
(i) It rains
(ii) The field is not wet
(iii) The field is wet
(iv) It did not rain
Which one of the options given below is NOT logically possible, based on the given fact?
(A) If (iii), then (iv). (B) If (i), then (iii).
(C) If (i), then (ii). (D) If (ii), then (iv).
Key: (C)

5. A window is made up of a square portion and an equilateral triangle portion above it. The base of the
triangular portion coincides with the upper side of the square. If the perimeter of the window is 6 m, the
area of the window in m2 is .
(A) 1.43 (B) 2.06 (C) 2.68 (D) 2.88

Key: (B) x
x
Exp:
3x+2x=6
5x=6
x=6/5 x
x
6 6 36
Area of square   
5 5 25
3 2 3 6 6 3 36
Area of triangle       .
4 4 5 5 4 25 x
 3  36
1     2.06
 4  25

 ICP–Intensive Classroom Program  eGATE-Live Internet Based Classes DLP  TarGATE-All India Test Series
Leaders in GATE Preparations  65+ Centers across India
© All rights reserved by Gateforum Educational Services Pvt. Ltd. No part of this booklet may be reproduced or utilized in any form without the written permission.

1
11/11
Visit : www.Civildatas.com
1/14
Visit : www.Civildatas.com

|ME| GATE-2016-PAPER-02 www.Civildatas.com

Q. No. 6 – 10 Carry Two Marks Each

6. Students taking an exam are divided into two groups, P and Q such that each group has the same number
of students. The performance of each of the students in a test was evaluated out of 200 marks. It was
observed that the mean of group P was 105, while that of group Q was 85. The standard deviation of
group P was 25, while that of group Q was 5. Assuming that the marks were distributed on a normal
distribution, which of the following statements will have the highest probability of being TRUE?
(A) No student in group Q scored less marks than any student in group P.
(B) No student in group P scored less marks than any student in group Q.
(C) Most students of group Q scored marks in a narrower range than students in group P.
(D) The median of the marks of group P is 100.
Key: (C)
68-95-97 rule
P Q

80 125 150 75 80 90 95
      2   2       2
105 85
 
95% of students in P scores between 65 to 150
95% of students in Q score between 75 to 95.
D is not correct
median = mean for normal distribution.
C is correct answer.
7. A smart city integrates all modes of transport, uses clean energy and promotes sustainable use of
resources. It also uses technology to ensure safety and security of the city, something which critics argue,
will lead to a surveillance state.
Which of the following can be logically inferred from the above paragraph?
(i) All smart cities encourage the formation of surveillance states.
(ii) Surveillance is an integral part of a smart city.
(iii) Sustainability and surveillance go hand in hand in a smart city.
(iv) There is a perception that smart cities promote surveillance.
(A) (i) and (iv) only (B) (ii) and (iii) only
(C) (iv) only (D) (i) only
Key: (C)

 ICP–Intensive Classroom Program  eGATE-Live Internet Based Classes DLP  TarGATE-All India Test Series
Leaders in GATE Preparations  65+ Centers across India
© All rights reserved by Gateforum Educational Services Pvt. Ltd. No part of this booklet may be reproduced or utilized in any form without the written permission.

2
22/12
Visit : www.Civildatas.com
2/14
Visit : www.Civildatas.com

|ME| GATE-2016-PAPER-02 www.Civildatas.com

8. Find the missing sequence in the letter series.


B, FH, LNP, _ _ _ _.
(A) SUWY (B) TUVW (C) TVXZ (D)TWXZ
Key: (C)

9. The binary operation  is defined as a  + b = ab+(a+b), where a and b are any two real numbers.
The value of the identity element of this operation, defined as the number x such that a  x = a, for any a,
is .
(A) 0 (B) 1 (C) 2 (D) 10
Key: (A)
Exp: ax  a  ax   a  x   a
 x 1  a   0  x  0 is the identity element

Which of the following curves represents the y  ln  e  for x  2 ?


 sin  x  

10. 
 
Here, x represents the abscissa and y represents the ordinate.

(A)

(B)

 ICP–Intensive Classroom Program  eGATE-Live Internet Based Classes DLP  TarGATE-All India Test Series
Leaders in GATE Preparations  65+ Centers across India
© All rights reserved by Gateforum Educational Services Pvt. Ltd. No part of this booklet may be reproduced or utilized in any form without the written permission.

3
33/13
Visit : www.Civildatas.com
3/14
Visit : www.Civildatas.com

|ME| GATE-2016-PAPER-02 www.Civildatas.com

(C)

(D)

Key: (C)

Mechanical Engineering

Q. No. 1 – 25 Carry One Mark Each

1. The condition for which the eigen values of the matrix


2 1 
A  are positive, is
1 k 
(A) k > 1/2 (B) k > −2 (C) k> 0 (D) k < −1/2
Key: (A)
Exp: By the properties of eigen values & eigen vectors, if all the principal minors of „A‟ are +Ve then all the
eigen values of „A‟ are also +Ve.
1
 A 22  0 for k 
2
1
So k 
2

 ICP–Intensive Classroom Program  eGATE-Live Internet Based Classes DLP  TarGATE-All India Test Series
Leaders in GATE Preparations  65+ Centers across India
© All rights reserved by Gateforum Educational Services Pvt. Ltd. No part of this booklet may be reproduced or utilized in any form without the written permission.

4
44/14
Visit : www.Civildatas.com
4/14
Visit : www.Civildatas.com

|ME| GATE-2016-PAPER-02 www.Civildatas.com

2. The values of x for which the function


x 2  3x  4
f (x)  is NOT continuous are
x 2  3x  4
(A) 4 and −1 (B) 4 and 1 (C) −4 and 1 (D) −4 and −1
Key: (C)
x 2  3x  4
Exp: The function f (x)  2 is not continuous at x  4 &1; since f(x) does not exists at x=-4 &1.
x  3x  4

3. Laplace transform of cos(ωt) is


s  s 
(A) (B) 
(C) (D)
s  2
2
s  2 s  2
2
s  2
2

Key: (A)
Exp: By the L.T of standard functions

4. A function f of the complex variable z  x  iy, is given as f (x, y)  u(x, y)  iv(x, y), where
u(x, y)  2kxy and v(x, y)  x 2  y 2 . The value of k, for which the function is analytic, is _____ .
Key: -1
Exp: From C-R equation; we have
u v u v
 & 
x y y dx
 u(x, y)  2kxy v(x, y)  x 2  y 2
u u v v
  2ky  2kx   2x;  2y
x y x y
u v
 
x y
 2ky  2y
 k  1

5. Numerical integration using trapezoidal rule gives the best result for a single variable function, which is
(A) linear (B) parabolic (C) logarithmic (D) hyperbolic
Key: (A)
6. A point mass having mass M is moving with a velocity V at an angle θ to the wall as shown in the figure.
The mass undergoes a perfectly elastic collision with the smooth wall and rebounds. The total change
(final minus initial) in the momentum of the mass is

(A) 2MV cos ˆj (B) 2MVsin ˆj (C) 2MVcos ˆj (D) 2MVsin ˆj
Key: (D)
 ICP–Intensive Classroom Program  eGATE-Live Internet Based Classes DLP  TarGATE-All India Test Series
Leaders in GATE Preparations  65+ Centers across India
© All rights reserved by Gateforum Educational Services Pvt. Ltd. No part of this booklet may be reproduced or utilized in any form without the written permission.

5
55/15
Visit : www.Civildatas.com
5/14
Visit : www.Civildatas.com

|ME| GATE-2016-PAPER-02 www.Civildatas.com

Exp: Let w is the velocity after collision

V sin j
w cos  i

V

 w

V cos  i  w sin  j
Beforecollision After collision

velocity of separation
e  1  e  1, for perfectly elastic collision
velocity of approach
 w sin ˆj  Vsin ˆj ______ (1)
Change in momentum = final momentum – initial momentum


P   w sin j  V sin j M  
 2MV sin  j  w sin j  V sin j
7. A shaft with a circular cross-section is subjected to pure twisting moment. The ratio of the
maximum shear stress to the largest principal stress is
(A) 2.0 (B) 1.0 (C) 0.5 (D) 0
Key: (B)
Exp: Mohr 's circle
xy xy

xy xy
xy
xy
max  xy
1  xy where, 1 is largest principal stress
max
 1
1

8. A thin cylindrical pressure vessel with closed-ends is subjected to internal pressure. The ratio of
circumferential (hoop) stress to the longitudinal stress is
(A) 0.25 (B) 0.50 (C) 1.0 (D) 2.0
Key: (D)

 ICP–Intensive Classroom Program  eGATE-Live Internet Based Classes DLP  TarGATE-All India Test Series
Leaders in GATE Preparations  65+ Centers across India
© All rights reserved by Gateforum Educational Services Pvt. Ltd. No part of this booklet may be reproduced or utilized in any form without the written permission.

6
66/16
Visit : www.Civildatas.com
6/14
Visit : www.Civildatas.com

|ME| GATE-2016-PAPER-02 www.Civildatas.com

pd
Exp: Circumferential stress  c  
2t
pd
Longitudinal stress    
4t
where, p is internal pressure
d is internal diameter
t is thickness
pd

 c  2t  2
 pd
4t

9. The forces F1 and F2 in a brake band and the direction of rotation of the drum are as shown in the figure.
The coefficient of friction is 0.25. The angle of wrap is 3π/2 radians. It is given that R = 1 m and
F2 = 1 N. The torque (in N-m) exerted on the drum is _____ .

Key: 2.248
F2
Exp:  exp   
F1
Torque   F2  F1  R

 1  exp   
 F2 R 

  3  
 1  1  1  exp  0.25     2.248 Nm
  2 
10. A single degree of freedom mass-spring-viscous damper system with mass m, spring constant k and
viscous damping coefficient q is critically damped. The correct relation among m, k, and q is

2k k
(A) q  2km (B) q  2 km (C) q  (D) q 
m m
Key: (B)
Exp: We know that
q
n
2m
k
q  2m   2 km
m

 ICP–Intensive Classroom Program  eGATE-Live Internet Based Classes DLP  TarGATE-All India Test Series
Leaders in GATE Preparations  65+ Centers across India
© All rights reserved by Gateforum Educational Services Pvt. Ltd. No part of this booklet may be reproduced or utilized in any form without the written permission.

7
77/17
Visit : www.Civildatas.com
7/14
Visit : www.Civildatas.com

|ME| GATE-2016-PAPER-02 www.Civildatas.com

11. A machine element XY, fixed at end X, is subjected to an axial load P, transverse load F, and a twisting
moment T at its free end Y. The most critical point from the strength point of view is

(A) a point on the circumference at location Y


(B) a point at the center at location Y
(C) a point on the circumference at location X
(D) a point at the center at location X
Key: (C)
Exp:
F.B.D
F
F T
T T
P P
P

Y MF M  F 
X

At location Y
At circumference
 Direct stress due to direct load P is 0  p A where A is cross section area.
 Due to shear load F is 1  F A
 d
 Due to torsion T is 2  16T d3 r  
 2
 Due to B.M F. dx is b  0
At centre of location Y
 0  P A

 1  F A

T 
 2  0 r  0  in relation 
J r
 b  0
At location X
At circumference
 0  P A

 1  F A

 ICP–Intensive Classroom Program  eGATE-Live Internet Based Classes DLP  TarGATE-All India Test Series
Leaders in GATE Preparations  65+ Centers across India
© All rights reserved by Gateforum Educational Services Pvt. Ltd. No part of this booklet may be reproduced or utilized in any form without the written permission.

8
88/18
Visit : www.Civildatas.com
8/14
Visit : www.Civildatas.com

|ME| GATE-2016-PAPER-02 www.Civildatas.com

 d
 2  16T / d 3  r  
 2
My 32M
 b   ( y  d / 2 and M  F  )
I d3
At center
 o  P / A
 1  F / A
 2  0  r  0 
 b  0  y  0 
 The most critical point is at circumference of location X.

12. For the brake shown in the figure, which one of the following is TRUE?

(A) Self energizing for clockwise rotation of the drum


(B) Self energizing for anti-clockwise rotation of the drum
(C) Self energizing for rotation in either direction of the drum
(D) Not of the self energizing type F
Key: (A) 
b
Exp: FBD of Lever
taking moment about hinge for clockwise rotation of wheel
f  N
Nb  F  Nc  0 c

N
F
 F N
 b  c  N

 b  c  
If b c  self energizing
So for clockwise rotation of the drum, the brake is self energizing.

13. The volumetric flow rate (per unit depth) between two streamlines having stream functions 1 and  2 is
(A) 1  2 (B) 1 2 (C) 1 /  2 (D) 1  2
Key: (D)
Exp: Volume flow rate per unit depth between two streamlines is given by 1   2
14. Assuming constant temperature condition and air to be an ideal gas, the variation in atmospheric pressure
with height calculated from fluid statics is
(A) linear (B) exponential (C) quadratic (D) cubic
Key: (B)
 ICP–Intensive Classroom Program  eGATE-Live Internet Based Classes DLP  TarGATE-All India Test Series
Leaders in GATE Preparations  65+ Centers across India
© All rights reserved by Gateforum Educational Services Pvt. Ltd. No part of this booklet may be reproduced or utilized in any form without the written permission.

9
99/19
Visit : www.Civildatas.com
9/14
Visit : www.Civildatas.com

|ME| GATE-2016-PAPER-02 www.Civildatas.com

15. A hollow cylinder has length L, inner radius r1, outer radius r2, and thermal conductivity k. The thermal
resistance of the cylinder for radial conduction is

ln  r2 / r1  ln  r1 / r2  2kL 2kL
(A) (B) (C) (D)
2kL 2kL ln  r2 / r1  ln  r1 / r2 
Key: (A)
Exp: For a hollow cylinder
2kL  T1  T2  k
Q
r 
ln  2  r1 r2
 r1 
T1  T2 T1  T2
 
 r2  R th
ln  
 r1 
2kL

16. Consider the radiation heat exchange inside an annulus between two very long concentric cylinders. The
radius of the outer cylinder is R0 and that of the inner cylinder is Ri. The radiation view factor of the
outer cylinder onto itself is
1/3
Ri R R  Ri
(A) 1  (B) 1 i (C) 1   i  (D) 1 
R0 R0  R0  R0

Key: (D) 2
1
Exp: F11  0
Ri
F12  1
A1 2R i L R i R0
A 2 F21  A1F1 2  F21   
A 2 2R 0 L R 0

Ri
 F2 2  1  F21  1 
R0

17. The internal energy of an ideal gas is a function of


(A) temperature and pressure
(B) volume and pressure
(C) entropy and pressure
(D) temperature only
Key: (D)
Exp: According to Joule‟s law,
Internal energy of an ideal gas is a function of temperature only.

 ICP–Intensive Classroom Program  eGATE-Live Internet Based Classes DLP  TarGATE-All India Test Series
Leaders in GATE Preparations  65+ Centers across India
© All rights reserved by Gateforum Educational Services Pvt. Ltd. No part of this booklet may be reproduced or utilized in any form without the written permission.

10
1010/
Visit : www.Civildatas.com
11010
Visit : www.Civildatas.com

|ME| GATE-2016-PAPER-02 www.Civildatas.com

18. The heat removal rate from a refrigerated space and the power input to the compressor are 7.2 kW and
1.8 kW, respectively. The coefficient of performance (COP) of the refrigerator is .
Key: 4
Q2 7.2
Exp: C.O.PR   4
W 1.8

Source

Q1

W  1.8kW
R
Q2  7.2kW

Refrigerated
Sink
space

19. Consider a simple gas turbine (Brayton) cycle and a gas turbine cycle with perfect regeneration. In both
the cycles, the pressure ratio is 6 and the ratio of the specific heats of the working medium is 1.4. The
ratio of minimum to maximum temperatures is 0.3 (with temperatures expressed in K) in the regenerative
cycle. The ratio of the thermal efficiency of the simple cycle to that of the regenerative cycle is ____ .
Key: 0.8021
Exp: Brayton cycle:-
3
rp  6
  1.4 T 2
1
brayton  1  1 4

(rp )
1 1
1 1.4 1
(6) 1.4
 0.4006 S
Gas Turbine cycle with perfect regeneration:-
rp  6
  1.4
  1  Cp  T5  T2   Cp  T3  T2  4
3
 T5  T3 2
T
T1
 0.3 5
T4
6
Heat supplied  Cp  T4  T3  1

 ICP–Intensive Classroom Program  eGATE-Live Internet Based Classes DLP  TarGATE-All India Test Series
Leaders in GATE Preparations  65+ Centers across India
© All rights reserved by Gateforum Educational Services Pvt. Ltd. No part of this booklet may be reproduced or utilized in any form without the written permission.

11
1111/
Visit : www.Civildatas.com
11111
Visit : www.Civildatas.com

|ME| GATE-2016-PAPER-02 www.Civildatas.com

Work done  WT  Wc
 Cp  T4  T5   Cp  T2  T1 
 Cp  T4  T3   Cp  T2  T1 

Work done Cp  T4  T3   Cp  T2  T1 
regenerative  
Heat supplied Cp  T4  T3 

T   T2 
T1  2  1  1 
 T2  T1   1
T   1  T1  T1
1   1 
 4
T  T3   T3  T4  1  T5 
T4 1    T 
 T4   4 

 
 1 
T1  p 
 r  1  1 1.4 1
  1   rp   1  0.3(6) 1.4  0.4994
T1
1  
T4  1  1  T4
 1

  rp   
 
brayton 0.4006
  0.8021
regenerative 0.4994

20. In a single-channel queuing model, the customer arrival rate is 12 per hour and the serving rate is 24 per
hour. The expected time that a customer is in queue is minutes.
Key: 2.5
 12
Exp:   12 hr ,   24 hr      0.5
 24
Let, expected time that a customer spend in queue is w q

  0.5  0.5
LqL   1 
wq   s   0.5  60
   12
0.5
  60  2.5mins
12

21. In the phase diagram shown in the figure, four samples of the same composition are heated to
temperatures marked by a, b, c and d.

At which temperature will a sample get solutionized the fastest?


(A) a (B) b (C) c (D) d
Key: (C)
 ICP–Intensive Classroom Program  eGATE-Live Internet Based Classes DLP  TarGATE-All India Test Series
Leaders in GATE Preparations  65+ Centers across India
© All rights reserved by Gateforum Educational Services Pvt. Ltd. No part of this booklet may be reproduced or utilized in any form without the written permission.

12
1212/
Visit : www.Civildatas.com
11212
Visit : www.Civildatas.com

|ME| GATE-2016-PAPER-02 www.Civildatas.com

22. The welding process which uses a blanket of fusible granular flux is
(A) tungsten inert gas welding (B) submerged arc welding
(C) electroslag welding (D) thermit welding
Key: (B)
Exp: Submerged arc welding uses a blanket of fusible granular flux

23. The value of true strain produced in compressing a cylinder to half its original length is
(A) 0.69 (B) − 0.69 (C) 0.5 (D) − 0.5
Key: (B)
Exp: Final length = L/2
Initial length = L
Lf L 1
T  ln  ln  ln  0.69
L0 2L 2

24. The following data is applicable for a turning operation. The length of job is 900 mm, diameter of job is
200 mm, feed rate is 0.25 mm/rev and optimum cutting speed is 300 m/min. The machining time (in min)
is ___.
Key: 7.539
Exp: L  900mm
d  200mm
f  0.25mm / rev
v  300m / min
t ?
v  DN m/min
1000
300 m/min =  0.2  N
300
N  478 RPM
 0.2
L 900
t   7.539min
fN 0.25  478

25. In an ultrasonic machining (USM) process, the material removal rate (MRR) is plotted as a function of
the feed force of the USM tool. With increasing feed force, the MRR exhibits the following behavior:
(A) increases linearly
(B) decreases linearly
(C) does not change
(D) first increases and then decreases
Key: (D)

 ICP–Intensive Classroom Program  eGATE-Live Internet Based Classes DLP  TarGATE-All India Test Series
Leaders in GATE Preparations  65+ Centers across India
© All rights reserved by Gateforum Educational Services Pvt. Ltd. No part of this booklet may be reproduced or utilized in any form without the written permission.

13
1313/
Visit : www.Civildatas.com
11313
Visit : www.Civildatas.com

|ME| GATE-2016-PAPER-02 www.Civildatas.com

Exp: In USM,

MRR Increase
Decrease due to Crushing
of abrasives

Feed force of tool

Q. No. 26 – 55 carry Two Marks Each

26. A scalar potential  has the following gradient.   yziˆ  xzjˆ  xyk.
ˆ Consider the integral

 
 .dr on the curve r  xiˆ  yjˆ  zk.
ˆ
c

x  t

The curve C is parameterized as follows:  y  t 2 and 1  t  3.
z  3t 2

The value of the integral is______ .

Key: 726

 .dr    yziˆ  xzjˆ  xykˆ .dxiˆ  dyjˆ  dzkˆ 



Exp:
c c

  yzdx  xzdy  xydz … (1)


c

 x  t; y  t 2 ; z  3t 2

 dx  dt  dy  2tdt  dz  6tdt


From (1);  .dr   t 2 (3t 2 )dt  t  3t 2  2tdt  t  t 2  6tdt
c c

  3t   6t 4  6t 4 dt
3

t 1

3
 t5 
  15t dt  15    3 35  1
3
4
t 1
 5 1

 726

 ICP–Intensive Classroom Program  eGATE-Live Internet Based Classes DLP  TarGATE-All India Test Series
Leaders in GATE Preparations  65+ Centers across India
© All rights reserved by Gateforum Educational Services Pvt. Ltd. No part of this booklet may be reproduced or utilized in any form without the written permission.

14
1414/
Visit : www.Civildatas.com
11414
Visit : www.Civildatas.com

|ME| GATE-2016-PAPER-02 www.Civildatas.com

3z  5
27. The value of  (z 1)(z 2) dz
r
along a closed path  is equal to (4i), where z=x+iy and i  1. The

correct path  is
(A) (B)

(C) (D)

Key: (B)
Exp: Since if z=1 lies inside the closed path  and z=2 lies outside of the closed path  then by cauchy‟s
formula.
3z  5
3z  5
M (z 1)(z 2) dz  n zz  12 dz
 3z  5 
 2i  
 z  2  at z 1
 2 
 2i    4i
 1 

28. The probability that a screw manufactured by a company is defective is 0.1. The company sells screws in
packets containing 5 screws and gives a guarantee of replacement if one or more screws in the packet are
found to be defective. The probability that a packet would have to be replaced is .
Key: 0.40951
Exp: Probability that a packet would have to be replaced i.e., P  X  1  ? [ Let „x‟ denote the number of
defective screws]
 P  X  1  1  P  X  1
 1  P  X  0
 1  5C0 (0.1)0 (0.9)5
 1   0.9   0.40951
5

Since by the Binomial distribution when P=probability of defective screw.



29. The error in numerically computing the integral   sin x  cos x  dx using the trapezoidal rule with three
0
intervals of equal length between 0 and  is_______ .
Key: 0.178
 ICP–Intensive Classroom Program  eGATE-Live Internet Based Classes DLP  TarGATE-All India Test Series
Leaders in GATE Preparations  65+ Centers across India
© All rights reserved by Gateforum Educational Services Pvt. Ltd. No part of this booklet may be reproduced or utilized in any form without the written permission.

15
1515/
Visit : www.Civildatas.com
11515
Visit : www.Civildatas.com

|ME| GATE-2016-PAPER-02 www.Civildatas.com

ba 0 
Exp: h   ; f (x)  sin x  cos x
n 3 3
 2
x 0 
3 3
y  f (x) 1 1.37 0.37 1
By trapezoidal rule; we have the approximate value of the integral is
 /3
0 (sin x  cos x)dx  2 1  (1)  2(1.37  0.37)
 1.822
Exact value of the integral is

(sinx  cosx)dx    cos x  sin x 0  1  (1)  2

0

Error  Exact value  Approximate value


 2  1.822  0.178

30. A mass of 2000 kg is currently being lowered at a velocity of 2 m/s from the drum as shown in the figure.
The mass moment of inertia of the drum is 150 kg-m2. On applying the brake, the mass is brought to rest
in a distance of 0.5 m. The energy absorbed by the brake (in kJ) is

Key: 14.11
Exp:
V  R
V 2.0
i  i   2rad/sec
R 1

J  i2  f2 
1
Loss in kinetic energy of Drum =
2
1
 KE drum  150   2   300 Joule
2

2
1
Loss in kinetic energy of block =  m vi2  vf2
2
 
1
  2000   2   02 
2

2  
KE block  4000 Joule
Loss of Potential energy of block = mgh
 2000 9.81  0.5
PE block  9810 Joule

 ICP–Intensive Classroom Program  eGATE-Live Internet Based Classes DLP  TarGATE-All India Test Series
Leaders in GATE Preparations  65+ Centers across India
© All rights reserved by Gateforum Educational Services Pvt. Ltd. No part of this booklet may be reproduced or utilized in any form without the written permission.

16
1616/
Visit : www.Civildatas.com
11616
Visit : www.Civildatas.com

|ME| GATE-2016-PAPER-02 www.Civildatas.com

Total energy loss  KEdrum  KE block  PE block


 300  4000  9810 14110 joule
 14.11 kJ

31. A system of particles in motion has mass center G as shown in the figure. The particle i has mass mi and
its position with respect to a fixed point O is given by the position vector ri. The position of the particle
with respect to G is given by the vector  i . The time rate of change of the angular momentum of the
system of particles about G is
(The quantity i indicates second derivative of  i with respect to time and likewise for ri ).

(A) i ri  mii (B) i i  miri (C) i ri  miri (D) i i  mii


Key: (B)
Exp: By definition of Torque
 
Torque  rPOR  Fext
where

rPOR = position vector with respect to point of rotation (POR) of the particle on which force is acting.

Fext  External force acting on ith particle

 miri
 
i  i  mi ri
For complete Rigid body
 
 i   i  mi ri
i

32. A rigid horizontal rod of length 2L is fixed to a circular cylinder of radius R as shown in the figure.
Vertical forces of magnitude P are applied at the two ends as shown in the figure. The shear modulus for
the cylinder is G and the Young‟s modulus is E.

The vertical deflection at point A is


(A) PL3 / R 4 G   (B) PL3 / R 4 E  (C) 2PL3 / R 4 E   (D) 4PL3 / R 4 G  
Key: (D)

 ICP–Intensive Classroom Program  eGATE-Live Internet Based Classes DLP  TarGATE-All India Test Series
Leaders in GATE Preparations  65+ Centers across India
© All rights reserved by Gateforum Educational Services Pvt. Ltd. No part of this booklet may be reproduced or utilized in any form without the written permission.

17
1717/
Visit : www.Civildatas.com
11717
Visit : www.Civildatas.com

|ME| GATE-2016-PAPER-02 www.Civildatas.com

Exp: F.B.D
T

T  2PL

T
Because of torsion angle of twist    will be there.
T 2  32 P2 4 P2
Where    
G.J G  d 4 R 4G
Due to angle of twist, A will reach at A ' and B will reach at B'
let A 'A'' be the vertical displacement
A' B
 
x

O
A '' 
A
B'
In  A'OA''
A'A'' x
sin    where, x is vertical deflection
A'O 
x
When  is very small, sin      x  

4P 3
x
R 4G
33. A simply supported beam of length 2L is subjected to a moment M at the mid-point x = 0 as shown in the
figure. The deflection in the domain 0 ≤ x ≤ L is given by
Mx
W  L  x  (x  c),
12EIL
where E is the Young‟s modulus, I is the area moment of inertia and c is a constant (to be determined) .

The slope at the center x = 0 is


(A) ML / (2EI) (B) ML / (3EI) (C) ML / (6EI) (D) ML / (12EI)
Key: (C)

 ICP–Intensive Classroom Program  eGATE-Live Internet Based Classes DLP  TarGATE-All India Test Series
Leaders in GATE Preparations  65+ Centers across India
© All rights reserved by Gateforum Educational Services Pvt. Ltd. No part of this booklet may be reproduced or utilized in any form without the written permission.

18
1818/
Visit : www.Civildatas.com
11818
Visit : www.Civildatas.com

|ME| GATE-2016-PAPER-02 www.Civildatas.com

Exp: As we know that



M2dx
U
0 2EI

U
and slope at mid-span where moment is applied will be (according to Costigliano‟s theorem).
M

Mx
M 0  x  
2

Mx
M M    x  2 
2 x

 
   Mx 2 2
 Mx 
2
 1
 U     dx     M  dx  
 0  2    2   2EI x
M M M

2
M2  x 2  x  2  dx  2
 2 2

U    2 dx    x
2EI  0 4  4 2 

M2   3   3  M 2 
     
2EI  4 2  3  3  12EI

 U  M
 slopeat  x  0     
 M  x 0 6EI
34. In the figure, the load P = 1 N, length L = 1 m, Young‟s modulus E = 70 GPa, and the cross-section of the
links is a square with dimension 10 mm × 10 mm. All joints are pin joints.

The stress (in Pa) in the link AB is ________ .


(Indicate compressive stress by a negative sign and tensile stress by a positive sign.)
Key: 0

 ICP–Intensive Classroom Program  eGATE-Live Internet Based Classes DLP  TarGATE-All India Test Series
Leaders in GATE Preparations  65+ Centers across India
© All rights reserved by Gateforum Educational Services Pvt. Ltd. No part of this booklet may be reproduced or utilized in any form without the written permission.

19
1919/
Visit : www.Civildatas.com
11919
Visit : www.Civildatas.com

|ME| GATE-2016-PAPER-02 www.Civildatas.com

Exp: F.B.D of point B


FBC

45
45 B

P
FAB

Applying Lame‟s theorem


FAB P F
  BC
sin180 sin135 sin 45
 FAB  0
Stress in AB  0

35. A circular metallic rod of length 250 mm is placed between two rigid immovable walls as shown in the
figure. The rod is in perfect contact with the wall on the left side and there is a gap of 0.2 mm between
the rod and the wall on the right side. If the temperature of the rod is increased by 200 o C, the axial stress
developed in the rod is MPa.
Young‟s modulus of the material of the rod is 200 GPa and the coefficient of thermal expansion is 10−5
per oC.

Key: 240
Exp: t  200C
  105 C
E  2 105 MPa
  0.2 mm
We know that, Axial stress gets induced in the rod when some gap „  ‟ is provided is
 t      105  200  250  0.2 
  E     2 10
5

    250 
 240 MPa

 ICP–Intensive Classroom Program  eGATE-Live Internet Based Classes DLP  TarGATE-All India Test Series
Leaders in GATE Preparations  65+ Centers across India
© All rights reserved by Gateforum Educational Services Pvt. Ltd. No part of this booklet may be reproduced or utilized in any form without the written permission.

20
2020/
Visit : www.Civildatas.com
12020
Visit : www.Civildatas.com

|ME| GATE-2016-PAPER-02 www.Civildatas.com

36. The rod AB, of length 1 m, shown in the figure is connected to two sliders at each end through pins. The
sliders can slide along QP and QR. If the velocity VA of the slider at A is 2 m/s, the velocity of the
midpoint of the rod at this instant is m/s.

Key: 1
VA =2m/sec 2cos60  1
Exp: Given AB  2 60 2sin60  3
A
Since Rod AB is rigid, so A
Axial velocity of A & B should be same C
C 2cos60  1
VA cos60  VB cos60 60 60 B
Q B
VA  VB  2m/sec VB

C is mid point of AB 2sin60  3

A 3
A

C
 C  Vc  1

B
1 B
 3

Velocity corresponding to Velocity corresponding to


pure translation part Rotational part

Alternate Method:
 2   2  x 2
2 2
y a
cos120  x
30o
2 2 2 c
2
2
1  x  120o
 1    30o
2 2 2 b o
2
3
x  2 22 3
2
x
y  3,so oc will be perpendicular toab.
2
v
sin  30   c  vc 1
2

 ICP–Intensive Classroom Program  eGATE-Live Internet Based Classes DLP  TarGATE-All India Test Series
Leaders in GATE Preparations  65+ Centers across India
© All rights reserved by Gateforum Educational Services Pvt. Ltd. No part of this booklet may be reproduced or utilized in any form without the written permission.

21
2121/
Visit : www.Civildatas.com
12121
Visit : www.Civildatas.com

|ME| GATE-2016-PAPER-02 www.Civildatas.com

37. The system shown in the figure consists of block A of mass 5 kg connected to a spring through a massless
rope passing over pulley B of radius r and mass 20 kg. The spring constant k is 1500 N/m. If there is no
slipping of the rope over the pulley, the natural frequency of the system is rad/s.

Key: 10
Exp: Displace the block “A” & Release

 k r r   
1 2
Mr  mr 2  
 0
2 
 
  k 
   0
 1 M m 
2 
 
 k  1500
n     10 rad/sec
 1Mm  10  5
2 

Alternate method:
1 1 1
Energy of system remain conserved, E  J2  mv 2  mg y  k  y    …(1)
2

2 2 2

Rotational Translational Gravitational P.E stored


K.E K.E K.E in spring
where,
  Static elongation of spring at equilibrium which is calculated as follows:

mg
mg  k   
k
Differentiating Eqn.(1)w.r.t time, which will be zero because E  constant
dE
0
dt
d dv
J  mv  mgv  k  y    v  0 ...(2)
dt dt
Since there is no slipping between rope & pulley
v  r

 ICP–Intensive Classroom Program  eGATE-Live Internet Based Classes DLP  TarGATE-All India Test Series
Leaders in GATE Preparations  65+ Centers across India
© All rights reserved by Gateforum Educational Services Pvt. Ltd. No part of this booklet may be reproduced or utilized in any form without the written permission.

22
2222/
Visit : www.Civildatas.com
12222
Visit : www.Civildatas.com

|ME| GATE-2016-PAPER-02 www.Civildatas.com

1 2  v  1 dv  dv
Mr     mv  mgv  k  y    v  0
2  r  r dt  dt
M d y
2

  m  2  ky  0
 2  dt
k 1500
n   10rad/sec
M  10  5
  m
 2 

38. In a structural member under fatigue loading, the minimum and maximum stresses developed at the
critical point are 50 MPa and 150 MPa, respectively. The endurance, yield, and the ultimate strengths of
the material are 200 MPa, 300 MPa and 400 MPa, respectively. The factor of safety using modified
Goodman criterion is

3 8 12
(A) (B) (C) (D) 2
2 5 7
Key: (D)
Exp: Given

max 150MPa, min  50MPa, m  100MPa

a  50MPa,Se  200MPa,Sut  300MPa

Sut  400MPa

Equation of line 1

a  m
 1 ...(1)
Se Sut

Equation of line 2

a 1
 tan   ...(2)
m 2

Solving above two equations to get co-ordinates of point P (Sm, Sa)

Sa S
 m 1
200 400

2 Sa  Sm  400 ...(3)

Sm  2Sa ...(4)

from (3) & (4)

Sa  100MPa
Sa 100
f.o.s   2
a 50

 ICP–Intensive Classroom Program  eGATE-Live Internet Based Classes DLP  TarGATE-All India Test Series
Leaders in GATE Preparations  65+ Centers across India
© All rights reserved by Gateforum Educational Services Pvt. Ltd. No part of this booklet may be reproduced or utilized in any form without the written permission.

23
2323/
Visit : www.Civildatas.com
12323
Visit : www.Civildatas.com

|ME| GATE-2016-PAPER-02 www.Civildatas.com

39. The large vessel shown in the figure contains oil and water. A body is submerged at the interface of oil
and water such that 45 percent of its volume is in oil while the rest is in water. The density of the body is
______kg/m3.
The specific gravity of oil is 0.7 and density of water is 1000 kg/m3.
Acceleration due to gravity g = 10 m/s2.

Key: 865
Exp: Given water  1000 kg m3
oil  700 kg m3
body  ?
Voil  0.45Vbody
Vwater  0.55Vbody
oil Voil g  water Vwater g  body Vbody g
700  0.45Vbody  1000  0.55Vbody  body Vbody
body  865kg m3

40. Consider fluid flow between two infinite horizontal plates which are parallel (the gap between them being
50 mm). The top plate is sliding parallel to the stationary bottom plate at a speed of 3 m/s. The flow
between the plates is solely due to the motion of the top plate. The force per unit area (magnitude)
required to maintain the bottom plate stationary is N/m2.
Viscosity of the fluid µ = 0.44 kg/m-s and density ρ = 888 kg/m3.
Key: 26.4
Exp: Vtop  3m / s
50 mm
  0.44 kg m  s
  888kg m3

du Vtop  Vbottom 3
   60
dy 0.05 0.05
du
J wall    0.44  60 kg m.s 2
dy
 26.4 N m2
 ICP–Intensive Classroom Program  eGATE-Live Internet Based Classes DLP  TarGATE-All India Test Series
Leaders in GATE Preparations  65+ Centers across India
© All rights reserved by Gateforum Educational Services Pvt. Ltd. No part of this booklet may be reproduced or utilized in any form without the written permission.

24
2424/
Visit : www.Civildatas.com
12424
Visit : www.Civildatas.com

|ME| GATE-2016-PAPER-02 www.Civildatas.com

41. Consider a frictionless, massless and leak-proof plug blocking a rectangular hole of dimensions 2R  L
the bottom of an open tank as shown in the figure. The head of the plug has the shape of a semi-cylinder
of radius R. The tank is filled with a liquid of density ρ up to the tip of the plug. The gravitational
acceleration is g. Neglect the effect of the atmospheric pressure.

The force F required to hold the plug in its position is


     2
(A) 2R 2 gL 1   (B) 2R 2 gL 1   (C) R 2gL (D) R gL
 4  4 2
Key: (A)
Exp:

2R
Downward force due to water = weight of water above curved surface
  R 2 L 
 2  g  R 2 L  
  4 
 
 2gR 2 L 1   N
 4
Weight of plug is neglected.

42. Consider a parallel-flow heat exchanger with area Ap and a counter-flow heat exchanger with area Ac.
In both the heat exchangers, the hot stream flowing at 1 kg/s cools from 80 ℃ to 50 ℃. For the cold
stream in both the heat exchangers, the flow rate and the inlet temperature are 2 kg/s and 10 ℃,
respectively. The hot and cold streams in both the heat exchangers are of the same fluid. Also, both the
heat exchangers have the same overall heat transfer coefficient. The ratio Ac / Ap is ________ .
Key: 0.928
Exp: mh  1kg / s
mc  2kg / s
CPh  Cpc (as both are same fluids)

 
mk Cpk t k1  t k 2  mc Cpc t c2  t c1  
 
1(80  50)  2 t c2  10  t c2  25 C o

 ICP–Intensive Classroom Program  eGATE-Live Internet Based Classes DLP  TarGATE-All India Test Series
Leaders in GATE Preparations  65+ Centers across India
© All rights reserved by Gateforum Educational Services Pvt. Ltd. No part of this booklet may be reproduced or utilized in any form without the written permission.

25
2525/
Visit : www.Civildatas.com
12525
Visit : www.Civildatas.com

|ME| GATE-2016-PAPER-02 www.Civildatas.com

Parallel flow
T
T Counter flow

t h1  80o C t h1  80o C
1  t k1  t c2
t h2  50o C t c2  25o C
t h 2  50o C
1  t k1  t c1 2  t k2  t c2 2  t k2  t c1
t c2  25o C t c2  10o C

t c1  10o C

L
L

1  70o C 1  80  25  55o C
2  25o C 2  50  10  40o C
  1  2
mp  1 2 mc 
   
ln  1  ln  1 
 2   2 
70  25 55  40
 
 70   55 
ln   ln  
 25   40 
 43.705  47.1
Q  U.Ap mp Q  U.A c .mc
 A p mp  A c .mc
A c mp 43.705
   0.928
A p mc 47.1

43. Two cylindrical shafts A and B at the same initial temperature are simultaneously placed in a furnace.
The surfaces of the shafts remain at the furnace gas temperature at all times after they are introduced into
the furnace. The temperature variation in the axial direction of the shafts can be assumed to be negligible.
The data related to shafts A and B is given in the following Table.

Quantity Shaft A Shaft B


Diameter (m) 0.4 0.1
Thermal conductivity (W/m-K) 40 20
Volumetric heat capacity (J/m3-K) 2×106 2×107

The temperature at the centerline of the shaft A reaches 400℃ after two hours. The time required
(in hours) for the centerline of the shaft B to attain the temperature of 400℃ is _____ .
Key: 2.4 – 2.6

 ICP–Intensive Classroom Program  eGATE-Live Internet Based Classes DLP  TarGATE-All India Test Series
Leaders in GATE Preparations  65+ Centers across India
© All rights reserved by Gateforum Educational Services Pvt. Ltd. No part of this booklet may be reproduced or utilized in any form without the written permission.

26
2626/
Visit : www.Civildatas.com
12626
Visit : www.Civildatas.com

|ME| GATE-2016-PAPER-02 www.Civildatas.com

44. A piston-cylinder device initially contains 0.4 m3 of air (to be treated as an ideal gas) at 100 kPa and
80oC. The air is now isothermally compressed to 0.1 m3. The work done during this process is
kJ.
(Take the sign convention such that work done on the system is negative)
Key: -55.45
Exp: V1  0.4m3
P1  100kPa
T1  80o C  353K
V2  0.1 m3
Ideal gas & process is isothermal.
V 
 W  P1V1 ln  2 
 V1 
 0.1 
 100  0.4ln  
 0.4 
 55.45kJ

45. A reversible cycle receives 40 kJ of heat from one heat source at a temperature of 127 ℃ and 37 kJ from
another heat source at 97 ℃. The heat rejected (in kJ) to the heat sink at 47 ℃ is .
Key: 64
Exp: Reversible cycle.
T1  127  273 T2  97  273
 400K  370K

Q1  40kJ Q2  37kJ

Q3

T3  47  273
 320K
Q
 0
T
Q1 Q2 Q3 40 37 Q3
  0   0
T1 T2 T3 400 370 320
Q3 1 320
  Q3   64kJ
320 5 5

46. A refrigerator uses R-134a as its refrigerant and operates on an ideal vapour-compression
refrigeration cycle between 0.14 MPa and 0.8 MPa. If the mass flow rate of the refrigerant is 0.05 kg/s,
the rate of heat rejection to the environment is kW.
Given data:
At P = 0.14 MPa, h = 236.04 kJ/kg, s=0.9322 kJ/kg-K
At P = 0.8 MPa, h = 272.05 kJ/kg (superheated vapour)
At P = 0.8MPa, h = 93.42 kJ/kg (saturated liquid)
Key: 8.9315
 ICP–Intensive Classroom Program  eGATE-Live Internet Based Classes DLP  TarGATE-All India Test Series
Leaders in GATE Preparations  65+ Centers across India
© All rights reserved by Gateforum Educational Services Pvt. Ltd. No part of this booklet may be reproduced or utilized in any form without the written permission.

27
2727/
Visit : www.Civildatas.com
12727
Visit : www.Civildatas.com

|ME| GATE-2016-PAPER-02 www.Civildatas.com

Exp: Given
h1  236.04kJ / kg P 0.8MPa 3
s1  0.9322 kJ / kg 2
h 2  272.05kJ / kg
h 3  93.42kJ / kg
0.14MPa 4 1
m  0.05kg / s

  h 2  h3 
 Heat Rejection to environment  m
 0.05  272.05  93.42  h

 8.9315kW

47. The partial pressure of water vapour in a moist air sample of relative humidity 70% is 1.6 kPa, the total
pressure being 101.325 kPa. Moist air may be treated as an ideal gas mixture of water vapour and dry air.
The relation between saturation temperature (Ts in K) and saturation pressure (ps in kPa) for water is
given by ln  ps / p0  14.317  5304 / Ts , where p0  101.325 kPa. The dry bulb temperature of the moist
air sample (in ℃) is
Key: 19.89
Exp: PV  1.6 kPa
T
  70% Ps  2.2857 kPa
Po  101.325 kPa PV  1.6 kPa
P  5304
ln  s  14.317   Ps  ? TSat
d.b.t  TSat  ?
 po  Ts  2
Pv

Psat
1 d.p.t
1.6 1.6
0.7   Ps   2.2857 kPa
Ps 0.7 S

 2.2857  5304
ln   14.317 
 101.325  Tsat
5304
Tsat   292.898 K
 2.2857 
14.317  n  
 101.325 
d.b.t  t sat  19.89o C

48. In a binary system of A and B, a liquid of 20% A (80% B) is coexisting with a solid of 70% A (30% B).
For an overall composition having 40% A, the fraction of solid is
(A) 0.40 (B) 0.50 (C) 0.60 (D) 0.75
Key: (A)

 ICP–Intensive Classroom Program  eGATE-Live Internet Based Classes DLP  TarGATE-All India Test Series
Leaders in GATE Preparations  65+ Centers across India
© All rights reserved by Gateforum Educational Services Pvt. Ltd. No part of this booklet may be reproduced or utilized in any form without the written permission.

28
2828/
Visit : www.Civildatas.com
12828
Visit : www.Civildatas.com

|ME| GATE-2016-PAPER-02 www.Civildatas.com

 0.2L  0.7S  0.4 20% A 70% A


 L  S 
80% B 30% B
0.2L  0.7S  0.4L  0.4S
Liquid (L) Solid (S)
0.2L  0.3S ...(1)
S
Fraction of solid 
S L  0.2L  0.7S % A


S  0.8L  0.3S % B
0.3
S S Mixture of liquid&solid
0.2
0.2
  0.40
0.5

49. Gray cast iron blocks of size 100 mm × 50 mm × 10 mm with a central spherical cavity of diameter 4 mm
are sand cast. The shrinkage allowance for the pattern is 3%. The ratio of the volume of the pattern to
volume of the casting is ______ .
Key: 1.08 – 1.10

50. The voltage-length characteristic of a direct current arc in an arc welding process is
V  100 + 40  , where l is the length of the arc in mm and V is arc voltage in volts. During a welding
operation, the arc length varies between 1 and 2 mm and the welding current is in the range 200-250 A.
Assuming a linear power source, the short circuit current is A.
Key: 424.6
Exp: V  100  40 1  1mm &  2  2mm
V1  140V I1  250A
V2  180V I2  200A
V I
 1
0CV SCC
140 250
 1 …(a)
OCV SCC
180 200
 1 …(b)
OCV SCC
On solving equation a and b, we have
S.C.C=424.6A

51. For a certain job, the cost of metal cutting is Rs. 18C/V and the cost of tooling is Rs. 270 C/(TV), where
C is a constant, V is the cutting speed in m/min and T is the tool life in minutes. The Taylor's tool life
equation is VT0.25  150. The cutting speed (in m/min) for the minimum total cost is .
Key: 57.9
Exp: Cost of metal cutting = Rs 18 C/V
Cost of Tooling = Rs 270 C/TV
 ICP–Intensive Classroom Program  eGATE-Live Internet Based Classes DLP  TarGATE-All India Test Series
Leaders in GATE Preparations  65+ Centers across India
© All rights reserved by Gateforum Educational Services Pvt. Ltd. No part of this booklet may be reproduced or utilized in any form without the written permission.

29
2929/
Visit : www.Civildatas.com
12929
Visit : www.Civildatas.com

|ME| GATE-2016-PAPER-02 www.Civildatas.com

C= Constant ,V = Cutting Speed , T = tool life

C = 150, VT0.25 = 150  T  150 V 


1 0.25

T  150 V 
4

C C
Total cost  18  270
V TV
18C 270CV 4
 
V 1504 V
18C 270C V3
 
V 1504
On, differentiating total cost
18C 270 C3V 2
 
V2 1504
18 1504
V4 
3  270
 V  57.914 m min

52. The surface irregularities of electrodes used in an electrochemical machining (ECM) process are 3 µm
and 6 µm as shown in the figure. If the work-piece is of pure iron and 12V DC is applied between the
electrodes, the largest feed rate is mm/min.
Conductivity of the electrolyte 0.02 ohm-1mm-1
Over-potential voltage 1.5 V
Density of iron 7860 kg/m3
Atomic weight of iron 55.85 gm
+2
Assume the iron to be dissolved as Fe and the Faraday constant to be 96500 Coulomb.

Key: 51.51
Exp: Given V = 12V V  1.5V

 ICP–Intensive Classroom Program  eGATE-Live Internet Based Classes DLP  TarGATE-All India Test Series
Leaders in GATE Preparations  65+ Centers across India
© All rights reserved by Gateforum Educational Services Pvt. Ltd. No part of this booklet may be reproduced or utilized in any form without the written permission.

30
3030/
Visit : www.Civildatas.com
13030
Visit : www.Civildatas.com

|ME| GATE-2016-PAPER-02 www.Civildatas.com

1
k  0.02 ohm 1 mm 1  0.02  0.2ohm 1 cm 1
ohm101 cm
  7860 kg m3
103
 7860  gm cm3
106
  55.85gm
 7.860 gm cm3
AI 55.85I
MRR : Q :   3.68 105 cm3 sec
ZF 7.86  2  96600
Inter electrode gap given  9 m  9 104 cm
k  V  V 
current density J 
y
0.2 12  1.5
  2333.33
9 104
I  J  S.A
I  2333.33  S.A  S.A  I 2333.33
Electrode feed rate = MRR surface area cm sec
3.68 105 I  2333.33
 cm sec
I
 0.086 10  60 mm min
 51.51mm min
53. For the situation shown in the figure below the expression for H in terms of r, R and D is

(A) H  D  r 2  R 2 (B) H  (R  r)  (D  r)

(C) H  (R  r)  D 2  R 2 (D) H  (R  r)  2D(R  r)  D2


Key: (D)

 ICP–Intensive Classroom Program  eGATE-Live Internet Based Classes DLP  TarGATE-All India Test Series
Leaders in GATE Preparations  65+ Centers across India
© All rights reserved by Gateforum Educational Services Pvt. Ltd. No part of this booklet may be reproduced or utilized in any form without the written permission.

31
3131/
Visit : www.Civildatas.com
13131
Visit : www.Civildatas.com

|ME| GATE-2016-PAPER-02 www.Civildatas.com

Exp:

a c
H

a  R  r, b  D   R  r  , C  a 2  b 2

 R  r   D   R  r 
2
C 
2

H  R  r  C  R  r  R  r   D2   R  r   2D  R  r 
2 2

H   R  r   2D  R  r   D2

54. A food processing company uses 25,000 kg of corn flour every year. The quantity-discount price of corn
flour is provided in the table below:

Quantity (kg) Unit price (Rs/kg)


1-749 70
750-1499 65
1500 and above 60

The order processing charges are Rs. 500/order. The handling plus carry-over charge on an annual basis
is 20% of the purchase price of the corn flour per kg. The optimal order quantity (in kg) is .
Key: 1500
Exp: D = 25000 kg, C0 = Rs 500/order, Ch = 20% of Cu

Qty (kg) Cu (Rs/kg) Ch (Rs/Kg/year)


1  Q1  750 70 0.2  70  14
750  Q2  1500 65 0.2  65  13
Q3  1500 60 0.2  60  12

This problem belongs to inventory model with two price break.


2DCo
Q 
Ch
 first checking for least unit price

2  25000  500
Q*3   1443.37
12
Now, 1443.37 < 1500 therefore, the company will not get the item at Rs 60/kg
Now, checking for second minimum unit price
 ICP–Intensive Classroom Program  eGATE-Live Internet Based Classes DLP  TarGATE-All India Test Series
Leaders in GATE Preparations  65+ Centers across India
© All rights reserved by Gateforum Educational Services Pvt. Ltd. No part of this booklet may be reproduced or utilized in any form without the written permission.

32
3232/
Visit : www.Civildatas.com
13232
Visit : www.Civildatas.com

|ME| GATE-2016-PAPER-02 www.Civildatas.com

2  25000  500
Q*2   1386.75
13
Since, 1386.75 lies between 750 and 1500
Therefore, we need to find

 
Total cost Q2  25000  65 
25000
1386.75
 500 
1386.75
2
13

 Rs1643027.755

 D Q 
T.C  D  Cu  Q  C0  2  Ch 
 
25000 1500
Totalcost 1500   25000  60   500  12  Rs 1517333.33
1500 2
Since, T.C (1500) < T.C Q*2 0  
Therefore, optimal order quantity is 1500.

55. A project consists of 14 activities, A to N. The duration of these activities (in days) are shown in brackets
on the network diagram. The latest finish time (in days) for node 10 is

Key: 14
Exp:
E5
E9
L  10
L  14
3 4
10
3 E  10
8 L  12
2 3
E2 6
L2 2 E 8
3
2 L  10 2 2
4 4 9 12
1 2 11
E0 E6 4 5 E  15 E  17 E  19
L0 L6 L  15 L  17 L  19
7
2 E  10 4
L  10

5
E4
L  11
The latest finish time for node 10 is 14 days.
 ICP–Intensive Classroom Program  eGATE-Live Internet Based Classes DLP  TarGATE-All India Test Series
Leaders in GATE Preparations  65+ Centers across India
© All rights reserved by Gateforum Educational Services Pvt. Ltd. No part of this booklet may be reproduced or utilized in any form without the written permission.

33
3333/
Visit : www.Civildatas.com
13333
Visit : www.Civildatas.com

|ME| GATE-2016-PAPER-03 www.Civildatas.com

General Aptitude
Q. No. 1 – 5 Carry One Mark Each

1. Based on the given statements, select the appropriate option with respect to grammar and
usage. Statements
(i) The height of Mr. X is 6 feet.
(ii) The height of Mr. Y is 5 feet.
(A) Mr. X is longer than Mr. Y.
(B) Mr. X is more elongated than Mr. Y.
(C) Mr. X is taller than Mr. Y.
(D) Mr. X is lengthier than Mr. Y.
Key: (C)

2. The students the teacher on teachers’ day for twenty years of dedicated teaching.
(A) facilitated (B) felicitated (C) fantasized (D) facillitated
Key: (B)

3. After India’s cricket world cup victory in 1985, Shrotria who was playing both tennis and
cricket till then, decided to concentrate only on cricket. And the rest is history.
What does the underlined phrase mean in this context?
(A) history will rest in peace (B) rest is recorded in history books
(C) rest is well known (D) rest is archaic
Key: (C)

Given  9 inches    0.25yards 


1/2 1/2
4. , which one of the following statements is TRUE?
(A) 3 inches = 0.5 yards (B) 9 inches = 1.5 yards
(C) 9 inches = 0.25 yards (D) 81 inches = 0.0625 yards
Key: (C)

5. S, M, E and F are working in shifts in a team to finish a project. M works with twice the
efficiency of others but for half as many days as E worked. S and M have 6 hour shifts in a
day, whereas E and F have 12 hours shifts. What is the ratio of contribution of M to
contribution of E in the project?
(A) 1:1 (B) 1:2 (C) 1:4 (D) 2:1
Key: (B)

 ICP–Intensive Classroom Program  eGATE-Live Internet Based Classes DLP  TarGATE-All India Test Series
Leaders in GATE Preparation  65+ Centers across India
© All rights reserved by Gateforum Educational Services Pvt. Ltd. No part of this booklet may be reproduced or utilized in any form without the written permission.

Visit : www.Civildatas.com
Visit : www.Civildatas.com

|ME| GATE-2016-PAPER-03 www.Civildatas.com

Q. No. 6 – 10 Carry Two Marks Each

6. The Venn diagram shows the preference of the student population for leisure activities.

29

From the data given, the number of students who like to read books or play sports is .
(A) 44 (B) 51 (C) 79 (D) 108
Key: (D)
Exp: From Venn diagram
n(A)  no of persons reading books  13  44  12  7  76
n(B)  no of persons playing  15  44  7  17  83
n(A  B)  51
n(A  B)  n(A)  n(B)  n(A  B)  76  83  51  108

7. Social science disciplines were in existence in an amorphous form until the colonial period
when they were institutionalized. In varying degrees, they were intended to further the colonial
interest. In the time of globalization and the economic rise of postcolonial countries like India,
conventional ways of knowledge production have become obsolete.
Which of the following can be logically inferred from the above statements?
(i) Social science disciplines have become obsolete.
(ii) Social science disciplines had a pre-colonial origin.
(iii) Social science disciplines always promote colonialism.
(iv) Social science must maintain disciplinary boundaries.
(A) (ii) only (B) (i) and (iii) only
(C) (ii) and (iv) only (D) (iii) and (iv) only
Key: (A)

8. Two and a quarter hours back, when seen in a mirror, the reflection of a wall clock without
number markings seemed to show 1:30. What is the actual current time shown by the clock?
(A) 8:15 (B) 11:15 (C) 12:15 (D) 12:45
Key: (D)
Exp: If reflection is seen as Actual will be

1: 30
10 : 30
Thus present time will be 10:30  2:15 12: 45

 ICP–Intensive Classroom Program  eGATE-Live Internet Based Classes DLP  TarGATE-All India Test Series
Leaders in GATE Preparation  65+ Centers across India
© All rights reserved by Gateforum Educational Services Pvt. Ltd. No part of this booklet may be reproduced or utilized in any form without the written permission.

Visit : www.Civildatas.com
Visit : www.Civildatas.com

|ME| GATE-2016-PAPER-03 www.Civildatas.com

9. M and N start from the same location. M travels 10 km East and then 10 km North-East. N
travels 5 km South and then 4 km South-East. What is the shortest distance (in km) between
M and N at the end of their travel?
(A) 18.60 (B) 22.50 (C) 20.61 (D) 25.00
Key: (C)

10. A wire of length 340 mm is to be cut into two parts. One of the parts is to be made into a
square and the other into a rectangle where sides are in the ratio of 1:2. What is the length of
the side of the square (in mm) such that the combined area of the square and the rectangle is a
MINIMUM?
(A) 30 (B) 40 (C) 120 (D) 180

Key: (B)
Exp: x  y  340
 x 2x 
Perimeter of rectangle  2     2x
3 3 
Perimeter of square  340  2x x 2x/ 3

340  2x
Length of square 
4 x x/ 3
 340  2x  2 2
2

Totalarea     x  f (x)
 4  9
Square Rectangle
4 2x  340
f '(x)  x  0
9 4
4 1
 x   340  2x   x  90
9 4
340  2x
Length of square   40mm
4
Mechanical Engineering
Q. No. 1 – 25 Carry One Mark Each

1 A real square matrix A is called skew-symmetric if


(A) AT = A
(B) AT = A-1
(C) AT = A
(D) AT = A+A-1
Key: (C)

log e (1  4 x)
2 lt is equal to
x 0 e3x  1
1 4
(A) 0 (B) (C) (D) 1
12 3

Key: (C)

 ICP–Intensive Classroom Program  eGATE-Live Internet Based Classes DLP  TarGATE-All India Test Series
Leaders in GATE Preparation  65+ Centers across India
© All rights reserved by Gateforum Educational Services Pvt. Ltd. No part of this booklet may be reproduced or utilized in any form without the written permission.

Visit : www.Civildatas.com
Visit : www.Civildatas.com

|ME| GATE-2016-PAPER-03 www.Civildatas.com

log e (1  4x) 0


Exp: lim  
x 0 e3x  1 0
1
.4
lim 1 3x4x 
4 4

x 0 e .3 (1  4.0)e .3 3
0

3. Solutions of Laplace’s equation having continuous second-order partial derivatives are called
(A) biharmonic functions
(B) harmonic functions
(C) conjugate harmonic functions
(D) error functions
Key: (B)

4. The area (in percentage) under standard normal distribution curve of random variable Z within
limits from −3 to +3 is _____
Key: 99.74
Exp:
3  99.74%

0
1  68.4% 
2  95.45%

5. The root of the function f(x) = x3+x1 obtained after first iteration on application of Newton-
Raphson scheme using an initial guess of x0=1 is
(A) 0.682 (B) 0.686 (C) 0.750 (D) 1.000
Key: (C)
f  xn 
Exp: We have x n 1  x n 
f  xn 
f  x0 
For n=0, x1  x 0 
f  x0 
f (x)  x 3  x  1  f (x)  3x 2  1
given x 0  1
f  x 0   f (1)  1, f   x 0   f (1)  4
1 3
 x1  1    0.75
4 4

 ICP–Intensive Classroom Program  eGATE-Live Internet Based Classes DLP  TarGATE-All India Test Series
Leaders in GATE Preparation  65+ Centers across India
© All rights reserved by Gateforum Educational Services Pvt. Ltd. No part of this booklet may be reproduced or utilized in any form without the written permission.

Visit : www.Civildatas.com
Visit : www.Civildatas.com

|ME| GATE-2016-PAPER-03 www.Civildatas.com

6. A force F is acting on a bent bar which is clamped at one end as shown in the figure.

The CORRECT free body diagram is

(A) (B)

(C) (D)

Key: (A)

7. The cross-sections of two solid bars made of the same material are shown in the figure. The
square cross-section has flexural (bending) rigidity I1, while the circular cross-section has
flexural rigidity I2. Both sections have the same cross-sectional area. The ratio I1/I2 is
(A) 1/ 
(B) 2 / 
(C)  / 3
(D)  / 6
Key: (C)
Exp: Flexural rigidity = EI
Both have same cross-section area

 a 2  d2
4
Where a is side of square and d is diameter of circle.
a4 2 d 4
E1 
 4 I 2
12  12  16  
 a4  d  1 ( E1  E2 because of same material)
16 I2 d 4  4 3
E2  d
64 64

 ICP–Intensive Classroom Program  eGATE-Live Internet Based Classes DLP  TarGATE-All India Test Series
Leaders in GATE Preparation  65+ Centers across India
© All rights reserved by Gateforum Educational Services Pvt. Ltd. No part of this booklet may be reproduced or utilized in any form without the written permission.

Visit : www.Civildatas.com
Visit : www.Civildatas.com

|ME| GATE-2016-PAPER-03 www.Civildatas.com

8. The state of stress at a point on an element is shown in figure (a). The same state of stress is
shown in another coordinate system in figure (b).
The components (xx,yy,xy) are given by


(A) p / 2,  p / 2, 0 
(B) (0, 0, p)

(C)  p, p, p / 2 
(D)  0,0, p / 2 
Key: (B) a  b
Exp: We know,
 x   y   x   y 
     cos 2  xy sin 2
 2   2 
Where,  is the location of any oblique plane which making an angle  in CCW direction.
 When   45o ,   xx ,  x  p,  y  p

pp pp
  xx      cos90  0
o
 2   2 
When   45o ,  yy

pp pp
      cos90  0
o
 2   2 
When   45o ,   xy

 x   y 
We know     sin 2  xy cos 2
 2 
pp
   xy    sin 90  p xx , yy , xy is 0, 0,p
 2 

9. A rigid link PQ is undergoing plane motion as shown in the figure (VP and VQ are non-zero). VQP
is the relative velocity of point Q with respect to point P.

Which one of the following is TRUE?


(A) VQP has components along and perpendicular to PQ
(B) VQP has only one component directed from P to Q
(C) VQP has only one component directed from Q to P
(D) VQP has only one component perpendicular to PQ
Key: (D)
 ICP–Intensive Classroom Program  eGATE-Live Internet Based Classes DLP  TarGATE-All India Test Series
Leaders in GATE Preparation  65+ Centers across India
© All rights reserved by Gateforum Educational Services Pvt. Ltd. No part of this booklet may be reproduced or utilized in any form without the written permission.

Visit : www.Civildatas.com
Visit : www.Civildatas.com

|ME| GATE-2016-PAPER-03 www.Civildatas.com

Exp: Let VP & VQ make an angle  and  with axis of link PQ respectively.

VQ cos  Q

 VQ sin 
VQ sin 
VQ VQ cos 
VP sin  VP sin 

VP
 P

VP cos  VP cos 

Since link PQ is rigid, so the distance between P & Q will never change. Hence relative
velocity between P & Q along axial direction should be zero.
Vp cos   VQ sin 
VQ sin 
Relative velocity between P & Q
 VP sin   PQ r to PQ
VP sin 
 VQ sin  PQ r to PQ
  VP sin   VQ sin   PQ r to PQ

10. The number of degrees of freedom in a planar mechanism having n links and j simple hinge
joints is
(A) 3(n  3)  2j (B) 3(n  1)  2j (C) 3n  2j (D) 2j  3n  4
Key: (B)
Exp: DOF F  3 n  1  2j
Where, n  total number of links
j = Effective number of binary points

11. The static deflection of a spring under gravity, when a mass of 1 kg is suspended from it, is 1
mm. Assume the acceleration due to gravity g =10 m/s2. The natural frequency of this spring-
mass system (in rad/s) is ______
Key: (100)
Exp: st  103 m
g 10
n    100 rad/sec
st 103

12. Which of the bearings given below SHOULD NOT be subjected to a thrust load?
(A) Deep groove ball bearing
(B) Angular contact ball bearing
(C) Cylindrical (straight) roller bearing
(D) Single row tapered roller bearing
Key: (C)

 ICP–Intensive Classroom Program  eGATE-Live Internet Based Classes DLP  TarGATE-All India Test Series
Leaders in GATE Preparation  65+ Centers across India
© All rights reserved by Gateforum Educational Services Pvt. Ltd. No part of this booklet may be reproduced or utilized in any form without the written permission.

Visit : www.Civildatas.com
Visit : www.Civildatas.com

|ME| GATE-2016-PAPER-03 www.Civildatas.com

13. A channel of width 450 mm branches into two sub-channels having width 300 mm and 200
mm as shown in figure. If the volumetric flow rate (taking unit depth) of an incompressible
flow through the main channel is 0.9 m3/s and the velocity in the sub-channel of width 200
mm is 3 m/s, the velocity in the sub-channel of width 300 mm is m/s.
Assume both inlet and outlet to be at the same elevation.

Key: 1
Exp: Apply Mass conservation and taking incompressibility
We have
300mm V2  ?
A1V1  A 2 V2  A3 V3 Given A1V1  0.9m3 /s
450mm
0.9  0.3V2  0.2  3
2

0.9  0.6  0.3V2 1


200mm
 V2  1m / s 3
0.9m /s 3

V3  3m/s

14. For a certain two-dimensional incompressible flow, velocity field is given by 2xyiˆ  y2ˆj . The
streamlines for this flow are given by the family of curves
(A) x 2 y 2  constant (B) xy 2  constant
(C) 2xy  y 2 = constant (D) xy  constant
Key: (B)
Exp: v  2xyiˆ  y 2 ˆj
 
u v
y x
2xy dy  d
On integrating
 = xy 2  f  x 

  y2  f '  x      y2 
x
 y2  f '  x   y2
f '  x   0  f  x   constant
  =xy 2  constant

 ICP–Intensive Classroom Program  eGATE-Live Internet Based Classes DLP  TarGATE-All India Test Series
Leaders in GATE Preparation  65+ Centers across India
© All rights reserved by Gateforum Educational Services Pvt. Ltd. No part of this booklet may be reproduced or utilized in any form without the written permission.

Visit : www.Civildatas.com
Visit : www.Civildatas.com

|ME| GATE-2016-PAPER-03 www.Civildatas.com

15. Steady one-dimensional heat conduction takes place across the faces 1 and 3 of a composite
slab consisting of slabs A and B in perfect contact as shown in the figure, where kA , kB denote
the respective thermal conductivities. Using the data as given in the figure, the interface
temperature T2 (in °C) is .

Key: 67.5
T1  T2 T2  T3
Exp: 
LA LB
kA  A kB  A
130  T2 T2  30

0.1 0.3
20 100
390  3T2  5T2  150
8T2  540
T2  67.5o C
 Interface temperature  67.5o C

16. Grashof number signifies the ratio of


(A) inertia force to viscous force
(B) buoyancy force to viscous force
(C) buoyancy force to inertia force
(D) inertia force to surface tension force
Key: (B)
Inertia force  Buoyant force
Exp: Grashof number (Gr) 
(Viscous force) 2

17. The INCORRECT statement about the characteristics of critical point of a pure substance is
that
(A) there is no constant temperature vaporization process
(B) it has point of inflection with zero slope
(C) the ice directly converts from solid phase to vapor phase
(D) saturated liquid and saturated vapor states are identical
Key: (C)
Exp: The process of conversion from solid phase to vapour phase is called sublimation and this
does not happen at critical point. All the other statements are true at Critical point.

 ICP–Intensive Classroom Program  eGATE-Live Internet Based Classes DLP  TarGATE-All India Test Series
Leaders in GATE Preparation  65+ Centers across India
© All rights reserved by Gateforum Educational Services Pvt. Ltd. No part of this booklet may be reproduced or utilized in any form without the written permission.

Visit : www.Civildatas.com
Visit : www.Civildatas.com

|ME| GATE-2016-PAPER-03 www.Civildatas.com

18. For a heat exchanger, ∆Tmax is the maximum temperature difference and ∆Tmin is the minimum
temperature difference between the two fluids. LMTD is the log mean temperature difference.
Cmin and Cmax are the minimum and the maximum heat capacity rates. The maximum possible
heat transfer (Qmax) between the two fluids is
(A) Cmin LMTD (B) Cmin ∆Tmax (C) Cmax ∆Tmax (D) Cmax ∆Tmin
Key: (B)
Exp: In a heat exchanger, maximum possible heat transfer will be.
Qmax  Cmin Tmax

19. The blade and fluid velocities for an axial turbine are as shown in the figure.

The magnitude of absolute velocity at entry is 300 m/s at an angle of 65 to the axial direction,
while the magnitude of the absolute velocity at exit is 150 m/s. The exit velocity vector has a
component in the downward direction. Given that the axial (horizontal) velocity is the same at
entry and exit, the specific work (in kJ/kg) is _____
Key: 52.80
Exp: Given:   90  65  25o.
Let ‘1’ & ‘2’ denotes inlet & outlet of vane.
Vane velocity (u) = u1  u 2  150m / s.
Inlet:
Velocity of whirling Vw1  V1 cos  
 300cos25  271.89m / s

 
Velocity of flow Vf1  V1 sin 
 300sin 25  126.7854m / s
 
Absolute velocity of Inlet = Vw1  Vf1
It is given that horizontal velocity is same at entry & exit.
So,
Vf1  Vf 2
126.7859  V2 cos 

126.7854 
  cos 1    32.3o
 150 
Vw2  V2 sin(32.3)  80.16m / s

 ICP–Intensive Classroom Program  eGATE-Live Internet Based Classes DLP  TarGATE-All India Test Series
Leaders in GATE Preparation  65+ Centers across India
© All rights reserved by Gateforum Educational Services Pvt. Ltd. No part of this booklet may be reproduced or utilized in any form without the written permission.

10

Visit : www.Civildatas.com
Visit : www.Civildatas.com

|ME| GATE-2016-PAPER-03 www.Civildatas.com


u  150m / s
Vf2


V2 Vw 2
Vr2
Vf1

u2

Vr1
Vw1 V1

u1

65

Absolute velocity at outlet


  
V2  Vf2  Vw 2
Specific work done on the fluid per unit weight (wsp )
 
 wsp   u. Vw2  Vw1
 
Let ĵ unit vector positive toward upward.
()Ve

Vw1  271.89m / sec ()Ve

u  150m / sec

Vw2  80.16

()Ve


w  (150 ˆj). (271.89 ˆj)  (80.16 ˆj)  52.8075kJ / kg 
20. Engineering strain of a mild steel sample is recorded as 0.100%. The true strain is
(A) 0.010% (B) 0.055% (C) 0.099% (D) 0.101%
Key: (C)
0.1
Exp: 
100
We know,
T  ln 1    , where T is True strain and  is Engineering strain
 0.1 
 T  ln 1    0.0009995 T %  0.0009995 100  0.099%
 100 

 ICP–Intensive Classroom Program  eGATE-Live Internet Based Classes DLP  TarGATE-All India Test Series
Leaders in GATE Preparation  65+ Centers across India
© All rights reserved by Gateforum Educational Services Pvt. Ltd. No part of this booklet may be reproduced or utilized in any form without the written permission.

11

Visit : www.Civildatas.com
Visit : www.Civildatas.com

|ME| GATE-2016-PAPER-03 www.Civildatas.com

21. Equal amounts of a liquid metal at the same temperature are poured into three moulds made of
steel, copper and aluminum. The shape of the cavity is a cylinder with 15 mm diameter. The
size of the moulds are such that the outside temperature of the moulds do not increase
appreciably beyond the atmospheric temperature during solidification. The sequence of
solidification in the mould from the fastest to slowest is
(Thermal conductivities of steel, copper and aluminum are 60.5, 401 and 237 W/m-K,
respectively. Specific heats of steel, copper and aluminum are 434, 385 and 903 J/kg-K,
respectively.
Densities of steel, copper and aluminum are 7854, 8933 and 2700 kg/m3, respectively.)
(A) Copper - Steel - Aluminum
(B) Aluminum - Steel – Copper
(C) Copper - Aluminum - Steel
(D) Steel - Copper - Aluminum
Key: (C)
Exp: K steel  60.5; steel  7854; Csteel  434 J/kg K
K copper  401; copper  8933; Ccopper  385 J/kg K
K A1  237; A1  2700; C Al  903 J/kg K

Heat capacity c


 c steel  3408.636 kJ/m3K
 c copper  3439.205 kJ/m3K
 c A1  2438.100 kJ/m3K
 k  60.5
  steel     103  0.0177 103 m 2 /s

 steel
c 3408
 k  401
  copper     103  0.1166 103 m 2 /s

 copper
c 3439
 k  23.7
  A1     103  0.1119 103 m 2 /s

 A1
c 2438
So, cooling rates would be copper, Aluminium, steel

22. In a wire-cut EDM process the necessary conditions that have to be met for making a
successful cut are that
(A) wire and sample are electrically non-conducting
(B) wire and sample are electrically conducting
(C) wire is electrically conducting and sample is electrically non-conducting
(D) sample is electrically conducting and wire is electrically non-conducting
Key: (B)
Exp: In this process, a thin metallic wire is fed on to the conducting workpiece, which is submerged
in a tank of dielectric fluid such as de-ionized water. Wire is fed in the programmed path &
material is cut from the workpiece accordingly. Material removal takes place by a series of
discrete discharges between the wire electrode & workpiece in the presence of a dielectric
fluid. The di-electric fluid gets ionized in between the tool electrode gap thereby creating a

 ICP–Intensive Classroom Program  eGATE-Live Internet Based Classes DLP  TarGATE-All India Test Series
Leaders in GATE Preparation  65+ Centers across India
© All rights reserved by Gateforum Educational Services Pvt. Ltd. No part of this booklet may be reproduced or utilized in any form without the written permission.

12

Visit : www.Civildatas.com
Visit : www.Civildatas.com

|ME| GATE-2016-PAPER-03 www.Civildatas.com

path for each discharge. The area wherein discharge takes place gets heated to very high
temperature such that the surface get melted & removed. The cut particles (debris) get flushed
away by continuous flowing dielectric fluid.
Generally, wire-cut EDM is used for cutting Aluminium, brass, etc. & wire material used for
quicker cutting action is zinc coated brass wires.

23. Internal gears are manufactured by


(A) hobbing (B) shaping with pinion cutter
(C) shaping with rack cutter (D) milling
Key: (B)

24. Match the following part programming codes with their respective functions

Part Programming Codes Functions

P. G01 I. Spindle stop

Q. G03 II. Spindle rotation, clockwise

R. M03 III. Circular interpolation,


anticlockwise
S. M05 IV. Linear interpolation

(A) P – II, Q – I, R – IV, S – III


(B) P – IV, Q – II, R – III, S – I
(C) P – IV, Q – III, R – II, S – I
(D) P – III, Q – IV, R – II, S – I

Key: (C)

25. In PERT chart, the activity time distribution is


(A) Normal (B) Binomial (C) Poisson (D) Beta
Key: (D)

Q. No. 26 – 55 carry Two Marks Each


2 1 0
26.
 
The number of linearly independent eigenvectors of matrix A   0 2 0  is ____ .
0 0 3
Key: 2
Exp: Here   2, 2,3

For   2, No. of L.I eigen vectors

 3  rank of  A  2I   3  2  1
For   3, No. of L.I eigen vectors =1
∴ Total L.I eigen vectors = 2

 ICP–Intensive Classroom Program  eGATE-Live Internet Based Classes DLP  TarGATE-All India Test Series
Leaders in GATE Preparation  65+ Centers across India
© All rights reserved by Gateforum Educational Services Pvt. Ltd. No part of this booklet may be reproduced or utilized in any form without the written permission.

13

Visit : www.Civildatas.com
Visit : www.Civildatas.com

|ME| GATE-2016-PAPER-03 www.Civildatas.com

4
27. The value of the line integral  F.r ds, where C is a circle of radius

units is .
C

Here, F(x, y)  yiˆ  2xjˆ and r is the UNIT tangent vector on the curve C at an arc length s
from a reference point on the curve. î and ĵ are the basis vectors in the x-y Cartesian
reference. In evaluating the line integral, the curve has to be traversed in the counter-clockwise
direction.

Key: 16
Exp: By Green’s theorem,

 F.r ' ds   ydx  2xdy    2  1 dxdy


c c
2
 4 
   16
 

28. l x  i 2
m    x
(A) 0 (B) ∞ (C) 1/2 (D) −∞
Key: (C)

Exp: lim
x 
 x2  x 1  x   x2  x 1  x
x2  x 1  x
x2  x 1  x2
 lim
x2  x 1  x
x 

 1
x 1  
 x 1 0 1
 lim  
x  1 1 1 0  0  1 2
x 1  2 1
x x

29. Three cards were drawn from a pack of 52 cards. The probability that they are a king, a queen,
and a jack is
16 64 3 8
(A) (B) (C) (D)
5525 2197 13 16575

Key: (A)
4
C1  4C1  4C1 64 16
Exp: Required probability  52
 
C3 22100 5525

30. An inextensible massless string goes over a frictionless pulley. Two


weights of 100 N and 200 N are attached to the two ends of the
string. The weights are released from rest, and start moving due to
gravity. The tension in the string (in N) is .
Key: 133.33
200N

100N

 ICP–Intensive Classroom Program  eGATE-Live Internet Based Classes DLP  TarGATE-All India Test Series
Leaders in GATE Preparation  65+ Centers across India
© All rights reserved by Gateforum Educational Services Pvt. Ltd. No part of this booklet may be reproduced or utilized in any form without the written permission.

14

Visit : www.Civildatas.com
Visit : www.Civildatas.com

|ME| GATE-2016-PAPER-03 www.Civildatas.com

Exp: From F.B.D


200
200  T  a …(1)
g

100
T  100  a …(2)
g
Adding equations (1) and (2) T
T
300 a
100  a  a  g/3 200
g
From equation (1) a 100
200 2 200
200  T    F.B.D F.B.D
g 3 3
T T
200
T  200   133.33N
3 a a

200 100

31. A circular disc of radius 100 mm and mass 1 kg, initially at rest at position A, rolls without
slipping down a curved path as shown in figure. The speed v of the disc when it reaches
position B is m/s.

Acceleration due to gravity g = 10 m/s2.


Key: 20
Exp: According to energy conservation principle
Total K.E + P.E = constant
 K.E A  P.E A  K.E B  P.E B
1 1
 0  mg.30  m  v 2  I2  0
2 2
1 1 1
 30 10  1 v 2    1  v 2
2 2 2
 mr 2
v
I  and   
 2 r
3 2
 30 10   v  v  20m/s
4

 ICP–Intensive Classroom Program  eGATE-Live Internet Based Classes DLP  TarGATE-All India Test Series
Leaders in GATE Preparation  65+ Centers across India
© All rights reserved by Gateforum Educational Services Pvt. Ltd. No part of this booklet may be reproduced or utilized in any form without the written permission.

15

Visit : www.Civildatas.com
Visit : www.Civildatas.com

|ME| GATE-2016-PAPER-03 www.Civildatas.com

32. A rigid rod (AB) of length L  2 m is undergoing translational as well as rotational motion
in the x-y plane (see the figure). The point A has the velocity V1  ˆi  2ˆjm / s. The end B is
constrained to move only along the x direction.
The magnitude of the velocity V2 (in m/s) at the end B is
Key: 3
Exp: v1  i  2j V 5 1
 v x  1 and v y  2 2
V2 cos 45
v1  1  2  5
2 2 =α+45°
45 V2
1
Let ϕ is angle between v1 and x-axis.
 tan   2   63.43 V1
V2 sin 45
   45  63.43    18.43
V1 sin  V1 cos
Since, the rod is rigid 
 v1 cos   v 2 cos 45 45

 5 cos18.43  v 2 cos 45  v 2  3m/s

33. A square plate of dimension L × L is subjected to a uniform pressure load p = 250 MPa on
its edges as shown in the figure. Assume plane stress conditions. The Young’s modulus E =
200 GPa.

The deformed shape is a square of dimension L  2. If L  2 m and   0.001 m, the


Poisson’s ratio of the plate material is ______
Key: 0.2
Exp: According to Hooke’s Law
 y
x  x 
E E
Where,  x   y   p and  x 
2

2  p p 
    
 E E 
21 p 250
  1     1       0.2
2000 E 2  105

 ICP–Intensive Classroom Program  eGATE-Live Internet Based Classes DLP  TarGATE-All India Test Series
Leaders in GATE Preparation  65+ Centers across India
© All rights reserved by Gateforum Educational Services Pvt. Ltd. No part of this booklet may be reproduced or utilized in any form without the written permission.

16

Visit : www.Civildatas.com
Visit : www.Civildatas.com

|ME| GATE-2016-PAPER-03 www.Civildatas.com

34. Two circular shafts made of same material, one solid (S) and one hollow (H), have the same
length and polar moment of inertia. Both are subjected to same torque. Here, S is the twist
and S is the maximum shear stress in the solid shaft, whereas H is the twist and H is the
maximum shear stress in the hollow shaft. Which one of the following is TRUE?
(A) s  H and s  H (B) s  H and s  H
(C) s  H and s  H (D) s  H and s  H
Key: (D)
Exp: According to pure torsion equation
T  G
 
J R 
Let ds and d1 are diameter of solid shaft and outer diameter of hollow shaft
Ts s

Js R s  
 s  h
Th h Rs Rh

Jh R h
  d
 s  h  s   h . s
d s d1 d1
Since, J s  J h
 d1 must be greater than ds
ds
 must be less than 1  s  h and
d1
Ts G s s

T G Js s
   s  h
J  T G 
h
 h h
Jh h

35. A beam of length L is carrying a uniformly distributed load w per unit length. The flexural
rigidity of the beam is EI. The reaction at the simple support at the right end is _____ .

wL 3wL wL wL
(A) (B) (C) (D)
2 8 4 8
Key: (B)

Exp: 

A B 

RB


2
A B 
1
RB
 ICP–Intensive Classroom Program  eGATE-Live Internet Based Classes DLP  TarGATE-All India Test Series
Leaders in GATE Preparation  65+ Centers across India
© All rights reserved by Gateforum Educational Services Pvt. Ltd. No part of this booklet may be reproduced or utilized in any form without the written permission.

17

Visit : www.Civildatas.com
Visit : www.Civildatas.com

|ME| GATE-2016-PAPER-03 www.Civildatas.com

4 R 3
1  ; 2  B
8EI 3EI
Since, net vertical deflection at B is zero
 1  2
 4 R B3 3
   RB 
8EI 3EI 8

36. Two masses m are attached to opposite sides of a rigid rotating shaft in the vertical plane.
Another pair of equal masses m1 is attached to the opposite sides of the shaft in the vertical
plane as shown in figure. Consider m = 1 kg, e = 50 mm, e1 = 20 mm, b = 0.3 m, a = 2 m and
a1 = 2.5 m. For the system to be dynamically balanced, m1 should be kg.

Key: 2
Exp: Couple due to m = couple due to m1
mea  m1e1a1
 50   2 
m1  1        2kg
 20   2.5 

37. A single degree of freedom spring-mass system is subjected to a harmonic force of constant
3k
amplitude. For an excitation frequency of , the ratio of the amplitude of steady state
m
response to the static deflection of the spring is .

Key: 0.5
1 1 1
Exp: M.F      0.5
  1 3 2
2
1 
n

 ICP–Intensive Classroom Program  eGATE-Live Internet Based Classes DLP  TarGATE-All India Test Series
Leaders in GATE Preparation  65+ Centers across India
© All rights reserved by Gateforum Educational Services Pvt. Ltd. No part of this booklet may be reproduced or utilized in any form without the written permission.

18

Visit : www.Civildatas.com
Visit : www.Civildatas.com

|ME| GATE-2016-PAPER-03 www.Civildatas.com

38. A bolted joint has four bolts arranged as shown in figure. The cross sectional area of each bolt
is 25 mm2. A torque T = 200 N-m is acting on the joint. Neglecting friction due to clamping
force, maximum shear stress in a bolt is MPa.

Key: 40
Exp: Let the resisting force in each bolt = F Newton
3
Net resisting torque (TR) = 4F  50 10 N-m
Applied torque (T) = Resisting Torque
T  4F  50103
200
F 1000  1000N
4  50
Let shear stress developed in each bolt =  MPa
It is given that resisting area (AR) = 25mm2
F    AR
1000
  40MPa
25

39. Consider a fully developed steady laminar flow of an incompressible fluid with viscosity µ
through a circular pipe of radius R. Given that the velocity at a radial location of R/2 from the
centerline of the pipe is U1, the shear stress at the wall is kµU1/R, where K is
Key: 2.667
Exp: Given, ,R u at R/2  U 1 ,
y wall  ku1 /R
K=?
Velocity profile in horizontal pipe flow is
 r2 
u  u max 1  2 
 R 
 R2 
u1  u max 1  2 
 4R 
3 4
u1  u max  u max  u1
4 3

 u
y wall   
 r r  R
2r 2 2 u max 2 4
   max 2
  u max    u1
R R R R 3
8 u1 8
  k   2.667
3 R 3
 ICP–Intensive Classroom Program  eGATE-Live Internet Based Classes DLP  TarGATE-All India Test Series
Leaders in GATE Preparation  65+ Centers across India
© All rights reserved by Gateforum Educational Services Pvt. Ltd. No part of this booklet may be reproduced or utilized in any form without the written permission.

19

Visit : www.Civildatas.com
Visit : www.Civildatas.com

|ME| GATE-2016-PAPER-03 www.Civildatas.com

40. The water jet exiting from a stationary tank


through a circular opening of diameter 300 mm
impinges on a rigid wall as shown in the figure.
Neglect all minor losses and assume the water
level in the tank to remain constant. The
net horizontal force experienced by the
wall is kN.
Density of water is 1000 kg/m3.
Acceleration due to gravity g = 10 m/s2

Key: 8.76
Exp: Force exerted by a jet of water striking fixed wall is av 2 N where a is area of jet.


 103   0.32  v 2  v  2gh  2 10  6.2
4


103   0.32  2  10  62  8.76kN
4

 x ˆ y ˆ
41. For a two-dimensional flow, the velocity field is u  i 2 j, where ˆi and ˆj are the
x y
2 2
x  y2
basis vectors in the x-y Cartesian coordinate system. Identify the CORRECT statements from
below.
(1) The flow is incompressible.
(2) The flow is unsteady.
y
(3) y-component of acceleration, a y 
 
2
x  y2
2

  x  y
(4) x-component of acceleration, a x 
x  y2 
2 2

(A) (2) and (3) (B) (1) and (3) (C) (1) and (2) (D) (3) and (4)

Key: (B)
x y
Exp: u ; v
x 2  y2 x 2  y2
 Given flow is independent of time of flow is steady
 Density ‘ρ’ is not present in velocity components, so flow is incompressible
Acceleration along x direction
u u
ax  u v
x y
x
 u
x  y2
2

u  x  y   x2x
2 2
y2  x 2 u 2xy
    
x x  y   x  y  y  x  y2 
2 2 2 2 2 2 2 2

 ICP–Intensive Classroom Program  eGATE-Live Internet Based Classes DLP  TarGATE-All India Test Series
Leaders in GATE Preparation  65+ Centers across India
© All rights reserved by Gateforum Educational Services Pvt. Ltd. No part of this booklet may be reproduced or utilized in any form without the written permission.

20

Visit : www.Civildatas.com
Visit : www.Civildatas.com

|ME| GATE-2016-PAPER-03 www.Civildatas.com

u
 u  2
x  y  x   xy  x
2 2 2 3
…(a)
x x  y2 x  y  x  y 
2 2 2 2 2 3

u y 2xy 2xy 2
 v  2  …(b)
y  x  y2   x 2  y 2 2  x 2  y 2 3

xy2  x 3   2xy 2  x  x 2  y 2  x
ax  a  b   
x 
2 3
x 
2 3
x  y2 
2
2
y 2
y 2

So, only statement 1 & 3 are correct.

42. Two large parallel plates having a gap of 10 mm in between them are maintained at
temperatures.
T1 = 1000 K and T2 = 400 K. Given emissivity values, 1  0.5, 2  0.25 and Stefan-
Boltzmann constant  = 5.67 × 10−8 W/m2-K4, the heat transfer between the plates (in
kW/m2) is
Key: 11.049
  T14  T24 
Exp: Q1 2  T1  1000K
1 1 T2  400K
 1
1 2
5.67  108 10004  4004 

1

1
1 1  0.5 2  0.25
0.5 0.25
 11049.696 W 2  11.049 kW 2
m m

43. A cylindrical steel rod, 0.01 m in diameter and 0.2 m in length is first heated to 750 0 C and
then immersed in a water bath at 1000C. The heat transfer coefficient is 250 W/m2-K. The
density, specific heat and thermal conductivity of steel are =7801 kg/m3, c = 473
J/kg-K, and k = 43 W/m-K, respectively. The time required for the rod to reach 3000C is
____ seconds.
Key: 43.49
Exp: d = 0.01 m
L = 0.2m, R=7801 kg/m3.
t i  750o C, C  473J / kgK
t   100o C, k  43 W / mK
 2
d
V 4 d
h  250 W / m K, Lc   2

A d 4
 t  t   hA h
ln     
 t i  t   Vc cLc
 t  t   h  4
ln   
 t i  t   c  d
 300  100  250  4
ln       43.49s
 750  100  7801 473  0.01

 ICP–Intensive Classroom Program  eGATE-Live Internet Based Classes DLP  TarGATE-All India Test Series
Leaders in GATE Preparation  65+ Centers across India
© All rights reserved by Gateforum Educational Services Pvt. Ltd. No part of this booklet may be reproduced or utilized in any form without the written permission.

21

Visit : www.Civildatas.com
Visit : www.Civildatas.com

|ME| GATE-2016-PAPER-03 www.Civildatas.com

44. Steam at an initial enthalpy of 100 kJ/kg and inlet velocity of 100 m/s, enters an
insulated horizontal nozzle. It leaves the nozzle at 200 m/s. The exit enthalpy (in kJ/kg)
is .
Key: 85 1 2
Exp: h1  100kJ / kg
C1  100m / s
C1 C2
C2  200m / s
h1 h2
2 2
C C
h1   h2 
1 2
2000 2000
1002 2002
h 2  100    85kJ / kg
2000 2000

45. In a mixture of dry air and water vapor at a total pressure of 750 mm of Hg, the partial
pressure of water vapor is 20 mm of Hg. The humidity ratio of the air in grams of water vapor
per kg of dry air (gw/kgda) is ____ .
Key: 17
Exp: Pt  750 mm of Hg
Pv  20 mm of Hg
Humidity ratio (or) specific Humidity
PV
w  0.622
Pt  PV
 20 
 0.622   
 750  20 
kg g
 0.017 w.v  17 w.v
kg d.a kg d.a

46. In a 3-stage air compressor, the inlet pressure is p1 , discharge pressure is p4 and the
intermediate pressures are p2 and p3  p2  p3 . The total pressure ratio of the compressor is 10
and the pressure ratios of the stages are equal. If p1  100 kPa, the value of the pressure p3 (in
kPa) is .
Key: 464.16
Exp: 3 stage compressor, P1= 100 kPa
Pressure ratios of all stages are equal
P2 P3 P4
 
P1 P2 P3
P4
Overall pressure ratio   10
P1
P2 P3 P4 3
 rp     10  2.154
P1 P2 P3
P3  (rp )2  P1
 P3  2.514  2.514 100  464.16kPa

 ICP–Intensive Classroom Program  eGATE-Live Internet Based Classes DLP  TarGATE-All India Test Series
Leaders in GATE Preparation  65+ Centers across India
© All rights reserved by Gateforum Educational Services Pvt. Ltd. No part of this booklet may be reproduced or utilized in any form without the written permission.

22

Visit : www.Civildatas.com
Visit : www.Civildatas.com

|ME| GATE-2016-PAPER-03 www.Civildatas.com

47. In the vapour compression cycle shown in the figure, the


evaporating and condensing temperatures are 260 K and 310 K,
respectively. The compressor takes in liquid-vapour mixture (state
1) and isentropically compresses it to a dry saturated vapour
condition (state 2). The specific heat of the liquid refrigerant is
4.8kJ/kg-K and may be treated as constant. The enthalpy of
evaporation for the refrigerant at 310 K is 1054 kJ/kg.
The difference between the enthalpies at state points 1 and 0 (in kJ/kg) is
Key: 1103.51
Exp: dQ  Cp dT
3
Cp dT h fg T
s 2  s0   
0 T 310
3 2
 310  1054
s1  s0  4.8ln   ( s1  s2 )
 260  310 0
4 1
h1  h 0  310  1054
 4.8ln   s
260  260  310
 h1  h 0  1103.51kJ kg

48. Spot welding of two steel sheets each 2 mm thick is carried out successfully by passing 4 kA
of current for 0.2 seconds through the electrodes. The resulting weld nugget formed between
the sheets is 5 mm in diameter. Assuming cylindrical shape for the nugget, the thickness of the
nugget is mm.

Latent heat of fusion for steel 1400 kJ/kg


Effective resistance of the weld joint 200 
Density of steel 8000 kg/m3
Key: 2.91
Exp: t s  2mm
I  4kA; t  0.2
d  5mm; t n  ?
L.H 1400
R  200
  8000 kg m3
Energy supplied = I2 Rt …(a)

 
2
 4  103  200  106  0.2  640J

Energy required for melting =  V   L.H  … (b)


 1400  103  8000   52  106  t n  219911.4858t n
4
Equating (a) & (b)
640 = 219911.4858t n
t n  2.91mm
 ICP–Intensive Classroom Program  eGATE-Live Internet Based Classes DLP  TarGATE-All India Test Series
Leaders in GATE Preparation  65+ Centers across India
© All rights reserved by Gateforum Educational Services Pvt. Ltd. No part of this booklet may be reproduced or utilized in any form without the written permission.

23

Visit : www.Civildatas.com
Visit : www.Civildatas.com

|ME| GATE-2016-PAPER-03 www.Civildatas.com

49. For an orthogonal cutting operation, tool material is HSS, rake angle is 22°, chip thickness is
0.8 mm, speed is 48 m/min and feed is 0.4 mm/rev. The shear plane angle (in degrees) is
(A) 19.24 (B) 29.70 (C) 56.00 (D) 68.75
Key: (B)
Exp: Given o  22
Thickness of chip = 0.8mm  t 2 
Feed = 0.4 mm/rev
Speed = 48m/min
In orthogonal cutting = feed = thickness of uncut chip.  t1 
t 0.8
K 2 2
t1 0.4
cos o cos 22
tan     0.57
K  sin o 2  sin 22
  tan 1  0.57   29.7

50. In a sheet metal of 2 mm thickness a hole of 10 mm diameter needs to be punched. The yield
strength in tension of the sheet material is 100 MPa and its ultimate shear strength is 80 MPa.
The force required to punch the hole (in kN) is
Key: 5.0265
Exp: Given
t  2mm
d  10mm
s  80MPa

Syt  100MPa
F  dt  s  10  2  80  5026.5N  5.0265kN

51. In a single point turning operation with cemented carbide tool and steel work piece, it is found
that the Taylor’s exponent is 0.25. If the cutting speed is reduced by 50% then the tool life
changes by ____ times
Key: 16
Exp: n = 0.25
V
V2  1
2
V1T1n  V2T2n
V
V1T1n  1 T2n
2
0.25
 T2  2
 T
 1

T2  T1  24
T2  16  T1
Tool life changes by 16 times

 ICP–Intensive Classroom Program  eGATE-Live Internet Based Classes DLP  TarGATE-All India Test Series
Leaders in GATE Preparation  65+ Centers across India
© All rights reserved by Gateforum Educational Services Pvt. Ltd. No part of this booklet may be reproduced or utilized in any form without the written permission.

24

Visit : www.Civildatas.com
Visit : www.Civildatas.com

|ME| GATE-2016-PAPER-03 www.Civildatas.com

52. Two optically flat plates of glass are kept at a small angle  as shown in the figure.
Monochromatic light is incident vertically.

If the wavelength of light used to get a fringe spacing of 1 mm is 450 nm, the wavelength of
light (in nm) to get a fringe spacing of 1.5 mm is _______.
Key: 675
Exp: Fringe space by (d1) = 1mm
h
1  450nm t

d2  1.5mm, 2  ?
x
1 
tan    2 L
2d1 2d 2
 1.5 
 2  450     675nm
 1 

53. A point P (1, 3, −5) is translated by 2iˆ  3jˆ  4kˆ and then rotated counter clockwise by 90
about the z-axis. The new position of the point is
(A) (−6, 3, −9) (B) (−6, −3, −9) (C) (6, 3, −9) (D) (6, 3, 9)
Key: (A)
Exp: We can write vector 2iˆ  3jˆ  4kˆ y

as coordinate form as  2, 3,  4  .

OP  ˆi  3jˆ  5kˆ P 1,3, 5 P'  3,6, 9

OP  3iˆ  6ˆj  9kˆ

Now rotate OP' vector about z axis through 90°
in counter clockwise direction.
Since, it is rotated about z-axis, therefore z-
x
coordinate remains same. 0
x n  x o cos   yo sin 
yn  x o sin   yo cos 
z
Where  x o , yo  are co-ordinates corresponding
to old values
 xn , yn  are co-ordinates corresponding to new values

After rotating vector OP through an angle θ in counter clockwise
x n  3 cos 90  6sin 90
x n  6
y n  3sin 90  6cos90
yn  3
Hence the new coordinate is  6, 3,  9

 ICP–Intensive Classroom Program  eGATE-Live Internet Based Classes DLP  TarGATE-All India Test Series
Leaders in GATE Preparation  65+ Centers across India
© All rights reserved by Gateforum Educational Services Pvt. Ltd. No part of this booklet may be reproduced or utilized in any form without the written permission.

25

Visit : www.Civildatas.com
Visit : www.Civildatas.com

|ME| GATE-2016-PAPER-03 www.Civildatas.com

54. The demand for a two-wheeler was 900 units and 1030 units in April 2015 and May
2015, respectively. The forecast for the month of April 2015 was 850 units. Considering a
smoothing constant of 0.6, the forecast for the month of June 2015 is
(A) 850 units (B) 927 units
(C) 965 units (D) 970 units
Key: (D)
Exp:
Month Demand Forecast    D t 1  1    Ft 1 
April 900 850
May 1030  0.6 900  0.4 850  880
June  0.6 1030  0.4 880  970
 FJune  970 units

55. A firm uses a turning center, a milling center and a grinding machine to produce two parts.
The table below provides the machining time required for each part and the maximum
machining time available on each machine. The profit per unit on parts I and II are Rs. 40 and
Rs. 100, respectively. The maximum profit per week of the firm is Rs. .

Machining time required for


the machine part (minutes) Maximum machining time
Type of machine
available per week (minutes)
I II
Turning Center 12 6 6000
Milling Center 4 10 4000
Grinding Machine 2 3 1800

x2
Key: 40,000
Exp: Linear program formulation is
max z  40x1  100x 2  0, 1000
Constraints are
12x1  6x 2  6000  0, 600
4x1  10x 2  4000
2x1  3x 2  1800  0, 400
x1 , x 2  0
375, 250
At  0,400  , z  40,000
At  375, 250  ,z  40,000 x1
500,0 900,0 1000,0
At  500,0  ,z  20,000
∴ Maximum profit per week is Rs. 40,000.

 ICP–Intensive Classroom Program  eGATE-Live Internet Based Classes DLP  TarGATE-All India Test Series
Leaders in GATE Preparation  65+ Centers across India
© All rights reserved by Gateforum Educational Services Pvt. Ltd. No part of this booklet may be reproduced or utilized in any form without the written permission.

26

Visit : www.Civildatas.com
Visit : www.Civildatas.com

|ME| GATE-2017-PAPER-I www.Civildatas.com

Mechanical Engineering
Q. No. 1 to 25 Carry One Mark Each

1. A motor driving a solid circular steel shaft transmits 40kW of power at 500 rpm. If the
diameter of the shaft is 40 mm, the maximum shear stress in the shaft is ________MPa.
Key: 60 to 61
Exp: P=40kW N=500rpm
16T
D= 40mm T=
d 3
P  60,000 40  60,000
T   763.94N  m
2N 2  500
16  763.94  103
  60.79MPa
   40 
3

2. Consider the following partial differential equation for u(x,y) with the constant c > 1 :
u u
c 0
y x
Solution of this equation is
(A) u  x, y   f  x  cy  (B) u  x, y   f  x  cy 
(C) u  x, y   f  cx  y  (D) u  x, y   f  cx  y 
Key: (B)
u u
Exp: Given c 0
y x
u
 f '  x  cy 
x
u
 c f '  x  cy 
x
u u
 0
x y
u  x, y   f  x  cy 

3. The following figure shows the velocity- time plot for a particle traveling along a straight line.
The distance covered by the particle from t = 0 to t= 5 s is __________m.

4
3
Velocity (m / s)
2
1

1 2 3 4 5 6
Time(s)

 ICP–Intensive Classroom Program  eGATE-Live Internet Based Classes DLP  TarGATE-All India Test Series
Leaders in GATE Preparations  65+ Centers across India
© All rights reserved by Gateforum Educational Services Pvt. Ltd. No part of this booklet may be reproduced or utilized in any form without the written permission.

Visit : www.Civildatas.com
Visit : www.Civildatas.com

|ME| GATE-2017-PAPER-I www.Civildatas.com

Key: 10 to 10
Exp: Area under V- T wave
S  a1  a 2  a 3  a 4  a 5
1    1  4    4  2  
  11   1 1     1     2
2    2    2  
 0.5  1  2.5  6  10m

4. The damping ratio for a viscously damped spring mass system, governed by the relationship
d2 x dx
m 2 C  kx  F  t  , is given by
dt dt
c c c c
(A) (B) (C) (D)
mk 2 km km 2mk
Key: (B)
dx 2 dx c c
Exp: m 2
 c  k x  F t ;  
dt dt cc 2 km

d2 y
5. The differential equation  16y  0 for y  x  with the two boundary conditions
dx 2
dy dy
 1 and  1 has
dx x 0 dx x  
2

(A) no solution (B) exactly two solutions


(C) exactly one solution (D) infinitely many solutions
Key: (A)
d2 y dy dy
Exp:  16y  0 1  1
dx 2 dx x 0 dx x  
2

 m  16  0
2

m  0  4i
yc  c1cos 4x  c2 sin 4x and yp  0
 y  c1 cos 4x  c 2 sin 4x
y'  x   4c1 sin 4x  4c2 cos 4x
1
y'  0   1  0  4c2  1  c2 
4
 1
y '    1  0  4c 2  1  c 2 
2 4
1 1
c 2  and both not possible
4 4
Hence there is no solution

6. Metric thread of 0.8 mm pitch is to be cut on a lathe. Pitch of the lead screw is 1.5 mm. If the
spindle rotates at 1500 rpm, the speed of rotation of the lead screw (rpm) will be _________
Key: 800 to 800
 ICP–Intensive Classroom Program  eGATE-Live Internet Based Classes DLP  TarGATE-All India Test Series
Leaders in GATE Preparations  65+ Centers across India
© All rights reserved by Gateforum Educational Services Pvt. Ltd. No part of this booklet may be reproduced or utilized in any form without the written permission.

Visit : www.Civildatas.com
Visit : www.Civildatas.com

|ME| GATE-2017-PAPER-I www.Civildatas.com

1500  0.8
Exp: Speed of rotation of lead screw   800rpm
1.5

7. The molar specific heat at constant volume of an ideal gas is equal to 2.5 times the universal
gas constant (8.314 J/mol.K). When the temperature increases by 100K, the change in molar
specific enthalpy is _______________ J/mol.
Key: 2908 to 2911
Exp: C v  2.5R v where  R v  8.314 J mol.K 
T 100K
H  ?
H  Cp T
Cp  C v  R v
[Cp  3.5R v ]
So, H  3.5  8.314 100
[H  2909.9 J mol]

8. A particle of unit mass is moving on a plane. Its trajectory, in polar coordinates, is given by
r(t) = t2,   t   t, where t is time. The kinetic energy of the particle at time t = 2 is
(A) 4 (B) 12 (C) 16 (D) 24
Key: (C)
Exp: r = t 2 ;θ = t
1
K.E = mv 2  ? at t  2 sec
2
 m 1kg

V  r(t) ˆ  dr rˆ  t 2 1(t)
ˆ  2trˆ
dt
ˆ  2t(r)
V  t 2 (t) ˆ
at t  2s
ˆ  4(r)
V  4(t) ˆ
V  16  16  32
1 1
K.E.  mv 2  1 32  16
2 2

9. The Poisson‟s ratio for a perfectly incompressible linear elastic material is


(A) 1 (B) 0.5 (C) 0 (D) infinity
Key: (B)

10. A heat pump absorbs 10 kW of heat from outside environment at 250 K while absorbing 15
kW of work. It delivers the heat to a room that must be kept warm at 300K. The Coefficient of
Performance (COP) of the heat pump is ___________.
Key: 1.66 to 1.70
 ICP–Intensive Classroom Program  eGATE-Live Internet Based Classes DLP  TarGATE-All India Test Series
Leaders in GATE Preparations  65+ Centers across India
© All rights reserved by Gateforum Educational Services Pvt. Ltd. No part of this booklet may be reproduced or utilized in any form without the written permission.

Visit : www.Civildatas.com
Visit : www.Civildatas.com

|ME| GATE-2017-PAPER-I www.Civildatas.com

Exp:
300K

25kW  QH QH 25
COP    1.66
Wi/ p 15
H.P 15kW

10kW

250K

11. Which one of the following is NOT a rotating machine?


(A) Centrifugal pump (B) Gear pump
(C) Jet pump (D) Vane pump
Key: (C)
Exp: In the given options all the pumps have rotating machine elements except Jet pump.

12. Consider the schematic of a riveted lap joint subjected to tensile load F, as shown below. Let d
be the diameter of the rivets, and Sf be the maximum permissible tensile stress in the plates.
What should be the minimum value for the thickness of the plates to guard against tensile
failure of the plates? Assume the plates to be identical.

F F F 2F
(A) (B) (C) (D)
Sf  W  2d  Sf W Sf  W  d  Sf W
Key: (A)
F F F
Exp: Sf   Sf  ; t
A t  W  2d  Sf  w  2d 
13. Water (density =1000 kg/m3) at ambient temperature flows through a horizontal pipe of
uniform cross section at the rate of 1 kg/s. If the pressure drop across the pipe is 100 kPa, the
minimum power required to pump the water across the pipe, in watts, is _______
Key: 100 to 100
Exp: given,
w 1000 kg m3

m 1kg s
P 10kPa

So, minimum power require  m gh f
  P 
 mg 
 g 
1  100 1000
 100Watts
1000

 ICP–Intensive Classroom Program  eGATE-Live Internet Based Classes DLP  TarGATE-All India Test Series
Leaders in GATE Preparations  65+ Centers across India
© All rights reserved by Gateforum Educational Services Pvt. Ltd. No part of this booklet may be reproduced or utilized in any form without the written permission.

Visit : www.Civildatas.com
Visit : www.Civildatas.com

|ME| GATE-2017-PAPER-I www.Civildatas.com

14. For steady flow of a viscous incompressible fluid through a circular pipe of constant diameter,
the average velocity in the fully developed region is constant. Which one of the following
statements about the average velocity in the developing region is TRUE?
(A) It increases until the flow is fully developed.
(B) It is constant and is equal to the average velocity in the fully developed region.
(C) It decreases until the flow is fully developed.
(D) It is constant but always lower than the average velocity in the fully developed region.
Key: (B)
Exp: The average velocity in pipe flow always be same either for developing flow or fully
developed flow.

15. Cylindrical pins of diameter 150.020 mm are being produced on a machine. Statistical quality
control tests show a mean of 14.995 mm and standard deviation of 0.004mm. The process
capability index Cp is
(A) 0.833 (B) 1.667 (C) 3.333 (D) 3.750
Key: (B)
USL- LSL 15.02 14.98
Exp: Cp    1.666
6σ 6  0.004

2 0 1 
16. The product of Eigen values of the matrix P is P   4 3 3
0 2 1
(A) -6 (B) 2 (C) 6 (D) -2
Key: (B)
2 0 1 
Exp: P   4 3 3 
0 2 1
We know that, product of eigen values of P = determinant of P
 2  3  6   0  18   6  8  2

17. Match the processes with their characteristics.


Process Characteristics
P : Electrical Discharge machining 1. No residual stress
Q : Ultrasonic machining 2. Machining of electrically conductive
materials
R : Chemical machining 3. Machining of glass
S : Ion Beam Machining 4. Nano-machining
(A) P – 2, Q – 3, R – 1, S – 4 (B) P – 3, Q – 2, R – 1, S – 4
(C) P – 3, Q – 2, R – 4, S – 1 (D) P – 2, Q – 4, R – 3, S – 1
Key: (A)

 ICP–Intensive Classroom Program  eGATE-Live Internet Based Classes DLP  TarGATE-All India Test Series
Leaders in GATE Preparations  65+ Centers across India
© All rights reserved by Gateforum Educational Services Pvt. Ltd. No part of this booklet may be reproduced or utilized in any form without the written permission.

Visit : www.Civildatas.com
Visit : www.Civildatas.com

|ME| GATE-2017-PAPER-I www.Civildatas.com

x 3  sin  x 
18. The Value of limx 0 is
x
(A) 0 (B) 3 (C) 1 (D) -1
Key: (D)

Exp: t
x 0
x 3  sin x
x x 0
 
 t x 2   t
 x 0 x 

sin x 
  0  1  1

19. In an arc welding process, welding speed is doubled. Assuming all other process parameters to
be constant, the cross sectional area of the weld bead will
(A) Increase by 25% (B) Increase by 50% (C) Reduce by 25% (D) Reduce by 50%
Key: (D)
Exp:  V.I  H m A.V
1
A
V
A 2 V1 V
 
A1 V2 2V
A1
A2 
2
By doubling welding speed, Area reduces by 50%

20. A six-face fair dice is rolled a large number of times. The mean value of the outcomes is
______.
Key: 3.5 to 3.5
Exp: The Probabilities corresponding to the outcomes are given below:

Face 1 2 3 4 5 6
Probability 1/6 1/6 1/6 1/6 1/6 1/6

mean  E  x    x.P  x 
1 1  6   2  16   3 16   4  16   5  16   6  16 
1 21
 1  2  3  4  5  6   3.5
6 6


21. Consider the two dimensional velocity field given by V  5  a1x  b1y  i   4  a 2 x  b2 y  j,
wher a1 , b1 , a 2 and b2 are constants. Which one of the following conditions needs to be
satisfied for the flow to be incompressible?
(A) a1  b1  0 (B) a1  b 2  0 (C) a 2  b2  0 (D) a 2  b1  0
Key: (B)

Exp: Given V   5  a1 x  b1y  i   4  a 2 x  b 2 y  j
 u V
For Incompressible V  0; i.e.,  0
x y
a1  b 2  0

 ICP–Intensive Classroom Program  eGATE-Live Internet Based Classes DLP  TarGATE-All India Test Series
Leaders in GATE Preparations  65+ Centers across India
© All rights reserved by Gateforum Educational Services Pvt. Ltd. No part of this booklet may be reproduced or utilized in any form without the written permission.

Visit : www.Civildatas.com
Visit : www.Civildatas.com

|ME| GATE-2017-PAPER-I www.Civildatas.com

22. Consider a beam with circular cross-section of diameter d. The ratio of the second moment of
area about the neutral axis to the section modulus of the area is.
d d
(A) (B) (C) d (D) d
2 2
Key: (A)
 4
d
I I 64 d
Exp: Z y  
y Z 
d3 2
32

23. Saturated steam at 100°C condenses on the outside of a tube. Cold fluid enters the tube at 20°
C and exists at 50°C. The value of the Log Mean Temperature Difference (LMTD) is
________°C.
Key: 63.5 to 64 S
o  i 80  50
Exp: LMTD=  100C 100C 
n  o i  n  80 50  i  50C  
LMTD  63.82C 50C  o  80C
20C 

24. In a metal forming operation when the material has just started yielding, the principal stresses
are 1  180 MPa, 2  100 MPa, 3  0. Following Von Mises criterion, the yield stress is
________ MPa.
Key: 245 to 246
Exp: As per Von-Mises criteria
 1  2    1  3    5  1   2m2
2 2 2

180  100    100  0    0  180   22m


2 2 2

m  245.76MPa

25. In the engineering stress-strain curve for mild steel, the Ultimate Tensile Strength (UTS) refers
to
(A) Yield stress (B) Proportional limit
(C) Maximum stress (D) Fracture stress.
Key: (C)

Q. No. 26 to 55 Carry Two Marks Each

 u   u 
26. A parametric curve defined by x  cos   , y  sin   in the range 0  u  1 is rotated
 2   2 
about the X – axis by 360 degrees. Area of the surface generated is.

(A) (B)  (C) 2  (D) 4 
2
Key: (C)
 ICP–Intensive Classroom Program  eGATE-Live Internet Based Classes DLP  TarGATE-All India Test Series
Leaders in GATE Preparations  65+ Centers across India
© All rights reserved by Gateforum Educational Services Pvt. Ltd. No part of this booklet may be reproduced or utilized in any form without the written permission.

Visit : www.Civildatas.com
Visit : www.Civildatas.com

|ME| GATE-2017-PAPER-I www.Civildatas.com

 u   u 
Exp: Given x  cos   , y  sin   0    1
 2   2 
dx   u 
 sin  
d 2  2 
dy  u
 cos
dx 2 2
We know that surface area when the curve revolved about X- axis of a parametric curve is
2 2 2 2
 u     u      u  
1 1
 dx   dy 
 2 y      du  2 sin    sin      cos    du
0  du   du  0  2   2  2   2  2 
1
 u 
1
u  
2
u  1cos 2 
1
2  
 2 sin du  2   sin dx   
2
    cos  cos 0 
2

2 4 20 2    2 
0
2 
 0
 2 cos 0  1  2

27. Assume that the surface roughness profile is triangular as shown schematically in the figure. If
the peak to valley height is 20 m, The central line average surface roughness Ra (in m ) is

(A) 5 (B) 6.67 (B) 10 (D) 20


Key: (A)
R max 20
Exp: Ra    5m
4 4

28. A thin uniform rigid bar of length L and mass M is hinged at point O, located at a distance of
L
from one of its ends. The bar is further supported using springs, each of stiffness k, located
3
M
at the two ends. A particle of mass m  is fixed at one end of the bar, as shown in the
4
figure. For small rotations of the bar about O, the natural frequency of the systems is.

5k 5k 3k 3k
(A) (B) (C) (D)
M 2M 2M M
Key: (B)
Exp: Max moment of inertia of Rod.

 ICP–Intensive Classroom Program  eGATE-Live Internet Based Classes DLP  TarGATE-All India Test Series
Leaders in GATE Preparations  65+ Centers across India
© All rights reserved by Gateforum Educational Services Pvt. Ltd. No part of this booklet may be reproduced or utilized in any form without the written permission.

Visit : www.Civildatas.com
Visit : www.Civildatas.com

|ME| GATE-2017-PAPER-I www.Civildatas.com

I0  Ic  mr 2 L
2 L
ML2  2L L  m
I0   M   3
12  3 2
ML2 ML2 ML2 0
I0    K K
12 36 9
Mass moment of inertia of particular mass
2
M  2L  ML2
I particular    
4  3  9
ML2 ML2 2(ML2 )
ITotal   
9 9 9
 M0  0
 L  L   2L  2L  
K      K      I  0
 3  3   3  3 
5L2
 2ML 2
   5L 2
 .k
9 5k
   K    0  n  
 9   9  2ML2 2M
9

29. A point mass of 100 kg is dropped onto a massless elastic bar (cross-sectional area = 100 mm2,
length = 1m, Young‟s moduls = 100 GPa) from a height H of 10mm as shown (Figure is not to
scale). If g = 10m/s2, the maximum compression of the elastic bar is _______ mm.
m  100kg
H  10mm

L  1m
Key: 1.50 to 1.52
Exp: Given that
m  100kg, g  10m sec 2 , E  100GPa
H  10mm, L  1m  100mm,
A  100mm 2
From the given figure, we can say that this is case of Impact loading,
We know that, stress due to Impact load is
2
P  P   2PhE 
I.L      
A  A   AL 
P  mg  100  10  1000N

 1000   2  1000  10  1  10 
2 5
1000
I.L    
    151.7745 N mm
2

100  100   100  1000 


I.L  L 151.7745  1000
Compression       1.5177 mm
E 1  105

 ICP–Intensive Classroom Program  eGATE-Live Internet Based Classes DLP  TarGATE-All India Test Series
Leaders in GATE Preparations  65+ Centers across India
© All rights reserved by Gateforum Educational Services Pvt. Ltd. No part of this booklet may be reproduced or utilized in any form without the written permission.

Visit : www.Civildatas.com
Visit : www.Civildatas.com

|ME| GATE-2017-PAPER-I www.Civildatas.com

30. One kg of an ideal gas (gas constant, R = 400 J/kg.K; specific heat at constant volume,
c  1000J/kg.K) at 1 bar, and 300 K is contained in a sealed rigid cylinder. During an
adiabatic process, 100kJ of work is done on the system by a stirrer. The increase in entropy of
the system is _________ J/K.
Key: 286 to 288
Exp: Given that m  1kg, R  400 kJ kgK, CV  1000 J kgK
P1  1 bar, T1  300K
Since the gas is contained in a sealed rigid cylinder, and given that adiabatic process is done to
the system, means no heat is transferred from/to the system, Q = 0
And we know from first law of thermo dynamics, Adiabatic wall, Q  0
Q  du  W
0  mCV  T2  T1   100  103  du  mC V  T2  T1  
mCV  T2  T1   100  103 still Q work
given  100 kJ
1  1000  T2  300   100  103
100  103
T2   300  400K
1000
We know that the first law of thermodynamics can be written as
Tds  mC V dT  pdV
dT P  mRT 
ds  mC V  dv  PV  mRT  P  
T T  V 
dT mRT dT dV
ds  mC V  dV  mC V  mR
T rT T V
T  V 
Entropy increase  S2  S1   mCV n  2   mR n  2 
 T1   V1 
Since the above process is a constant volume process that is V2  V1
 400 
 S2  S1   1  1000   n    287.6821 J K
 300 

31. For an inline slider-crank mechanism, the lengths of the crank and connecting rod are 3m and
4m, respectively. At the instant when the connecting rod is perpendicular to the crank, if the
velocity of the slider is 1m/s, the magnitude of angular velocity (upto 3 decimal points
accuracy) of the crank is _________ radian/s.
Key: 0.26 to 0.27
Exp: Given that velocity of slider
VB  1m / sec A
Length of crank (OA) = 3m 4mm
Length of connecting rod (AB) = 4m 3mm
From the configuration diagram  B
A O 5mm
sin      53.130o C
5

 ICP–Intensive Classroom Program  eGATE-Live Internet Based Classes DLP  TarGATE-All India Test Series
Leaders in GATE Preparations  65+ Centers across India
© All rights reserved by Gateforum Educational Services Pvt. Ltd. No part of this booklet may be reproduced or utilized in any form without the written permission.

10

Visit : www.Civildatas.com
Visit : www.Civildatas.com

|ME| GATE-2017-PAPER-I www.Civildatas.com

The velocity diagram for the above configuration diagramO,c


is 1m sec
Oa b
 From velocity diagram sin 53.13o 
Ob 53.130o
 Oa  1 sin 53.130o  0.8
 VA  Oa  OA  OA  0.8
a
0.8
Angular velocity of crank  OA    0.267 rad sec
3

32. In an epicyclic gear train, shown in the figure, the outer


ring gear is fixed, while the sun gear rotates
counterclockwise at 100rpm. Let the number of teeth on
the sun, planet and outer gears to be 50, 25, and 100,
respectively. The ratio of magnitudes of angular velocity
of the planet gear to the angular velocity of the carrier
arm is _________.

Key: 3 to 3
Exp: TS = 50
TP=25
TR=100

Arm
S.No Condition of motion Gear S Gear P Gear R
Gear A
Arm is fixed gear S Ts Ts
1 0 +1  
with +1 revolution TP TR
Arm is fixed Gear S Ts Ts
2 0 +x -x x
with +x revolution TP TR
Arm with + y
3 y y y y
revolution
Ts Ts
4 Total y x+y yx y x
TP TR

NS  x  y  100 ...(1)
50
NR  y  x0
100
y  0.5x ...  2 
x  0.5x  100
100
x  66.66rpm
1.5
y  33.34 rpm
 50 
N P  33.34   66.66    99.99rpm
 25 
NP 99.99
  3  approx 
N arm 33.32

 ICP–Intensive Classroom Program  eGATE-Live Internet Based Classes DLP  TarGATE-All India Test Series
Leaders in GATE Preparations  65+ Centers across India
© All rights reserved by Gateforum Educational Services Pvt. Ltd. No part of this booklet may be reproduced or utilized in any form without the written permission.

11

Visit : www.Civildatas.com
Visit : www.Civildatas.com

|ME| GATE-2017-PAPER-I www.Civildatas.com

33. Moist air is treated as an ideal gas mixture of water vapor and dry air (molecular weight of air
= 28.84 and molecular weight of water = 18). At a location, the total pressure is 100 kPa, the
temperature is 30°C and the relative humidity is 55%. Given that the saturation pressure of
water at 30°C is 4246 Pa, the mass of water vapor per kg of dry air is _____________ grams.
Key: 14.7 to 15.1
Exp: Ma  28.84
M w 18
P 100kPa, T  30C, RH  55%,
Ps  4246Pa
Pv P
  0.55  v  Pv  2335.3Pa
Ps 4246
 P   2335.3 
 0.622  v   0.622   
 P  Pv   100000  2335.3 
 0.01487 kg of vapour kg of D.A
14.87 gm of vapour kg of D.A

34. Following data refers to the jobs (P, Q, R, S) which have arrived at a machine for scheduling.
The shortest possible average flow time is ___________ days.

Job Processing Time (days)


P 15
Q 9
R 22
S 12

Key: 31 (not matching with IIT key)


Exp: For shortest avg. flow time SPT rule is used
Job Sequence Processing Time In Out Flow Time
Q 9 0 9 9
S 12 9 21 21
P 15 21 36 36
R 22 36 58 58
9  21  36  58
Min Avg. Flow time   31 days
4

35. Two models, P and Q, of a product earn profits of Rs. 100 and Rs. 80 per piece, respectively.
Production times for P and Q are 5 hours and 3 hours, respectively, while the total production
time available is 150 hours. For a total batch size of 40, to maximize profit, the number of
units of P to be produced is ____________.
Key: 15 to 15
Exp: Let x1  No. of units of P
x 2  No. of units of Q

 ICP–Intensive Classroom Program  eGATE-Live Internet Based Classes DLP  TarGATE-All India Test Series
Leaders in GATE Preparations  65+ Centers across India
© All rights reserved by Gateforum Educational Services Pvt. Ltd. No part of this booklet may be reproduced or utilized in any form without the written permission.

12

Visit : www.Civildatas.com
Visit : www.Civildatas.com

|ME| GATE-2017-PAPER-I www.Civildatas.com


x2
max. z  100 x1  80x 2  0,50 
5x1  3x 2  150
x1  x 2  40  0, 40  15, 25 
Z 0,40  Rs.3200
Z15,225  Rs.3500  max .profit  0,0 
x1
 30,0   40,0 
So, for maximum profit, No. of units of P produced is 15 units.

36. Circular arc on a part profile is being machined on a vertical CNC milling machine. CNC part
program using metric units with absolute dimensions is listed below:
--------------------------------
N60 G01 X 30 Y 55 Z – 5 F 50
N70 G02 X 50 Y 35 R 20
N80 G01 Z 5
--------------------------------
The coordinates of the centre of the circular arc are :
(A) (30, 55) (B) (50, 55) (C) (50, 35) (D) (30, 35)
Key: (D)
Exp: y

 30,56 

20

 50,35 
 30,35 

x
20

Centre of circular arc is (30, 35)


37. Two black surfaces, AB and BC, of lengths 5m and 6m,
respectively, are oriented as shown. Both surfaces extend
infinitely into the third dimension. Given that view factor
F12=0.5, T1=800K, T2=600K, Tsurrounding=300K and Stefan
Boltzmann constant,   5.67 108 W /  m2 K 4  , the heat
transfer rate from Surface 2 to the surrounding environment is
____________ kW.
Key: 13.7 to 13.9
Exp: Given that two black surfaces „AB‟ and „BC‟

Length of AB = 5m, BC = 6 m A
And temperature of
Surface „1‟  TBC  T1   800o K Surface '1'
Surroundings'3'
Temperature of surface „2‟  TAB  T2  600 K  o

Temperature of surroundings  T3   300o K


B C

 ICP–Intensive Classroom Program  eGATE-Live Internet Based Classes DLP  TarGATE-All India Test Series
Leaders in GATE Preparations  65+ Centers across India
© All rights reserved by Gateforum Educational Services Pvt. Ltd. No part of this booklet may be reproduced or utilized in any form without the written permission.

13

Visit : www.Civildatas.com
Visit : www.Civildatas.com

|ME| GATE-2017-PAPER-I www.Civildatas.com

  5.67  108 W / m 2 K 4
F12  0.5
W.K.T. F11  F22  0, sin ce they areflat surfaces
F21  F22  F23  1
A1F12  A 2F21
  6  0.5    5  F21  Assume unit width for surfaces 
3
 F21   0.6
5
0.6  0  F23  1  F23  1  0.6  0.4
Using resistance concept we can draw as follows
Since surfaces are black and area of surrounding is large we can write  1 
E b1  J1 , E b2  J 2 , E b3  J 3  E b2  J 2   A 2 F23   E b3  J 3 
E  E b3   T2  T3 
4 4

Q 23  b2  2 3
 1  1
  A 2 F23 1 1
 A 2 F23 
 5.67  108   6004  300 4   5  0.4
A2 F21 A1F13

 13.778kW / metre
1
 1 0 1  E b1  J1
 2 2 

38. Consider the matrix P   0 1 0 .
 1 0 1 

 2 2 
Which one of the following statements about P is INCORRECT?
(A) Determinant of P is equal to 1.
(B) P is orthogonal.
(C) Inverse of P is equal to its transpose.
(D) All Eigen values of P are real numbers
Key: ( D)
 1 0 1 
 2 2 

Exp: P  0 1 0 
 1 0 1 
 
 2 2 
1  1  1  1  1 1
P    0  0      1
2 2  2 2 2 2
 1 0 1  1 0 1 
 2    
 2 2 2  1 0 0 
P.P T  0 1 0  0 1 0   0 1 0 
 1 0 1  1 0 
1  0 0 1 
 
 2 2   2 2 

 ICP–Intensive Classroom Program  eGATE-Live Internet Based Classes DLP  TarGATE-All India Test Series
Leaders in GATE Preparations  65+ Centers across India
© All rights reserved by Gateforum Educational Services Pvt. Ltd. No part of this booklet may be reproduced or utilized in any form without the written permission.

14

Visit : www.Civildatas.com
Visit : www.Civildatas.com

|ME| GATE-2017-PAPER-I www.Civildatas.com

P is an orthogonal matrix
(A) Is correct  Inverse of P is its transpose only
(B) and (C) both are correct
(D) is incorrect

39. The Pressure ratio across a gas turbine (for air, specific heat at constant pressure,
c p  1040J / kg.K and ratio of specific heats,   1.4) is 10. If the inlet temperature to the
turbine is 1200K and the isentropic efficiency is 0.9, the gas temperature at turbine exit is
______ K.
Key: 675 to 684
Exp: Cp  1040 J kg.K , r 1.4
P2 P1  10, T3 1200K
is  0.9
T
P2
 Isentropic Expansion,
r 1 3 P1
 P3  r  T3  1200 2
      10 
0.4/1.4

 P4   T4  T4
41
T4  621.5K 4
1
T3  T 1
S
iso  4

T3  T4
So, T41  T3  iso  T3  T4   1200  09 1200  621.5 
T4  679.38K

40. An initially stress-free massless elastic beam of length L and circular cross-section with
diameter d (d << L) is held fixed between two walls as shown. The beam material has Young‟s
modulus E and coefficient of thermal expansion  .

If the beam is slowly and uniformly heated, the temperature rise required to cause the beam to
buckle is proportional to
(A) d (B) d2 (C) d3 (D) d4
Key: (B)
EI
Exp: P  EA  T  
 2e
 1  I
T    
  A
 4 d
d L
 1 64
T   
   d2
4
T  d 2
 ICP–Intensive Classroom Program  eGATE-Live Internet Based Classes DLP  TarGATE-All India Test Series
Leaders in GATE Preparations  65+ Centers across India
© All rights reserved by Gateforum Educational Services Pvt. Ltd. No part of this booklet may be reproduced or utilized in any form without the written permission.

15

Visit : www.Civildatas.com
Visit : www.Civildatas.com

|ME| GATE-2017-PAPER-I www.Civildatas.com

 
41. For the vector V  2yzi  3xzj  4xyk,
 the value of .   V is ____________
 
Key: 0 to 0

Exp: V  2yzi  3xzj  4xyk
we know that .   V   0 for any vector V

42. A 10 mm deep cylindrical cup with diameter of 15mm is drawn from a circular blank.
Neglecting the variation in the sheet thickness, the diameter (upto 2 decimal points accuracy)
of the blank is _________ mm.
Key: 28.71 to 28.73

Exp: D  d2  4dh  152  4 10 15  28.72 mm

43. A machine element has an ultimate strength  u  of 600 N/mm2, and endurance limit   en  of
250 N/mm2. The fatigue curve for the element on log-log plot is shown below. If the element
is to be designed for a finite of 10000 cycles, the maximum amplitude of a completely
reversed operating stress is _________ N/mm2.

Key: 370 to 390 (0.8 u )


A
Exp: u  600MPa

en  250MPa B
e
N  10000 cycle
log  0.8u   log  250  log  0.8u   log   

36 34
log  480   log  250  log  480   log    103 104 106

3 1
3log  480   log  480   log  250  2log  480   log  250 
log     
3 3
max  386.19MPa

44. A sprue in a sand mould has a top diameter of 20mm and height of 200mm. The velocity of
the molten metal at the entry of the sprue is 0.5m/s. Assume acceleration due to gravity as 9.8
m/s2 and neglect all losses. If the mould is well ventilated, the velocity (upto 3 decimal points
accuracy) of the molten metal at the bottom of the sprue is ________ m/s.

Key: 2.04 to 2.07

 ICP–Intensive Classroom Program  eGATE-Live Internet Based Classes DLP  TarGATE-All India Test Series
Leaders in GATE Preparations  65+ Centers across India
© All rights reserved by Gateforum Educational Services Pvt. Ltd. No part of this booklet may be reproduced or utilized in any form without the written permission.

16

Visit : www.Civildatas.com
Visit : www.Civildatas.com

|ME| GATE-2017-PAPER-I www.Civildatas.com

Exp: Apply Bernoulli‟s between (1) and (2)


P1 V12 P V2 1
  z1  2  2  z 2
 2g  2g
0.52 V22
 0.2  200 mm
2g 2g
V2  2.042 m s
2

45. Air contains 79% N2 and 21% O2 on a molar basis. Methane (CH4) is burned with 50% excess
air than required stoichiometrically. Assuming complete combustion of methane, the molar
percentage of N2 in the products is ________________
Key: 73 to 74
 79  79
Exp: Stoichiometric reaction CH 4  2. O 2  N 2  2H 2O  CO 2 2   N 2
 21  21
50% excess air
 79  79
CH 4  3. O 2  N 2   2H 2O  CO 2  3   N 2  O 2
 21  21
3  79
%N 2  21 100  73.83%
79
2 11 3
21

46. P(0,3), Q(0.5, 4), and R (1,5) are three points on the curve defined by f(x). Numerical
integration is carried out using both Trapezoidal rule and Simpson‟s rule within limits x = 0
and x =1 for the curve. The difference between the two results will be.
(A) 0 (B) 0.25 (C) 0.5 (D) 1
Key: (A)
Exp: Let x 0 0.5 1
y 3 4 5
Trapezoidial rule
1
0.5 0.5
 f  x  dx 
0
2 
 3  5   2  4   
2
 16  4

Simpsons rule
1
0.5 0.5
 f  x  dx 
0
3
 3  5   0  4  4   
3
 24  4

Difference  0

47. Heat is generated uniformly in a long solid cylindrical rod (diameter = 10mm) at the rate of
4×107 W/m3. The thermal conductivity of the rod material is 25W/m.K. Under steady state
conditions, the temperature difference between the centre and the surface of the rod is
_________ °C.
Key: 10 to 10

 ICP–Intensive Classroom Program  eGATE-Live Internet Based Classes DLP  TarGATE-All India Test Series
Leaders in GATE Preparations  65+ Centers across India
© All rights reserved by Gateforum Educational Services Pvt. Ltd. No part of this booklet may be reproduced or utilized in any form without the written permission.

17

Visit : www.Civildatas.com
Visit : www.Civildatas.com

|ME| GATE-2017-PAPER-I www.Civildatas.com

Exp: Given that, heat is generated uniformly i.e., g  4 107 W/ m3

Diameter at the rod (d) = 10 mm


Thermal conductivity of the rod (K) = 25 W/mK

dT To
0
dr
Temperatureprofile
TS T3
g  4 107 W / m3
K  25W / m3K
d  10mm

W.K.T for steady state, with internal heat generation, the conduction equation will be,
1 d  dT  g
 r.    0
r dr  dr  k
d  dT  gr
 r  0
dr  dr  K
After Integration once
dT gr 2 dT gr  c
r.   C1    1  (1)
dr 2K dr 2K r
After second Integration
 2
gr
Tr   C1n  r   C 2  (2)
4K
dT
Substituting boundary condition of  0at r  0in eq(1)
dr
That gives C1  0
 2
gr
 Tr   C 2 , substitute at r =0, T(r) = To
4k
 To   o   C 2  C 2  To
 2
gr
T  r    To
4K
10
Substitute r  ro   5mm  0.005m andT  r   TS
2
The above equation will become
 2
gr
TS  o  T o
4K
4  107  0.0052
 To  Ts   10o C
4  25

 ICP–Intensive Classroom Program  eGATE-Live Internet Based Classes DLP  TarGATE-All India Test Series
Leaders in GATE Preparations  65+ Centers across India
© All rights reserved by Gateforum Educational Services Pvt. Ltd. No part of this booklet may be reproduced or utilized in any form without the written permission.

18

Visit : www.Civildatas.com
Visit : www.Civildatas.com

|ME| GATE-2017-PAPER-I www.Civildatas.com

48. Two disks A and B with identical mass (m) and radius (R) are initially at rest. They roll down
from the top of identical inclined planes without slipping. Disk A has all of its mass
concentrated at the rim, while Disk B has its mass uniformly distributed. At the bottom of the
plane, the ratio of velocity of the center of disk A to the velocity of the center of disk B is.
3 3
(A) (B) (C) 1 (D) 2
4 2
Key: (A)
Exp: IA  MR 2
MR 2
IB  A,B
2
m

P.E   K.E T  K.E R A   K.E T  K.E R B
1 1 1 1 H
 MVA2  IA2  MVB2  IB2
2 2 2 2
1 1 1 1 MR 2 2
 MVA2  MR 2 2A  MVB2  B
2 2 2 2 2
VB2
VA2  VA2  VB2 
2
3 V 3
2VA2  VB2  A 
2 VB 4

49. A block of length 200mm is machined by a slab milling cutter 34mm in diameter. The depth of
cut and table feed are set at 2mm and 18mm/minute, respectively. Considering the approach
and the over travel of the cutter to be same, the minimum estimated machining time per pass is
_____________ minutes.
Key: 12 to 12
2  Dd  d 2  L 2  (34  2)  2  200
2
Exp: Milling Time   12
f 18
Where, D  Dia of cutter  mm  , d  depth of cut  mm 
L  length f  feed  mm min 

50. A horizontal bar, fixed at one end (x = 0), has a length of 1 m, and cross-sectional area of
100mm2. Its elastic modulus varies along its length as given by E(x) = 100e -x GPa, Where x is
the length coordinate (in m) along the axis of the bar. An axial tensile load of 10 kN is applied
at the free end (x=1). The axial displacement of the free end is _______ mm.
Key: 1.70 to 1.72
Exp: Given that

P  10kN  10  103 N, A  100mm 2 Stripof lengthdx


x
100  e  10 9
x
E  x   100e x Gpa   105  e x dx
106
P  10kN
P  dx 
Change in the length of small strip   x0 x 1
AE x
L  1mm

 ICP–Intensive Classroom Program  eGATE-Live Internet Based Classes DLP  TarGATE-All India Test Series
Leaders in GATE Preparations  65+ Centers across India
© All rights reserved by Gateforum Educational Services Pvt. Ltd. No part of this booklet may be reproduced or utilized in any form without the written permission.

19

Visit : www.Civildatas.com
Visit : www.Civildatas.com

|ME| GATE-2017-PAPER-I www.Civildatas.com

L
Pdx
Total change in the length of the bar     
0
AE x
L L L 1
P dx P dx P P P
 x
 5  x
 5 
e x dx  5 
e x dx  e1  1
5 
0
A 100e A  10 0 e A  10 0 A 10 0 A 10
10  103
  2.7183  1 1000  1.7183mm
100  105

51. Consider steady flow of an incompressible fluid through two long and straight pipes of
diameters d1 and d2 arranged in series. Both pipes are of equal length and the flow is turbulent
in both pipes. The friction factor for turbulent flow though pipes is of the form, f  K(Re) n
where K and n are known positive constants and Re is the Reynolds number. Neglecting minor
 P 
losses, the ratio of the frictional pressure drop in pipe 1 to that in pipe 2,  1  , is given by
 P2 
 5 n  5  3 n   5 n 
d  d  d  d 
(A)  2  (B)  2  (C)  2  (D)  2 
 d1   d1   d1   d1 
Key: (A)
 f L Q2  12d 52g  f1  d 2   Re1   d 2 
5 n 5
P1  64 
Exp:   1 15  2 
    n   ......(1)  f  
P2  12d1 g  f 2 L2 Q  f 2  d1   Re2   d1   Re 

Vd   4Q 
Re  & Q  AV  d 2 V   V  2 
 4  d 
4Q d 4Q
So, Re  2  
d  d
d1 d2
5 L1  L2
1 P d n  d 
Re  From(1) 1  1 n  2 
d P2 d 2  d1 
5 n
P1  d 2  L1 L2
 
P2  d1 

52. The velocity profile inside the boundary layer for flow over a flat plate is given as
u  y
 sin   , where U is the free stream velocity and  is the local boundary layer
U 2 
*
thickness. If  * is the local displacement thickness, the value of is

2 2 2
(A) (B) 1  (C) 1+ (D) 0
  
Key: (B)
U  y 
Exp:  sin  
U  2 
displacement thickness
  y 
*   1  U U  dy  1  sin  dy
0
 2 

 ICP–Intensive Classroom Program  eGATE-Live Internet Based Classes DLP  TarGATE-All India Test Series
Leaders in GATE Preparations  65+ Centers across India
© All rights reserved by Gateforum Educational Services Pvt. Ltd. No part of this booklet may be reproduced or utilized in any form without the written permission.

20

Visit : www.Civildatas.com
Visit : www.Civildatas.com

|ME| GATE-2017-PAPER-I www.Civildatas.com


cos   y 2   0
  
*

  2 
2
*    cos  2  cos 0

 * 2 
     
 
   1  2  
*


53. For a steady flow, the velocity field is V   x 2  3y  i   2xy  j. The magnitude of the
acceleration of a particle at (1, -1) is
(A) 2 (B) 1 (C) 2 5 (D) 0
Key: (C)

Exp: V   x 2  3y  i   2xy  j
U   x 2  3y
 
V  2xy

U U
a x  U   V     x 2  3y   2x    2xy  3  2x 3  6xy  6xy  2x 3
 x  y               
 
V V
a y  U   V     x 2  3y   2y    2xy  2x   2x 2 y  6y 2  4x 2 y  2x 2 y  6y 2
 x  y                 
x y
but x  1, y  1
but x  1, y  1

ax  2
   2       2  
a y  2 1  1  6  1  6  2  4
    
a  a 2x  a 2y  4  16  20

a2 5

54. Two cutting tools with tool life equations given below are being compared:
Tool 1: VT0.1=150
Tool 2: VT0.3=300
Where V is cutting speed in m/minute and T is tool life in minutes. The breakeven cutting
speed beyond which Tool 2 will have a higher tool life is ________ m/minute.
Key: 105 to 107
Exp: At Breakeven point
T1  T2
1 0.1 1 0.3
 150   300 
   
 V   V 
V  106.121 m min

 ICP–Intensive Classroom Program  eGATE-Live Internet Based Classes DLP  TarGATE-All India Test Series
Leaders in GATE Preparations  65+ Centers across India
© All rights reserved by Gateforum Educational Services Pvt. Ltd. No part of this booklet may be reproduced or utilized in any form without the written permission.

21

Visit : www.Civildatas.com
Visit : www.Civildatas.com

|ME| GATE-2017-PAPER-I www.Civildatas.com

55. A rectangular region in a solid is in a state of plane strain. The (x,y) coordinates of the corners
of the under deformed rectangle are given by P(0,0), Q (4,0), S (0,3). The rectangle is
subjected to uniform strains,  xx  0.001,  yy  0.002,  xy  0.003. The deformed length of the
elongated diagonal, up to three decimal places, is _________ units.
Key: 5.013 to 5.015
Exp: Given that

 xx  0.001,  yy  0.002
rxy  0.003

Length of the diagonal  PR   42  32  5m


y
 yy
S  0,3 R  4,3 

5mm  xx
 xx rxy
x
P  0,0  Q  4,0 
3  yy
tan 1  
4
To find the diagonal (PR) strain, the direction of the plane angle from the +ve x-axis will from
„R‟ towards „P‟
  ?
Where   180  tan 1  3 / 4   216.87o
 xx   yy  xx   yy rxy
 216.87o   cos 2 
sin 2
2 2 2
0.001  0.002 0.001  0.002
sin  2  216.87 o 
0.003
  cos  2  216.87  
2 2 2
 0.0015  1.4  104  1.44  103
 2.8  103
Elongation of the diagonal = 216.87  5  2.8 103 • 5  0.014
Defined length of diagonal  5  0.014  5.014

 ICP–Intensive Classroom Program  eGATE-Live Internet Based Classes DLP  TarGATE-All India Test Series
Leaders in GATE Preparations  65+ Centers across India
© All rights reserved by Gateforum Educational Services Pvt. Ltd. No part of this booklet may be reproduced or utilized in any form without the written permission.

22

Visit : www.Civildatas.com
Visit : www.Civildatas.com

|ME| GATE-2017-PAPER-I www.Civildatas.com

General Aptitude
Q. No. 1 - 5 Carry One Mark Each
1. A right – angled cone (with base radius 5cm and height 12cm), as shown in the figure below,
is rolled on the ground keeping the point P fixed until the point Q (at the base of the cone, as
shown) touches the ground again.

By what angle (in radians) about P does the cone travel?


5 5 24 10
(A) (B) (C) (D)
12 24 5 13
Key: (D)
Exp: L  52  122 13cm 
L
Circumference of base circle = length of arc QR
R P
(2r)  R (R  Slant height of the Cone  13 cm)
(2  5) 13
10 R

13
2. In a company with 100 employees, 45 earn Rs. 20,000 per month, 25 earn Rs. 30,000, 20 earn
Rs. 40,000,8 earn Rs. 60,000, and 2 earn Rs. 150,000. The median of the salaries is
(A) Rs. 20,000 (B) Rs.30,000 (C) Rs. 32,300 (D) Rs. 40,000
Key: (B)
Exp: All the values put either in ascending or descending order first.
Now number of observations equal to 100 [even]
The median of these values = Avg of two middle most observations.
50 th observation  51st observation 30,000  30,000
   30,000
2 2

3. As the two speakers became increasingly agitated, the debate became __________.
(A) lukewarm (B) poetic (C) forgiving (D) heated
Key: (D)

4. P,Q, and R talk about S‟s car collection. P states that S has at least 3 cars. Q believes that S has
less than 3 cars. R indicates that to his knowledge, S has at least one Car. Only one of P, Q and
R is right the number cars owned by S is.
(A) 0 (B) 1 (C) 3 (D) Cannot be determined
Key: (A)
Exp: P States that S has atleast 3 cars, i.e.,  3
Q believes that S has less than 3 cars, i.e.,  3
R indicates that S has atleast one car  1
P‟s and Q‟s statements are exactly opposite in nature and R‟s statement is proportional to P‟s
statement.

 ICP–Intensive Classroom Program  eGATE-Live Internet Based Classes DLP  TarGATE-All India Test Series
Leaders in GATE Preparations  65+ Centers across India
© All rights reserved by Gateforum Educational Services Pvt. Ltd. No part of this booklet may be reproduced or utilized in any form without the written permission.

23

Visit : www.Civildatas.com
Visit : www.Civildatas.com

|ME| GATE-2017-PAPER-I www.Civildatas.com

From the given data, only one person statement is right as it mean that two persons statements
are wrong, i.e., P and R wrong when S has zero cars.

5. He was one of my best __________ and I felt his loss _________.


(A) friend, keenly (B) friends, keen (C) friend, keener (D) friends, keenly
Key: (D)
Q. No. 6- 10 Carry Two Marks Each

6. Two very famous sportsmen Mark and Steve happened to be brothers, and played for country K.
Mark teased James, an opponent from country E, “There is no way you are good enough to play
for your country.‟‟ James replied, “Maybe not, but at least I am the best player in my own family.”
Which one of the following can be inferred from this conversation?
(A) Mark was known to play better than James
(B) Steve was known to play better than Mark
(C) James and Steve were good friends
(D) James played better than Steve
Key: (B)

7. “Here, throughout the early 1820s, Stuart continued to fight his losing battle to allow his
sepoys to wear their caste-marks and their own choice of facial hair on parade, being again
reprimanded by the commander-in-chief. His retort that „A stronger instance than this of
European prejudice with relation to this country has never come under my observations‟ had
no effect on his superiors.”
According to this paragraph, which of the statements below is most accurate?
(A) Stuart‟s commander – in chief was moved by this demonstration of his prejudice.
(B) The Europeans were accommodating of the sepoys‟ desire to wear their caste – marks.
(C) Stuart‟s losing battle‟ refers to his inability to succeed in enabling sepoys to wear caste-
marks.
(D) The commander– in – Chief was exempt from the European preiudice that dictated how
the sepoys were to dress.
Key: (C)

8. The growth of bacteria (lactobacillus) in milk leads to curd formation. A minimum bacterial
population density of 0.8(in suitable units) is needed to form curd. In the graph below, the
population density of lactobacillus in 1 litre of milk is plotted as a function of time, at two
different temperatures, 25°C and 37°C.

Consider the following statements based on the data shown above:


 ICP–Intensive Classroom Program  eGATE-Live Internet Based Classes DLP  TarGATE-All India Test Series
Leaders in GATE Preparations  65+ Centers across India
© All rights reserved by Gateforum Educational Services Pvt. Ltd. No part of this booklet may be reproduced or utilized in any form without the written permission.

24

Visit : www.Civildatas.com
Visit : www.Civildatas.com

|ME| GATE-2017-PAPER-I www.Civildatas.com

(i) The growth in bacterial population stops earlier at 37°C as compared to 25°C
(ii) The time taken for curd formation at 25°C is twice the time taken at 37°C
Which one of the following options is correct?
(A) Only i (B) only ii (C) Both i and ii (D) Neither i nor ii
Key: (A)
Exp: From the graph, Statement (i) is correct,
The time taken for curd formation @25o C= 120 min,
the time taken for curd formation @ 37o C= 80 min, hence (ii) is incorrect.

9. Let S1 be the plane figure consisting of the points (x,y) given by the inequalities x  1  2 and
y  2  3. Let S2 be the plane figure given by the inequalities x  y  2, y 1, and x  3 Let
S be the union of S1 and S2. The area of S is.
(A) 26 (B) 28 (C) 32 (D) 34
Key: (C)

10. What is the sum of the missing digits in the subtraction problem below?
5_ _ _ _
4 8 _ 8 9
1111
(A) 8 (B) 10 (C) 11 (D) Cannot be determined
Key: (D)

 ICP–Intensive Classroom Program  eGATE-Live Internet Based Classes DLP  TarGATE-All India Test Series
Leaders in GATE Preparations  65+ Centers across India
© All rights reserved by Gateforum Educational Services Pvt. Ltd. No part of this booklet may be reproduced or utilized in any form without the written permission.

25

Visit : www.Civildatas.com
Visit : www.Civildatas.com

|ME| GATE-2017-PAPER-II www.Civildatas.com

Mechanical Engineering
Q. No. 1 to 25 Carry One Mark Each

1. A mass m of a perfect gas at pressure p1 and volume V1 undergoes an isothermal process. The
final pressure is p2 and volume is V2. The work done on the system is considered positive. If R
is the gas constant and T is the temperature, then the work done in the process is
V2 p1 V2 p2
(A) p1V1 ln (B) p1V1 ln (C) RT ln (D) mRT ln
V1 p2 V1 p1
Key: (B)
Exp: Isothermal work done, W   pdV
For isothermal, pV= C
p1V1  pV  C  constant 
V2
dV
so, W  p1V1 
V1
V
V  p 
W  p1V1 n  2   p1V1 n  1  ( p1V1  p 2 V2 )
 V1   p2 

2. Which one of the following statements is TRUE for the ultrasonic machining (USM) process?
(A) In USM, the tool vibrates at subsonic frequency.
(B) USM does not employ magnetostrictive transducer.
(C) USM is an excellent process for machining ductile materials.
(D) USM often uses a slurry comprising abrasive-particles and water.
Key: (D)

3. The standard deviation of linear dimensions P and Q are 3 m and 4 m, respectively. When
assembled, the standard deviation (in m ) of the resulting linear dimension (P+Q) is ________
Key: 5 to 5
Exp: Given that
Standard deviate of P is 3 m  Variance of P is 9 m
Standard deviation of Q is 4 m  Variance of Q is 16 m
Variance of P + Q = Var (P+Q) = Variance P + Variance Q
= 9 + 16= 25
 Standard deviation of P + Q = + Variance = + 25 = 5

4. The emissive power of a blackbody is P. If its absolute temperature is doubled, the emissive
power becomes.
(A) 2P (B) 4P (C) 8P (D) 16P
Key: (D)
Exp: Emissive power of black body  E b   T 4

Given  E b 1  P  T 4

 ICP–Intensive Classroom Program  eGATE-Live Internet Based Classes DLP  TarGATE-All India Test Series
Leaders in GATE Preparations  65+ Centers across India
© All rights reserved by Gateforum Educational Services Pvt. Ltd. No part of this booklet may be reproduced or utilized in any form without the written permission.

Visit : www.Civildatas.com
Visit : www.Civildatas.com

|ME| GATE-2017-PAPER-II www.Civildatas.com

now T  2T
 Eb 2    2T   16T 4   E b 2  16P
4

5. The state of stress at a point is x   y  z  xz  zx   yz  zy  0 and  xy   yx  50MPa .


The maximum normal stress (in MPa) at that point is _________
Key: 49.9 to 50.1
Exp: 
 0,  
It is a pure torsion case in 2D
xy   yx  50MPa
xy

 for pure Torsion, 


xy   yx  1
0,   xy
1
So maximum Normal stress 1  50MPa

6. The determinant of a 2×2 matrix is 50. If one eigenvalue of the matrix is 10, the other
eigenvalue is ___________
Key: 5 to 5
Exp: Given that det of 2×2 Matrix is 50 and are Eigen Value is 10.
 Other Eigen value is 5  det  product of eigenvalues 

7. Which one of the following statement is TRUE?


(A) Both Pelton and Francis turbines are impulse turbines.
(B) Francis turbine is a reaction turbine but Kaplan turbine is an impulse turbine.
(C) Francis turbine is an axial – flow reaction turbine.
(D) Kaplan turbine is an axial – flow reaction turbine.
Key: (D)

8. Two coins are tossed simultaneously. The probability (upto two decimal points accuracy) of
getting at least one head is ____________
Key: 0.75 to 0.75
Exp: Total No of outcomes when two coins are tossed is 4 and sample space
S  HH, HT,TH,TT
Favorable out comes for existence of at least one head are HH, HT, TH.
3
Required probability =  0.75
4

9. A cantilever beam of length L and flexural modulus EI is subjected to a point load P at the free
end. The elastic strain energy stored in the beam due to bending (neglecting transverse shear)
P 2 L3 P 2 L3 PL3 PL3
(A) (B) (C) (D)
6EI 3EI 3EI 6EI
Key: (A)

 ICP–Intensive Classroom Program  eGATE-Live Internet Based Classes DLP  TarGATE-All India Test Series
Leaders in GATE Preparations  65+ Centers across India
© All rights reserved by Gateforum Educational Services Pvt. Ltd. No part of this booklet may be reproduced or utilized in any form without the written permission.

Visit : www.Civildatas.com
Visit : www.Civildatas.com

|ME| GATE-2017-PAPER-II www.Civildatas.com

Exp: M x   P.x
x P
L 2 2 2
M dx P x dx
U   x

0
2EI 2EI L
x x
P 2 L3
U
6EI

10. It is desired to make a product having T-shaped cross-section from a rectangular aluminium
block. Which one of the following processes is expected to provide the highest strength of the
product?
(A) Welding (B) Casting (C) Metal Forming (D) Machining
Key: (C)

11. The heat loss from a fin is 6W. The effectiveness and efficiency of the fin are 3 and 0.75,
respectively. The heat loss (in W) from the fin, keeping the entire fin surface at base
temperature, is __________.
Key: 7.9 to 8.1
Exp: Given QLoss  6W
  3,   0.75
Q Qloss
  act 
Q max Heat loss keep entire fin as base temperature
6
Q max   8W
0.75

12. For a single server with Poisson arrival and exponential service time, the arrival rate is 12 per
hour. Which one of the following service rates will provide a steady state finite queue length?
(A) 6 per hour (B) 10 per hour (C) 12 per hour (D) 24 per hour
Key: (D)
Exp:   12 / hour
For steady state finite queue length

So,   24 / hour

13. For the stability of a floating body the


(A) centre of buoyancy must coincide with the centre of gravity
(B) centre of buoyancy must be above the centre of gravity
(C) centre of gravity must be above the centre of buoyancy
(D) metacentre must be above the centre of gravity
Key: (D)
Exp: Stability of floating body is measure with the help of Meta center.
Floating body to be stable, Meta center must be above C.G
I
 MG   BG ,

For stability MG > 0

 ICP–Intensive Classroom Program  eGATE-Live Internet Based Classes DLP  TarGATE-All India Test Series
Leaders in GATE Preparations  65+ Centers across India
© All rights reserved by Gateforum Educational Services Pvt. Ltd. No part of this booklet may be reproduced or utilized in any form without the written permission.

Visit : www.Civildatas.com
Visit : www.Civildatas.com

|ME| GATE-2017-PAPER-II www.Civildatas.com

14. The divergence of the vector yi  xj __________


Key: 0 to 0

Exp: Let F   yi  xj
  
divergence of F    y    x   0
x y

15. For a loaded cantilever beam of uniform cross-section, the bending moment (in N.mm) along
the length is M(x) = 5x2+10x, where x is the distance (in mm) measured from the free end of
the beam. The magnitude of shear force (in N) in the cross-section at x =10 mm is ________.
Key: 110 to 110
d
Exp:  S.F  (B.M)
dx
S.F   5x 2  10x 
d
dx
S.f  10x  10   S.F  x 10mm  10  10   10  110N

16. A sample of 15 data is a follows: 17, 18, 17, 17, 13, 18, 5, 5, 6, 7, 8, 9, 20, 17, 3. The mode of
the data is
(A) 4 (B) 13 (C) 17 (D) 20
Key: (C)
Exp: We know that mode is the value of the data which occurred most of
17 is mode

17. If a mass of moist air contained in a closed metallic vessel is heated, then its
(A) relative humidity decreases (B) relative humidity increases
(C) specific humidity increases (D) specific humidity decreases
Key: (A)
Exp: Given that mass of moist air contained in a closed metallic vessel is heated, means its specific
humidity   is constant.
So, from the psychometric chart,

1
Relative humidity 2
    100%

1 2

DBT
We can say that at constant specific humidity, as temperature increases relative humidity
decreases i.e., 2  1
So, Answer is (A)

 ICP–Intensive Classroom Program  eGATE-Live Internet Based Classes DLP  TarGATE-All India Test Series
Leaders in GATE Preparations  65+ Centers across India
© All rights reserved by Gateforum Educational Services Pvt. Ltd. No part of this booklet may be reproduced or utilized in any form without the written permission.

Visit : www.Civildatas.com
Visit : www.Civildatas.com

|ME| GATE-2017-PAPER-II www.Civildatas.com

18. In a slider-crank mechanism, the lengths of the crank and the connecting rod are 100mm and
160mm, respectively. The crank is rotating with an angular velocity of 10 radian/s counter-
clockwise. The magnitude of linear velocity (in m/s) of the piston at the instant corresponding
to the configuration shown in the figure is _____________

Key: 0.99 to 1.01


 sin 2 
Exp: V  r sin  
 n 
if   90
 sin180o 
V  r  sin 90    0.1 1 sin 90   1m sec
 2n 

19. A machine component made of a ductile material is subjected to a variable loading with
min  50 MPa and max  50 MPa. If the corrected endurance limit and the yield strength for
the material are 'e  100 MPa and  y  300MPa, the factor of safety is __________
Key: 1.99 to 2.01
Exp:  max  50MPa
 min   50MPa
 y  300MPa
e  100MPa
max   min 50  50
max   0
2 2
   min 50   50 
 v  max   50
2 2
max  v 1
 
 yt e F.S
0 50 1
 
300 100 F.S
F.S  2

20. The crystal structure of aluminium is


(A) body-centred cubic (B) face-centred cubic
(C) close-packed hexagonal (D) body-centred tetragonal
Key: (B)

21. A steel bar is held by two fixed supports as shown in the figure and is subjected to an increases
of temperature T  100C. and 200GPa, respectively, the magnitude of thermal stress (in
MPa) induced in the bar is __________.

 ICP–Intensive Classroom Program  eGATE-Live Internet Based Classes DLP  TarGATE-All India Test Series
Leaders in GATE Preparations  65+ Centers across India
© All rights reserved by Gateforum Educational Services Pvt. Ltd. No part of this booklet may be reproduced or utilized in any form without the written permission.

Visit : www.Civildatas.com
Visit : www.Civildatas.com

|ME| GATE-2017-PAPER-II www.Civildatas.com

Key: 218 to 222


Exp: Thermal Stress,
Th  ET  200  103  11 106  100  220MPa

22. The Laplace transform of tet is


s 1 1 s
(A) (B) (C) (D)
 s  1  s  1  s  1 s 1
2 2 2

Key: (B)
 Leat f  t   F  s  a  
Lte  
1
Exp: t
;  
 s  1  where F  s   Lf  t  
2

23. Consider a laminar flow at zero incidence over a flat plate. The shear stress at the wall is
denoted by  w . The axial positions x1 and x2 on the plate are measured from the leading edge
in the direction of flow. If x2 > x1, then
(A) w x1
 w x2
 0 (B) w x1
 w x2
 0 (C) w x1
 w x2
(D) w x1
 w x2

Key: (C)
Exp:  w 1   w 2
 u   u   u 
       
 y  w 
1  y
y 0  y 0 
2  y y 0 

24. A mass m is attached to two identical springs having spring constant k as shown in the figure.
The natural frequency  of this single degree of freedom system is
2k k
(A) (B)
m m

k 4k
(C) (D)
2m m

Key: (A)
Exp: Equivalent stiffness keq = k + k = 2k
k eq 2k
Natural frequency,  n    n 
m m

25. Given the atomic weight of Fe is 56 and that of C is 12, the weight percentage of carbon in
cementite (Fe3C) is _________.
Key: 6.3 to 7.0
12
Exp: Percentage of carbon by weight in cementite =  100  6.67%
56  3  12

 ICP–Intensive Classroom Program  eGATE-Live Internet Based Classes DLP  TarGATE-All India Test Series
Leaders in GATE Preparations  65+ Centers across India
© All rights reserved by Gateforum Educational Services Pvt. Ltd. No part of this booklet may be reproduced or utilized in any form without the written permission.

Visit : www.Civildatas.com
Visit : www.Civildatas.com

|ME| GATE-2017-PAPER-II www.Civildatas.com

Q. No. 26 to 55 Carry Two Marks Each


26. In an orthogonal machining with a tool of 9° orthogonal rake angle, the uncut chip thickness is
0.2mm. The chip thickness fluctuates between 0.25 mm and 0.4 mm. The ratio of the
maximum shear angle to the minimum shear angle during machining is ___________
Key: 1.45 to 1.53
Exp:   9
t1  0.2mm
t c  0.25mm to 0.4mm
r cos  t
 tan   , where r  1
1  r sin  tc
if t c  0.25mm, r  0.8 &   42.08o
if t c  0.4mm, r  0.5 &   28.18o
max 42.08
  1.493
min 28.18

0.005
27. A cylindrical pin of 250.020
0.010 mm diameter is electroplated. Plating thickness is 2.0 mm.
Neglecting the gauge tolerance, the diameter (in mm, up to 3 decimal points accuracy) of the
GO ring gauge to inspect the plated pin is _________.
Key: 29.030 to 29.030
Exp: Diameter of GO end of ring gauge = 25.02 + 2(2.005) = 29.030mm

28. A helical compression spring made of wire of circular cross-section is subjected to a


compressive load. The maximum shear stress induced in the cross-section of the wire is 24
MPa. For the same compressive load, if both the wire diameter and the mean coil diameter are
doubled, the maximum shear stress (in MPa) induced in the cross-section of the wire is _____.
Key: 6 to 6
8P.D
Exp: max 
d 3
Given, 1  24MPa
P1  P2 , d 2  2d1 , D 2  2D1
1 D1 d 32
 2 
24 1 3
so,  . 3
2 D 2 d1 2 2
24
2  MPa
4
2  6MPa

29. In a counter-flow heat exchanger, water is heated at the rate of 1.5kg/s from 40°C to 80°C by
an oil entering at 120°C and leaving at 60°C. The specific heats of water and oil are
4.2kJ/kg.K and 2kJ/kg.K respectively. The overall heat transfer coefficient is 400 W/m2.K.
The required heat transfer surface area (in m2) is
(A) 0.104 (B) 0.022 (C) 10.4 (D) 21.84
Key: (D)
Exp: Given counter flow Heat Exchanger

 ICP–Intensive Classroom Program  eGATE-Live Internet Based Classes DLP  TarGATE-All India Test Series
Leaders in GATE Preparations  65+ Centers across India
© All rights reserved by Gateforum Educational Services Pvt. Ltd. No part of this booklet may be reproduced or utilized in any form without the written permission.

Visit : www.Civildatas.com
Visit : www.Civildatas.com

|ME| GATE-2017-PAPER-II www.Civildatas.com


Water – m c  1.5kg / s Oil - Th i  120C
Tci  40C Th o  60C
Tco  80C C po  2kJ / kg.K
C pc  4.2KJ / kg.K
Overall Heat Transfer Coefficient = 400 W/m2K
Area (A) =?
Q  UA  LMTD  Thi  120C
 
i  40 
Tco  80C

 

mc Cpc Tco  Tci  UA  LMTD  
 Th o  60C

 

mc Cpc Tco  Tci   o  20
A Tci  40C 

   o 
U i 
 n(i / o ) 
1.5  4.2   80  40   103
  21.83m 2  A  21.83m 2
 
40  20
400  
 n 40
 
20  

30. The rod PQ of length L = 2 m, and uniformly distributed mass of M = 10 kg, is released
from rest at the position shown in the figure. The ends slide along the frictionless faces OP and
OQ. Assume acceleration due to gravity, g = 10 m/s2. The mass moment of inertia of the rod
about its centre of mass and an axis perpendicular to the plane of the figure is (ML 2/12). At
this instant, the magnitude of angular acceleration (in radian/s2) of the rod is ____________

Key: 7.25 to 7.75


Exp: M  10kg, g 10m sec2
0.5
ML2
Ic  I
12 1

 2
2
2 2
M  1  2M M 2M
II   M     1m
12  2 12 2 3
2M
T  I  Mg  0.5   Mg
3
2
 10  0.5    1m
3
15
   7.5rad sec 2
2
 ICP–Intensive Classroom Program  eGATE-Live Internet Based Classes DLP  TarGATE-All India Test Series
Leaders in GATE Preparations  65+ Centers across India
© All rights reserved by Gateforum Educational Services Pvt. Ltd. No part of this booklet may be reproduced or utilized in any form without the written permission.

Visit : www.Civildatas.com
Visit : www.Civildatas.com

|ME| GATE-2017-PAPER-II www.Civildatas.com

31. A steel plate, connected to a fixed channel using three identical bolts A, B and C, carries a
load of 6kN as shown in the figure. Considering the effect of direct load and moment, the
magnitude of resultant shear force (in kN) on bolt C is.

(A) 13 (B) 15 (C) 17 (D) 30


Key; (C)
6
Exp: Pr imary shear   2kN
3
Secondaryshear 1500 kN  mm
 Pe  1500
M 2 2   0.3
 r1  r2   50  50 
2 2

R1A  R1C  C  r1  0.3  50  15kN


Resultant shear at 'C'  2  15 17 kN.
Resultant shear  15  2  17 kN.

32. The volume and temperature of air (assumed to be an ideal gas) in a closed vessel is 2.87 m3
and 300K, respectively. The gauge pressure indicated by a manometer fitted to the wall of the
vessel is 0.5bar. If the gas constant of air is R = 287 J/kg. K and the atmospheric pressure is 1
bar, the mass of air (in kg) in the vessel is
(A) 1.67 (B) 3.33 (C) 5.00 (D) 6.66
Key: (C)
Exp: V  2.87m3 ; T  300K
Pgauge  0.5bar
R  287J / kg.K
Patm  1 bar
Pabs  Pg  Patm  1.5bar
PV  mRT
PV 1.5  105  2.87
So, m    5kg
RT 287  300

33. For the laminar flow of water over a sphere, the drag coefficient CF is defined as
CF  F /  U2 D2  , where F is the drag force,  is the fluid density, U is the fluid velocity and
D is the diameter of the sphere. The density of water is 1000 kg/m3. When the diameter of the
sphere is 100mm and the fluid velocity is 2m/s, the drag coefficient is 0.5. If water now flows
over another sphere of diameter 200mm under dynamically similar conditions, the drag force
(in N) on this sphere is _____________

 ICP–Intensive Classroom Program  eGATE-Live Internet Based Classes DLP  TarGATE-All India Test Series
Leaders in GATE Preparations  65+ Centers across India
© All rights reserved by Gateforum Educational Services Pvt. Ltd. No part of this booklet may be reproduced or utilized in any form without the written permission.

Visit : www.Civildatas.com
Visit : www.Civildatas.com

|ME| GATE-2017-PAPER-II www.Civildatas.com

Key: 19.9 to 20.1


Exp: Given that the condition is dynamic similarity, and in the given condition, Inertia and viscous
force plays major role, hence Reynold‟s number should be same for both model and prototype.
 Re 1   Re 2
In the first case: U1  2m sec, D1  100 mm,   1000 kg m3
In the second case: U2  2m sec, D2  200mm,   1000 kg m3
 UD   UD 
   
  1    2
Since same water is flowing over both sphere
1   2 , 1   2
U1D1  U 2 D 2
  2 100    V2  200 
U 2  1m sec
So, Drage force in second case will be
F2  CFU22 D22   0.51000 (1)2 0.2   20N
2

34. A rod of length 20mm is stretched to make a rod of length 40 mm. Subsequently, it is
compressed to make a rod of final length 10mm. Consider the longitudinal tensile strain as
positive and compressive strain as negative. The total true longitudinal strain in the rod is
(A) –0.5 (B) –0.69 (C) –0.75 (D) –1.0
Key: (B)
Exp:  i  20mm  f  10mm
 truestrain,
   10 
True  n  f   n    0.69
 i   20 

35. A gear train shown in the figure consists of gears P, Q, R and S. Gear Q and gear R are
mounted on the same shaft. All the gears are mounted on parallel shafts and the number of
teeth of P, Q, R and S are 24, 45, 30 and 80, respectively. Gear P is rotating at 400 rpm. The
speed (in rpm) of the gear S is _________.

Key: 120 to 120


Exp: TP  24, TQ  45, TR  30, TS  80
N P  400rpm
NP .TP  NQTQ

 ICP–Intensive Classroom Program  eGATE-Live Internet Based Classes DLP  TarGATE-All India Test Series
Leaders in GATE Preparations  65+ Centers across India
© All rights reserved by Gateforum Educational Services Pvt. Ltd. No part of this booklet may be reproduced or utilized in any form without the written permission.

10

Visit : www.Civildatas.com
Visit : www.Civildatas.com

|ME| GATE-2017-PAPER-II www.Civildatas.com

400  24
NQ   213.33rpm
45
N Q TQ  NSTS
213.33  45
NS   120rpm
80

36. In the Rankine cycle for a steam power plant the turbine entry and exit enthalpies are 2803 kJ/kg
and 1800 kJ/kg, respectively. The enthalpies of water at pump entry and exit are 121 kJ/kg and 124
kJ/kg, respectively. The specific steam consumption (in kg/k W.h) of the cycle is ______
Key: 3.5 to 3.7
Exp: Given that, T
h1  2803 kJ / kg, h 2  1800kJ / kg 1
h 3  121kJ / kg, h 4  124kJ / kg
4
WT  h1  h 2 3
2
 2803  1800  1003kJ / kg
Wp  h 4  h 3
S
 124  121  3kJ / kg
Wnet  WT  WP  1000kJ / kg
3600
Specific Steam consumption   3.6kg / kWh
Wnet

37. A calorically perfect gas (specific heat at constant pressure 1000 J/kg.K) enters and leaves a gas
turbine with the same velocity. The temperatures of the gas at turbine entry and exit are 1100 K
and 400 K. respectively. The power produced is 4.6 MW and heat escapes at the rate of 300 kJ/s
through the turbine casing. The mass flow rate of the gas (in kg/s) through the turbine is.
(A) 6.14 (B) 7.00 (C) 7.50 (D) 8.00
Key: (B)
Exp: Given that,
Cp  1000J / kgK
T1  1100K, P  4.6MW P
T2  400K, Q L  300kJ / s
 
E in  E out
V12 V2
h1   gz1  h 2  2  gz 2  Q L  P
2 2
1 2
 V1  V2 , Z1  Z2
So, h1  h 2  Q L  P QL

m Cp  T1  T2   Q L  P

m
 300 10    4.6 10 
3 6

1000  1100  400 



m  7kg / sec

 ICP–Intensive Classroom Program  eGATE-Live Internet Based Classes DLP  TarGATE-All India Test Series
Leaders in GATE Preparations  65+ Centers across India
© All rights reserved by Gateforum Educational Services Pvt. Ltd. No part of this booklet may be reproduced or utilized in any form without the written permission.

11

Visit : www.Civildatas.com
Visit : www.Civildatas.com

|ME| GATE-2017-PAPER-II www.Civildatas.com

38. Three masses are connected to a rotating shaft supported on bearings A and B as shown in the
figure. The system is in a space where the gravitational effect is absent. Neglect the mass of
shaft and rods connecting the masses. For m1 = 10kg, m2 = 5kg and m3 = 2.5 kg and for a shaft
angular speed of 1000 radian/s, the magnitude of the bearing reaction (in N) at location B is
_________.

Key: 0 to 0
Exp: m1  10kg, r1  0.1m
m 2  5kg , r2  0.2m
m3  2.5kg , r3  0.4m

F x  m1r1 cos 1  m 2 r2 cos 2  m 3r3 cos 3


 10  0.1 cos 0o   5  0.2  cos120o  2.5  0.4  cos 240o
 1  0.5  0.5  0
F y m1r1 sin 1  m 2 r2 sin 2  m3 r3 sin 3  0   5  0.2  sin120o   2.5  0.4sin 240o   0
R A  R B  0kN

39. A strip of 120 mm width and 8mm thickness is rolled between two 300 mm-diameter rolls to
get a strip of 120 mm width and 7.2 mm thickness. The speed of the strip at the exit is 30
m/min. There is no front or back tension. Assuming uniform roll pressure of 200 MPa in the
roll bite and 100% mechanical efficiency, the minimum total power (in kW) required to drive
the two rolls is _________.
Key: 9.4 to 9.8
Exp: Width = 120mm
Initial thickness to = 8mm
Diameter of Roller = 300mm
Radius of Roller = 150mm
Final thickness = 7.2mm
h  t i  t f  8  7.2  0.8 mm
Power require to drive one roller
P  T.  F  L p    .A.L p . Here, A  LP  b

P 2pL .  h e r p e , L

V
P    Rh  .b.  
R
30
 200  106  0.8  103  0.12   4.8 kW
60
So, power require to drive 2 roller = 2P  2  4.8 kW  9.6 kW

 ICP–Intensive Classroom Program  eGATE-Live Internet Based Classes DLP  TarGATE-All India Test Series
Leaders in GATE Preparations  65+ Centers across India
© All rights reserved by Gateforum Educational Services Pvt. Ltd. No part of this booklet may be reproduced or utilized in any form without the written permission.

12

Visit : www.Civildatas.com
Visit : www.Civildatas.com

|ME| GATE-2017-PAPER-II www.Civildatas.com

40. A product made in two factories p and Q, is transported to two destinations, R and S. The per
unit costs of transportation (in Rupees) from factories to destinations are as per the following
matrix:
Destination
R S
Factory
P 10 7
Q 3 4

Factory P produces 7 units and factory Q produces 9 units of the product. Each destination
requires 8 units. If the north-west corner method provides the total transportation cost as X (in
Rupees) and the optimized (the minimum) total transportation cost Y (in Rupees), then (X-Y),
in Rupees, is
(A) 0 (B) 15 (C) 35 (D) 105
Key: Answer is not matched with IIT Key
Exp: R S
10 7
P 7
3 4
Q 9
8 8
By North West corner Rule
R S
10
P 7 7
Q 3 1 4 8

Total cost, x = 10×7+7×1+4×8 = Rs.105


By VAM
V1  6 V2  7
u1  0 10

7
7 7 1
u 2  3 3
8 4
1 9 1
8 8
7  3
Total minimum Cost, y = 3×8+4×1+7×7=Rs.77
So, x–y = Rs.28

41. One kg of an ideal gas (gas constant R = 287 J/kg.K) undergoes an irreversible process from
state-1 (1 bar, 300 K) to state -2 (2 bar, 300 K). The change in specific entropy (s2 – s1) of the
gas (in J/kg. K) in the process is ___________
Key: –201 to –197
Exp: Given m  1kg, R  287J / kg.K
P1  1bar, P2  2bar
T1  300K, T2  300K
Same Temperature

S2  S1  Rn  P P   287n  21   198.93J / kgK


2
1

 ICP–Intensive Classroom Program  eGATE-Live Internet Based Classes DLP  TarGATE-All India Test Series
Leaders in GATE Preparations  65+ Centers across India
© All rights reserved by Gateforum Educational Services Pvt. Ltd. No part of this booklet may be reproduced or utilized in any form without the written permission.

13

Visit : www.Civildatas.com
Visit : www.Civildatas.com

|ME| GATE-2017-PAPER-II www.Civildatas.com

42. A 60 mm-diameter water jet strikes a plate containing a hole of 40mm diameter as shown in
the figure. Part of the jet passes through the hole horizontally, and the remaining is deflected
vertically. The density of water is 1000 kg/m3. If velocities are as indicated in the figure, the
magnitude of horizontal force (in N) required to hold the plate is _________

Key: 627 to 629


Exp: Force in X–direction = Rate of change of momentum
  Pi  Pf  x  direction

  

  m1 V1  m 2 V2  D1  6cm
  x direction D2  4cm
 V  A1V1  A 2 V2  V1  V2 
 V 2  A1  A 2 
 20m / s  V2
 1000  20  20   0.06    0.04  
2 2

4   20m / s  V1
 628.32N

43. The arrangement shown in the figure measures the


velocity V of a gas of density 1 kg/m3 flowing
through a pipe. The acceleration due to gravity is
9.81 m/s2. If the manometric fluid is water (density
1000 kg/m3) and the velocity V is 20 m/s, the
differential head h (in mm) between the two arms of
the manometer is ____________
Key: 19 to 21
Exp: Given
g  1kg / m3 , g  9.81m / s 2
m  1000kg / m3 , V  20m / s

 
V  2gH Where H  h  m 1 
  
V 2
 
So,  h  m 1
2g   
  20 2 
   h 1000  1
 2  9.81 
h  0.0204m  2.04cm  h  20.4mm

 ICP–Intensive Classroom Program  eGATE-Live Internet Based Classes DLP  TarGATE-All India Test Series
Leaders in GATE Preparations  65+ Centers across India
© All rights reserved by Gateforum Educational Services Pvt. Ltd. No part of this booklet may be reproduced or utilized in any form without the written permission.

14

Visit : www.Civildatas.com
Visit : www.Civildatas.com

|ME| GATE-2017-PAPER-II www.Civildatas.com

44. A metal ball of diameter 60mm is initially at 220 °C. The ball is suddenly cooled by an air jet
of 20°C. The heat transfer coefficient is 200 W/m2.K and 9000kg/m3, respectively. The ball
temperature (in °C) after 90 seconds will be approximately.
(A) 141 (B) 163 (C) 189 (D) 210
Key: (A)
Exp: D = 60mm = 0.06m
Ti  220C, h  200W / m 2 K,   9000kg / m3
T  20C, Cp  400J / kgK, K  400W / mK
t  90sec T?
 h.A 
 200903 
Ti  T  .t
 .VC  220  20  0.039000400 
 e p   e
Tf  T Tf  20
Tf  141.3C

45. A single – plate clutch has a friction disc with inner and outer radii of 20mm and 40 mm,
respectively. The friction lining in the disc is made in such a way that the coefficient of
friction  varies radially as   0.01r, where r is in mm. The clutch needs to transmit a friction
torque of 18.85kN.mm. As per uniform pressure theory, the pressure (in MPa) on the disc is
_________
Key: 0.49 to 0.51
Exp: ri  20mm, ro  40mm,   0.01r dr
T  18.85kN  mm
p?
P   p2rdr r
T   p 2r 2 dr   p  0.01r  2r 2 dr   0.0628r 3 .dr  ri
40 r0
40
 r4 
T  0.0628p  r dr  0.0628p  
3

20  4  20
 404 204 
18.85  103  0.0628p   
 4 4 
p  0.5MPa

46. The surface integral  F.ndS


S
over the surface S of the sphere x 2  y 2  z 2  9, where

F=(x+y) i+ (x+z) j+(y+z) k and n is the unit outward surface normal, yields ______.
Key: 225 to 227

Exp: F   x  y i   x  z J   y  z k
   
divF   x  y    x  z    y  z   1  0  1  2
x y z
By divergence theorem,
 ^ 
S
F.n dS   divFdV
V
where V is volume of given surface of sphere x 2  y2  z2  9

4  27 
  2dV  2V  2   72  226.1947
V
3

 ICP–Intensive Classroom Program  eGATE-Live Internet Based Classes DLP  TarGATE-All India Test Series
Leaders in GATE Preparations  65+ Centers across India
© All rights reserved by Gateforum Educational Services Pvt. Ltd. No part of this booklet may be reproduced or utilized in any form without the written permission.

15

Visit : www.Civildatas.com
Visit : www.Civildatas.com

|ME| GATE-2017-PAPER-II www.Civildatas.com

47. Block 2 slides outward on link 1 at a uniform velocity of 6 m/s as shown in the figure. Link 1
is rotating at a constant angular velocity of 20 radian/s counterclockwise. The magnitude of
the total acceleration (in m/s2) of point P of the block with respect to fixed point O is
________

Key: 243 to 244


Exp: Acceleration of the block,
v  6m / sec,   20rad / sec

  r    2v
2
a  a r 2  a cr 2  2 2

r  OP,
a cr  2v  2  6  20  240m / s 2
a r  2 r   20   0.1  40
2

a r  2402  402  243.31m / sec 2

48. During the turning of a 20mm-diameter steel bar at a spindle speed of 400 rpm, a tool life of
20 minute is obtained. When the same bar is turned at 200 rpm, the tool life becomes 60
minute. Assume that Taylor‟s tool life equation is valid. When the bar is turned at 300 rpm, the
tool life (in minute) is approximately.
(A) 25 (B) 32 (C) 40 (D) 50
Key: (B)
Exp: Taylors Tool life equation,
VT n  C
 V1T1n  V2 T2 n
n
 T2  V1
  
 T1  V2
n
 60  400
  
 20  200
n  0.63
at N 3  300rpm, T3  ?
n
 T3  V1
  
 T1  V3
0.63
 T3  400
 20   300
 
T3  31.57 min utes
T3  32 min utes
 ICP–Intensive Classroom Program  eGATE-Live Internet Based Classes DLP  TarGATE-All India Test Series
Leaders in GATE Preparations  65+ Centers across India
© All rights reserved by Gateforum Educational Services Pvt. Ltd. No part of this booklet may be reproduced or utilized in any form without the written permission.

16

Visit : www.Civildatas.com
Visit : www.Civildatas.com

|ME| GATE-2017-PAPER-II www.Civildatas.com

50 70 
49. Consider the matrix A =   whose eigenvectors corresponding to eigenvalues 1 and  2
70 80 
 70    80
are x1    and x 2   2 T
 . respectively. The value of x1 x 2 is_________

 1  50   70 
Key: 0 to 0
50 70 
Exp: A 
70 80 
 70    2  80 
Eigen vectors are X1    ; X2   
 1  50   70 
Where 1 ,  2 Eigen values of A
   80 
X1T X 2  70 1  50   2   70   2  80    1  50  70
 70 
 70 2  5600  701  3500  70  1   2   9100
 70 130   9100  9100  9100  0

 sum of eigen values  1   2 


 
 Trace  50  80  130 

50. The radius of gyration of a compound pendulum about the point of suspension is 100mm. The
distance between the point of suspension and the centre of mass is 250mm. Considering the
acceleration due to gravity as 9.81 m/s2, the natural frequency (in radian/s) of the compound
pendulum is _________.
Key: 15 to 16

Exp: k  100mm  0.1m


L  250mm  0.250m
g  9.81m / sec

I  mk 2  m  0.1
2

mgL m  9.81 0.250


Wn    15.66rad / sec
m  0.1
2
I

51. Consider the differential equation 3y ''  x   27y  x   0 with initial conditions y(0) = 0 and
y'(0)  2000. The value of y at x = 1 is ________.
Key: 93 to 95
Exp: 3y ''  x   27y  x   0, y  0   0, y '  0   2000
Auxillary equation, 3m 2  27  0  m 2  9  0  m  0  3i
y c  c1 cos3x  c 2 sin 3x and y p  0
 y c  c1 cos3x  c 2 sin 3x
y  0   0  c1  0  0  c1  0
 y  c2 sin 3x
y '  3c2 cos3x

 ICP–Intensive Classroom Program  eGATE-Live Internet Based Classes DLP  TarGATE-All India Test Series
Leaders in GATE Preparations  65+ Centers across India
© All rights reserved by Gateforum Educational Services Pvt. Ltd. No part of this booklet may be reproduced or utilized in any form without the written permission.

17

Visit : www.Civildatas.com
Visit : www.Civildatas.com

|ME| GATE-2017-PAPER-II www.Civildatas.com

2000
y '  0   2000  2000  3c 2  c 2 
3
2000 2000
y  sin 3x, y 1  sin 3  94.08
3 3

52. If f(z) = (x2+ay2) + ibxy is a complex analytic function of z = x + iy, where i = 1, then
(A) a = –1, b= –1 (B) a = –1, b = 2 (C) a = 1, b = 2 (D) a = 2, b = 2
Key: (B)
Exp: Given f  z    x 2  ay2   i bxy is analytic

u  x 
  , 
x y y x
value u  x 2  ay 2 ,   bxy
u 
 2x  by
x x
u 
 2ay  bx
y y
Clearly for b = 2 and a= –1 above Cauchy-Riemann equations holds

53. A project starts with activity A and ends with activity F. The precedence relation and durations
of the activities are as per the following table:

Duration
Activity Immediate Predecessor
(days)
A - 4
B A 3
C A 7
D B 14
E C 4
F D,E 9

The minimum project completion time (in days) is ______


Key: 30 to 30
Exp: 3 D
B
3 14
1 A 2 5 F 6
4 4 9
C, 7 4 E
Minimum project completion Time = Length of longest path = 4+3+14+9=30 Days.

54. Maximize Z = 5x1+3x2


Subject to
x1+2x2  10,
x1–x2  8,
x1, x2  0.
In the starting Simplex tableau, x1 and x2 are non-basic variables and the value of Z is zero.
The value of Z in the next Simplex tableau is _________.
 ICP–Intensive Classroom Program  eGATE-Live Internet Based Classes DLP  TarGATE-All India Test Series
Leaders in GATE Preparations  65+ Centers across India
© All rights reserved by Gateforum Educational Services Pvt. Ltd. No part of this booklet may be reproduced or utilized in any form without the written permission.

18

Visit : www.Civildatas.com
Visit : www.Civildatas.com

|ME| GATE-2017-PAPER-II www.Civildatas.com

x2
Key: 40 to 40
  26 2 
 26 2   0,5   , 
Exp: Zmax at  ,   3 3
 3 3

10,0  x1
1st Table at  0,0   Z  0  0,0   8,0 
2nd Table at 8,0   Z  40
  0, 8 

55. The principal stresses at a point in a critical section of a machine component are
1  60MPa, 2  5MPa and 3  40 MPa. For the material of the component, the tensile
yield strength is  y  200 MPa. According to the maximum shear stress theory, the factor of
safety is
(A) 1.67 (B) 2.00 (C) 3.60 (D) 4.00
Key: (B)
1  3 w y
Exp:   1  3 
2 2 FOS
200
60   40  
FOS
FOS  2
General Aptitude
Q. No. 1 - 5 Carry One Mark Each

1. If you choose plan P, you will have to _______ plan Q, as these two are mutually _________.
(A) forgo, exclusive (B) forget, inclusive
(C) accept, exhaustive (D) adopt, intrusive
Key: (A)

2. P looks at Q while Q looks at R. P is married, R is not. The number of people in which a


married person is looking at an unmarried person is
(A) 0 (B) 1 (C) 2 (D) Cannot be determined
Key: (B)

3. If a and b are integers and a – b is even, which of the following must always be even?
(A) ab (B) a2 + b2 + 1 (C) a2 + b + 1 (D) ab – b
Key: (D)
Exp: According to the given relation of a–b = even, there is a possibility of odd-odd (or) even-even
is equal to even. From the options, Option (D) is correct. Since, odd × odd–odd (or) even ×
even–even → is always even number.

Alternate Method:
Let a – b =2K  a  b  2K
ab  b   b  2K  b  b  b2  2Kb  b   b2  b   2Kb  b  b  1  2Kb
Even; since 2Kb always even & b (b-1) also even, if with b is odd or even.

 ICP–Intensive Classroom Program  eGATE-Live Internet Based Classes DLP  TarGATE-All India Test Series
Leaders in GATE Preparations  65+ Centers across India
© All rights reserved by Gateforum Educational Services Pvt. Ltd. No part of this booklet may be reproduced or utilized in any form without the written permission.

19

Visit : www.Civildatas.com
Visit : www.Civildatas.com

|ME| GATE-2017-PAPER-II www.Civildatas.com

4. A couple has 2 children. The probability that both children are boys if the older one is a boy is
(A) 1/4 (B) 1/3 (C) 1/2 (D) 1
Key: (C)
No.of Favourablecases
Exp: Probability 
Total No.of Possible cases
1
The probability that both children are boys if the order one is a boy =
2

5. The ways in which this game can be played __________ potentially infinite.
(A) is (B) is being (C) are (D) are being
Key: (C)

Q. No. 6- 10 Carry Two Marks Each

6. “If you are looking for a history of India, or for an account of the rise and fall of the British
Raj, or for the reason of the cleaving of the subcontinent into two mutually antagonistic parts
and the effects this mutilation will have in the respective sections, and ultimately on Asia, you
will not find it in these pages; for though I have spent a lifetime in the country, I lived too near
the seat of events, and was too intimately associated with the actors, to get the perspective
needed for the impartial recording of these matters.”
Which of the following closest in meaning to „cleaving‟?
(A) Deteriorating (B) Arguing (C) Departing (D) Splitting
Key: (D)

7. There are 4 women P, Q, R, S, and 5 men V, W, X, Y, Z in a group. We are required to form


pairs each consisting of one woman and one man. P is not to be paired with Z, and Y must
necessarily be paired with someone. In how many ways can 4 such pairs be formed?
(A) 74 (B) 76 (C) 78 (D) 80
Key: (C)
Exp: If P is paired with y; they Q has 4 choices
R has 3 choices
S has 2 choices
Total 24 choices
(or)
If Q is paired with y; then P has 3 choices
R has 3 choices
S has 2 choices
Total 18 choices
(or)
If R is paired with y; then P has 3 choices
Q has 3 choices
S has 2 choices
Total 18 choices

 ICP–Intensive Classroom Program  eGATE-Live Internet Based Classes DLP  TarGATE-All India Test Series
Leaders in GATE Preparations  65+ Centers across India
© All rights reserved by Gateforum Educational Services Pvt. Ltd. No part of this booklet may be reproduced or utilized in any form without the written permission.

20

Visit : www.Civildatas.com
Visit : www.Civildatas.com

|ME| GATE-2017-PAPER-II www.Civildatas.com

(or)
If S is paired with y; then P has 3 choices
Q has 3 choices
S has 2 choices
Total 18 choices
 Total number of ways = 24+18+18+18=78

8. In the graph below, the concentration of a particular pollutant in a lake is plotted over
(alternate) days of a month in winter (average temperature 10°C) and a month in summer
(average temperature 30°C).

Consider the following statements based on the data shown above:


(i) Over the given months, the difference between the maximum and the minimum pollutant
concentrations is the same in both winter and summer.
(ii) There are at last four days in the summer month such that the pollutant concentrations on
those days are within 1 ppm of the pollutant concentrations on the corresponding days in
the winter month.
Which one of the following options is correct?
(A) Only i (B) Only ii (C) Both i and ii (D) Neither i nor ii
Key: (B)
Exp: The difference between the maximum and the minimum pollutant concentrations
(i) in winter = 8-0=8 ppm,
(ii) in summer = 10.5-1.5=9 ppm
 (i) is false & (ii) is correct from the graph.

9. All people in a certain island are either „Knights‟ or „Knaves‟ and each person knows every
other person‟s identity. Knights NEVER lie, and knaves ALWAYS lie.
P says “Both of us are knights”. Q says “None of us are knaves”.
Which one of the following can be logically inferred from the above?
(A) Both P and Q are knights
(B) P is a knight; Q is a knave
(C) Both P and Q are knaves
(D) The identities of P, Q cannot be determined
Key: (D)

 ICP–Intensive Classroom Program  eGATE-Live Internet Based Classes DLP  TarGATE-All India Test Series
Leaders in GATE Preparations  65+ Centers across India
© All rights reserved by Gateforum Educational Services Pvt. Ltd. No part of this booklet may be reproduced or utilized in any form without the written permission.

21

Visit : www.Civildatas.com
Visit : www.Civildatas.com

|ME| GATE-2017-PAPER-II www.Civildatas.com

10. X bullocks and Y tractors take 8 days to plough a field. If we halve the number of bullocks and
double the number of tractors, it takes 5days to plough the same field. How many days will it
take X bullocks alone to plough the field?
(A) 30 (B) 35 (C) 40 (D) 45
Key: (A)
Exp: Given Number of days required that X bullocks and Y tractors to plough a field = 8 days 
(1)  i.e, X  Y  8D  8X  8Y  1day Number of days required that
X
bullocks and 2y tractors
2
to plough field  5   2 
 X 5 
i.e, 2  2Y  5D  2 X  10Y  1D 
 
From (1) & (2); we have
5X 11X
8X  8Y   10Y  Y 
2 4
15X
 From 1 X  Y  8D   8D  X  30days
4

 ICP–Intensive Classroom Program  eGATE-Live Internet Based Classes DLP  TarGATE-All India Test Series
Leaders in GATE Preparations  65+ Centers across India
© All rights reserved by Gateforum Educational Services Pvt. Ltd. No part of this booklet may be reproduced or utilized in any form without the written permission.

22

Visit : www.Civildatas.com
Downloaded From : www.EasyEngineering.net

ww
w .Ea
syE
ngi
nee
rin
g.n
et

**Note : Other Websites/ Blogs Owners we requested you, Please do not Copy
(or) Republish this Material.
This copy is NOT FOR SALE.
**Disclimers : EasyEngineering does not own this book/materials, neither created nor
scanned. we provide the links which is already available on the internet. For any quarries,
Disclaimer are requested to kindly contact us. We assured you we will do our best. We DO
NOT SUPPORT PIRACY, this copy was provided for students who are financially troubled but deserving
to learn. Downloaded From : www.EasyEngineering.net
Thank You and God Bless!
Downloaded From : www.EasyEngineering.net

|ME-2019-Forenoon|

GENERAL APTITUDE

Q. No. 1 - 5 Carry One Mark Each

1. The minister avoided any mention of the issue of women’s reservation in the private sector. He
was accused of _____ the issue.
(A) belting (B) skirting (C) tying (D) collaring
Key: (B)

2. _____ I permitted him to leave, I wouldn’t have had any problem with him being

ww absent _____ I?
(B) Have, wouldn’t

w.E
(A) Had, would
(C) Have, would (D) Had, wouldn’t

asy
Key: (A)

3. En
A worker noticed that the hour hand on the factory clock had moved by 225 degrees during her

gin
stay at the factory. For how long did she stay in the factory?
(A)
(C)
3.75 hours
4 hours and 15minseer (B) 7.5 hours
(D) 8.5 hours
Key: (B)

Sol:
in
Number of hours in a clock = 12 hours
g.n
One rotation hour hand covers 360°
360 degree = 12 hours

1 degree 
12
hours
10
9
11 12 1
2
3
et
360 4
8
225° = ? 7 6 5
12
  225  7.5 hours
360

4. John Thomas, an ______ writer, passed away in 2018.


(A) imminent (B) prominent (C) dominant (D) eminent
Key: (D)

© All rights reserved by Thinkcell Learning Solutions Pvt. Ltd. No part of this booklet may be reproduced or utilized in any form without the written permission.

Downloaded From : www.EasyEngineering.net


Downloaded From : www.EasyEngineering.net

|ME-2019-Forenoon|

5. The sum and product of two integers are 26 and 165 respectively. The difference between these
two integers is ________.
(A) 3 (B) 6 (C) 2 (D) 4
Key: (D)

Sol: Let us take two number are a & b


Given that
a  b  26, ab  165
ab?
a  b
2
 262  a 2  b 2  2ab  262
a 2  b 2  262  2ab
a  b   a 2  b 2   2ab
2

ww  a  b   262  2ab  2ab


2

 a  b   262  4ab   a  b   262  4  165


w.E
2 2

 a  b   16  a  b  4
2

asy Q. No. 6 - 10 Carry Two Marks Each

En
gin
6. A person divided an amount of Rs. 100,000 into two parts and invested in two different
schemes. In one he got 10% profit and in the other he got 12%. If the profit percentages are

eer
interchanged with these investments he would have got Rs. 120 less. Find the ratio between his
investments in the two schemes.

Key: (D)
(A) 37:63
in (B) 9:16 (C) 11:14 (D) 47:53

g.n
Sol: Considering first scheme as x second scheme as y
Given that
x  y  1,00,000  1
et
Assume profit of sum before interchanging percentage = z
1.1x  1.12y  z   2
After interchanging profit percentages
1.12x  1.1y  z  120   3
Solving (2) and (3)
1.12x  1.1y  z  120
1.1x  1.12y  z
  
0.02x  0.02y  120

© All rights reserved by Thinkcell Learning Solutions Pvt. Ltd. No part of this booklet may be reproduced or utilized in any form without the written permission.

Downloaded From : www.EasyEngineering.net


Downloaded From : www.EasyEngineering.net

|ME-2019-Forenoon|

120
xy   6000   4 
0.02
By solving (4) & (1)
x  y  100000
x  y  6000
2x  94000
x  47000
y  100000  47000  53000
x 47000 47
 
y 53000 53

7. Under a certain legal system, prisoners are allowed to make one statement. If their statement

ww turns out to be true then they are hanged. If the statement turns out to be false then they are shot.
One prisoner made a statement and the judge had no option but to set him free. Which one of

(A)
w.E
the following could be that statement?
I will be shot (B) I committed the crime
(C)
asy
I did not commit the crime (D) You committed the crime

En
Key: (A)

8.
gin
A firm hires employees at five different skill levels P, Q, R, S, T. The shares of employment at
these skill levels of total employment in 2010 is given in the pie chart as shown. There were a

eer
total of 600 employees in 2010 and the total employment increased by 15% from 2010 to 2016.

in
The total employment at skill levels P, Q and R remained unchanged during this period. If the
employment at skill level S increased by 40% from 2010 to 2016, how many employees were
there at skill level T in 2016? g.n
Percentage share of skills in 2010

T
et
S 5 P
25 20

Q
R
25
25

(A) 30 (B) 72 (C) 35 (D) 60

© All rights reserved by Thinkcell Learning Solutions Pvt. Ltd. No part of this booklet may be reproduced or utilized in any form without the written permission.

Downloaded From : www.EasyEngineering.net


Downloaded From : www.EasyEngineering.net

|ME-2019-Forenoon|

Key: (D)

Sol: 2010 2016


T
5% 60
S 210 T
D 20% P
25% S
120
R Q R Q
25% 25% 150 150

In 2010:
Total number of employees = 600
Number of employees of skills

ww Q  R  S  25% of 600  150


Number of employees of skill P = 20% of 600 = 120

In 2016:
w.E
Number of employees of skill T= 5% of 600 = 30

asy
Total number of employees increased by 15%
Total number of employees  1.15  600  690

En
As there is no change in skill level of P, Q, and R

gin
Number of employees of skill level P = 120
Number of employees of skill level Q = 150

eer
Number of employees of skill level R=150
Number of employees at skill level S  40% increases  1.4 150  210

in
Number of employees at skill level T  690  120  150  150  210   60.

M and N had four children P, Q, R and S. Of them, only P and R were married. They had g.n
et
9.
children X and Y respectively. If Y is a legitimate child of W, which one of the following
statement is necessarily FALSE?
(A) M is the grandmother of Y (B) W is the wife of R
(C) W is the wife of P (D) R is the father of Y
Key: (C) M and N
Sol:
Children

P Q R W S

Wife & husband


X
Y
Child
Child
© All rights reserved by Thinkcell Learning Solutions Pvt. Ltd. No part of this booklet may be reproduced or utilized in any form without the written permission.

Downloaded From : www.EasyEngineering.net


Downloaded From : www.EasyEngineering.net

|ME-2019-Forenoon|

10. Congo was named by Europeans. Congo’s dictator Mobuto later changed the name of the
country and the river to Zaire with the objective of Africanising names of persons and spaces.
However, the name Zaire was a Portuguese alteration of Nzadi o Nzere, a local African term
meaning ‘River that swallows Rivers’. Zaire was the Portuguese name for the Congo river in the
16th and 17 centuries. Which one of the following statements can be inferred from the
paragraph above?
(A) The term Nzadi o Nzere was of Portuguese origin
(B) As a dictator Mobuto ordered the Portuguese to alter the name of the river to Zaire
(C) Mobuto’s desire to Africanise names was prevented by the Portuguese
(D) Mobuto was not entirely successful in Africanising the name of his country
Key: (D)

ww
w.E MECHANICAL ENGINEERING
Q. No. 1 to 25 Carry One Mark Each

1.
asy
Consider the stress-strain curve for an ideal elastic-plastic strain hardening metal as shown in

En
the figure. The metal was loaded in uniaxial tension starting from O. Upon loading, the stress-

gin
strain curve passes through initial yield point at P, and then strain hardens to point Q, where the
loading was stopped. From point Q, the specimen was unloaded to point R, where the stress is

eer
zero. If the same specimen is reloaded in tension from point R, the value of stress at which the

in
material yields again is _______MPa.

210 Q g.n
180
P et
Stress (MPa)

O R Strain
Key: (210)

Sol: Strain hardening improve tensile strength, yield strength and hardness at the expense of
reduced ductility.

© All rights reserved by Thinkcell Learning Solutions Pvt. Ltd. No part of this booklet may be reproduced or utilized in any form without the written permission.

Downloaded From : www.EasyEngineering.net


Downloaded From : www.EasyEngineering.net

|ME-2019-Forenoon|

2. The length, width and thickness of a steel sample are 400 mm, 410 mm, 40 mm and 20 mm,
respectively. Its thickness needs to be uniformly reduced by 2 mm in a single pass by using
horizontal slab milling. The milling cutter (diameter: 100 mm, width: 50 mm) has 20 teeth and
rotates at 1200 rpm. The feed per tooth is 0.05 mm. The feed direction is along the length of the
sample. If the over-travel distance is the same as the approach distance, the approach distance
and time taken to complete the required machining task are
(A) 14mm, 21.4 s (B) 21 mm, 39.4 s (C) 21 mm, 28.9s (D) 14mm, 18.4 s
Key: (A)

Sol: L  400mm
b  40 mm
t  20 mm

ww d  2 mm
D  100 mm, Z  20, N  1200 rpm

w.E
f  0.05 mm tooth

Ca  d  D  d   2 100  2   2  98  14 mm
F  fNZ
asy
 0.05  1200  20  1200 mm min  20 mm s

tm  e En
L  Ca  Co 400  14  14
  21.4s
F
gin 20

3.
flow rate, are eer
As per common design practice, the three types of hydraulic turbines, in descending order of

(A)
(C)
in
Francis, Kaplan, Pleton
Pelton, Kaplan, Francis
(B) Kaplan, Francis, Pelton
(D) Pelton, Francis, Kaplan
g.n
Key: (B)

Sol: Kaplan turbine is operating under high flow rates.


Francis turbine is operating under medium flow rates.
et
Pelton turbine is operating under low flow rates

4. The table presents the demand of a product. By simple three-months moving average method,
the demand-forecast of the product for the month of September is
Month Demand
January 450
February 440
March 460
April 510
May 520

© All rights reserved by Thinkcell Learning Solutions Pvt. Ltd. No part of this booklet may be reproduced or utilized in any form without the written permission.

Downloaded From : www.EasyEngineering.net


Downloaded From : www.EasyEngineering.net

|ME-2019-Forenoon|

June 495
July 475
August 560

(A) 490 (B) 536.67 (C) 510 (D) 530


Key: (C)

495  475  560


Sol: Forecast of the product of the month September =  510
3

5. The lengths of a large stock of titanium rods follow a normal distribution with a mean    of

ww 440 mm and a standard deviation    of 1 mm. What is the percentage of rods whose lengths lie

between 438 mm and 441 mm?


(A)
w.E
86.64% (B) 68.4% (C) 99.75% (D) 81.85%

asy
Key: (D)

Sol: Given , Mean     440mm, S.D     1 mm

En
The random variable ‘X’ denotes lengths of rods.
P  438  X  441  ?
gin
The standard normal variable Z 
X

438  440 eer 


Z  2 Z  1 Z  0 Z 1 Z  2
If X  438  Z 

If X  441  Z 
1
441  440
1
1
in
 2

68.26%
g.n
 P  438  X  441  P  2  Z  1

 95.44 
 P  2  Z  0  P 0  Z  1
 68.26 
95.44%

et
  %  %   47.72  %   34.13 %
 2   2 
 P  438  X  441  81.85%

6. During a non-flow thermodynamic process (1-2) executed by a perfect gas, the heat interaction

is equal to the work interaction  Q1 2  W12  when the process is

(A) Isentropic (B) Isothermal (C) Polytropic (D) Adiabatic

© All rights reserved by Thinkcell Learning Solutions Pvt. Ltd. No part of this booklet may be reproduced or utilized in any form without the written permission.

Downloaded From : www.EasyEngineering.net


Downloaded From : www.EasyEngineering.net

|ME-2019-Forenoon|

Key: (B)
1
Sol: First law of thermodynamics for non flow (closed) system,
dQ  du  dW T  Constant
 dQ   mc dT   dW
v  1 P
2
When process is isothermal, dT = 0  Q12  W12
Shaded area shows equal amount of heat & work.
V

4
7. Evaluation of 2
x 3 dx using a 2-equal-segment trapezoidal rule gives a value of __________.

Key: (63)

ww
Sol: Using Trapezoidal rule, we have
b
h
 f  x  dx  2  y  y n   2  y1  y 2  ...  y n 1  ;
a
w.E
where h  step size 
0

ba

asy n
Let f  x   x 3 ; a  2; b  4; n  number of intervals  2

h
42
2
1
En
x
gin
2 3 4

f  x   x3
8
eer 27
 y0   y1   y2 
64

4
1
in
1
  x 3dx  8  64   2  27   72  54  63
2
2 2
g.n
et
4
  x dx  63 3

8. The natural frequencies corresponding to the spring-mass systems I and II are I and II ,

I
respectively. The ratio is
II
k k

SYSTEM  I

© All rights reserved by Thinkcell Learning Solutions Pvt. Ltd. No part of this booklet may be reproduced or utilized in any form without the written permission.

Downloaded From : www.EasyEngineering.net


Downloaded From : www.EasyEngineering.net

|ME-2019-Forenoon|

k
SYSTEM  II
1 1
(A) (B) 4 (C) 2 (D)
2 4
Key: (A)

ww
Sol:
k k

w.E M

1 1 1
  
1

asy
Since springs are in series
2
k eq k k k eq k
En
k eq 
k
2
and I 
gin k eq
m

k
2m

eer k

in m
g.n
Since springs are in parallel k eq  k  k  k eq  2k
k
et
k eq 2k
II  
m m
k
I 2m  k  m  1

II 2k 2m 2k 2
m

9. A solid cube of side 1 m is kept at a room temperature of 32° C. The coefficient of linear
thermal expansion of the cube material is 1×10–5/°C and the bulk modulus is 200 GPa. If the

© All rights reserved by Thinkcell Learning Solutions Pvt. Ltd. No part of this booklet may be reproduced or utilized in any form without the written permission.

Downloaded From : www.EasyEngineering.net


Downloaded From : www.EasyEngineering.net

|ME-2019-Forenoon|

cube is constrained all around and heated uniformly to 42°C, then the magnitude of volumetric
(mean) stress (in MPa) induced due to heating is ________.
Key: (60)

Sol: a  1m K  200GPa
T1  32C Tf  42C
5
  1  10 v  ?
x  T  1 105  42  32   1  104
v  3 x  3  104
v
K   v  3  104  200  103  60MPa
v

ww
10. For a hydro dynamically and thermally fully developed laminar flow through a circular pipe of
constant cross-section, the Nusselt number at constant wall heat flux (Nu q) and that at constant

w.E
wall temperature (NuT) are related as

Nu q  Nu T (B) Nu q   Nu T 
2
(C) Nu q  Nu T (D) Nu q  Nu T

asy
(A)

Key: (D)

Sol:
En
Since average convective heat transfer coefficient h in case of constant heat flux h q is   
gin
more that from constant wall temperature h T .

 Nuq  NuT
 

eer
For fully developed laminar flow,
Nu q  4.36, Nu T  3.66
in g.n
11. A flat-faced follower is driven using a circular eccentric cam rotating at a constant angular
velocity . At time t = 0, the vertical position of the follower is y(0) = 0, and the system is in
the configuration shown below
et
yt

© All rights reserved by Thinkcell Learning Solutions Pvt. Ltd. No part of this booklet may be reproduced or utilized in any form without the written permission.

Downloaded From : www.EasyEngineering.net


Downloaded From : www.EasyEngineering.net

|ME-2019-Forenoon|

The vertical position of the follower face, y(t) is given by

(A) e 1  cos 2t  (B) esin t

(C) esin 2t (D) e 1  cos t 

Key: (D)

Sol: t  0, y  0   0, 
B
A

Q
 e
P S
O

ww
w.E
asy
x  AB  OS  OQ  QS
 OQ  PQ cos 

En
 OQ  OQ cos 

gin
y  OQ 1  cos    e 1  cos    e 1  cos  t  

12.
eer
In a casting process, a vertical channel through which molten metal and flows downward from

Key: (A)
(A) sprue in
pouring basin to runner for reaching the mold cavity is called
(B) pin hole (C) riser
g.n
(D) blister

13. Air of mass 1 kg, initially at 300K and 10 bar, is allowed to expand isothermally till it reaches a
pressure of 1 bar. Assuming air as an ideal gas with gas constant of 0.287 kJ/kg.K, the change in
et
entropy of air (in kJ/kg.K, round off to two decimal places) is __________.
Key: (0.66)

Sol: Given that for Air, initially m  1kg, T1  300K, P1  10 bar


Finally; T2  300K  Isothermal 
P2  1 bar
R  0.287 KJ kg. K
 T P 
Change in entropy, S  m C p n 2  R n 2   1
 T1 P1 

© All rights reserved by Thinkcell Learning Solutions Pvt. Ltd. No part of this booklet may be reproduced or utilized in any form without the written permission.

Downloaded From : www.EasyEngineering.net


Downloaded From : www.EasyEngineering.net

|ME-2019-Forenoon|

Since T2  T1
P2 1
S  mR n  1  0.287  n
P1 10
S  0.66084
S  0.66 kJ kg  K

14. A block of mass 10 kg rests on a horizontal floor. The acceleration due to gravity is 9.81 m/s 2.
The coefficient of static friction between the floor and the block is 0.2.

10N
10kg

ww A horizontal force of 10 N is applied on the block as shown in the figure. The magnitude of

Key: (10)
w.E
force of friction (in N) on the block is _____.

Sol:
asy
m  10kg,   0.2, F  10N
g  9.81 m sec2 ,
En 10kg 10N

gin F  W

eer
F  0.2  10  9.81  19.62  10
Hence friction force = 10N.
in g.n
15.
1 1 0 
Consider the matrix P  0 1 1 
et
0 0 1 

The number of distinct eigenvalues of P is


(A) 0 (B) 1 (C) 3 (D) 2
Key: (B)

Given 1 1 0 
P  0 1 1 
0 0 1 

Upper triangular matrix

© All rights reserved by Thinkcell Learning Solutions Pvt. Ltd. No part of this booklet may be reproduced or utilized in any form without the written permission.

Downloaded From : www.EasyEngineering.net


Downloaded From : www.EasyEngineering.net

|ME-2019-Forenoon|

 Eigen values of P are 1, 1, 1; since the Eigen values of upper triangular matrix are it’s
diagonal elements.
 Number of distinct eigen values of P = 1.

16. During a high cycle fatigue test, a metallic specimen is subjected to cyclic loading with a mean
stress of +140 MPa, and a minimum stress of –70 MPa. The R-ratio (minimum stress to
maximum stress) for this cycle loading is _____ (round off to one decimal place).
Key: (–0.2)

Sol: mean  140MPa


min
min  70MPa; ?
max

ww mean 
max   min
2
 140

max

 
max 350 5
w.E
 280  70   max  350
 min 70 1
 0.2

asy
17.

En
A slender rod of length L, diameter d (L >> d) and thermal conductivity k1 is joined with
another rod of identical dimensions, but of thermal conductivity k2, to form a composite

gin
cylindrical rod of length 2L. The heat transfer in radial direction and contact resistance are

eer
negligible. The effective thermal conductivity of the composite rod is
2k1k 2 k1 k 2
(A) k1  k 2 (B) (C) (D)

in
k1k 2
k1  k 2 k1  k 2
Key: (C)
g.n
Sol: Two rods of equal length L are conductivity k1 and k 2 are connected.
No radial heat transfer
 Equivalent resistance per unit area Rod1 Rod2
et
L L k1 k2
R eq =   i 
k1 k 2
If this is single composite rod; Temp
Resistance, of composite rod per unit area
2L
R comp    ii 
k eq
From (I) and (II)

L1  L2  L

© All rights reserved by Thinkcell Learning Solutions Pvt. Ltd. No part of this booklet may be reproduced or utilized in any form without the written permission.

Downloaded From : www.EasyEngineering.net


Downloaded From : www.EasyEngineering.net

|ME-2019-Forenoon|

2L L L
 
k eq k1 k 2
2k1k 2
k eq 
k1  k 2

18. Consider an ideal vapor compression refrigeration cycle. If the throttling process is replaced by
an isentropic expansion process, keeping all the other processes unchanged, which one of the
following statements is true for the modified cycle?
(A) Coefficient of performance is the same as that of the original cycle
(B) Coefficient of performance is lower than that of the original cycle
(C) Refrigerating effect is lower than that of the original cycle

ww
Key: (D)
(D) Coefficient of performance is higher than that of the original cycle

Sol:
w.E
Figure 1: Isenthalpic expansion
Figure 2: Isentropic expansion

asy
En
3
2
3'
2'

T
gin T hc

4
eer 1 4'
1'

in S S
g.n
Form both figures we see
Figure1 Figure 2
et
h1'  h 4'  h1  h 4 (Refrigerating effect)
 COP in case (ii) is more than (i)because of more refrigerating effect.
 COP of isentropic expansion is more than isenthalpic expansion.

© All rights reserved by Thinkcell Learning Solutions Pvt. Ltd. No part of this booklet may be reproduced or utilized in any form without the written permission.

Downloaded From : www.EasyEngineering.net


Downloaded From : www.EasyEngineering.net

|ME-2019-Forenoon|

19. The position vector OP of point P(20, 10) is rotated anti-clockwise in X-Y plane by an angle
  30 such that point P occupies position Q, as shown in the figure. The coordinates (x, y) of
Q are Y
Q

P

X
O

(A) (13.40, 22.32) (B) (12.32, 18.66) (C) (22.32, 8.26) (D) (18.66, 12.32)

ww
Key: (B)

Sol: P   20,10  ,   30o Y

w.E
Q   x ', y '
 x ' cos 
 y '   sin 
 sin    20 
Q

  
asy cos   10 
 3 2 1 2   20 
 P


 1 2
En  
3 2  10  O
X


gin
0.866 0.5   20  12.32 
    
 0.5 0.866  10  18.66 

eer
 x ', y '  12.32,18.66 

20.
in
A cylindrical rod of diameter 10 mm and length 1.0 m fixed at one end. The other end is twisted
g.n
by angle of 10° by applying a torque. If the maximum shear strain in the rod is p×10 –3, then p is
equal to _____ (round off to two decimal places).
Key: (0.8726)
et
Sol: D  10mm,   10
L  1m
  P  103
L  R

1  1000  P  103  5  10 
180
50
P  0.8726
180

© All rights reserved by Thinkcell Learning Solutions Pvt. Ltd. No part of this booklet may be reproduced or utilized in any form without the written permission.

Downloaded From : www.EasyEngineering.net


Downloaded From : www.EasyEngineering.net

|ME-2019-Forenoon|

21. Which one of the following welding methods provides the highest heat flux (W/mm2)?
(A) Plasma are welding (B) Tungsten inert gas welding
(C) Oxy-acetylene gas welding (D) Laser beam welding
Key: (D)

Sol:
Welding process  W cm  2 Temp(oC)

Heat density
1. Gas welding 102  103 2500-3500
2. Shielded metal Arc 104 >6000
welding
3. Gas metal Arc 105 8000-10,000

ww welding
4. Plasma Arc welding 106 15000-30000

w.E
5. Electron beam
welding
107  108 20000-30000

asy
6. Laser beam welding 109 >30,000

En 4 
22.
gin
Water flows through a pipe with a velocity given by V    x  y  ˆj m s, where ˆj is the unit
t 

eer
vector in the y direction, t(>0) is in seconds, and x and y are in meters. The magnitude of total
acceleration at the point (x, y) = (1, 1) at t = 2s is ___________ m/s2.
Key: (3)

Sol:
4 
V    x  y  j m sec
in g.n
t 
V  ui  vj  wk
Acceleration at any point x, y, z and any time‘t’ is given as
et
dV dV dV dV
a  x,y,z,t   u V w
dt dx dy dz

4 
Given that x  1, y  1, t  2, z  0 and u=0, V    x  y  , w  0
t 
then,
dV d  4  4
   x  y  j.j   2
dt dt  t  t
dV d  4 
   x  y  j.j  1
dx dx  t 

© All rights reserved by Thinkcell Learning Solutions Pvt. Ltd. No part of this booklet may be reproduced or utilized in any form without the written permission.

Downloaded From : www.EasyEngineering.net


Downloaded From : www.EasyEngineering.net

|ME-2019-Forenoon|

dV d  4 
   x  y  j.j  1
dy dy  t 
dV d  4 
   x  y  j.j  0
dz dz  t 
4 4 
 a 1,1,0,2  2   0 1    x  y  1   0   0 
t t 
4 4
 2  xy
t t
4 4
 2   1  1  1  2  1  1  3m sec 2
2 2

A parabola x  y with 0  x  1 is shown in the figure. The volume of the solid of rotation
2
23.

ww obtained by rotating the shaded area by 360° around the x-axis is

w.E y

asy x  y2

En
gin
eer O 1 x

Key: (B)
(A)

4 in (B)

2
(C) 2 (D) 

g.n
Sol: Volume of the solid of rotation obtained by rotating around the x – axis is given by

V
b

 y dx
2 y
et
x a x  y2
1
V  xdx 
x 0
y 2  x 

1
 x2  
   
 2 0 2
0  x
1

© All rights reserved by Thinkcell Learning Solutions Pvt. Ltd. No part of this booklet may be reproduced or utilized in any form without the written permission.

Downloaded From : www.EasyEngineering.net


Downloaded From : www.EasyEngineering.net

|ME-2019-Forenoon|

24. A spur gear with 20° full depth teeth is transmitting 20 kW at 200 rad/s. The pitch circle
diameter of the gear is 100mm. The magnitude of the force applied on the gear in the radial
direction is
(A) 1.39 kN (B) 2.78 kN (C) 0.36 kN (D) 0.73 kN
Key: (D)

Sol:   20, P  20kW,   200rad sec.


D  100mm, FR  ?
P 20  103
T   100Nm
 200
T 100
Ft    2000N
R 50  103

ww FR  Ft tan   2000 tan 20  727.9N  0.73kN

25. For the equation


w.E dy
dx
 7x 2 y  0, if y  0   3 7, then the value of y(1) is

(A)
7 7 3
3
e
asy (B)
3 7 3
7
e (C)
3 3 7
7
e (D)
7 3 7
3
e

Key: (B)
En
Sol: Given D.E is
gin
dy
dx
 7x 2 y  0, y  0   3 7

The value of y (1) is ____.


dy eer
 7x 2  y   0  1

dx
in
 The equation (1) is linear D.E, Where P  7x 2 ; Q  0
7 x 3
g.n
I.F  e  e
et
2 3
P dx 7x dx
e
Solution of equation (1) is
y. I.F    Q. I.F  dx  C
7x 3 7x 3
 ye 3
  0.  I.F  dx  C  ye 3
C
 7x 3
yCe 3
  2
Given y  3 7 at x  0
 2  3 7  C
3  7 x3 3
From  2  , y  e 3  y 1  e 7 3
7 7

© All rights reserved by Thinkcell Learning Solutions Pvt. Ltd. No part of this booklet may be reproduced or utilized in any form without the written permission.

Downloaded From : www.EasyEngineering.net


Downloaded From : www.EasyEngineering.net

|ME-2019-Forenoon|

Q. No. 26 to 55 Carry Two Marks Each

26. A cube of side 100 mm is placed at the bottom of an empty container on one of its faces. The
density of the material of the cube is 800 kg/ m3. Liquid of density 1000 kg/m3 is now poured
into the container. The minimum height to which the liquid needs to be poured into the
container for the cube to just lift up is _____ mm.
Key: (80)

Sol:

ww h

w.E a  100

Given that density of cube material ecube  800kg m3

asy
Density of liquid poured e water  1000kg m3

En
Weight of cube  ecube  volume of cube  g
 800  0.1 0.1 0.1 g  0.8g N

gin
To just lift the cube, weight of cube = buoyancy force buoyancy force = weight of liquid
displaced

eer
 eliquid  volume of liquid  g  1000  0.1 0.1 h  g  10hg

in
Where h = height of water poured
By equating weight of cube = buoyancy force
0.8g  10hg g.n
h
0.8
10
 0.08m  80 mm
et
27. A project consists of six activities. The immediate predecessor of each activity and the
estimated duration is also provided in the table below:
Activity Immediate predecessor Estimated duration (weeks)
P - 5
Q - 1
R Q 2
S P, R 4
T P 6
U S,T 3

© All rights reserved by Thinkcell Learning Solutions Pvt. Ltd. No part of this booklet may be reproduced or utilized in any form without the written permission.

Downloaded From : www.EasyEngineering.net


Downloaded From : www.EasyEngineering.net

|ME-2019-Forenoon|

If all activities other than S take the estimated amount of time, the maximum duration (in
weeks) of the activity S without delaying the completion of the project is ____________.
Key: (6)

Sol: From the given data, we can represent network flow as follows

1
P 5 T 6
U  3
1 6
1
Dummy  0 

Q 1 R  2 S 4
1 1

ww Considering path 1-2-5-6, time taken will be = 5+ 6+ 3= 14 weeks

w.E
Considering path 1-2-4-5-6, time taken will be=5+0+4+5=12 weeks.
Considering path 1-3-4-5-6, time taken will be = 1+2+4+3 =10 weeks

asy
Maximum time taken is 14 weeks, so ‘2’ weeks can be delayed so that 1-2-4-5-6 path will also
take 14 weeks.

En
So answer is 4 weeks +2 weeks = 6 weeks
Duration can be given for activities without delay the project.

gin
Consider an elastic straight beam of length L  10m, with square cross-section of side a=5

eer
28.
mm, and Young’s modulus E = 200 GPa. This straight beam was bent in such a way that the

in
two ends meet, to form a circle of mean radius R. Assuming that Euler-Bernoulli beam theory is
applicable to this bending problem, the maximum tensile bending stress in the bent beam is
___________ MPa. g.n
L R
et
Key: (100)
Ends of the beam
Sol: L  10 mts; a  5mm, E  200GPa
10
L  2R, R   5mts  5000mm.
2
a 5
y 
2 2
E 200  103  5 
 y     100MPa
R 5000 2

© All rights reserved by Thinkcell Learning Solutions Pvt. Ltd. No part of this booklet may be reproduced or utilized in any form without the written permission.

Downloaded From : www.EasyEngineering.net


Downloaded From : www.EasyEngineering.net

|ME-2019-Forenoon|

29. A truss is composed of members AB, BC, CD, AD and BD, as shown in the figure. A vertical
load of 10 kN is applied at point D. The magnitude of force (in kN) in the member BC is
________. 10kN
D

45 45
A C
B
Key: (5)

ww
Sol: Due symmetry

RA  RC 

Joint C
w.E 10kN
2
 5kN

asy 45
R DC
135

En R BC 90 90

gin RC

eer 5kN
R BC

R BC 

R DC

5
sin 225 sin 90 sin 45
5 sin 225
in
 5kN  Tension  g.n
sin 45

et
30. A gas is heated in a duct as it flows over a resistance heater. Consider a 101 kW electric heating
system. The gas enters the heating section of the duct at 100 kPa and 27°C with a volume flow
rate of 15m3/s. If heat is lost from the gas in the duct to the surroundings at a rate of 51kW, the
exit temperature of the gas is
(Assume constant pressure, ideal gas, negligible change in kinetic and potential energies and
constant specific heat; Cp = 1 kJ/kg.K; R = 0.5 kJ/kg.K).
(A) 53°C (B) 32°C (C) 37°C (D) 76°C

© All rights reserved by Thinkcell Learning Solutions Pvt. Ltd. No part of this booklet may be reproduced or utilized in any form without the written permission.

Downloaded From : www.EasyEngineering.net


Downloaded From : www.EasyEngineering.net

|ME-2019-Forenoon|

Key: (B)

Sol: Q

W
Heater

Inlet conditions
P1  100kPa, T1  27C  300K
V1  V2  15m3 s

ww Q  51 kW
W  101 kW

w.E
Now, mass flow rate m 
P1V1
RT1

m asy
100  15
0.5  300
 10 kg s

En
From 1st law of thermodynamics and steady flow energy equation
Q  m  h 2  h1   W
gin
51  m  h 2  h1   101
m  h 2  h1   50
eer
10  h 2  h1   50  h 2  h1  5

in
Cp  T2  T1   5  T2  27  5  32 C

g.n
31. A harmonic function is analytic if it satisfies the Laplace equation. If u  x, y   2x  2y  4xy

is a harmonic function, then its conjugate harmonic function v  x, y  is


2 2

et
(A)  4xy  2y2  2x 2  constant (B) 4xy  2x 2  2y2  constant

(C) 2x 2  2y2  xy  constant (D) 4y2  4xy  constant


Key: (B)

Given, u  x, y   2x  2y  4xy is a harmonic function.


2 2
Sol:
u u
  4x  4y;  4y  4x
x y
The conjugate harmonic function v  x, y  is obtained as follows:

© All rights reserved by Thinkcell Learning Solutions Pvt. Ltd. No part of this booklet may be reproduced or utilized in any form without the written permission.

Downloaded From : www.EasyEngineering.net


Downloaded From : www.EasyEngineering.net

|ME-2019-Forenoon|

v v
dv  dx  dy  from total differential of v  x, y 
x y
 u   u 
 dv     dx    dy ,using C  R equations
 y   x 
 dv    4y  4x  dx   4x  4y  dy

Exact D.E

  dv     4y  4x  dx    4x  4y  dy
  x 2 
 v  x, y     4yx  4     4  y 2 2   C
  2 
 v  x, y   4xy  2x  2y 2  C
2

ww
32. A uniform thin disk of mass 1 kg and radius 0.1 m is kept on a surface as shown in the figure. A

w.E
spring of stiffness k1 = 400 N/m is connected to the disk center A and another spring of stiffness
k2 = 100 N/m is connected at point B just above point A on the circumference of the disk.

asy
Initially, both the springs are unstretched. Assume pure rolling of the disk. For small
disturbance from the equilibrium, the natural frequency of vibration of the system is ______

En
rad/s (round off to one decimal place).

gin B
k2

eer k1

in A

g.n
Key: (23.1) et
Sol:
k2
B

k1

Give a small displacement to Disc about ‘O’

© All rights reserved by Thinkcell Learning Solutions Pvt. Ltd. No part of this booklet may be reproduced or utilized in any form without the written permission.

Downloaded From : www.EasyEngineering.net


Downloaded From : www.EasyEngineering.net

|ME-2019-Forenoon|

k 2 2r sin 
2r sin 
B
2r cos 
r
r sin
k1 rsin 
A
r cos 
I0 
O 
O'
m0  0

ww I0    k1r sin   r cos     k 2 2r sin   2r cos    0


mr 2 3 3

w.E
I0  IC.G.  mr 2 
2
 mr 2  mr 2   1 0.12  0.015kg m 2
2
m  1kg , r  0.1 m, k1  400 N m, k 2  100 N m
2

asy
Assume sin   , cos   1
0.015    400  0.1   100  2  0.1   2  0.1  0
2

0.015  8  0
En
n 
8
0.015 gin
 23.09  23.1 rad sec

33. eer
In ASA system, the side cutting and end cutting edge angles of a sharp turning tool are 45° and

in
10°, respectively. The feed during cylindrical turning is 0.1 mm/rev. The center line average
surface roughness (in m, round off to one decimal place) of the generated surface is _______.
g.n
Key: (3.747)

Sol: Given, Cs  45, Ce  10


f  0.1 mm / rev
et
f
R a or CLA 
4  tan Cs  cot Ce 
0.1
  3.747  103 mm  3.747 m.
4  tan 45  cot10 

34. Consider a prismatic straight beam of length L  m, pinned at the two ends as shown in the
figure.

© All rights reserved by Thinkcell Learning Solutions Pvt. Ltd. No part of this booklet may be reproduced or utilized in any form without the written permission.

Downloaded From : www.EasyEngineering.net


Downloaded From : www.EasyEngineering.net

|ME-2019-Forenoon|

The beam has a square cross-section of side p = 6mm. The Young’s modulus E = 200 GPa, and
the coefficient of thermal expansion   3 106 K1. The minimum temperature rise required to
cause Euler buckling of the beam is ___________K.
Key: (1)

Sol: L  , T  ?
64
Area  6  6  36 mm 2 , I   108mm 4
12
E  200GPa
  3  106 K 1
2 EI 2  200  103  108
PE    21.6 MN
2
e 2  106

ww PE  EAT
21.6  200  103  36  3  10 6  T
T  1K.
w.E
35.
x  y  z 1 asy
The set of equations

ax  ay  3z  5
5x  3y  az  6 En
gin
has infinite solutions, if a =

Key: (A)
(A) 4
eer (B) –4 (C) –3 (D) 3

Sol:
x  y  z 1 
in
ax  ay  3z  5    Non  homogeneous 
g.n
5x  3y  az  6 

1 1 1 1 
Augmented matrix, [A|B]   a a 3 5 
et
 5 3 a 6 

R2  R3
1 1 1 1 
5 3 a 6 
 A | B  
a a 3 5 

Applying R 2  R 2  5R1; R 3  R 3  aR1

© All rights reserved by Thinkcell Learning Solutions Pvt. Ltd. No part of this booklet may be reproduced or utilized in any form without the written permission.

Downloaded From : www.EasyEngineering.net


Downloaded From : www.EasyEngineering.net

|ME-2019-Forenoon|

1 1 1 1 
0 8 a  5 1 

0 2a 3  a 5  a 
R 3  4R 3  a R 2
1 1 1 1 
0 8 a 5 1 

0 0 a 2  a  12 20  5a 
To have infinite number of solutions,
a 2  a  12  0 & 20  5a  0
  a  4  a  3  0 a 4
 a  4  or  a  3 & a4

ww  a must be equal to '4' only.

36.
w.E
In a UTM experiment, a sample of length 100 mm, was loaded in tension until failure. The
failure load was 40 kN. The displacement, measured using the cross-head motion, at failure,

asy
was 15 mm. The compliance of the UTM is constant and is given by 5 × 10–8 m/N. The strain at
failure in the sample is ___________%.
Key: (13)
En    100mm
Sol: Sample length

gin
Failure load  p   40 kN 40kN

eer
Deformation at failure total  15mm
Force
 Total strain 
in
 total

15
100
 0.15  15%

1 g.n
Compliance of the UTM  5  108 m N 

For axial loaded specimen, stiffness  


stiffness
AE  1 
8 
 5  10 
N m  permanent
et
recoverable
 total
Deformation recoverable at failure load Deformation
re cov erable 2
 Recoverable strain    0.02  2%
100
Permanent strain = Total strain – Recoverable = 15 – 2 = 13%

37. A plane-strain compression (forging) of a block is shown in the figure. The strain in the z-
direction is zero. The yield strength (Sy) in uniaxial tension/compression of the material of the
block is 300 MPa and it follows the Tresca (maximum shear stress) criterion. Assume that the

© All rights reserved by Thinkcell Learning Solutions Pvt. Ltd. No part of this booklet may be reproduced or utilized in any form without the written permission.

Downloaded From : www.EasyEngineering.net


Downloaded From : www.EasyEngineering.net

|ME-2019-Forenoon|

entire block has started yielding. At a point where x  40 MPa (compressive) and xy  0, the

stress component  y is
y
Moving platen

O x

Fixed platen

(A) 260 MPa (tensile) (B) 340 MPa (compressive)


(C) 260 MPa (compressive) (D) 340 MPa (tensile)

ww
Key: (B)

 y  300MPa,  x  40MPa  compressive  ,  xy  0,  y  ?


Sol:

w.E
For plane strain
z  y
z  0 
E
asy
v x v
E
 z  v  40   y 
E
 0  z  v  x   y 

En
 x   y  y  z z   x 
max  M ax 
 2
gin
,
2
,
2 

 40   y  y  v   y  40  v   y  40   40 
max 
Sy
2  F.S
 Max 
eer
 2
,
2
,
2



in
From the above equation Maximum will be the first one i.e.,
40   y
2

Sy

g.n
2  F.S
40   y
2

2  F.S
40   y  300
Sy
or
40   y
2

2  F.S
40   y  300
Sy
et
 y  340MPa  y  260MPa
 y  340MPa(Compressive) or  y  260MPa (Tensile)

40   y  300   y  260 MPa  Tension 

But in the forging operation  y can’t be tensile hence the answer is 340MPa (compressive).

38. Match the following sand mold casting defects with their respective causes.
Defect Cause
(P) Blow hole 1. Poor collapsibility
(Q) Misrun 2. Mold erosion

© All rights reserved by Thinkcell Learning Solutions Pvt. Ltd. No part of this booklet may be reproduced or utilized in any form without the written permission.

Downloaded From : www.EasyEngineering.net


Downloaded From : www.EasyEngineering.net

|ME-2019-Forenoon|

(R) Hot tearing 3. Poor permeability


(S) Wash 4. Insufficient fluidity
Codes:
(A) P-3, Q-4, R-2, S-1 (B) P-4, Q-3, R-1, S-2
(C) P-2, Q-4, R-1, S-3 (D) P-3, Q-4, R-1, S-2
Key: (D)

39. A steam power cycle with regeneration as shown below on the T-s diagram employs a single
open feedwater heater for efficiency improvement. The fluids mix with each other in an open
feedwater heater. The turbine is isentropic and the input (bleed) to the feedwater heater from the
turbine is at state 2 as shown in the figure. Process 3-4 occurs in the condenser. The pump work

ww is negligible. The input to the boiler is at state 5.

w.E
The following information is available from the steam tables:

State 1 2 3 4 5 6

asy Enthalpy (kJ/kg) 3350 2800 2300 175 700 1000

En
gin
1

eer T

in 5 2
4
3
g.n
s

The mass flow rate of steam bled from the turbine as a percentage of the total mass flow rate at
the inlet to the turbine at state 1 is _______.
et
Key: (20)

Sol: 1
6
m2
T
5 2
m1
4
3

© All rights reserved by Thinkcell Learning Solutions Pvt. Ltd. No part of this booklet may be reproduced or utilized in any form without the written permission.

Downloaded From : www.EasyEngineering.net


Downloaded From : www.EasyEngineering.net

|ME-2019-Forenoon|

Let m 2 mass is bled at state 2 and m1 mass goes to the condenser.


m1h 4
Assuming no heat loss from the feed water heater,
m1h 4  m 2 h 2   m1  m 2  h 5  1
h 4  175 kJ kg, h 2  2800kJ kg, h 5  700 kJ kg Q0
175 m1  2800m 2   m1  m 2  700 m2 h 2
 2100 m 2  525 m1
m2 m2
 0.25   0.2. Hence 20%
m1 m1  m2
 m1  m 2  h 5
40. The rotor of turbojet engine of an aircraft has a mass 180 kg and polar moment of inertia 10
kg.m2 about the rotor axis. The rotor rotates at a constant speed of 1100 rad/s in the clockwise

ww direction when viewed from the front of the aircraft. The aircraft while flying at a speed of 800

w.E
km per hour takes a turn with a radius of 1.5 km to the left. The gyroscopic moment exerted by
the rotor on the aircraft structure and the direction of motion of the nose when the aircraft

asy
turns, are
(A) 1629.6 N.m and the nose goes up
(B)
(C) En
1629.6 N.m and the nose goes down
162.9 N.m and the nose goes down
(D)
gin
162.9 N.m and the nose goes up
Key: (B)

Sol: eer
m  180kg   1100rad sec, V  800kmph R  1.5 mts
I  10kg  m 2
5
800 
in g.n
V
p   18  0.148rad sec
R 1.5  1000
C  Ip  1100  10  148.148  1629.628 Nm
et
a
O

Dip the nose and Raise the tail. So answer is option ‘B’.

41. The wall of a constant diameter pipe of length 1 m is heated uniformly with flux q" by
wrapping a heater coil around it. The flow at the inlet to the pipe is hydrodynamically fully

© All rights reserved by Thinkcell Learning Solutions Pvt. Ltd. No part of this booklet may be reproduced or utilized in any form without the written permission.

Downloaded From : www.EasyEngineering.net


Downloaded From : www.EasyEngineering.net

|ME-2019-Forenoon|

developed. The fluid is incompressible and the flow is assumed to be laminar and steady all
through the pipe. The bulk temperature of the fluid is equal to 0°C at the inlet and 50°C at the
exit. The wall temperatures are measured at three locations, P, Q and R, as shown in the figure.
The flow thermally develops after some distance from the inlet. The following measurements
are made:

Point P Q R
Wall Temp (°C) 50 80 90

0.2 m 0.2 m 0.2 m 0.2 m 0.2 m

ww
0 C w.E P Q R
50C

asy
En Constant wall flux

gin
P and Q only eer
Among the locations P, Q and R, the flow is thermally developed at:
(A) (B) P, Q and R (C) R only (D) Q and R only

Key: (D)
in g.n
Sol:
et
0 C 50C

P Q R

From heat balace


q" d  x  m.c.  TB  Tin   1
Where mc = heat capacity of fluid and
 q" d 
TB  bulk mean temperature  TB    x  Tin
 mc 
At inlet z  0, Tin  0

© All rights reserved by Thinkcell Learning Solutions Pvt. Ltd. No part of this booklet may be reproduced or utilized in any form without the written permission.

Downloaded From : www.EasyEngineering.net


Downloaded From : www.EasyEngineering.net

|ME-2019-Forenoon|

x  1m, TB  50 C
q" d
  Z  constant   50
mc
 TB  50x   ii 

Now : q" d  h*  Tw  TB  At any section 


q" d
   Tw  TB 
h
q" d
  TB  Tw
h
 q" d 
 Tw  C  TB C     constant for fully developed flow.
 h 

ww Tw  C  50x.
At P, Tw  50C, x  0.4 50
50
For thermally

w.E
 50  50  0.4  C  30
At Q, Tw  80C, x  0.6
Tw

TB
30
developed region
Tw  TB  C

asy
 80  50  0.6  50  C
At R, Tw  90, x  0.8
is constant at Q & R

En
90  50  0.8  50  C P Q R

gin
 Clearly we see that Tw  TB is constant from Q
 Flow will be thermally developed between Q &R

42. eer
At a critical point in a component, the state of stress is given as xx  100 MPa,

in
 yy  220 MPa, xy   yx  80 MPa and all other stress components are zero. The yield strength
of the material is 468 MPa. The factor of safety on the basis of maximum shear stress theory is g.n
Key: (1.8)
_________ (round off to one decimal place).
et
Sol: xx  100MPa,  yy  220MPa,  xy   yx  80MPa,  yt  468MPa.
 yt  yt     2 1  2 
  1 , , 
F.O.S 2F.0.S  2 2 2
2
x  y  x  y 
1,2       xy
2

2  2 
2
100  220  100  220 
     80
2

2  2 
 160  100  260MPa, 60MPa
yt  260  60 260 60  468
 Max  , ,   F.O.S   1.8
2  F.O.S  2 2 2 260

© All rights reserved by Thinkcell Learning Solutions Pvt. Ltd. No part of this booklet may be reproduced or utilized in any form without the written permission.

Downloaded From : www.EasyEngineering.net


Downloaded From : www.EasyEngineering.net

|ME-2019-Forenoon|

43. A gas turbine with air as the working fluid has an isentropic efficiency of 0.70 when operating
at a pressure ratio of 3. Now, the pressure ratio of the turbine is increased to 5, while
maintaining the same inlet conditions. Assume air as a perfect gas with specific heat ratio
  1.4. If the specific work output remains the same for both the cases, the isentropic
efficiency of the turbine at the pressure ratio of 5 is _____ (round off to two decimal places).
Key: (0.51)

P1 P 1 P1
Sol:  3, l  5, rp  pressure ratio
P2 P3
Work done by turbine 1 for rp  3 P2
2
W1 2'  mc p  T1  T2   mc p  1 2'   T1  T2  T 2' P3
3'

ww 

 m  c p  1 2'   T1 

T1 

1 

 
rp1  
3

w.E 



1 
 i 
S

asy
W1 2'   m  c p  T1   1 2' 1  1 
 r 
 P1 

En
Since W12'  W13'


13' 1 
1 
gin

1 

 

 1 

 1 2' 1  1 

 rP2  

eer  r 
 P1 

in
Putting 1 2  0.7, rP1  3, rP2  5

13'  0.5115  13'  51.15% g.n


Hence efficiency  0.5115
et
44. The value of the following definite integral is ________ (round off to three decimal places)
e

x
1
n x  dx

Key: (2.097)
e
Sol: x
1
nx  dx

Let n x  t  x  e t  dx  e t dt

© All rights reserved by Thinkcell Learning Solutions Pvt. Ltd. No part of this booklet may be reproduced or utilized in any form without the written permission.

Downloaded From : www.EasyEngineering.net


Downloaded From : www.EasyEngineering.net

|ME-2019-Forenoon|

If x  1  t  0
If x  e  t  n e  1
e 1 1
  x nxdx   e t te t dt   te 2t dt
1 0 0

1
  e 2t   e2t    e2 e2   1 2 1 1 1
  t            0    e      2.097.
  2   4  0  2 4   4 2 4 4

45. Taylor’s tool life equation is given by VTn = C, where V is in m/min and T is in min. In a
turning operation, two tools X and Y are used. For tool X, n = 0.3 and C = 60 and for tool Y,
n = 0.6 and C = 90. Both the tools will have the same tool life for the cutting speed (in m/min,

ww
round off to one decimal place) of _____.
Key: (40.5)

Sol: Tool  X
n  0.3
C  60
w.E Tool  Y
n  0.6
C  90
VTX0.3  60
asy VTY0.6  90

En
1 0.3 1 0.6
 60   90 
TX    TY   
 VX   VY 

gin
For same toll life at breakeven  VX  VY  V 

 60 
Tx  TY  
 Vx 
eer
10 6
 90 
 
 VY 
10 6

 V  40.5m min

46.
in
In a four bar planar mechanism shown in the figure, AB = 5 cm, AD = 4 cm and DC = 2 cm. In
g.n
the configuration shown, both AB and DC are perpendicular to AD. The bar AB rotates with an
angular velocity of 10 rad/s. The magnitude of angular velocity (in rad/s) of bar DC at this
instant is
B
et
10 rad s C

A D

(A) 25 (B) 15 (C) 10 (D) 0

© All rights reserved by Thinkcell Learning Solutions Pvt. Ltd. No part of this booklet may be reproduced or utilized in any form without the written permission.

Downloaded From : www.EasyEngineering.net


Downloaded From : www.EasyEngineering.net

|ME-2019-Forenoon|

Key: (A)
B I 23
Sol: AB = 5 cm
AD = 4 cm and DC = 2cm, 10 rad s
AB  10rad sec, DC  ? C I
34

I13   so the velocity of link


BC is zero and now VB  VC A I 24
D
rAB . AB  rCD .CD I12 I 41
5  10  2  CD
CD  25rad sec I13  

47. If one mole of H2 gas occupies a rigid container with a capacity of 1000 liters and the

ww temperature is raised from 27°C to 37°C, the change in pressure of the contained gas (round off
to two decimal places), assuming ideal gas behavior, is _____ Pa. (R = 8.314 J/mol.K).
Key: (83.14)
w.E
Initially T1  27C  300K
Sol:

asy
n1  1 mole

En
R  8.314 KJ mol  K

V1  1000 litres  1m 3

gin
Finally, T2  37C  310K
P2  ?
From ideal Gas relation eer
P1V1   nR  T1
P1  8.314  300 pascal
P1  300 R
in g.n
Now since the volume of container is constant hence.
V1  V2
et
 R  TP   nR  TP
1 2

1 2

T2 310
P2  P1  300R 
T1 300
 P2  310R  Pascal 
 Change in pressure,  P2  P1    310  300  R  10R  10  8.314
P  83.14 pascal

© All rights reserved by Thinkcell Learning Solutions Pvt. Ltd. No part of this booklet may be reproduced or utilized in any form without the written permission.

Downloaded From : www.EasyEngineering.net


Downloaded From : www.EasyEngineering.net

|ME-2019-Forenoon|

48. Three slabs are joined together as shown in the figure. There is no thermal contact resistance at
the interfaces. The center slab experience a non-uniform internal heat generation with an
average value equal to 10000 Wm–3, while the left and right slabs have no internal heat
generation.

Left extreme
face T1  100C T2
1m 1m 1m

100 W m2 .K 100 W m2 .K
30C 30C

ww
w.E
All slabs have thickness equal to 1 m and thermal conductivity of each slab is equal to 5 Wm –1
K . The two extreme faces are exposed to fluid with heat transfer coefficient 100 Wm–2K–1 and
–1

asy
bulk temperature 30°C as shown. The heat transfer in the slabs is assumed to be one

En
dimensional and steady, and all properties are constant. If the left extreme face temperature T1 is
measured to be 100°C, the right extreme faced temperature T2 is _____ °C.
Key: (60)
gin
Sol:
eer
Left extreme
face T1  100C in 1m 1m 1m
T2
Right
entrance face
g.n
100 W m2 .K
T1  30C
100 W m 2 .K
T2  30C
et
100  30    T1  T   7 kN m
Heat flowing from left slab to left extreme face  1 2

 1  1
   
 100  h
Heat generated in the central slab = 10 kW m3

For 1m length heat generated in the central slab  10 kN m3 1m  10 kW m2

© All rights reserved by Thinkcell Learning Solutions Pvt. Ltd. No part of this booklet may be reproduced or utilized in any form without the written permission.

Downloaded From : www.EasyEngineering.net


Downloaded From : www.EasyEngineering.net

|ME-2019-Forenoon|

Out of 10 kW m2 heat generated in the central slab, 7 kW m2 will be flowing out through
left slab remaining 3kW m 2 should flow through right slab.
Applying heat flow equation at right extreme face
 
T T 
2
3000   
2

  1  
 h 
   

3000  2
T  30   T  60C
 
2
1
 
 100 

ww
49. Five jobs (J1, J2, J3, J4 and J5) need to be processed in a factory. Each job can be assigned to any
of the five different machines (M1, M2, M3, M4 and M5). The time duration taken (in minutes)

w.E
by the machines for each of the jobs, are given in the table. However, each job is assigned to a
specific machine in such a way that the total processing time is minimum. The total processing

asy
time is _____ minutes.

En M1 M2 M3 M4 M5

gin J1 40 30 50 50 58

eerJ2 269 38 60 26 38

in
J3

J4
40

28
34

40
28

40
24

32
30

48
g.n
Key: (146)

Sol:
J5 28

This problem can be solved by assignment problem


32 38 22 44

et
M1 M2 M3 M4 M5

J1 40 30 50 50 58

J2 26 38 60 26 38

J3 40 34 28 24 30

J4 28 40 40 32 48

J5 28 32 38 22 44

Row minimization matrix is

© All rights reserved by Thinkcell Learning Solutions Pvt. Ltd. No part of this booklet may be reproduced or utilized in any form without the written permission.

Downloaded From : www.EasyEngineering.net


Downloaded From : www.EasyEngineering.net

|ME-2019-Forenoon|

10 0 20 20 28
0 12 34 0 12
16 10 4 0 6
0 12 12 4 20
6 10 16 0 22

Column minimization matrix is


10 0 16 20 22
0 12 30 0 6
16 10 0 0 0
0 12 8 4 14
6 10 12 0 16

ww In the above matrix all zeros can be coved with only four lines as follows

10 w.E 0 16 20 22
0
16
asy
12 30 0 6
10 0 0 0
0
6
En
12 8 4 14
10 12 0 16

gin
The least value in the uncrossed calls is 8. It is subtracted from the uncrossed cell and added

eer
for the intersection of the vertical line and horizontal lines
18 0 16 28 22
0 4 22 0 6
24 10 0 8 0
0 4 0 4 14
in g.n
6 2 4 0 16
Since the above matrix can only be covers with ‘5’ lines the assignment can be done as
follows
et
M1 M2 M3 M4 M5
J1 18 0 16 28 22
J2 0 4 22 0 6
J3 24 0 0 8 0
J4 0 4 0 4 14
J5 6 2 4 0 16

 Assignments of jobs are


J1  M 2 J 2  M1 J3  M5 J 4  M3 J5  M4

30 38 28 28 22
© All rights reserved by Thinkcell Learning Solutions Pvt. Ltd. No part of this booklet may be reproduced or utilized in any form without the written permission.

Downloaded From : www.EasyEngineering.net


Downloaded From : www.EasyEngineering.net

|ME-2019-Forenoon|

 Total processing time = 30 + 38 + 28 + 28 + 22 = 146 minutes

50. In orthogonal turning of a cylindrical tube of wall thickness 5mm, the axial and the tangential
cutting forces were measured at 1259 N and 1601 N, respectively. The measured chip thickness
after machining was found to be 0.3 mm. The rake angel was 10° and the axial feed was 100
mm/min. The rotational speed of the spindle was 1000 rpm. Assuming the material to be
perfectly plastic and Merchant’s first solution, the shear strength of the martial is closest to
(A) 722 MPa (B) 875 MPa (C) 200 MPa (D) 920 MPa
Key: (A)

Sol: Ft  1259 N, FC  1601 N, t C  0.3 mm


  10, F  100 mm min , N  1000 rpm

ww f
F 100

N 1000
 0.1mm / rev

w.E
Since it is orthogonal machining
t  f  0.1 mm

r 
t 0.1
t c 0.3 asy
 0.33

tan  
En
r cos 

0.33cos10
1  r sin  1  0.33 sin10
 0.348

  19.18
gin
FS  FC cos   Ft sin 

eer
 1601 cos 19.18  1259 sin 19.18  1098.42 N

FS   bt sin   1098.42 

  721.74 MPa
in
  5  0.1
sin 19.18 

g.n
51. A single block brake with a short shoe and torque capacity of 250 N-m is shown. The
cylindrical brake drum rotates anticlockwise at 100 rpm and the coefficient of friction is 0.25.
et
1.5a a
P

a 4

© All rights reserved by Thinkcell Learning Solutions Pvt. Ltd. No part of this booklet may be reproduced or utilized in any form without the written permission.

Downloaded From : www.EasyEngineering.net


Downloaded From : www.EasyEngineering.net

|ME-2019-Forenoon|

The value of a, in mm (round off to one decimal place), such that the maximum actuating force
P is 2000 N, is _____.
Key: (212.5)

Sol: FBD of drum is


RN
R N  Ff

ww Given that Braking torque b  250 N.m


b  Ff  a

w.E
250  Ff  a  Ff 

F
250
a
N

250 1000

asy
Ff  R N  R N  f 
 a  0.25 

a
N

En
FBD of lever is

a 1.5a a

gin
eer o a 4

in RN
F

g.n
M 0  0  P  2.5a  R N  a  Ff 

 2000   2.5a 
1000
a 
250 a

a
4 et
a a 4
5000a  1000  62.5
a  0.2125m  212.5 mm

0.01
52. A circular shaft having diameter 65.000.05 mm is manufactured by turning process. A 50 m
thick coating of TiN is deposited on the shaft Allowed variation in TiN film thickness is 5m.
The minimum hole diameter (in mm) to just provide clearance fit is
(A) 65.12 (B) 64.95 (C) 65.01 (D) 65.10
Key: (A)

© All rights reserved by Thinkcell Learning Solutions Pvt. Ltd. No part of this booklet may be reproduced or utilized in any form without the written permission.

Downloaded From : www.EasyEngineering.net


Downloaded From : www.EasyEngineering.net

|ME-2019-Forenoon|

Sol: Shaft  65.120.01


0.05

Coating thickness  50  5m


55m  0.055mm
or 45m  0.045 mm
Clearance Fit

t max t max
Hole
ULS

ULH

Maximum
LLH

ww Minimum
clearance
clearance

ULS
Basic size

w.E Shaft
LLS

asy
En Hole

gin
For just clearance Fit
Minimum clearance  zero eer
 LLH  ULS
in
 ULS before electro plating = 65.01
g.n
 ULS after electroplating  65.01  2  0.055  65.12mm

53. Two immiscible, incompressible, viscous fluids having same densities but different viscosities
et
are contained between two infinite horizontal parallel plates, 2 m apart as shown below. The
bottom plate is fixed and the upper plate moves to the right with a constant velocity of 3 m/s.
With the assumptions of Newtonian fluid, steady, and fully developed laminar flow with zero
pressure gradient in all directions, the momentum equations simplify to

y 1
2  21
2m
2 1m

d2u x
 0.
dy 2

© All rights reserved by Thinkcell Learning Solutions Pvt. Ltd. No part of this booklet may be reproduced or utilized in any form without the written permission.

Downloaded From : www.EasyEngineering.net


Downloaded From : www.EasyEngineering.net

|ME-2019-Forenoon|

If the dynamic viscosity of the lower fluid,  2 , is twice that of the upper fluid, 1 , then the
velocity at the interface (round off to two decimal places) is ___________ m/s.
Key: (1)

Sol:

y 1
2  21
2m
2 1m

d2u
Given that  0  1
dy 2

ww Integrating once, the above equation becomes


du
 C1   2 
dy
w.E
Integrating equation  2  ,

asy
u  C1 y  C2   3
From equation (3) we can say that, velocity is linearly varying so the shear stress will be

En
constant at the interface of two viscous fluids
i.e., shear stress at y=1m, from fixed plate = shear stress at 1m from moving plate.
 v 0
2  2
 1 
  1 
 1  gin
 v  vi 

eer
where v= velocity of moving plate, vi = velocity at interface of two fluids
v 
 
1
 3  vi 
21  i   1 
 1 
in
  2vi  3  vi  vi  1m / sec

Then the velocity profile ill be as follows g.n


1
3m / sec

1m
et
2m
2 1m / sec 1m

© All rights reserved by Thinkcell Learning Solutions Pvt. Ltd. No part of this booklet may be reproduced or utilized in any form without the written permission.

Downloaded From : www.EasyEngineering.net


Downloaded From : www.EasyEngineering.net

|ME-2019-Forenoon|

54. A car having weight W is moving in the direction as shown in the figure. The centre of gravity
(CG) of the car is located at height h from the ground, midway between the front and rear
a
wheels.
Direction
of motion
CG

h W

l
Rr Rf

The distance between the front and rear wheels is . The acceleration of the car is a, and
acceleration due to gravity is g. The reactions on the front wheels (Rf) and rear wheels (Rr) are

ww given by

(A)
w.E
Rf  Rr 

W Wh
W Wh
2
  a
g  
W Wh
(B) Rf 
asy2
   a; R r 
g  
W Wh
2
  a
g  

(C) Rf  Rr 
En 2
  a
g  

(D) Rf 
2 gin
W Wh
   a; R r 
g  
W Wh
2
  a
g  
Key: (D)

Sol: eer W

h
in W
g
a

g.n
R
l F et
Rr Rf
M y  0

Rr  Rf  W  1
M R  0
 L W 
 W     a  h   Rf  0
 2  g 
W W h 
Rf    a
2 g L
W Wh 
Rr     a.
2 g L

© All rights reserved by Thinkcell Learning Solutions Pvt. Ltd. No part of this booklet may be reproduced or utilized in any form without the written permission.

Downloaded From : www.EasyEngineering.net


Downloaded From : www.EasyEngineering.net

|ME-2019-Forenoon|

55. The variable x takes a value between 0 and 10 with uniform probability distribution. The
variable y takes a value between 0 and 20 with uniform probability distribution. The probability
of the sum of variables (x + y) being greater than 20 is
(A) 0.33 (B) 0.50 (C) 0.25 (D) 0
Key: (C)

Sol: x  0, 10 y Required portion


y  0, 20
20
A
y  x  20

Area of 'S' at  x, y   S

ww 0

w.E
P  x  y  20  ?

 f  x, y  
1

1
10 20

asy Area of S 200


 P  x  y  20  P  y  20  x 

area of A 2
1
En
 10  10


area of S

200
gin   0.25
4
 or 
eer
P  x  y  20   f  x, y  dxdy  
1
dx dy


1

200 A  dx dy 
1
200
A

in
 Area of triangle
A
200

g.n

1 1 
   10  10  0.25.
200  2  et

© All rights reserved by Thinkcell Learning Solutions Pvt. Ltd. No part of this booklet may be reproduced or utilized in any form without the written permission.

Downloaded From : www.EasyEngineering.net


Downloaded From : www.EasyEngineering.net

ww
w.E
asy
En
gin
eer
in g.n
et

**Note : Other Websites/ Blogs Owners we requested you, Please do not Copy
(or) Republish this Material.
This copy is NOT FOR SALE.
**Disclimers : EasyEngineering does not own this book/materials, neither created nor
scanned. we provide the links which is already available on the internet. For any quarries,
Disclaimer are requested to kindly contact us. We assured you we will do our best. We DO
NOT SUPPORT PIRACY, this copy was provided for students who are financially troubled but deserving
to learn. Downloaded From : www.EasyEngineering.net
Thank You and God Bless!
Downloaded From : www.EasyEngineering.net

ME-GATE-2019-Afternoon

GENERAL APTITUDE

Q. No. 1 - 5 Carry One Mark Each

1. If IMHO = JNIP; IDK=JEL; and SO = TP, then IDC= ________.


(A) JDE (B) JDC (C) JCD (D) JED
Key: (D)
Sol: IMHO  JNIP IDK  JEL SO  TP
I M H O I D K S O I D C
1 1 1 1 1 1 1 1 1  1  1  1

ww
2.
J N I P J E L T P

Once the team of analysts identify the problem, we ______ in a better position to comment on
J E D

the issue.
w.E
Which one of the following choices CANNOT fill the given blank?

Key: (D)
(A) might be
asy (B) were to be (C) are going to be (D) will be

En
3. gin
The product of three integers X, Y and Z is 192. Z is equal to 4 and P is equal to the average of

7 eer
X and Y. What is the minimum possible value of P?
(A) (B) 6 (C) 9.5 (D) 8
Key: (A)
ing
Sol: Given X, Y, Z = 192
Z4 .ne
XY 
192
4
 48

Possible values of X & Y for XY=48 are


t
X  48 or 1, Y  1 or 48
X  16or 3, Y  3 or 16
X  12 or 4, Y  4 or 12
X  8 or 6, Y  6 or 8
X  Y 68
min values of P   7
2 2

4. A final examination is the _____ of a series of evaluations that a student has to go through.
(A) insinuation (B) culmination (C) desperation (D) consultation
Key: (B)

© All rights reserved by Thinkcell Learning Solutions Pvt. Ltd. No part of this booklet may be reproduced or utilized in any form without the written permission.

Downloaded From : www.EasyEngineering.net


Downloaded From : www.EasyEngineering.net

ME-GATE-2019-Afternoon

5. Are there enough seats here? There are _____ people here than I expected.
(B) least (C) many (A) most (D) more
Key: (D)
Q. No. 6 - 10 Carry Two Marks Each

6. X is an online media provider. By offering unlimited and exclusive online content at attractive
prices for a loyalty membership, X is almost forcing its customers towards its loyalty
membership. If its loyalty membership continues to grow at its current rate, within the next
eight years more households will be watching X than cable television.
Which one of the following statements can be inferred from the above paragraph?
(A) The X is cancelling accounts of non-members

ww (B) Non-members prefer to watch cable television

(C)
w.E
Most households that subscribe to X’s loyalty membership discontinue watching cable
television
Cable television operators don’t subscribe to X’s loyalty membership
Key: (C)
(D)

asy
7. En
Two pipes P and Q can fill a tank in 6 hours and 9 hours respectively, while a third pipe R can

gin
empty the tank in 12 hours. Initially, P and R are open for 4 hours. Then P is closed and Q is

eer
opened. After 6 more hours R is closed. The total time taken to fill the tank (in hours) is ____.
(A) 16.50 (B) 14.50 (C) 13.50 (D) 15.50
Key: (B)
ing
Sol: P & Q can fill tank in 6 hours and 9 hours respectively
In 1 hour
.ne
P along can fill =

Q along can fill =


1
6
1
t
9
1
R along can fill =
12
In first four hours P and R are opened
1 1 2 1 1
4  P  4R  4   4   
6 12 3 3 3
After 4 hours P is closed and Q and R opened for 6 more hours
1 1 2 1 1
6  Q  6R  6   6    
9 12 3 2 6

© All rights reserved by Thinkcell Learning Solutions Pvt. Ltd. No part of this booklet may be reproduced or utilized in any form without the written permission.

Downloaded From : www.EasyEngineering.net


Downloaded From : www.EasyEngineering.net

ME-GATE-2019-Afternoon

1 1 1
In 10 hours tank filled =  
3 6 2
1
Remaining = tank
2
9
Q will take to fill remain half tank = hour  4.5 hours
2
Total time taken = 4  6  4.5  14.5hrs.

8. Mola is a digital platform for taxis in a city. It offers three types of rides – Pool, Mini and
Prime. The table below presents the number of rides for the past four months. The platform
earns one US dollar per ride. What is the percentage share of revenue contributed by Prime to

ww the total revenues of Mola, for the entire duration?

Month

w.E Type January February March April

asy Pool

Mini
170

110
320

220
215

180
190

70

En
gin Prime 75 180 120 90

Key: (B)
(A) 16.24
eer (B) 23.97 (C) 25.86 (D) 38.74

Sol:
ing
Month
.ne
Type Jan Feb Mar Apr Total no. of rides Revenu 1$ per ride
Pool 170 320 215 190
Mini 110 220 180 70
Prime 75 180 120 90
895
580
465
Total revenue
t 895$
580$
465$
1940$

Percentage Share of revenue contributed by prime


Revenue of prime 465
= 100  100  23.97%
Total revenue 1940

9. Fiscal deficit was 4% of the GDP in 2015 and that increased to 5% in 2016. If the GDP
increased by 10% from 2015 to 2016, the percentage increase in the actual fiscal deficit is
(A) 37.50 (B) 25.00 (C) 35.70 (D) 10.00
Key: (A)
Sol: Let us take, In 2015, Fiscal deficit =X, GDP = y

© All rights reserved by Thinkcell Learning Solutions Pvt. Ltd. No part of this booklet may be reproduced or utilized in any form without the written permission.

Downloaded From : www.EasyEngineering.net


Downloaded From : www.EasyEngineering.net

ME-GATE-2019-Afternoon

Given
in 2015 x  4% of y  0.04y
in 2016 y '  increases by10%  1.10y
x '  increases to5% of y '  0.05  y '
x '  0.05  1.1y  0.055y
0.055y  0.04y
Percentage increase   100  37.5%
0.04y

10. While teaching a creative writing class in India, I was surprised at receiving stories from the
students that were all set in distant places: in the American West with cowboys and in
Manhattan penthouses with clinking ice cubes. This was, till an eminent Caribbean writer gave

ww
the writers in the once-colonised countries the confidence to see the shabby lives around them
as worthy of being “told”.

w.E
The writer of this passage is surprised by the creative writing assignments of his students
because ______.
(A)
asy
None of the students had written stories set in India
(B)
(C)
En
Some of the students had written about ice cubes and cowboys
Some of the students had written stories set in foreign places

Key: (A)
(D)
gin
None of the students had written about ice cubes and cowboys

eer
ing MECHANICAL ENGINEERING

.ne Q. No. 1 to 25 Carry One Mark Each

1.
Key: (3)
Sol: Given data values are 3, x, 2 and 4
t
If x is the mean of data 3, x, 2 and 4, then the mode is

3 x  2 4 9x
 Mean  x  Mean  x 
4 4
 4x  9  x  3x  9
x 3
 Data value sare3, 3, 2, 4
Mode = 3 [most frequently repeated observation]

2. The cold forming process in which a hardened tool is pressed against a workpiece (when there
is relative motion between the tool and the workpiece) to produce a roughened surface with a
regular pattern is
(A) Chamfering (B) Roll forming (C) Knurling (D) Strip rolling
© All rights reserved by Thinkcell Learning Solutions Pvt. Ltd. No part of this booklet may be reproduced or utilized in any form without the written permission.

Downloaded From : www.EasyEngineering.net


Downloaded From : www.EasyEngineering.net

ME-GATE-2019-Afternoon

Key: (C)
Sol:

Knurled surface

3. The figure shows an idealized plane truss. If a horizontal force of 300N is applied at point A,
then the magnitude of the force produced in member CD is _____N.

A 300N

ww B C

w.E
asy D E

En F
75 75 G

gin
eer
Key: (0)
Sol: Joint A
255
A ing 300

30
75
.ne
R AB R AC
t
300 R AC R AB
 
sin 30 sin255 sin 75
300 sin 255
R AC   579.55 N  C 
sin 30
R AB  579.55N  T 

From the figure AB and BD are collinear. So, R BC  0 and AC and CE are collinear then
RCD=0.

© All rights reserved by Thinkcell Learning Solutions Pvt. Ltd. No part of this booklet may be reproduced or utilized in any form without the written permission.

Downloaded From : www.EasyEngineering.net


Downloaded From : www.EasyEngineering.net

ME-GATE-2019-Afternoon

4. The fluidity of molten metal of cast alloys (without any addition of fluxes) increases with
increase in
(A) viscosity (B) degree of superheat
(C) surface tension (D) freezing range
Key: (B)

5. Consider a linear elastic rectangular thin sheet of metal, subjected to uniform uniaxial tensile
stress of 100 MPa along the length direction. Assume plane stress condition in the plane normal
to the thickness. The Young’s modulus E = 200 MPa and Poisson’s ratio v  0.3 are given. The
principal strains in the plane of the sheet are
(A) (0.35, –0.15) (B) (0.5, –0.5) (C) (0.5, 0.0) (D) (0.5, –0.15)

ww
Key: (D)
Sol: 1  100MPa,   0.3, E  200MPa, 2  0

1 
1
E

w.E

 2  1 


E
100
200
 0.5

0.3100
E E
asy
2  2   1  2 
200
 0.15

1 , 2    0.5,  0.15  option  D 

En
6.
gin
The state of stress at a point in a component is represented by a Mohr’s circle of radius 100
MPa centered at 200 MPa on the normal stress axis. On a plane passing through the same point,

eer
the normal stress is 260 MPa. The magnitude of the shear stress on the same plane at the same

ing
point is ____ MPa.
Key: (80)
Sol:
.ne
0
C
t D

R  100MPa, C  200MPa,   260MPa,   ?


OC  200, CD  100, OE  260, DE  ?
DE  CD 2  CE 2  100 2   260  200   100 2  60 2  80MPa
2

7. A wire of circular cross-section of diameter 1.0 mm is bent into a circular are of radius 1.0 m
by application of pure bending moments at its ends. The Young’s modulus of the material of
the wire is 100 GPa. The maximum tensile stress developed in the wire is ____ MPa.
© All rights reserved by Thinkcell Learning Solutions Pvt. Ltd. No part of this booklet may be reproduced or utilized in any form without the written permission.

Downloaded From : www.EasyEngineering.net


Downloaded From : www.EasyEngineering.net

ME-GATE-2019-Afternoon

Key: (50)
Sol: d  1mm, R  1.0mts, E  100GPa,   ?
E  1 
 y  y   0.5mm 
R  2 
2100  10  1 
3
     50MPa
1000 2

8. A two-dimensional incompressible frictionless flow field is given by u  xi  y j. If  is the


density of the fluid, the expression for pressure gradient vector at any point in the flow field is
given as

(A) 
 xi  y j  
(B)  x 2 i  y 2 j  
(C)  xi  y j  
(D)  xi  y j 
ww
Key: (D)

Sol:
w.E
Given that, flow field V  xiˆ  yiˆ
V  uiˆ  vjˆ

u du u asy
Euler’s equation of motion in ‘x’ and ‘y’ directions are
u 1 p
t
u v w
x dy z
En
 xx 
 x
v
t
v v v
 u  v  w  xy 
x x z
gin
1 p
 y

eer
Where u = x, v = -y, w = 0 xx, xy are body forces in ‘x’ and ‘y’ directions and they are equal
to zero since there is no extra forces acting on fluid.
p
x ing p
 pressure gradient in x-directoin;  pressure gradient in y-directoin
y


  .ne
  
 x    x   x    y   x    0  0 
1 p
t

0x0
x
1 p
 x

p
x
t
 y
 x
  x

   1 p
  y  x  y   y  y   0  0 
t x y  y
1 p p
y   y
 y y

Pressure gradient vector 


p ˆ p ˆ
x
i  j  xiˆ  yjˆ   xiˆ  yjˆ
y
 
So answer is option ‘D’.

© All rights reserved by Thinkcell Learning Solutions Pvt. Ltd. No part of this booklet may be reproduced or utilized in any form without the written permission.

Downloaded From : www.EasyEngineering.net


Downloaded From : www.EasyEngineering.net

ME-GATE-2019-Afternoon

9. One-dimensional steady state heat conduction takes place through a solid whose cross-sectional
area varies linearly in the direction of heat transfer. Assume there is no heat generation in the
solid and the thermal conductivity of the material is constant and independent of temperature.
The temperature distribution in the solid is
(A) Logarithmic (B) Quadratic (C) Linear (D) Exponential
Key: (A)
Sol: 1-D steady state with no heat generation
d  dT 
 k A 0
dx  dx 
dT
 kA  C1
dx
A  Cx  B  linear variation 

ww 2
  dT  
2
C1
k  Cx  B 
dx
1

T2  T1  w.E 1

C1  Cx 2  B 
K
n 
 Cx1  B 

asy
 Temperature distribution in the solid is logarithmic

10.
En
Endurance limit of a beam subjected to pure bending decreases with
(A)
(B) gin
decrease in the surface roughness and increase in the size of the beam
increase in the surface roughness and decrease in the size of the beam
(C)
eer
increase in the surface roughness and increase in the size of the beam

ing
(D) decrease in the surface roughness and decrease in the size of the beam
Key: (C)
Sol: e  k a .k b .k c . k d .  'e
.ne
 'e  endurance strength, e  corrected endurance strength

t
K a  size factor, K b  surface factor, K c  load factor, K d  temperature factor
With the increase in surface roughness, size factor etc the endurance strength drops.

11. Which one of the following modifications of the simple ideal Rankine cycle increases the
thermal efficiency and reduces the moisture content of the steam at the turbine outlet?
(A) Decreasing the condenser pressure
(B) Increasing the boiler pressure
(C) Decreasing the boiler pressure
(D) Increasing the turbine inlet temperature

© All rights reserved by Thinkcell Learning Solutions Pvt. Ltd. No part of this booklet may be reproduced or utilized in any form without the written permission.

Downloaded From : www.EasyEngineering.net


Downloaded From : www.EasyEngineering.net

ME-GATE-2019-Afternoon

Key: (D)
Sol:
T
1

4
3 2'
2

S
Increasing the inlet temperature to the turbine improves of quality of steam at the outlet of the
turbine.

ww This is the reason we use reheat cycle.

12.
w.E
A rigid triangular body, PQR, with sides of equal length of 1 unit moves on a flat plane. At the
instant shown, edge QR is parallel to the x-axis, and the body moves such that velocities of

asy
points P and R are V P and VR, in the x and y directions, respectively. The magnitude of the
angular velocity of the body is

En VP
P
y

gin x

eer VR

ing
.ne Q R

Key: (D)
(A) VR 3
t (B) VP 3 (C) 2VP (D) 2VR

Sol: Locating the I-centre A


Vp
AI  AC sin 60
CI  AC cos 60 VR
2
Vp   AI      Vp
3 60
VR   CI      2VR  Option  D  C
I

© All rights reserved by Thinkcell Learning Solutions Pvt. Ltd. No part of this booklet may be reproduced or utilized in any form without the written permission.

Downloaded From : www.EasyEngineering.net


Downloaded From : www.EasyEngineering.net

ME-GATE-2019-Afternoon

13. For a simple compressible system, v, s, p and T are specific volume, specific entropy, pressure
 v 
and temperature, respectively. As per Maxwell’s relation,   is equal to
 s  P

 T   T   s   p 
(A)   (B)    (C)   (D)  
 p  s  p p  T  P  v T
Key: (A)

 V   T 
Sol:    
 S  P  P S
dh  Tds  Vdp
dx  Mdy  Ndz
Maxwell’s relation can be obtained by using

ww  M   N 
  
 T   V 
    
 Z  y  y z  P s  S  p

14.
w.E
The most common limit gage used for inspecting the hole diameter is

Key: (B)
(A)
asy
Snap gage (B) Plug gage (C) Ring gage (D) Master gage

En
15.
gin
The directional derivative of the function f  x, y   x 2  y 2 along a line directed from (0, 0) to

eer
(1, 1), evaluated at the point x = 1, y = 1 is

Key: (C)
(A) 4 2

ing (B) 2 (C) 2 2 (D) 2

.ne
Sol: Given, Scalar valued function f  x,y   x 2  y 2

Line directed from (0, 0) to (1, 1) is 1  0  ˆi  1  0  ˆj


t
∴ Directional derivative of f  x,y  at (1, 1) in the direction of ˆi  ˆj is given by

 ˆi  ˆj
i.e.,  DD of f 1, 1 in  
ˆi  ˆj direction   f  .
1,1 ˆi  ˆj
... 1

 2
 f  ˆi
x
 x  y2   ˆj y  x 2  y2 
 f  ˆi  2x   ˆj  2y    f 1,1  2iˆ  2jˆ

 ˆi  ˆj 
∴ From (1), D.D of f = 2iˆ  2ˆj .   ˆi  ˆj
22
1 1
2 2

4
2
2 2

© All rights reserved by Thinkcell Learning Solutions Pvt. Ltd. No part of this booklet may be reproduced or utilized in any form without the written permission.

10

Downloaded From : www.EasyEngineering.net


Downloaded From : www.EasyEngineering.net

ME-GATE-2019-Afternoon

16. Water enters a circular pipe of length L = 5.0 m and diameter D = 0.20 m with Reynolds
number ReD  500. The velocity profile at the inlet of the pipe is uniform while it is parabolic at
the exit. The Reynolds number at the exit of the pipe is _______.
Key: (500)
Sol:

D  0.20m

L  5.0m

ww Given that Reynold’s number(ReD) = 500


We know that Reynold’s number for flow through pipes is,

Re D 
 w.E
Vavg D

  density of fluid, Vavg  average velocity of fluid

asy
D  diameter of pipe,   dynamic vis cosity of fluid

Vavg 
4Q
D 2
En
, where Q  discharge


 Re D  
 4Q 
2 
D 
D 
  gin
 4Q 

D 
 
eer
So Reynold’s number is a function of density, discharge, diameter and dynamic viscosity.

ing
Since density, diameter and dynamic viscosities are same at entrance and exit and discharge is

.ne
same hence the Reynold’s number is also same at entrance and exit.

17. In matrix equation [A]{X} = {R},


4 8 4  2  t 32 
 A   8 16 4 ,X  1  and R  16 .
   

 4 4 15  4  64 
   
One of the eigenvalues of matrix [A] is
(A) 8 (B) 16 (C) 15 (D) 4
Key: (B)
4 8 4  2  32 
     
Sol: Given  A   8 16 4  , x  1  and R  16 
   
 4 4 15  4  64 

© All rights reserved by Thinkcell Learning Solutions Pvt. Ltd. No part of this booklet may be reproduced or utilized in any form without the written permission.

11

Downloaded From : www.EasyEngineering.net


Downloaded From : www.EasyEngineering.net

ME-GATE-2019-Afternoon

4 8 4 2  32  2
     
Clearly,  A  x  R  8 16 4 

1   16   16 1 
 4 4 15       
4  64  4 
∴ The above equation is of the form AX  X; where '  ' isan eigen value
∴ One of the Eigen values of matrix must be '16'.

18. Sphere 1 with a diameter of 0.1 m is completely enclosed by another sphere 2 of diameter 0.4
m. The view factor F12 is
(A) 0.0625 (B) 0.5 (C) 1.0 (D) 0.25

ww
Key: (C)

Sol: d1  0.1 m
2
d 2  0.4 m
F11  0 w.E 1

Summation rule

asy
F11  F1 2  1
 F11  0 
 F1 2  1
En
19.
gin
In an electrical discharge machining process, the breakdown voltage across inter electrode gap
(IEG) is 200V and the capacitance of the RC circuit is 50F. The energy (in J) released per
spark across the IEG is ____eer
Key: (1) ing
Sol: IEG = 200
.ne
C  50F
Energy  e   ?
1
E  CV 2
t
2
E 1J

20. A thin vertical flat plate of height L, and infinite width perpendicular to the plane of the figure,
is losing heat to the surroundings by natural convection. The temperature of the plate and the
surroundings, and the properties of the surrounding fluid, are constant. The relationship
between the average Nusselt and Rayleigh numbers is given as Nu  KRa1 4 , where K is a
constant. The length scales for Nusselt and Rayleigh numbers are the height of the plate. The
height of the plate is increased to 16L keeping all other factors constant

© All rights reserved by Thinkcell Learning Solutions Pvt. Ltd. No part of this booklet may be reproduced or utilized in any form without the written permission.

12

Downloaded From : www.EasyEngineering.net


Downloaded From : www.EasyEngineering.net

ME-GATE-2019-Afternoon

Vertical Plate

L
g  9.8 m s 2

If the average heat transfer coefficient for the first plate is h1 and that for the second plate is h 2 ,

ww
Key: (2)
the value of the ratio h1 h 2 is___________.

Sol: w.E
Nu  K  R a 
 N u   Ra 
14

14
Rayleigh number R a  G r Pr

asy
 Nu   Gr.Pr 
14
As Prandtl number can be considered as a property. It remains constant

En
gin
 N u   Gr 

hL  gTL3 
14

eer 14

 
K  v2 

ing  hL  L3 4

h
L3 4
L
3
1

.ne
 h  L4  h  L1 4

h L 
 2  1 
h1  L 2 

h L 
14

14
t 14
 16L 
  24 
14 h1
and 1   2     2
h 2  L1   L  h2

21. A spur gear has pitch circle diameter D and number of teeth T. The circular pitch of the gear is
D 2 D D T
(A) (B) (C) (D)
T T T D
Key: (C)
D
Sol: Circular pitch = m 
T

© All rights reserved by Thinkcell Learning Solutions Pvt. Ltd. No part of this booklet may be reproduced or utilized in any form without the written permission.

13

Downloaded From : www.EasyEngineering.net


Downloaded From : www.EasyEngineering.net

ME-GATE-2019-Afternoon

22. An analytic function f(z) of complex variable z = x + iy may be written as f(z) = u(x, y) + iv(x,
y). Then u(x, y) and v(x, y) must satisfy
u v u v u v u v
(A)  and  (B)   and 
x y y x x y y x
u v u v u v u v
(C)   and  (D)  and 
x y y x x y y x
Key: (D)
Sol: Given an analytic function f  z   u  x,y   iv  x,y 

If f  z   u  iv is analytic then u, v must satisfy Cauchy –Riemann equations i.e,

u v u v
 & 

ww x y y x

23.
(A) w.E
Hardenability of steel is a measure of
the ability to retain its hardness when it is heated to elevated temperatures
(B)
(C) asy
the ability to harden when it is cold worked
the depth to which required hardening is obtained when it is austenitized and then

(D)
quenched
En
the maximum hardness that can be obtained when it is austenitized and then quenched
Key: (C)
gin
24. eer
The transformation matrix for mirroring a point in x-y plane about the line y = x is given by

(A)
0 1 
1 0 
  ing  0 1
(B)  
 1 0 
 1 0 
(C) 
 0 1

1 0 
(D)  
0 1
Key: (A)
.ne
t dy
25. The differential equation  4y  5 is valid in the domain 0    1 with y  0   2.25.
dx
The solution of the differential equation is
(A) y  e4x  1.25 (B) y  e4x  1.25

(C) y  e4x  5 (D) y  e4x  5


Key: (A)
dy
Sol: Given differential equation  4y  5, 0  x  1 with y  0   2.25
dx
Clearly the above differential equation is linear, where P=4, Q=5

 I.Fe
4dx
 e4x

© All rights reserved by Thinkcell Learning Solutions Pvt. Ltd. No part of this booklet may be reproduced or utilized in any form without the written permission.

14

Downloaded From : www.EasyEngineering.net


Downloaded From : www.EasyEngineering.net

ME-GATE-2019-Afternoon

Solution of equation (1) is


e 4x
y  e 4x    5.e 4x .dx  C  ye 4x  5 C ...  2 
4
Given y=2.25 at x = 0
5
(2)  2.25 1   C  C  2.25  1.25  1  C  1
4
∴ From (2), the required solution is
5 4x
ye 4x  e 1
4
5
y   e 4x
4
 y  e 4x  1.25

ww
w.E Q. No. 26 to 55 Carry Two Marks Each

26.
asy
In an orthogonal machining with a single point cutting tool of rake angle 10°, the uncut chip
thickness and the chip thickness are 0.125 mm and 0.22 mm, respectively. Using Merchant’s

En
first solution for the condition of minimum cutting force, the coefficient of friction at the chip-

gin
tool interface is _____ (round off to two decimal places).

Key: (0.74)
eer
Sol:

tan  
cos 10

ing
  10, t1  0.125mm t 2  0.22mm
0.98
0.22
0.125 .ne
 sin10 1.76  0.1736

tan   0.6177    31.706


For minimum cutting force

t
2       90
2
2  31.706    10  90
  36.59
tan     0.7423    0.74

© All rights reserved by Thinkcell Learning Solutions Pvt. Ltd. No part of this booklet may be reproduced or utilized in any form without the written permission.

15

Downloaded From : www.EasyEngineering.net


Downloaded From : www.EasyEngineering.net

ME-GATE-2019-Afternoon

27. Given a vector u 


1
3
 
 y3 i  x 3 j  z3 k and n as the unit normal vector to the surface of the

hemisphere  x 2  y2  z 2  1; z  0  , the value of integral     u  .n ds evaluated on the curved

surface of the hemisphere S is


  
(A) (B) (C)  (D) 
2 3 2
Key: (A)
Sol: Method –I:
x 2  y2 z 2 1
Given u 
1
3

 y3ˆi  x 3ˆi  z 3 kˆ and 
n̂  Unit normal vector to the surface of the hemi-sphere

ww x 2  y2  z 2  1; z  0.

    u  . nds
w.E
Using Stoke's theorem, we have

ˆ   u . dr     dx  dy  dz 
 y3 x3 z3 
0,1
C
s

 
y 3
x
dx  dy asy  3
3
c
3 3 


c

z  0 on 'c'
c
3

M
3
En 
N

1,0
R r

1,0
gin
   x 2  y 2  dx dy  Using Green 's theorem  
R

eer
2 1
  r
2
r dr d  Using polar co  ordination x 2  y 2  r 2 , J  r 
 0 r  0
 0,1
2
1  2
 r4  1 2 
ing
    r 3dr  d     d   0 
1

 0 

.ne
 0  r  0  4 0 4 2

Method –II:

Given,  
u 
1
3
ˆi
t
 y3ˆi  x 3ˆj  z 3 kˆ

ˆj kˆ
  
u   kˆ  x 2  y 2 
x y z
y3 x3 z3

3 3 3

 n̂ 


 xiˆ  yjˆ  zkˆ    ˆi  2x   ˆj  2y   kˆ  2z  
  4x 2  4y 2  4z 2  4 1  2
ˆ   z  x
    u  .nds
2
 y 2  ds ... 1

© All rights reserved by Thinkcell Learning Solutions Pvt. Ltd. No part of this booklet may be reproduced or utilized in any form without the written permission.

16

Downloaded From : www.EasyEngineering.net


Downloaded From : www.EasyEngineering.net

ME-GATE-2019-Afternoon


Let us parameterize ‘S’ in spherical coordinates, with colatitude 0    and longitude
2
0    2 ;
x  sin  cos  ; y sin  sin ; z  cos ; ds  sin  dd
/ 2 2
     u  .nds
ˆ    cos  sin
2
 cos 2   sin 2  sin 2   sin  dd
 0  0
/ 2 2
   cos . sin  dd   sin 2   cos 2   1
3

 0  0
/ 2 / 2
  3  1 
cos  sin 3   0 .d   2 
2
   cos  sin . d  2 
3
;
 0  0  4 2 
 / 2
 m  1 m  3)....    n  1 n  3 ..... 
  sin n x. cos n xdx   

ww 


0  m  n  m  n  2  ......
k

Where k   / 2; if n & m both are even; k  1; otherwise.

w.E  2 
2
4 2


asy
    u  .nˆ ds  2

En
The derivative of f  x   cos  x  can be estimated using the approximation

gin
28.

f x  h  f x  h
f ' x  

eer
2h
 Exact value  Aprroximate value 
. The percentage error is calculated as


 Exact value
ing
  100. The percentage error in the derivative of

.ne
f  x  at x   6 radian, choosing h  0.1 radian, is

Key: (B)
(A) >5%

Sol: Given f  x   cos x


t (B) > 0.1% and <1% (C) <0.1% (D) >1 % and <5%

 f '  x    sin x
   1
 f '  x  x  /6   sin       0.5 
6 2
∴ Exact value of the derivative = 0.5 ... 1

Approximate value of derivative

© All rights reserved by Thinkcell Learning Solutions Pvt. Ltd. No part of this booklet may be reproduced or utilized in any form without the written permission.

17

Downloaded From : www.EasyEngineering.net


Downloaded From : www.EasyEngineering.net

ME-GATE-2019-Afternoon

f x  h  f x  h cos  x  h   cos  x  h 
f ' x    f ' x  
2h 2h
   
cos   0.1  cos   0.1
 6  6  ; since h  0.1
 f '  
6 2  0.1
         
cos  6  .cos  0.1  sin  6  .sin  0.1   cos  6  cos  0.1  sin  6  sin  0.1 
         
 
2  0.1

2sin   .sin  0.1
6   0.5 sin  0.1
    0.4992
2  0.1  0.1

 f '     0.4992  Approximate value  ... 2 

ww 6

 0.5   0.4992 
Percentage error     100  0.16% ie.  0.1% and  1%
w.E  0.5 

29.
asy
Two masses A and B having mass m a and m b , respectively, lying in the plane of the figure

En
shown, are rigidly attached to a shaft which revolves about an axis through O perpendicular to
the plane of the figure. The radii of rotation of the masses m a and m b are ra and rb , respectively.

gin
The angle between lines OA and OB is 90°. If ma = 10 kg, mb  20 kg ra  200 mm and

eer
rb  400mm, then the balance mass to be placed at a radius of 200 mm is ____ kg (round off to
two decimal places)

ing mb B

.ne
t rb

90
ra ma
A
O

Key: (41.231)
Sol: Method-1:
ma  10kg, m b  20kg, ra  200mm,  0.2mts,
rb  400mm  0.4mts, rr  200mm  0.2mts.

 mr resultant   mr a   mr b 10  0.2    20  0.4 


2 2 2 2

8.24
m resultant   41.231 kg
0.2

© All rights reserved by Thinkcell Learning Solutions Pvt. Ltd. No part of this booklet may be reproduced or utilized in any form without the written permission.

18

Downloaded From : www.EasyEngineering.net


Downloaded From : www.EasyEngineering.net

ME-GATE-2019-Afternoon

Method-2:
m a  10kg ra  200mm B
mb
m b  20kg rb  400mm
at r  200 mm m  ? for balancing
Fx  0 rb
m a ra  m b rb cos  90   mr cos   0 90
ra ma
m a ra  0  mr cos   0
A
Fy  0 O
m b rb  mr sin   0
sin  m b rb

cos  m a ra
20  0.4

ww tan  
10  0.2
1
  tan 4    75.9640
mr
w.E
m  b b  41.23kg
r sin 

30. asy
A through hole is drilled in an aluminum alloy plate of 15 mm thickness with a drill bit of

En
diameter 10 mm, at a feed of 0.25 mm/rev and a spindle speed of 1200 rpm. If the specific

gin
energy required for cutting this material is 0.7 N  m mm3 , the power required for drilling is
___ W (round off to two decimal places).
Key: (274.889)
eer
Sol: t  15mm
d  10mm ing
f  0.25mm rev
N  1200rpm .ne
t
Specific energy required = 0.7 Nm mm 2
Power = ?
 2 N
Power  Df   SER   274.889 W
4 60

31. A horizontal cantilever beam of circular cross-section, length 1.0 m and flexural rigidity
EI  200 N  m 2 is subjected to an applied moment MA  1.0 N-m at the free end as shown in
the figure.
MA

1.0m
The magnitude of the vertical deflection of the free end is ____ mm (round off to one decimal
place)

© All rights reserved by Thinkcell Learning Solutions Pvt. Ltd. No part of this booklet may be reproduced or utilized in any form without the written permission.

19

Downloaded From : www.EasyEngineering.net


Downloaded From : www.EasyEngineering.net

ME-GATE-2019-Afternoon

Key: (2.5)
Sol:
A
MA
L

EI  200N  m 2 , L  1.0mts, M A  1.0N  mts


moment of Area underB.M.D ML L ML2
   
EI EI 2 2EI

ww 
1 12
2  200
 2.5mm

32. w.E
Consider two concentric circular cylinders of different materials M and N in contact with each

asy
other at r = b, as shown below. The interface at r = b is frictionless. The composite cylinder
system is subjected to internal pressure P. Let  u M
r ,u   and  rr,   denote the radial and
M M M

En
tangential displacement and stress components, respectively, in material M. Similarly
u N
r
gin
, u N  and  rrN , 
N
 denote the radial and tangential displacement and stress components,

eer
respectively, in material N. The boundary condition that need to be satisfied at the frictionless
interface between the two cylinders are:

ing N
r

.ne M

t b

r  u r and rr  rr and u   u  and   


uM N M N M N M N
(A)

(B) u M  u N and 


m
 
N
only

(C) M
rr  rr and    only
N M N

r  u r and rr  rr only


uM N M N
(D)

© All rights reserved by Thinkcell Learning Solutions Pvt. Ltd. No part of this booklet may be reproduced or utilized in any form without the written permission.

20

Downloaded From : www.EasyEngineering.net


Downloaded From : www.EasyEngineering.net

ME-GATE-2019-Afternoon

Key: (D)
r
Sol:

N
M

At the radial location r=b, there will be a contact pressure and element at interface of cylinder
M and N the radial displacement will be same

r  u r , RR  RR
uM N M N

ww u M  u N , 
M
 
N

33. w.E
A slender uniform rigid bar of mass m is hinged at O and supported by two springs, with

asy
stiffnesses 3k and k, and a damper with damping coefficient c, as shown in the figure. For the
system to be critically damped, the ratio c / km should be

En
gin
3k eer c k

ing L 2

.ne L 4
O

(A) 4 7
t (B) 4
L

(C) 2 7 (D) 2
Key: (A)
2
mL2 mL2  L  7mL
2
Sol: IC  , I0  IC  mr 2   m  
12 12 4 48
Taking moment about 0, Mo  0

 3L  3L   L  L  L  L
I0   k      3k      C       0
 4  4   4  4  4 4
2 2 2
7mL CL 12kL
  0
48 16 16
7m C 12k
   0
48 16 16

© All rights reserved by Thinkcell Learning Solutions Pvt. Ltd. No part of this booklet may be reproduced or utilized in any form without the written permission.

21

Downloaded From : www.EasyEngineering.net


Downloaded From : www.EasyEngineering.net

ME-GATE-2019-Afternoon

Critical damping, the roots of above differential equation are real and equal so we can use
b2  4ac  0
7m C 12k
a , b , c
48 16 16
(Critical damping)C  16  7mk
2

C  4 7mk   C
mk

4 7

34. An air standard Otto cycle has thermal efficiency of 0.5 and the mean effective pressure of the
cycle is 1000 kPa. For air, assume specific heat ratio   1.4 and specific gas constant
R = 0.287 kJ/kg.K. If the pressure and temperature at the beginning of the compression stroke

ww are 100 kPa and 300 K, respectively, then the specific net work output of the cycle is ___ kJ/kg
(round off to two decimal places).
Key: (708.6)

Sol: OHO  0.5 w.E


  1.4 asy
Pm  1000KPa P1  100KPa
T1  300K
R  0.287 KJ kgk
En
 1
1
 rc 
1
gin
0.5  1 
1
 rc  eer
1.4 1

1
 rc 
 0.5  rc   ing
 1  0.4
  5.65
1

 0.5 
.ne
0.4

P1V1  mRT
V1 RT1
m

P1
0.287  300
t
V1   0.861 m 3 kg
100
V1
 rc 
V2
V1 0.861
 V2  
rc 5.65
 V2  0.1524 m3 kg swept volume
Vs  V1  V2
w.D
Pm   W.D  Pm   V1  V2 
V1  V2
 1000   0.681  0.1524   708.6KJ kg

© All rights reserved by Thinkcell Learning Solutions Pvt. Ltd. No part of this booklet may be reproduced or utilized in any form without the written permission.

22

Downloaded From : www.EasyEngineering.net


Downloaded From : www.EasyEngineering.net

ME-GATE-2019-Afternoon

35. An idealized centrifugal pump (blade outer radius of 50mm) consumes 2kW power while
running at 3000 rpm. The entry of the liquid into the pump is axial and exit from the pump is
radial with respect to impeller. If the losses are neglected, then the mass flow rate of the liquid
through the pump is ________kg/s (round off to two decimal places).
Key: (8.1057)
Sol: From the given data, since the inlet flow is axial, whirl component at inlet is zero VW1  0

And also because of radial discharge whirl component exit is VW2  u 2

Power input to the pump is P=2kW and N=3000rpm and d2  2r2  100mm  0.1mm

  0.1 3000
 VW2  u 2   5m/sec
60

 
ww Power input to the pump 'P' = m VW2 u 2  VW1 u1

where m  mass flow rate of liquid in kg/sec

w.E
2000  m((5)(5)) ( Vw1  0)  m  8.1057kg / sec

36.
asy
A ball of mass 3 kg moving with a velocity of 4 m/s undergoes a perfectly-clastic direct-central
impact with a stationary ball of mass m. After the impact is over, the kinetic energy of the 3 kg

En
ball is 6J. The possible value (s) of m is/are

Key: (B)
(A) 6 kg only
gin (B) 1kg, 9 kg (C) 1 kg, 6 kg (D) 1 kg only

Sol:
eer
m  3kg, V1  4m / sec, m2  m u 2  0 , kE1  65, e  1
Conservation of Momentum
m1u1  m 2 u 2  m1v1  m 2 v 2ing
 3  4    m2  0   3v1  mv2
.ne ... 1
3v1  mv 2  12

e
v 2  v1
u1  u 2
 1  v 2  v1  4 t ...  2 

Conservation of Energy
 KE1 i   KE 2 i   KE1 f   KE 2 f
1 2 1
  3  4   0  6  mv 2  mv 2  36
2 2

2  2
From (1) and (2)
3  v 2  4   mv 2  12
 24 
3v 2  mv 2  24  v 2   
 3 m 
2
 24 
m   36
 3 m 
© All rights reserved by Thinkcell Learning Solutions Pvt. Ltd. No part of this booklet may be reproduced or utilized in any form without the written permission.

23

Downloaded From : www.EasyEngineering.net


Downloaded From : www.EasyEngineering.net

ME-GATE-2019-Afternoon

 576m   9  m2  6m  36
36m2  360m  324  0  m2  10m  9  0
 m  9  m  1  0  m  1,9kg

37. The annual demand of valves per year in a company is 10,000 units. The current order quantity
is 400 valves per order. The holding cost is Rs. 24 per valve per year and the ordering cost is
Rs. 400 per order. If the current order quantity is changed to Economic order quantity, then the
saving in the total cost of inventory per year will be Rs____ (round of to two decimal places).
Key: (943.60)
Sol: D  1000 units
Q  400 Values / order

ww C n  24 rs value / year
C0  400 Rs order

EOQ  w.E 2DCo


Cn

 asy
2  10000  400
24
Q  EOQ  577.35 En
D gin
TIC  Q   2DCo Cn  2 10000  400  24  13856
Q
TIC  Q  
Q
eer
C0  Cn
2


10000
400
 400 
400
2
24
ing
 10000  4800  14800

.ne
saving  14800  13856.41  Rs.943.60

38. t
Water flowing at the rate of 1 kg/s through a system is heated using an electric heater such that
the specific enthalpy of the water increases by 2.50 kJ/kg and the specific entropy increases by
0.007 kJ/kg K. The power input to the electric heater is 2.50 kW. There is no other work or heat
interaction between the system and the surroundings, Assuming an ambient temperature of 300
K, the irreversibility rate of the system is _______kW (round off to two decimal places).
Key: (2.1)

Sol: m  1kg s
h 2  h1  2.5 kJ kg , S2  S1  0.007 kj kg k
w  2.5kW, T0  300K
irreversibility  I   T0  S   300  0.007  2.1 kJ kg
 I  m  2.1  1  2.1  2.1 kW

© All rights reserved by Thinkcell Learning Solutions Pvt. Ltd. No part of this booklet may be reproduced or utilized in any form without the written permission.

24

Downloaded From : www.EasyEngineering.net


Downloaded From : www.EasyEngineering.net

ME-GATE-2019-Afternoon

39. The activities of a project, their duration and the precedence relationship are given in the table.
For example, in a precedence relationship “X <Y, Z” means that X is predecessor of activities
Y and Z. The time to complete the activities along the critical path is _______weeks,

Activity Duration (weeks) Precedence Relationship


A 5 A<B, C,D
B 7 B<E,F,G
C 10 C<I
D 6 D <G
E 3 E<H
F 9 F<I

ww G
H
7
4
G<I
H<I
……

w.E
I 2

(A) 21 (B) 23 (C) 17 (D) 25


Key: (B)
asy C 0
Sol:

En
gin
A  5 B7
E  3 H  4
I  2

eer D 6 F 9

ing G 7

ACI  5  10  2  17days
.ne
ABEHI  5  7  3  4  2  21days
ABFI  5  7  9  2  23 days
ABGI  5  7  7  2  21days
t
Time required = 23 days

40. A differential equation is given as


d2 y dy
x2 2
 2x  2y  4.
dx dx
The solution of the differential equation in terms of arbitrary constants C1 and C2 is
C1
(A) y  C1x 2  C2 x  4 (B) y   C2 x  4
x2
C1
(C) y  C2 x  2 (D) y  C1x 2  C2 x  2
x2

© All rights reserved by Thinkcell Learning Solutions Pvt. Ltd. No part of this booklet may be reproduced or utilized in any form without the written permission.

25

Downloaded From : www.EasyEngineering.net


Downloaded From : www.EasyEngineering.net

ME-GATE-2019-Afternoon

Key: (D)
Sol: Given differential equation
d2 y dy
x2 2
 2x  2y  4 ...Cauchy Euler LinearD.E
dx dx
  x D  2xD  2  y  4
2 2
... 1
d
ConsiderxD  ; x 2 D 2      1 where   & x  ez
dz
 From 1 , 
    1  2  2  y  4
 2    2  2  y  4
 2  3  2  y  4 ...  2 
A.E m 2  3m  2  0   m  2  m  1  0

ww  m  2, 1  real &distinct
 y C  C1e 2z  C 2 e z ...  3

 yp 
1
w.E
  3  2
2
4.e0z

1
yp  4  yp  2 asy
substitute   0; then

2
En
 Complete solution is y  y C  y p

gin
 y  C1e2z  C2 ez  2  y  C1x 2  C2 x  2  x  e z 

eer
41.
ing
Water flows through two different pipes A and B of the same circular cross-section but at

.ne
different flow rates. The length of pipe A is 1.0 m and that of pipe B is 2.0 m. The flow in both
the pipes is laminar and fully developed. If the frictional head loss across the length of the pipes

Key: (0.5)
t
is same, the ratio of volume flow rate QB QA is (round off to two decimal places).

Sol: Given that, diameter of pipe ‘A’ (dA) = diameter of pipe ‘B’ (dB)
Length of pipe ‘A’ is  A   1m
Length of pipe ‘B’ is  B   2m
32v
Frictional head loss in laminar flow is h f 
gd 2

It is mentioned in the question as  h f A   h f B

© All rights reserved by Thinkcell Learning Solutions Pvt. Ltd. No part of this booklet may be reproduced or utilized in any form without the written permission.

26

Downloaded From : www.EasyEngineering.net


Downloaded From : www.EasyEngineering.net

ME-GATE-2019-Afternoon

32VA 32VB
 A
 B
 dA  dB 
gd 2A gd B2
VA A  VB B

VA V 1
VA 1  VB  2   2 B 
VB VA 2
 2
Flow rate pipe ‘A’ is QA  VA dA
4
 2
Flow rate in pipe ‘B’ is Q B  VB dB
4
 2
 VB  
dB 
QB
  4   VB  1  0.5

QA
 VA   d 2A  VA 2
ww 4 

42.
w.E
A prismatic, straight, elastic, cantilever beam is subjected to a linearly distributed transverse
load as shown below. If the beam length is L, Young’s modulus E, and are moment of inertia I,

asy
the magnitude of the maximum deflection is

En q
y

gin
eer L
x

(A)
qL4 ing (B)
qL4
(C)
qL4
(D)
qL4

Key: (D)
10EI
.ne 15EI 60EI 30EI

Sol:
q t y

L
x
B.M.D L

qL2
6 3rd degree curve

© All rights reserved by Thinkcell Learning Solutions Pvt. Ltd. No part of this booklet may be reproduced or utilized in any form without the written permission.

27

Downloaded From : www.EasyEngineering.net


Downloaded From : www.EasyEngineering.net

ME-GATE-2019-Afternoon

1 qx x qx 3
Mxx  x  
2 L 3 6L
1 L
M A   q  L  (at A, x  L)
2 3
Moment of area under BMD about free end
Deflection 
EI
 1 
 Area of spandral  n  1 bh 
  n 3
centriod of spandral about apex  x   n  1 b 
 n  2 
Ax  1   qL2   3 1  L qL4
    . L .   
EI  3  1   6   3  2  EI 30EI

ww
43. A four bar mechanism is shown in the figure. The link numbers are mentioned near the links,

links are: w.E


input link 2 is rotating anticlockwise with a constant angular speed 2 . Length of different

asy
O 2 O 4  O 2 A  L,
AB  Q 4 B  2L A
3

En 2 4

gin 2
O2
4

O4

eer
The magnitude of the angular speed of the output link 4 is 4 at the instant when link 2 makes

ing
an angle of 90° with O2 O4 as shown. The ratio
4
is ______(round off to two decimal

places). .ne 2

Key: (0.788)
t B
Sol: O2 A  O2O4  L I 34
60
AB  O 4 B  2L

A I 23
60
75
45 2L

60 75

15 90
12 45 I 41

I 24 I12 O4
© All rights reserved by Thinkcell Learning Solutions Pvt. Ltd. No part of this booklet may be reproduced or utilized in any form without the written permission.

28

Downloaded From : www.EasyEngineering.net


Downloaded From : www.EasyEngineering.net

ME-GATE-2019-Afternoon

I 24 I12 2  I 24 I 414
I 24 I12  L tan 75  from  e I 24 I12 I 23 
I 24 I 41  L  L tan 75
4 I 24 I12 L tan 75
   0.788
2 I24 I 41 L  Ltan 750

44. A gas tungsten are welding operation is performed using a current of 250 A and an arc voltage
of 20 V at a welding speed of 5 mm/s. Assuming that the arc efficiency is 70%the net heat input
per unit length of the weld will be______ kJ/mm (round off to one decimal place).
Key: (0.7)
Sol: Given that, V = 20V, I = 250A

ww Welding speed = 5 mm/sec


Arc efficiency = 70% = 0.7

w.E
Power given to the welding operation  VI   20  250   5000 watts  5000 Joules sec

Since arc efficiency is 70%, net heat input will be  0.7  5000  3500 Joules sec

asy
Net heat input per unit length of the weld 
net heat input

3500  J / sec  En welding speed


50  mm / sec 
gin 700 J mm  0.7 kJ mm

eer
Three sets of parallel plate LM, NR and PQ are given in Figures 1, 2 and 3. The view factor FIJ

ing
45.
is defined as the fraction of radiation leaving plate I that is intercepted by plate J. Assume that

.ne
the values of FLM and FNR are 0.8 and 0.4 respectively. The value of FPQ (round off to one
decimal place) is________.

1m
t 1m 1m
N P
1m 1m 1m
M
R
Q
1m 1m 1m 1m 1m 1m

Figure1 Figure 2 Figure 3

Key: (0.6)

Sol: From the figures we can say that view factor between o parallel 1m plates  0.4  FNR
View factor between one 1m plate to another two equally inclined

© All rights reserved by Thinkcell Learning Solutions Pvt. Ltd. No part of this booklet may be reproduced or utilized in any form without the written permission.

29

Downloaded From : www.EasyEngineering.net


Downloaded From : www.EasyEngineering.net

ME-GATE-2019-Afternoon

plates  FLM  FNR  0.8  0.4  0.4


View factor between one 1m 4 plate to and this equally inclined.
0.4
Plate   0.2
2
 For figure ‘3’ view factor form one 1m plate to another parallel plate and another equally
inclined plate is
FPQ  0.4  0.2  0.6

46. A uniform disc with radius r and a mass of m kg is mounted centrally on a horizontal axle of
negligible mass and length of 1.5r. P

ww
w.E 

P asy Q

RP En RQ

gin g

eer
ing
.ne
The disc spins counter-clockwise about the axle with angular speed , when viewed from the

t
right-hand side bearing Q, a The axle processes about a vertical axis at p   / 10 in the

clockwise direction when viewed from above. Let R p and R Q (positive upwards) be the
resultant reaction forces due to the mass and the gyroscopic effect, at bearings P and Q,
respectively. Assuming 2 r  300m / s2 and g  10m s 2 , the ratio of the larger to the smaller
bearing reaction force (considering appropriate signs) is________
Key: (-3)
Sol: RP

15mr
RQ
© All rights reserved by Thinkcell Learning Solutions Pvt. Ltd. No part of this booklet may be reproduced or utilized in any form without the written permission.

30

Downloaded From : www.EasyEngineering.net


Downloaded From : www.EasyEngineering.net

ME-GATE-2019-Afternoon


p  , 2 r  300m/sec 2 , g  10m/sec 2 ,  1.5r
10
Gyroscope couple,C  Ip
 mr 2 
C   .p
 2 
 mr  r mr 300
2
  .    15mr
 2  10 2 10

mg 10m
Reaction due to weight, R P  R Q    5m (due to symmetry)
2 2
15MR
Reaction due to gyroscopic couple, R Q   10m, R P  10m
1.5R
Now net reaction, R Q  10m  5m  15m

ww R P  5m  10m  5m

w.E RQ
RP

15m
5m
 3

47.
asy
The figure shows a heat engine (HE) working between two reservoirs. The amount of heat

En
 Q 2  rejected by the heat engine is drawn by a heat pump (HP).
entire work output (W) of the heat engine. If temperatures, T1  T3  T2 , then the relation
The heat pump receives the

gin
between the efficiency   of the heat engine and the coefficient and the coefficient of

eer
performance (COP) of the heat pump is

ing
.ne
t T1 T3

Q3
Q1
W
HE HP

Q2 Q2

T2

(A) COP   (B) COP  1  1 (C) COP  1 (D) COP  1  
Key: (C)

© All rights reserved by Thinkcell Learning Solutions Pvt. Ltd. No part of this booklet may be reproduced or utilized in any form without the written permission.

31

Downloaded From : www.EasyEngineering.net


Downloaded From : www.EasyEngineering.net

ME-GATE-2019-Afternoon

Sol:

T1 T3

Q1 Q3

W
HE HP

Q2 Q2

T2

T1  T3  T2  Given 

ww COPH.P 
Q3
W


W w.E
W  Q1  Q 2  Q 2  Q1  W
Q
or 1  2  W   Q1
Q1
asy
Q1
Also Q3  Q 2  W

COPH.P 
Q2  W
W En
Q
 2 1  1
W
Q W
W
Q
gin
1  1 11
W
 COP  1

eer
48.
ing
The aerodynamic drag on a sports car depends on its shape. The car has a drag coefficient of

.ne
0.1 with the windows and the roof closed. With the windows and the roof open, the drag
coefficient becomes 0.8. The car travels at 44 km/h with the windows and roof closed. For the

t
same amount of power needed to overcome the aerodynamic drag, the speed of the car with the
windows and roof open (round off to two decimal places), is ____ km/h. (The density of air and
the frontal area may be assumed to be constant.)
Key: (22)
Sol: Co-efficient of drag with doors and roof closed  C d C  0.1

Co-efficient of drag with doors and roof open  Cd O  0.8

Velocity of car with doors and roof closed VC  44km/sec

Velocity of car with doors and roof open VO  ?


Power needed in overcoming aerodynamic drag with doors and roof closed =
Power needed in overcoming aerodynamic with doors and roof open

© All rights reserved by Thinkcell Learning Solutions Pvt. Ltd. No part of this booklet may be reproduced or utilized in any form without the written permission.

32

Downloaded From : www.EasyEngineering.net


Downloaded From : www.EasyEngineering.net

ME-GATE-2019-Afternoon

 Drag force  velocity closed   Drag force  velocity open


1  1 
   Cd C  eA  VC2  VC     Cd O  eA  VO2  VO 
2  2 
 0.1  44  0.8  V0
3 3

V0  22 km / hr

49. The binary phase diagram of metals P and Q is shown in the figure. An alloy X containing 60%
P and 40% Q (by weight) is cooled from liquid to solid state. The fractions of solid and liquid
(in weight percent) at 1250°C, respectively, will be

Liquidus

ww
1500 X
Liquid

w.E Temperature
 C 
1250
Solidus

asy 1000
Solid

En
gin 100% P 20 32 40 60 68 80 100%Q

Concentration of Q

eer  in weight % 

ing
(A) 22.2% and 77.8%
.ne (B) 68.0% and 32.0%

Key: (A)
(C) 32.0% and 68.0%

co  c 40  32
t (D) 77.8% and 22.2%

Sol: ms    22.2%
cs  c 68  32
m  77.8%

© All rights reserved by Thinkcell Learning Solutions Pvt. Ltd. No part of this booklet may be reproduced or utilized in any form without the written permission.

33

Downloaded From : www.EasyEngineering.net


Downloaded From : www.EasyEngineering.net

ME-GATE-2019-Afternoon

50. The crank of a slider-crank mechanism rotates counter clockwise (CCW) with a constant
angular velocity , as sown. Assume the length of the crank to be r.


1

2 1

ww Using exact analysis. The acceleration of the slider in the y-direction, at the instant shown,
where the crank is parallel to x-axis, is given by

Key: (C)
(A)
w.E
22 r (B) 22 r (C) 2 r (D) 2 r

Sol:
O
asy  A

En
gin B

eer
Velocity of OA ing
 V OA  r

Velocity of piston B
.ne


r
Radial acceleration of OA a OA
t
Since both the vectors are parallel VOA  VB and VAB  0

 r2   
t
Tangential acceleration of AB a AB

t
a 0B

aB

a O
r
a 0A

© All rights reserved by Thinkcell Learning Solutions Pvt. Ltd. No part of this booklet may be reproduced or utilized in any form without the written permission.

34

Downloaded From : www.EasyEngineering.net


Downloaded From : www.EasyEngineering.net

ME-GATE-2019-Afternoon

aB
tan 45  r
a OA
a B  r2 tan 45  r2

51. The probability that a part manufactured by a company will be defective is 0.05. If such parts
are selected randomly and inspected, then the probability that at least two parts will be
defective is ____ (round off to two decimal places).
Key: (0.17)
Sol: Given, the probability of manufactured part will be defective i.e,
P  D   0.05  q  1  0.05  0.95

Number of trials = n= 15 (15 parts are selected)

ww The R.V X → denote the number of defective parts


 P  X  2  ?

w.E
 P  X  2  1  P  x  2   1  P  X  0   P  X  1

 
 1  15Co .p O q15  15C1 p1q14 , Using Binomial distribution

asy 
 1  q 15
 15pq14  1   0.95  15  0.05  0.95 
15 14

En
 1  0.46  0.37  1  0.83  0.17

gin
52.
eer
The figure shows a pouring arrangement for casting of a metal block. Frictional losses are

ing
negligible. The acceleration due to gravity is 9.81 m / s2 . The time (in s, round off to two
decimal places) to fill up the mold cavity  of size 40 cm  30 cm  15 cm  is_____

.ne Atmospheric pressure

t
Filing height 2
1

20 cm Diameter 2cm

3
30 cm

Open to atmosphere

Volume to be casted
15 cm
 mold cavity 

40 cm

© All rights reserved by Thinkcell Learning Solutions Pvt. Ltd. No part of this booklet may be reproduced or utilized in any form without the written permission.

35

Downloaded From : www.EasyEngineering.net


Downloaded From : www.EasyEngineering.net

ME-GATE-2019-Afternoon

Key: (28.94)

Sol: Vg  2gh t  2  9.81 0.2


Vg  1.98m s
30  40  15 1200  4  15
t f1  
   4  198
  2   198
2

4
t f 1  28.94 sec.

53. Hot and cold fluids enter a parallel flow double tube heat exchanger at 100 °C and 15 °C,
respectively. The heat capacity rates of hot and cold fluids are Ch  200 W k and
Cc  1200 W K, respectively. If the outlet temperature of the cold fluid is 45°C, the log mean

ww temperature difference (LMTD) of the heat exchanger is ___________ K (round of to two


decimal places).
Key: (57.71)
w.E T
Sol:

asy
t h1  100C

En t h 2  82C

gin t c2  45C

t c1  15C
eer
Ch  2000 W K
ing L

Cc  1200 W K
.ne

Ch t h1  t h 2  Cc t c2  t c1  t
Heat lost by hot fluid = heat gained by cold fluid
 
 
2000 100  t h 2  1200  45  15   t h 2  82C
where 1  t h1  t c1  100  15  85o C
2  t h 2  t c2  82  45  37 o C

1  2
LMTD  m  
n  1 2 
85  37
  57.71o C
n  85 37 
 LMTD  m   57.71K

© All rights reserved by Thinkcell Learning Solutions Pvt. Ltd. No part of this booklet may be reproduced or utilized in any form without the written permission.

36

Downloaded From : www.EasyEngineering.net


Downloaded From : www.EasyEngineering.net

ME-GATE-2019-Afternoon

54. The thickness of a sheet is reduced by rolling (without any change in width) using 600 mm
diameter rolls. Neglect elastic deflection of the rolls and assume that the coefficient of friction
at the roll-workpiece interface is 0.05. The sheet enters the rotating rolls unaided. If the initial
sheet thickness is 2 mm, the minimum possible final thickness that can be produced by this
process in a single pass is _____ mm (round of to two decimal places).
Key: (1.25)
Sol: D  600mm
R  300mm;   0.05
h i  2mm
hf  ?
h i  h f  2 R

ww h i  h f  0.052  300
h i  h f  0.75

w.E
h f  2  0.75
h f  1.25mm

55.
asy
A short shoe external drum brake is shown in the figure. The diameter of the brake drum is 500

En
mm. The dimensions a = 1000 mm, b = 500 mm and c = 200 mm. The coefficient of friction
between the drum and the shoe is 0.35. The force applied on the lever F = 100 N as shown in

gin
the figure. The drum is rotating anti-clockwise. The braking torque on the drum is ____ N-m
(round off to two decimal places).

F eer
ing a

.ne
Brake
shoe
t 
c

Key: (20.34)
Sol: F

R N
c
H
RN

b
© All rights reserved by Thinkcell Learning Solutions Pvt. Ltd. No part of this booklet may be reproduced or utilized in any form without the written permission.

37

Downloaded From : www.EasyEngineering.net


Downloaded From : www.EasyEngineering.net

ME-GATE-2019-Afternoon

D  500mm, a  1000mm, b  500mm


C  200mm,   0.35, F  100N

M H  0
 F  a    R N  c    R N  b   0
100  1000    0.35  R N  200   R N  500   0
R N  232.55N
Braking torque  TB  .R N .R
500
 0.35  232.55  55   103  20.34N
2

ww
w.E
asy
En
gin
eer
ing
.ne
t

© All rights reserved by Thinkcell Learning Solutions Pvt. Ltd. No part of this booklet may be reproduced or utilized in any form without the written permission.

38

Downloaded From : www.EasyEngineering.net

Vous aimerez peut-être aussi